Sat Reading PDF

January 26, 2017 | Author: Neil Sethi | Category: N/A
Share Embed Donate


Short Description

Download Sat Reading PDF...

Description

 

Learnerator’s SAT Evidence-Based Reading & Writing Review Guide

Copyright © 2015 by Learnerator Education, Inc. All rights reserved. Neither this book nor any portion thereof may be reproduced or used in any manner whatsoever without the express written permission. Published in the United States of America.

Take  your  S   AT  prep  to  the  next  level.  Visit  www.learnerator.com   4  

 

Table of Contents Introduction  ...............................................................................................................................  4   About Us  .....................................................................................................................................  5   The 11 Things You Need to Know about the New 2016 SAT Exam  ........................  6   SAT vs. ACT  ..............................................................................................................................  9   Determining Explicit Meaning in Passages  .................................................................  19   Determining Implicit Meanings  ......................................................................................  28   Analogical  Reasoning  ...........................................................................................................  39   Citing Textual Evidence  ......................................................................................................  50   Determining Main Ideas  ....................................................................................................  59   Summarizing  ...........................................................................................................................  72   Understanding Relationships  ...........................................................................................  83   Interpreting Words and Phrases in Context  ..............................................................  102   Analyzing Word Choice  ...................................................................................................  110   Analyzing Overall Text Structure  ................................................................................  117   Analyzing Part-Whole Relationships  ..........................................................................  139   Analyzing Point of View  .................................................................................................  156   Analyzing Claims and Counterclaims  ........................................................................  173   Assessing Reasoning  ........................................................................................................  184   Analyzing  Evidence  ...........................................................................................................  193   Analyzing Quantitative Information  ..........................................................................  214   Development  .......................................................................................................................  221   Organization  ........................................................................................................................  231   Precision and Concision  ..................................................................................................  243   Style and Tone  ....................................................................................................................  248   Syntax  .....................................................................................................................................  260   Sentence Formation  ...........................................................................................................  267   Shifts in Construction  ......................................................................................................  272   Pronouns  ...............................................................................................................................  276   Possessive Determiners  ...................................................................................................  284  

Take  your  S   AT  prep  to  the  next  level.  Visit  www.learnerator.com   5  

  Grammatical Agreement  .................................................................................................  291   Frequently Confused Words  ..........................................................................................  300   Logical Comparisons  ........................................................................................................  310   Conventional Expressions  ...............................................................................................  315   Conventions of Punctuation  ...........................................................................................  323   Sign  up  for  Learnerator  SAT  Prep!  ...............................................................................  340  

Take  your  S   AT  prep  to  the  next  level.  Visit  www.learnerator.com   6  

4    

Introduction Hi  there!  We  created  this  book  to  serve  as  an  overview  of  all  the  major  concepts  that   will  be  asked  of  you  in  the  new  SAT  reading  section.  This  is  a  conglomeration  of  a   series  of  review  posts  we  originally  published  on  our  blog.  Whether  you’re  new  to   the  SAT  or  a  veteran,  we  hope  you’ll  find  this  book  helpful  in  your  preparation  for   the  exam!       -­‐The  Learnerator  Team     E-­‐mail  us  at  [email protected]  if  you  have  any  questions,  comments,  or   suggestions.  

 

Take  your  S   AT  prep  to  the  next  level.  Visit  www.learnerator.com   4  

5    

About Us Learnerator  bridges  the  gap  between  learning  and  mastery  by  curating  high  quality   practice  materials  for  a  variety  of  difficult  academic  subject  areas.  Our  online  test   prep  offers:   •

Over 300 SAT Integrated Reading & Writing practice questions



Over 300 SAT Math practice questions



Hundreds of practice questions for SAT IIs: Biology, Chemistry, French, Literature, Math 1 & 2, Physics, US History, World History



Detailed personalized statistics based on your performance



A competitive online leaderboard to see how you stand compared to others



Convenience with anytime access from an Internet-enabled device

   

Visit  us  at  www.learnerator.com   Use  SAT2016  for  10%  off  any  Learnerator  SAT  Subject  Guide  

    Take  your  S   AT  prep  to  the  next  level.  Visit  www.learnerator.com   5  

6    

The 11 Things You Need to Know about the New 2016 SAT Exam It's been a while since the College Board has revamped its flagship exam, the SAT. You may be wondering what the new 2016 SAT exam will entail. Below are a list of key differences between the old exam and the one being introduced in Spring 2016: 1. Exam  length.  It's  shorter  (sort  of)!  The  exam  is  being  cut  from  a  3  hour,  45   minute  exam  to  a  3  hour  exam  with  an  OPTIONAL  "Essay"  portion  lasting   50  minutes.     2. Section  reorganization.  The  "Writing"  and  "Critical  Reading"  sections   have  been  morphed  into  "Evidence-­‐Based  Reading  and  Writing"  (EBR&W)   with  two  subsections  for  reading  and  writing  &  language.     3. Vocab.  Remember  the  days  when  you'd  have  to  memorize  endless  flash   cards  with  words  like  "adroit"  and  "celerity"?  Probably  not,  but  I  do   because  I  am  getting  old  (I  took  the  SAT  in  2007!).  Now,  instead  of  focusing   on  rarely-­‐used,  arcane  words,  the  new  SAT  focuses  more  on  words  that   you'd  actually  use  in  college  and  in  your  career.  An  example  would  be   "synergy",  which  is  a  common  business  word.     4. Essay.  Since  it  is  now  optional,  whether  or  not  you  take  it  will  be  based  on   the  requirements  of  the  colleges  to  which  you  apply.  The  essay  is  also  no   longer  about  expressing  your  views  on  an  issue,  but  rather,  analyzing  a   source  text  of  another  person's  argument  using  evidence  from  the  passage.     5. Reading  passages.  The  passages  will  have  more  diversity,  with  the   following  weighting:  20%  U.S.  and  World  Literature,  40%  History  and   Social  Studies,  and  40%  Science  and  Math.  Each  passage  or  paired  set  will   Take  your  S   AT  prep  to  the  next  level.  Visit  www.learnerator.com   6  

7     be  500-­‐750  words  and  the  total  length  of  all  passages  will  be   approximately  3,250  words.     6. Graphical  representations.  The  Reading  &  Writing  section  is  going  to   oriented  around  command  of  evidence  in  context  (i.e.,  words  in  a  history   vs.  a  science  context  have  different  meanings).  This  also  applies  to   graphical  representations  of  facts,  which  is  going  to  be  emphasized  more  in   this  exam.     7. Historical  documents.  Each  exam's  Evidence-­‐Based  Reading  and   Writing  section  will  feature  either  a  passage  from  an  important  U.S.   historical  document  (like  the  Declaration  of  Independence),  or  from   landmark  speeches  or  arguments  from  important  people  like  JFK,  Martin   Luther  King,  or  Winston  Churchill  (they  are  not  necessarily  all  Americans).   This  may  present  a  disadvantage  to  international  students;  so  if  that  is  you,   make  sure  to  note  this.  Even  though  the  College  Board  says  no  prior   knowledge  is  needed  to  answer  these  questions,  it  certainly  helps  to  have   the  context  of  the  documents  ahead  of  time.     8. The  math  section.  The  old  math  section  of  the  SAT  was  a  bit  willy-­‐nilly   and  spanned  many  areas.  The  new  Math  section  is  more  narrowly  defined   and  introduces  a  section  called  "Problem  Solving  and  Data  Analysis",  which   introduces  some  basic  concepts  of  statistics.  For  more  on  this,  go  to   our  SAT  Review  page.     9. Calculator  usage.  There  are  now  some  portions  of  the  new  SAT  that  does   not  allow  for  the  use  of  calculators.     10. Delivery.  The  SAT  will  be  available  in  both  digital  as  well  as  paper  forms.    

Take  your  S   AT  prep  to  the  next  level.  Visit  www.learnerator.com   7  

8     11. Scoring.  Hooray!  No  more  guessing  penalty!  You  will  be  given  points  for   correct  answers,  but  blank  and  incorrect  answers  will  not  earn  you  any   deductions.  The  scoring  returns  to  400-­‐1600,  from  the  current  600-­‐2400   range.  You  are  also  going  to  get  separate  test  scores  and  subscores,  shown   below:  

Summary: It

is interesting to note that 2013 was the first year that the ACT

surpassed the SAT in terms of the number of exams taken. The ACT has become viewed as more representative of the skills necessary for college and career success, and this new SAT is the College Board's response. The result is an SAT that is closer to the ACT with perhaps an even greater emphasis on the ability to analyze sources, which is also where the CB is taking their AP exams.

 

 

Take  your  S   AT  prep  to  the  next  level.  Visit  www.learnerator.com   8  

9    

SAT vs. ACT Among  the  Internet’s  most  oft-­‐Googled  questions  is:  “What  is  the  difference  between   the  ACT  and  the  SAT?”  Now  that  the  SAT  is  changing,  the  answer  will  no  doubt   become  more  nuanced.   Below  is  a  table  summarizing  the  differences  and  similarities  between  the  two  tests.   Read  on  for  a  more  detailed  explanation  of  what’s  in  the  table.    

ACT  

SAT  

LOGISTICS  

*Offered  6x/year  

*Offered  5x/year  

*$38.00  without  essay  

*$52.50  now  (may  change)  

*$54.50  with  essay   ACHIEVEMENT  v.  

*Measures  what  you've  

*Measures  how  well  you  

APTITUDE  

learned  in  school  

are  likely  to  do  in  school  in   the  future  

ENGLISH  

*75  multiple-­‐choice  

*44  multiple-­‐choice  

questions,  4  answer  

questions,  4  answer  

choices  per  question.  

choices  per  question.  

*How  much  have  you  

*How  good  are  your  

learned  about  

grammar/rhetoric  skills?  

grammar/usage?  

*Called  “Writing  and   Language,”  integrated  with   reading  

READING  

*40  multiple-­‐choice  

*52  multiple-­‐choice  

questions,  4  answer  

questions,  4  answer  

choices  per  question  

choices  per  question  

*4-­‐8  passages  about  

*4-­‐6  passages  about  

science,  humanities,  

history/social  studies,  

history/social  studies,  

science,  literature.  

literature*Measures  

*Measures  reading  

Take  your  S   AT  prep  to  the  next  level.  Visit  www.learnerator.com   9  

10     reading  comprehension  

comprehension   *Contains  charts,  graphs,   visuals  

SCIENCE  

*40  multiple-­‐choice  

N/A  

questions   *Data  Representation,   Research  Summaries,  and   Conflicting  Viewpoints.   *Charts/graph/visual   analysis   MATH  

*60  questions,  5  answer  

*57  questions,  4  answer  

choices*All  multiple  choice   choices  

ESSAY  

*Pre-­‐algebra  and  Plane  

*12  grid-­‐ins  

Geometry  most  heavily  

*Algebra  most  heavily  

represented  

represented  

*Can  use  calculator  on  

*Can  use  calculator  on  

whole  test  

most  of  test  

*Optional  

*Optional  

*Give  your  opinion  on  a  

*Analyze  someone  else’s  

topic*Scored  out  of  12  

opinion  on  a  topic   *Scored  out  of  8  

SCORING  

*Scored  out  of  36  (average   *Scored  out  of  1600  (adds   of  all  sections)  

both  area  scores)  

*7  subscores  

*7  subscores  

  The  SAT’s  redesign  is  due  to  a  number  of  factors,  including  the  fact  that  the  ACT  is   now  slightly  more  popular  and  –  many  say  –  more  fair,  or  at  least  a  more  accurate   predictor  of  college  and  career  readiness.  But  even  with  the  redesign,  there  are  

Take  your  S   AT  prep  to  the  next  level.  Visit  www.learnerator.com   10  

11     some  key  differences  –  and  some  new  key  similarities.    

LOGISTICS:     Timing   Both  tests  can  be  taken  more  than  once,  in  either  junior  or  senior  year.  The  ACT  is   administered  six  times  a  year  in  the  United  States  (in  September,  October,   December,  February,  April,  and  June),  while  the  SAT  is  administered  5  times  (in   December,  January,  March,  May,  and  June).  The  new  SAT  will  be  administered  for   the  first  time  in  2016.     For  both  the  ACT  and  the  SAT,  you  can  take  the  test  in  your  junior  and  senior  year.   (In  fact,  we  recommend  taking  the  test  twice  if  you’re  not  thrilled  with  your  original   junior-­‐year  score,  as  ACT  Math  covers  typical  12th-­‐grade  topics  and  SAT  Integrated   Reading  and  Writing  will  include  some  college-­‐level  passages  –  so  it  will  be  helpful   to  go  into  the  test  feeling  more  confident  and  prepared!)     Costs   The  ACT  costs  $38.00  without  the  writing  portion  and  $54.50  with  the  writing.   Currently,  the  SAT  costs  $52.50,  though  this  might  change  now  that  the  new  SAT’s   Writing  portion  is  optional.     OK,  BUT  WHAT’S  THE  REAL  DIFFERENCE?     The  common  answer  to  this  is  that  the  SAT  measures  aptitude  while  the  ACT   measures  achievement.     Aptitude   Aptitude  tests  measure  how  well  you  are  likely  to  perform  a  task  based  on  the  skills   you  already  have,  or  have  developed  over  12  years  of  formal  education.  “Aptitude”  

Take  your  S   AT  prep  to  the  next  level.  Visit  www.learnerator.com   11  

12     has  been  compared  to  talent  or  intelligence,  and  aptitude  tests  ask  questions  that   measure  how  you  reason  or  think.  (Of  course,  there  are  multiple  ways  to  be  talented   and  intelligent,  and  multiple  ways  to  nurture  and  extend  these  qualities  –  which  is   one  reason  that  the  SAT,  which  measures  and  privileges  only  a  few  types  of  aptitude,   has  drawn  so  much  criticism.)     Achievement   On  the  other  hand,  achievement  tests,  like  the  ACT,  measure  how  well  you   can  already  perform  certain  tasks  based  on  what  you  have  already  learned  over  12   years  of  formal  education.  They  measure  skills  or  knowledge,  and  as  such  have  been   called  a  more  accurate  predictor  of  whether  students  are  ready  for  college  and  the   workplace.   However,  now  that  the  SAT  is  changing,  it  will  have  much  more  in  common  with  the   ACT  than  it  did  previously.  The  debate  over  aptitude  versus  achievement  is  a  hot   topic  in  education,  and  well  beyond  the  scope  of  this  post.  The  essential  thing  to   keep  in  mind  is  this:  if  you  work  hard  in  school  and  out  of  it,  you  can  and  will  learn   the  skills  that  are  essential  to  succeeding  on  both  the  SAT  and  the  ACT,  as  well  as  in   college,  the  workplace,  and  life.    

Take  your  S   AT  prep  to  the  next  level.  Visit  www.learnerator.com   12  

13     OK,  BUT  WHAT  DOES  THIS  ACTUALLY  MEAN  FOR  ME?     Unlike  the  new  SAT,  which  is  divided  into  two  mandatory  and  one  optional  section,   the  ACT  is  separated  into  four  mandatory  portions  and  one  optional  section.  Those   portions  are:     ACT  ENGLISH  which  features  75  questions  about  a  variety  of  strategically  “messed   up”  passages  to  measure  a  student’s  ability  to  recognize,  diagnose,  and  correct   problems  of  grammar  and  usage,  punctuation,  sentence  structure,  strategy,   organization,  and  style.  In  other  words,  the  ACT  English  test  measures  how  much   you’ve  learned  about  the  conventions  of  the  English  language  and  strategies  for   good  writing.  There  are  four  answer  choices  per  question.     ACT  READING  uses  four  to  eight  passages  and  forty  questions  to  measure  your   ability  to  comprehend  main  ideas,  significant  details,  implied  and  stated  ideas,   specific  vocabulary  words,  and  text  structures  like  comparison  and  sequence  of   events.  The  passages  might  be  single,  long  selections  or  paired,  short  selections.  The   test  promises  that  this  is  “the  type  of  reading  required  in  first  year  college  courses,”   and  the  topics  center  on  social  studies,  humanities,  natural  sciences,  and  literary   narratives.  In  other  words,  ACT  Reading  measures  how  well  you  have  learned  to   read  a  diverse  array  of  texts  over  the  past  11  years.  There  are  four  answer  choices   per  question.     Now  that  the  SAT  has  collapsed  its  Reading  and  Writing  sections  into  one  integrated   section  (“Evidence-­‐Based  Reading  and  Writing”)  its  structure  is  remarkably  similar   to  ACT  English  and  ACT  Reading.  SAT  Evidence-­‐Based  Reading  and  Writing  is   now  comprised  of  44  Writing  and  Language  questions  and  52  Reading  questions   about  4-­‐6  passages  (most  long,  but  with  some  paired  shorter  selections)  across   topics  in  history  and  social  studies,  science,  and  US  and  world  literature.  SAT   EBRW  measures  your  understanding  of  words  in  context  (no  more  of  those  pesky   “SAT  words”  like  “adamantine”  or  “vitreous”),  your  command  of  evidence,  and  your   Take  your  S   AT  prep  to  the  next  level.  Visit  www.learnerator.com   13  

14     ability  to  analyze  text  structure,  assess  implied  and  explicit  meanings,  and   understand  purpose.  The  new  SAT  EBRW  section  will  now  have  four  answer  choices   per  question  instead  of  five.  Notably,  the  new  SAT  EBRW  section  will  include  charts   and  graphs;  understanding  and  analyzing  visuals  is  a  type  of  literacy,  or  reading,   too!     The  ACT  Reading  section  does  not  offer  charts  or  graphs,  because  there  is  a  whole   section  dedicated  almost  exclusively  to  visual  data.  ACT  Science  is  its  own  40-­‐ question  section,  comprised  of  Data  Representation,  Research  Summaries,  and   Conflicting  Viewpoints.  This  section  doesn’t  actually  measure  how  much  you  know   about  science.  Instead,  it  measures  how  well  you  can  understand,  analyze,  and   synthesize  scientific  data  –  how  well  you  can  read  a  chart  or  a  graph,  how  deeply   you  understand  two  different  ideas  about  scientific  concepts,  whether  or  not  you   can  think  critically  and  make  an  inference  about  the  data  in  front  of  you.  Each   answer  has  four  options.     Last,  ACT  Math  has  sixty  questions  with  five  options  each  (not  four  like  on  all  the   other  sections  of  the  test)  about  topics  that  range  from  pre-­‐Algebra  through   Trigonometry.  The  most  heavily  represented  subjects  on  the  test  are  Pre-­‐Algebra   and  Plane  Geometry.  The  ACT  measures  your  mathematical  reasoning  skills  as  well   as  much  you’ve  absorbed  in  your  math  classes  over  the  past  12  years.  You  will  be   able  to  use  a  calculator  on  every  question  on  the  test.     The  SAT  Math  section  has  57  questions  total,  45  with  four  answer  choices  and  12   “Student-­‐Produced  Responses”  (i.e.  “grid-­‐ins”),  one  of  which  involves  showing  your   work  and  demonstrating  your  thinking  in  response  to  a  word  problem.  Of  the  four   content  domains  (“Heart  of  Algebra,”  “Passport  to  Advanced  Math,”  “Problem  Data   Analysis,”  and  “Additional  Topics”),  Heart  of  Algebra  is  the  biggest,  so  make  sure  you   know  your  algebra  before  you  take  this  test.  Also,  the  new  SAT  will  now  present   “item  sets”  –  a  group  of  related  questions  that  ask  you  to  respond  to  the  same  set  of   data/graphs/stimuli.  You  won’t  be  able  to  use  your  calculator  for  20  questions  on   Take  your  S   AT  prep  to  the  next  level.  Visit  www.learnerator.com   14  

15     the  SAT  Math  test.     Perhaps  the  biggest  difference  between  the  two  tests  is  the  Optional  Essay.  The   ACT  will  give  you  two  perspectives  on  a  common  topic  relevant  to  high  school   students  (such  as  curfews  and  drivers’  licenses)  and  ask  you  to  contribute  your   opinion.  You  will  have  to  demonstrate  that  you  understand  both  perspectives,  that   your  opinion  is  based  in  evidence  and  reasoning,  and  that  your  writing  conforms  to   standard  English  conventions.     The  SAT’s  optional  essay  will  ask  you  to  analyze  a  “source  text”  (which  will  have   very  little  to  do  with  common  topics  relevant  to  high  school  students)  and  figure  out   what  the  writer  is  saying  and  how  s/he  builds  an  argument.  You  will  have  to  prove   that  you  understand  how  a  writer  constructs  an  effective  argument,  using  evidence   from  the  text.  The  SAT  is  NOT  looking  for  your  opinion,  but  for  how  adroitly  you   marshal  and  cite  evidence,  and  how  fluently  you  write.     These  are  very  different  types  of  essays,  so  be  sure  you  have  plenty  of  practice  for   both.      

Take  your  S   AT  prep  to  the  next  level.  Visit  www.learnerator.com   15  

16     HOW  WILL  MY  TEST  BE  SCORED?     SAT  Scoring   The  SAT  is  going  back  to  its  original  1600-­‐point  scale  (#oldschool),  which  means   that  the  highest  score  you  can  earn  is  a  1600  (composite)  with  an  800  EBRW  score   (area  score)  and  an  800  Math  (area  score).  However,  it  gets  a  little  more   complicated  after  this:     You  get  3  test  scores  (from  10-­‐40)  for  (1)  Reading,  (2)  Writing  and  Language,  and   (3)  Math.     You  get  2  cross-­‐test  score  (from  10-­‐40)  for  (1)  Analysis  in  Science  and  (2)  Analysis   in  History/Social  Studies.     You  get  7  subscores  (from  1-­‐15)  for  (1)  Command  of  Evidence  and  (2)  Relevant   Words  in  Context  (Reading),  (3)  Expression  of  Ideas  and  (4)  Standard  English   Conventions  (Writing  and  Language),  (5)  Heart  of  Algebra,  (6)  Problem  Solving  and   Data  Analysis,  and  (7)  Passport  to  Advanced  Math.     Essays  are  scored  from  2-­‐8,  with  two  readers  who  will  assign  you  a  score  of  1-­‐4.     ACT  Scoring   On  the  ACT,  you  will  receive  a  Composite  Score  (out  of  36)  that  averages  each  of   your  four  test  scores  in  Math,  English,  Reading,  and  Science.  The  highest  score  you   can  receive  on  each  section  as  well  as  on  the  test  itself  is  a  36.     You  will  receive  seven  subscores:  (1)  Usage  and  Mechanics,  (2)  Rhetorical  Skills,   (3)  Pre-­‐Algebra  and  Elementary  Algebra,  (4)  Intermediate  Algebra/Coordinate   Geometry,  (5)  Plane  Geometry/Trigonometry,  (6)  Social  Studies/Natural  Sciences   reading  skills,  and  (7)  Arts/Literature  reading  skills.     Take  your  S   AT  prep  to  the  next  level.  Visit  www.learnerator.com   16  

17     These  are  scored  from  1-­‐18,  and  have  no  mathematical  bearing  on  your  composite   score.   Your  essay  is  scored  out  of  12,  with  two  readers  giving  you  a  score  between  1-­‐6.  

   

 

Take  your  S   AT  prep  to  the  next  level.  Visit  www.learnerator.com   17  

18    

            Part  I:  Information  &  Ideas  (Reading)    

 

Take  your  S   AT  prep  to  the  next  level.  Visit  www.learnerator.com   18  

19    

Determining Explicit Meaning in Passages “Determining explicit meanings” is the College Board’s highfalutin’ way of saying, “reading the text closely and understanding what it says.” All the SAT wants you to do is: 1) Pay

attention to detail, and

2) Understand

all the information that is already there.

This applies both to the passage and to the question and all of the answer choices. The beauty of the multiple-choice test is that all the answers are already there; you just have to know where to look. This means you have to consider every scrap of information the College Board gives you. The little blurb before each passage is crucial – they are giving you free information about

the passage, which will help you understand the passage better

and answer questions about it. There are two equally important ways to go about this.

1. Annotate the text. Underline, star, or make notes next to all the information that seems like it might be relevant. For example, take the blurb and the first paragraph of this passage, taken directly from the SAT website: This passage is adapted from a novel written by a woman in 1899. The novel was banned in many places because of its unconventional point of view:

Take  your  S   AT  prep  to  the  next  level.  Visit  www.learnerator.com   19  

20    

It was eleven o'clock that night when Mr. Pontellier returned from Klein's hotel. He was in an excellent humor, in high spirits, and very talkative. His entrance awoke his wife, who was in bed and fast asleep when he came in. He talked to her while he undressed, telling her anecdotes and bits of news and gossip that he had gathered during the day. From his trousers pockets he took a fistful of crumpled bank notes and a good deal of silver coin, which he piled on the bureau indiscriminately with keys, knife, handkerchief, and whatever else happened to be in his pockets. She was overcome with sleep, and answered him with little half utterances.

The first thing you’ll want to do is circle all the important information:

a)

This is a novel (ok, so fiction – that means you’ll have to be prepared for

questions on characterization, plot, literary devices…) b)

Written by a woman in 1899 (ok, so old-fashioned, probably – be prepared for

some old-fashioned language) c)

Banned in many places for being unconventional (which undoubtedly means

there will be a question about something related to this because the College Board wouldn’t drop that juicy morsel unless it had a reason – so this might contain themes considered “unconventional” in 1899) See how much information you can glean just from the little blurb? Now turn your attention to the passage. Take note of:

1)

What time it was when Mr. P came home (pretty late! 11pm, and his wife was

fast asleep!) 2)

Where Mr. P was coming from (a hotel… why was he at a hotel at 11 pm

without his wife? Work event? Party?) 3) Mr.

P’s mood (clearly good – something good must have happened. Take

special note of this because the SAT loves to ask about tone and mood!) 4) Other

thoughts: so his wife is in bed asleep at 11 pm but he decides this is the

Take  your  S   AT  prep  to  the  next  level.  Visit  www.learnerator.com   20  

21     time to tell her about “anecdotes and bits of news and gossip”? Take note of how she is reacting to him, too. Annotations aren’t the only way to record your observations of the text. It’s also crucial that you adopt a second strategy:

2. Translate the text. This won’t always be necessary, but the more complex, old-fashioned, or specialized a passage is, the more likely you will need to translate at least parts of it into plain English. You can do this while you annotate, to save time. Let’s look at the passage again, this time with some translations and annotations in red:

It was eleven o'clock that night when Mr. Pontellier returned from Klein's hotel. He was in an excellent humor (good mood), in high spirits, and very talkative. His entrance awoke his wife, who was in bed and fast asleep when he came in. He talked to her while he undressed, telling her anecdotes (stories) and bits of news and gossip that he had gathered during the day. From his trousers pockets he took a fistful of crumpled bank notes and a good deal of silver coin (lots of money), which he piled on the bureau indiscriminately with keys, knife, handkerchief, and whatever else happened to be in his pockets (piling all of his stuff up – so he’s messy). She was overcome with sleep (tired), and answered him with little half utterances (halfsentences/incoherent).

When it comes to translating the text, sometimes you’ll just get stuck on a word. Say you didn’t know “anecdotes” in the fourth line. You probably know “bits of news and gossip,” though, so it’s possible to figure out from context that he’s giving her a very chatty, thorough account of his day. You won’t need to know what every word means to figure out a general picture from the text, or even to do a translation. Take  your  S   AT  prep  to  the  next  level.  Visit  www.learnerator.com   21  

22     Now that you’ve annotated and translated the text, you’ll want to turn your attention to the question, and do the same, making sure you understand completely what is being asked. Question: “Mr. Pontellier’s conduct in this passage could most likely be described as…” A) Generous B) Good-natured C) Passionate D) Self-centered

Remember to annotate and/or translate. What is the question asking you? “Mr. Pontellier’s conduct (behavior) in this passage could most likely be described as…” So they’re asking about his behavior. Based on the few things we know about him – he got home late, he’s very chatty though his wife is asleep, he’s in a good mood – it’s pretty easy to answer this question, so let’s consider each answer choice. Option A, Generous.

Is there any evidence to support this?

Yes, there’s mention of the money in his pockets, which may serve to distract you, but the passage doesn’t say anything about him giving away or otherwise doing anything with that money. Cross out A. Option B, Good-natured.

Well, he is in “an excellent humor,” (good mood) but that

doesn’t necessarily make him good-natured. And remember – the question asks

Take  your  S   AT  prep  to  the  next  level.  Visit  www.learnerator.com   22  

23     about his behavior that evening, not his general personality. This isn’t a terrible choice, but leave it for now and consider the others. Option C, Passionate.

Is there any evidence that he’s being passionate? Certainly he

seems happy and excited, but there’s little evidence of passion – especially as passion generally tends to be directed towards something or someone, whereas a good mood is general. Cross that one off, and then move on to the last answer. Option D, Self-centered.

Remember that the question is asking about how heacted that

night. Think back to all the actions you noted when you were reading the text: he came in late, woke his sleeping wife with his chatter, and piled all his stuff everywhere in their shared room. Though he may befeeling good-natured (B), he’s acting pretty self-centered. Clearly, that is the only correct answer to what the College Board is asking here. All you need to do is pay attention to detail. -------------That’s a fairly simple passage, though, so let’s do this once more with a slightly harder passage. The following passage is from the same book. An indescribable oppression, which seemed to generate in some unfamiliar part of her consciousness, filled her whole being with a vague anguish. It was like a shadow, like a mist passing across her soul's summer day. It was strange and unfamiliar; it was a mood. She did not sit there inwardly upbraiding her husband, lamenting at Fate, which had directed her footsteps to the path, which they had taken. She was just having a good cry all to herself. The mosquitoes made merry over her, biting her firm, round arms and nipping at her bare insteps.

The little stinging, buzzing imps succeeded in dispelling a mood which might have held her there in the darkness half a night longer.

Take  your  S   AT  prep  to  the  next  level.  Visit  www.learnerator.com   23  

24     Again, let’s start annotating and translating:

An indescribable oppression (really bad feeling), which seemed to generate (come from) in some unfamiliar part of her consciousness (unfamiliar part of her consciousness – so she doesn’t know why she’s unhappy, or where the unhappiness is coming from… maybe her subconscious?), filled her whole being with a vague anguish.(She was sad) It was like a shadow, like a mist passing across her soul's summer day. (Imagery/sadness) It was strange and unfamiliar; it was a mood. She did not sit there inwardly upbraiding (criticizing) her husband, lamenting (being sad at) at Fate, which had directed her footsteps to the path which they had taken (so she’s NOT sitting there criticizing her husband and hating her fate? This is an interesting thing for the narrator to tell us… And it’s interesting that she would even think in terms of “fate,” instead of her own decisions, “directing her footsteps.”) She was just having a good cry all to herself. (as one does late at night) The mosquitoes made merry over her (mosquito bloodsucking party), biting her firm, round arms and nipping at her bare insteps (feet).

The little stinging, buzzing imps (ie mosquitos) succeeded in dispelling a mood (they got rid of her bad mood?!) which might have held her there in the darkness half a night longer. (guess they made her so itchy she forgot to be sad…?)

On the actual test, you won’t necessarily have time to write yourself insanely detailed notes like I did here. That’s OK! Instead of writing “really bad feeling” or making notes about “the unfamiliar part of her consciousness” you can use shorthand – a little frowny face, a question mark, an underline, an exclamation, a star. And it’s OK if you don’t know all the words, like “upbraiding” or “lamenting.” The rest of the passage – with its words like “oppression,” “anguish,” “shadow,” “mist passing across her soul,” “good cry” – should serve to cement the impression that whatever “upbraid” or “lament” might mean, it’s not good.

Take  your  S   AT  prep  to  the  next  level.  Visit  www.learnerator.com   24  

25     So now we can turn our attention to a question based on this passage: By depicting in such detail Mrs. Pontellier’s emotional state, the narrator is trying to suggest that… A) Mr.

Pontellier is worldly and callous

B) Mrs.

Pontellier is clearly upset about something that happened before Mr.

Pontellier came back home C) Mrs.

Pontellier does not enjoy being a mother

D) Mrs.

Pontellier is unused to confronting her own unhappiness

Remember to really think about what the test is asking you. The question asks about the narrator’s intentions here in showing you Mrs. Pontellier’s state – i.e., Based on this unhappy episode, what does the narrator want to tell you about this character? You can easily answer this question based on explicit textual evidence. Consider option A. Mr. Pontellier may indeed be worldly and callous (not very nice), but there is no evidence from this particular part of the passage (which is all the question asks about!) that this is why his wife is so upset. You can easily cross this one out – remember, you only want to look for what the text says explicitly, and here the question asks only about one particular part of the passage, in which Mr. Pontellier is NOT described in any detail. (In fact, all it says about him is that Mrs. Pontellier did NOT use this time to criticize him in her mind – but it gives no specific descriptions as to his personality.) Next, try option B. As with option A, this MIGHT be a possibility – except that there is absolutely no indication of this in the actual text. It says nothing about

Take  your  S   AT  prep  to  the  next  level.  Visit  www.learnerator.com   25  

26     what happened before Mr. Pontellier came home, so you can cross this one out immediately. Look at option C. Again, remember to look only at the part of the text that the question is asking about. There is no mention about being a mother in this part of the text, and in fact no mention of children at all. This clearly cannot be the answer. Now we come to option D: Mrs. Pontellier is unused to confronting her own unhappiness. Consider the evidence here, because the text has pretty much told you that this is the case: She feels “indescribably oppressed” (as though she lacks the words/experience to describe it – presumably, if she was very familiar with and deeply aware of this feeling, she would know exactly how to describe it and what the reasons for it are). Her unhappiness “generates from an unfamiliar part of her consciousness” – i.e., she has no idea where it came from. And if she doesn’t know WHY she’s unhappy, then this is pretty good evidence that she doesn’t think a whole lot about her own unhappiness. Clearly, this is the only reasonable choice. Now, a critical reader will no doubt make the connection between Mrs. Pontellier’s anguished crying and her inconsiderate husband. A good critical reader might also connect this to the blurb before the passage, which tells you that this book had an “unconventional point of view in 1899” – a point of view that will no doubt have something to do with the fact that the marriage between the main characters seems very unfulfilling.

Take  your  S   AT  prep  to  the  next  level.  Visit  www.learnerator.com   26  

27     These critical reading skills will be very helpful in the next chapter, Determining Implicit Meanings.

 

Take  your  S   AT  prep  to  the  next  level.  Visit  www.learnerator.com   27  

28    

Determining Implicit Meanings The College Board defines “determining implicit meanings” as “drawing reasonable inferences and logical conclusions from the text.” If determining explicit meanings is figuring out what the text says, then determining implicit meanings is figuring out what the text suggests or implies. This sounds difficult, and in some cases it can be. But there are tricks to drawing inferences and reaching conclusions, and they are: 1) Read  closely.     Just as you do when you’re trying to figure out an “explicit” meaning, you’ll want to take notes, translate, and pay attention to what the text is saying. Understanding the surface meaning of a text will help you understand it on a deeper level, too. 2) Find  evidence.  

Every time you choose an answer, match up your choice with actual evidence (meaning, a quote) from the text. If you can’t do that, or if the evidence you’ve chosen doesn’t convince you, then you are probably not making a supportable conclusion and you should pick another answer. Try this strategy with the following passage from the Public Library of Science blog:

As Ebola rears its ugly head in the U.S., there has been a lot of discussion about how afraid we really should be. While health officials have remained relatively calm, insisting in a tense

Take  your  S   AT  prep  to  the  next  level.  Visit  www.learnerator.com   28  

29     congressional hearing that there is no need to panic, some members of the American public have gone in the opposite direction, proclaiming a state of emergency on social media channels such as Twitter and Facebook. Donald Trump has been an especially noisy voice on the topic on Twitter, demanding that Obama stop all flights to and from West Africa and proclaiming that Ebola is actually much more contagious than the CDC says it is. At the end of September, with the announcement of the first suspected case of Ebola in the U.S., Trump expressed panic on Twitter, writing:

"The United States must immediately institute strong travel restrictions or Ebola will be all over the United States--a plague like no other! Ebola is much easier to transmit than the CDC and government representatives are admitting. Spreading all over Africa - and fast. Stop flights."

Trump’s comments suggest not only a distrust of health authorities but also a kind of hysteria, with the use of the word “plague,” that is reminiscent of the infectious disease panic seen in many other outbreaks in American history, including SARS in 2003 and bubonic plague in 1924, the latter an outbreak that famously led to the frantic firing of thousands of Latino workers and destruction of homes in low-income Latino neighborhoods because they were seen as the “source” of the disease.

The first thing you will want to do is annotate your text. Make notes next to anything that seems interesting, unusual, or otherwise notable. And if there’s something you don’t understand, translate it for yourself into plain English as best as you can. My translations and comments are in red:

As Ebola rears its ugly head in the U.S., (negative) there has been a lot of discussion about how afraid we really should be. While health officials have remained relatively calm, insisting in a tense congressional hearing that there is no need to panic, some members of the American public have gone in the opposite direction, proclaiming a state of emergency on social media channels such as Twitter and Facebook. (debate here) Donald Trump has been an especially noisy (negative-sounding) voice on the topic on Twitter, demanding that Obama stop all flights to and from West Africa and proclaiming that Ebola is actually much more contagious than the CDC says it is. At the end of September, with the announcement of the first suspected case of Ebola in the U.S., Trump expressed panic on Twitter, writing:

"The United States must immediately institute strong travel restrictions or Ebola will be all over

Take  your  S   AT  prep  to  the  next  level.  Visit  www.learnerator.com   29  

30     the United States--a plague like no other! Ebola is much easier to transmit than the CDC and government representatives are admitting. Spreading all over Africa - and fast. Stop flights."

Trump’s comments suggest not only a distrust of health authorities but also a kind of hysteria, (negative) with the use of the word “plague,” that is reminiscent of the infectious disease panic seen in many other outbreaks in American history, including SARS in 2003 and bubonic plague in 1924, the latter an outbreak that famously led to the frantic firing of thousands of Latino workers and destruction of homes in low-income Latino neighborhoods because they were seen as the “source” of the disease.(Trump is panicking in the same way that people panicked in 1924, which led to thousands racially-based firings… negative).

These are just preliminary notes, but they will help us answer the next two questions using the second strategy, matching up every option with a quote / evidence from the text.

How does the author of this passage most likely feel about Trump’s comments on Twitter?

A. She finds Trump’s comments laudable and important. B. She finds Trump’s comments irrelevant and pointless. C. She is distrustful of Trump’s comments. D. She is subtly disdainful of Trump’s comments.

Now, the author never explicitly expresses an opinion about Trump. However, if you read closely, you can answer this question by matching up each option with evidence from the text. Consider A: She finds Trump’s comments laudable and important. Even if you don’t know that “laudable” means “worthy of praise,” you can figure out that important is a positive thing. Comb over the text. Is there any mention that the author finds Trump’s contribution important, other than the fact that she is mentioning him? There’s no evidence of this. In fact, all of the

Take  your  S   AT  prep  to  the  next  level.  Visit  www.learnerator.com   30  

31     words that she uses to describe Trump’s contributions are NEGATIVE – she calls him “noisy” and says he is demonstrating “hysteria,” neither of which are positive words. Go ahead and cross out Option A. Now, Option B – Again, look at these words: “irrelevant and pointless.” Both are negative, which matches with the negative tone the author uses to describe Trump’s comments. But is there any evidence that she finds Trump “irrelevant”? Surely if he was “pointless,” the author wouldn’t mention him at all. Nor would she say that “Trump’s comments suggest… a kind of hysteria… reminiscent of the infectious disease panic… in 1924… that famously led to the frantic firing of thousands of Latino workers and destruction of homes in low-income Latino neighborhoods.” In fact, Trump’s comments are the opposite of “irrelevant” – if panics like his have led to people being fired and homes being destroyed, then clearly words like his have some resonance, even if they are a negative rather than a positive part of this national conversation.

Clearly, Option B can’t be correct.

Check out Option C and try to find evidence for the fact that the author is distrustful of Trump’s comments. True, the word “distrustful” shows up in the passage… although the passage suggests that it is Trump who is distrustful of authorities, not the author. True, the author seems like she does not agree with Trump, calling him “noisy” and “hysterical” and subtly suggesting that his comments could lead to widespread panic, firing, and destruction of homes. So this might be a good answer – there is some evidence here. Let’s consider Option D and see if there is more or better evidence: The author is “subtly disdainful” – i.e. she has a negative impression of Trump’s comments. Clearly, by describing Trump as “noisy,” “hysterical,” and “having an attitude

Take  your  S   AT  prep  to  the  next  level.  Visit  www.learnerator.com   31  

32     “reminiscent of widespread panics that led to firings and destructions of homes,” the author is demonstrating a disdain of Trump. So it’s between C and D. Which word better describes the author’s attitude? Is there evidence that she doesn’t trust Trump, or is there more evidence that she thinks negatively of him? While C may be a possible answer, D is a better answer because there is more evidence

for it. Which brings us to Strategy #3 for Determining Implicit

Meanings:

When in doubt, go with the option that has more evidence.

---------------------------------Let’s try one more example from the same article:

At the same time, a good deal of the coverage of the Ebola crisis in West Africa has focused on the “irrationality” and “ignorance” of West Africans, choosing to visit traditional healers instead of doctors and ignoring warnings from health officials that traditional burial rituals can hasten the spread of the disease. Indeed, this “lack of faith in Western medicine” is now being addressed by local and foreign health officials who are running educational programs in places like Liberia and Sierra Leone in order to ensure that the people exposed to the illness have the correct scientific information.

But while certain cultural practices and lack of education have certainly played a role in the rapid spread of Ebola in West Africa, the amount of attention we have paid to the “ignorance” and “irrationality” of people living in these developing countries has not been balanced with an equal amount of attention to our own developed nations’ manifestations of similar irrational health beliefs. Are we really that much more knowledgeable and rational than our West African counterparts, or do our irrationalities and psychological missteps simply take different cultural forms? And do our ignorance levels and irrationalities have the potential to be just as dangerous to our health as seeking out a local healer instead of a trained healthcare professional in response to a life-threatening viral illness?

Take  your  S   AT  prep  to  the  next  level.  Visit  www.learnerator.com   32  

33     Remember, annotate and translate anything that sticks out to you:

At the same time, a good deal of the coverage of the Ebola crisis in West Africa has focused on the “irrationality” and “ignorance” of West Africans, choosing to visit traditional healers instead of doctors and ignoring warnings from health officials that traditional burial rituals can hasten the spread of the disease. Indeed, this “lack of faith in Western medicine” is now being addressed by local and foreign health officials who are running educational programs in places like Liberia and Sierra Leone in order to ensure that the people exposed to the illness have the correct scientific information.

But while certain cultural practices and lack of education have certainly played a role in the rapid spread of Ebola in West Africa, the amount of attention we have paid to the “ignorance” and “irrationality” of people living in these developing countries has not been balanced with an equal amount of attention to our own developed nations’ manifestations of similar irrational health beliefs. Are we really that much more knowledgeable and rational than our West African counterparts, or do our irrationalities and psychological missteps simply take different cultural forms? And do our ignorance levels and irrationalities have the potential to be just as dangerous to our health as seeking out a local healer instead of a trained healthcare professional in response to a life-threatening viral illness?

As you can see, I bolded a lot of information – all because it stood out to me in some way. The words in quotes hold a lot of interest – clearly, the author is trying to communicate that SOMEONE believes that West Africans are “irrational” and “ignorant,” but by putting the words in quotes, she is demonstrating distance from that opinion. I also bolded “now being addressed” because this “lack of faith in Western medicine” is clearly important enough that local AND foreign health officials feel the need to address it. Next, I bolded parts of the first sentence of the next paragraph, paying special attention to those words “but while” – transition words like BUT, HOWEVER, WHILE, and YET (among others) always demonstrate a shift in thinking or a

Take  your  S   AT  prep  to  the  next  level.  Visit  www.learnerator.com   33  

34     contradiction, and I wanted to be prepared to answer a question about this shift in thinking. In translation, the author is saying “There has been some ignorance in West Africa, which helps spread Ebola, but we (meaning, Americans) have not paid attention to our own ignorance.” Now the purpose of the article shifts slightly to OUR ignorance and irrationalities, rather than West African ignorance and irrationalities. She finished by saying that OUR ignorance levels can be “dangerous to our health.” See if you can use our strategies (read closely, annotate, find evidence) to answer the following question:

Based on the passage, the author would most likely agree with which of the following aphorisms?

A. Don’t count your chickens before they are hatched. B. Don’t throw stones if you live in a glass house. C. Be the change you wish to see in the world. D. Even if sickness distracts from thoughts, all that is needed is the will to love.

Even if you don’t know what the word “aphorism” means (it is a fancy word for “common saying”), chances are that you recognize these idiomatic English expressions. This question will require you to make some inferences based on the passage because clearly, the author never uses any of these sayings. So let’s examine them one by one: Option A: “Don’t count your chickens before they’re hatched” means “don’t make plans based on things that haven’t happened yet.” Skim the text. What evidence is there that the author agrees with this advice? Other than the fact that this is good general advice, the author makes no mention of plans or hopes for the future. Cross this one off.

Take  your  S   AT  prep  to  the  next  level.  Visit  www.learnerator.com   34  

35     Option B: “Don’t throw stones if you live in a glass house” means “Don’t criticize others for faults that you have yourself.” Skim the text. Is there mention of criticism? Yes! We have criticized West Africans for being “irrational” and “ignorant” while ignoring our own “missteps and irrationalities” that are “just as dangerous to our health.” There is certainly evidence that the author thinks that the American public is “throwing stones” despite living in a glass house made of our own irrationalities and ignorance. Keep this as an option and consider the next option: Option C: “Be the change you wish to see in the world.” This can be interpreted as “If you want to see a change in the world, then you yourself should try and make this change.” Look at the text. Is the author advocating for a specific change? Maybe – by asking “And do our ignorance levels and irrationalities have the potential to be just as dangerous to our health” she

might be subtly advocating that we stop being

ignorant and irrational. But the general tone of the article isn’t focused on making changes. Instead, the point of the article seems to be more focused on illustrating a phenomenon rather than giving a directive (in this case: stop or change). It’s still an option, but the evidence isn’t quite rock-solid. Consider Option D: This one mentions sickness (which is one of the topics of the article). But the author never says anything about the will to “love.” You could make a very weak argument that by saying that Americans can be as irrational as West Africans when it comes to health, the author is advocating that Americans start loving their West African counterparts because we all have the same irrationalities. You COULD make this argument. But read it back to yourself. It sounds pretty flimsy. Cross off D.

Take  your  S   AT  prep  to  the  next  level.  Visit  www.learnerator.com   35  

36     Now you’re left debating between B and C. Which has the most evidence? Is the author advocating for change here, or is she pointing out that Americans can be pretty hypocritical in thinking West Africans are irrational when the American public has the potential to be just as irrational? Clearly, B has the most evidence, and therefore can be the only right answer. ---------------------------------Let’s try this one more time, using the same passage. At the same time, a good deal of the coverage of the Ebola crisis in West Africa has focused on the “irrationality” and “ignorance” of West Africans, choosing to visit traditional healers instead of doctors and ignoring warnings from health officials that traditional burial rituals can hasten the spread of the disease. Indeed, this “lack of faith in Western medicine” is now being addressed by local and foreign health officials who are running educational programs in places like Liberia and Sierra Leone in order to ensure that the people exposed to the illness have the correct scientific information.

But while certain cultural practices and lack of education have certainly played a role in the rapid spread of Ebola in West Africa, the amount of attention we have paid to the “ignorance” and “irrationality” of people living in these developing countries has not been balanced with an equal amount of attention to our own developed nations’ manifestations of similar irrational health beliefs. Are we really that much more knowledgeable and rational than our West African counterparts, or do our irrationalities and psychological missteps simply take different cultural forms? And do our ignorance levels and irrationalities have the potential to be just as dangerous to our health as seeking out a local healer instead of a trained healthcare professional in response to a life-threatening viral illness?

Which of the following American practices would the author most likely deride as a “psychological misstep” akin to an Ebola patient visiting a local healer?

A. Thinking that West Africans are irrational B. Panicking about Ebola C. Refusing a flu shot D. Staying up-to-date on “correct scientific information”

Take  your  S   AT  prep  to  the  next  level.  Visit  www.learnerator.com   36  

37     To answer this question, you first need to understand it. There are two blocks here – the fact that the question uses the word “deride” (which means criticize), and the fact that the question is so long and could potentially be confusing. So, even if you’re not sure of the word “deride,” try your best to translate the question. It is using terminology from the text (“psychological misstep”) to ask: Which of the following American practices would the author most likely deride (criticize) as a “psychological misstep” (irrational/bad thing) akin to (like) an Ebola patient visiting a local healer? I.e.,

“Which AMERICAN practice would the author think is AS irrational as visiting a local healer to cure Ebola”? In other words, which action is irrational AND potentially dangerous to public health? The key here are the words “misstep” and “akin” – clearly, the answer will be negative (misstep) in a similar way (akin = comparison word) as thinking Ebola can be cured by a local healer. Looking at the answer options, you can cross one off right from the start, because the answer will clearly be negative and at least one of the answers (D) is positive. Staying up to date on correct scientific information (which is a quote from the text, but don’t let that fool you!) is clearly a POSITIVE thing, not a dangerous, irrational one. Now look at Option A: “Thinking that West Africans are irrational.” The question is asking: what is an irrational thing that Americans do? While there is evidence from the text that Americans view West Africans as irrational, and that this view is wrong, is there evidence from the text to support the second part of the question – that this behavior is akin to visiting a local healer to cure Ebola (i.e. dangerous to public health)? Think back to the earliest part of the text, in which the author subtly criticized Donald Trump for potentially inciting mass hysteria that could lead to firing and destruction of homes. That is both irrational AND problematic/destructive – so for now, let’s keep Option A. There’s some evidence for it.

Take  your  S   AT  prep  to  the  next  level.  Visit  www.learnerator.com   37  

38     Try Option B next: “Panicking about Ebola.” Is panicking about Ebola irrational? The author suggests that it is, by saying in the very first paragraph “health officials have remained relatively calm,” which suggests that the average American should probably be a bit calmer too. So yes, the author would probably agree that panicking is irrational. But is it as irrational as treating Ebola incorrectly and allowing it to spread (as suggested by that “akin”)? Probably not. You can go ahead and cross this off based on lack of evidence. Now Option C: “Refusing a flu shot.” The author doesn’t explicitly mention flu in this article. But the flu, like Ebola, is a disease, and refusing to treat a disease is a fairly irrational thing to do – especially because like Ebola, the flu is contagious. The key to determining whether this is the right answer lies in that “akin” – which of the options are actively dangerous in the same way as not treating Ebola properly? Clearly, the best and most likely answer is C: both actions (not treating Ebola OR the flu properly) are irrational AND allow a disease to spread in a dangerous way. True, we had some evidence for Option A, but the scenario in Option C corresponds most directly with the scenario outlined in the text, so that’s the best answer – there is more evidence for it based on what the text says. This is a form of “analogical reasoning,” which is a skill you will develop further in the next section.  

Take  your  S   AT  prep  to  the  next  level.  Visit  www.learnerator.com   38  

39    

Analogical Reasoning K,  pop  quiz:     Analogies  :  SAT  ::  VCRs  :  ____________________   A,  Entertainment   B,  DVDs   C,  Theaters   D,  Limited-­‐edition  boxed  sets     Give  up?     At  the  risk  of  sounding  old,  analogies  were  huge  on  the  SAT  Verbal  section  when  I   was  a  wee  lass,  nervously  sharpening  my  #2  pencil  before  the  day  of  the  big  test.  (By   the  way,  the  answer  to  that  quiz  is  A  –  just  as  this  form  of  analogical  reasoning  is  no   longer  relevant  to  the  SATs,  VCRs  are  no  longer  relevant  to  the  wider  world  of   entertainment.)     Now  that  analogies  have  gone  the  way  of  pogs,  dinosaurs,  and  the  VCR,  you  may   think  that  you  don’t  need  to  worry  about  them  anymore.     Unfortunately,  though  the  question  format  has  changed,  the  SAT  will  still  test  you  on   whether  or  not  you  can  think  analogically:  whether  you  can  deftly  and  accurately   compare  one  thing  to  another,  whether  you  can  draw  conclusions  based  on   similarities  and  differences,  whether  you  can  logically  apply  information  from  one   scenario  into  another.  (The  word  analogy  comes  from  the  Greek  word  for   “proportion,”  which  makes  sense  if  you  think  of  an  analogy  as  setting  up  some  sort   of  relationship  between  two  or  more  things.)     You  won’t  have  to  memorize  lists  of  words  and  figure  out  if  “chthonic:  stygian  ::   celestial  :  effulgent”,  because  let’s  face  it,  those  words  are  fairly  useless  unless  you’re   Take  your  S   AT  prep  to  the  next  level.  Visit  www.learnerator.com   39  

40     a  huge  word  nerd/studying  for  a  PhD  in  mythology  /a  passionate  H.P.  Lovecraft   fan/an  awesome  combination  of  the  three.  (The  analogy  works,  by  the  way).     But  analogies  themselves  are  a  crucial  way  to  organize  information,  to  marshal   support  for  an  argument,  to  understand  a  relationship,  or  to  impose  an  orderly   framework  on  a  narrative.  In  short,  the  College  Board  finally  wised  up,  threw  out  the   thesaurus,  and  decided  to  test  you  on  applicable  skills,  like  whether  you  can  use   critical  reasoning  to  figure  out  what  information  can  and  should  be  usefully   extrapolated  and  applied  to  a  new  situation.  A  lot  of  this  sounds  more  complicated   than  it  is;  chances  are,  if  you’ve  ever  had,  made,  or  defended  an  argument,  you’ve   already  practiced  this  skill.   Now  you  just  need  to  hone  this  skill  in  the  context  of  reading.  It  may  help  to   remember  that  making  an  analogy  is  just  another  form  of  making  an  inference  –   extrapolating  from  the  text  to  figure  out  how  the  information  applies  to  a  new   situation.     Ready  to  practice?  The  following  passage  from  Plato’s  “Apology”  details  the   conviction  and  execution  of  Socrates.   The  question  may  be  asked,  Why  will  he  persist  in  following  a  profession  which   leads  him  to  death?  Why?-­‐-­‐because  he  must  remain  at  his  post  where  the  god   has  placed  him,  as  he  remained  at  Potidaea,  and  Amphipolis,  and  Delium,   where  the  generals  placed  him.  Besides,  he  is  not  so  over  wise  as  to  imagine   that  he  knows  whether  death  is  a  good  or  an  evil;  and  he  is  certain  that   desertion  of  his  duty  is  an  evil.  Anytus  is  quite  right  in  saying  that  they  should   never  have  indicted  him  if  they  meant  to  let  him  go.  For  he  will  certainly  obey   God  rather  than  man;  and  will  continue  to  preach  to  all  men  of  all  ages  the   necessity  of  virtue  and  improvement;  and  if  they  refuse  to  listen  to  him  he  will   still  persevere  and  reprove  them.  This  is  his  way  of  corrupting  the  youth,  which   he  will  not  cease  to  follow  in  obedience  to  the  god,  even  if  a  thousand  deaths   await  him.     Take  your  S   AT  prep  to  the  next  level.  Visit  www.learnerator.com   40  

41      Mark  up  your  text  however  you  like.  Here’s  how  I  annotated  and  translated  mine:     The  question  may  be  asked,  Why  will  he  (Socrates)  persist  in  following  a   profession  which  leads  him  to  death?  Why?-­‐-­‐because  he  must  remain  at  his  post   where  the  god  has  placed  him,  as  he  remained  at  Potidaea,  and  Amphipolis,  and   Delium,  where  the  generals  placed  him.  (Socrates  was  a  military  man)   Besides,  he  is  not  so  overwise  as  to  imagine  that  he  knows  whether  death  is  a   good  or  an  evil;  and  he  is  certain  that  desertion  of  his  duty  is  an  evil.  (Would   rather  face  death,  which  may  or  may  not  be  “evil,”  than  desert  his  duty,   which  he  knows  is  “evil.”)  Anytus  is  quite  right  in  saying  that  they  should   never  have  indicted  him  if  they  meant  to  let  him  go.  For  he  will  certainly  obey   God  rather  than  man;  and  will  continue  to  preach  to  all  men  of  all  ages  the   necessity  of  virtue  and  improvement;  and  if  they  refuse  to  listen  to  him  he  will   still  persevere  and  reprove  (reprimand)  them.  This  is  his  way  of  corrupting  the   youth,  which  he  will  not  cease  to  follow  in  obedience  to  the  god,  even  if  a   thousand  deaths  await  him.  (Stands  up  for  what  he  believes  in  despite  what   other  people  say)     Your  annotations  and  translations  may  look  different;  that’s  okay.  Different  things   jump  out  to  different  readers.  Practice  your  analogical  reasoning  skills  with  the   following  question:     1.  Based  on  the  excerpt,  which  of  the  following  behaviors  would  Socrates  most   disdain?   A.  A  teacher  corrupting  his  students   B.  The  worship  of  false  gods   C.  Registered  voters  neglecting  to  cast  their  ballots   D.  Students  returning  library  books  late     The  passage  doesn’t  mention  any  of  these  scenarios,  so  based  on  what  you  know   about  Socrates,  draw  an  analogy  between  the  scenarios  described  in  the  answer   Take  your  S   AT  prep  to  the  next  level.  Visit  www.learnerator.com   41  

42     choices  and  the  positions  outlined  in  the  excerpt,  and  see  which  relationship  fits   best.     Let’s  look  at  A.  The  words  “corrupting  the  youth”  appear  in  the  passage  –  in  the   context  of  something  that  Socrates  himself  “did  not  cease,”  so  there’s  no  need  to   even  draw  an  analogy  here  –  we  are  flat-­‐out  told  that  he  engaged  in  “corruption,”   whatever  this  might  mean,  so  we  this  can’t  be  a  scenario  that  he  disdained.     Check  out  B,  “The  worship  of  false  gods.”  There  is  mention  of  gods  and  obedience  to   gods  in  the  passage,  but  nowhere  is  it  mentioned  that  Socrates  believed  only  in  one   specific  type  of  god  and/or  cared  about  other  people’s  beliefs.  In  fact,  there’s  no   mention  at  all  of  Socrates  as  a  critic  of  other  people’s  faiths,  ideas,  or  dogmas,  so  this   can’t  be  it  –  we  can’t  even  draw  an  analogy  between  this  answer  choice  and  any   behavior  outlined  in  the  passage.     Look  at  C,  “Registered  voters  neglecting  to  cast  their  ballots.”  Think  about  the   relationship  between  a  voter  and  the  vote;  once  you  have  registered,  it  is  arguably   your  “civic  duty”  to  cast  your  ballot.  If  you  don’t,  you  are  “neglecting  your  duty.”   Though  there  is  no  mention  of  voting  here,  a  sizable  portion  of  the  passage  is   devoted  to  Socrates’  hatred  of  dereliction  (the  shirking  of  duty.)  In  fact,  he  would   rather  die  than  neglect  his  duty  –  which  he  views  as  “evil.”  If  we  take  the  premise   that  not  voting  =  shirking  your  duty,  then  this  relationship  could  fit.  But  let’s  view   Option  D  before  we  come  to  any  decisions…   Option  D  is  a  contender;  one  might  argue  that  returning  a  library  book  in  a  timely   fashion  is  also  a  duty.  However,  the  books  still  get  returned  in  this  scenario,  even  if   they  get  returned  late,  so  the  duty  isn’t  necessarily  “shirked,”  just  “delayed.”  Though   this  is  somewhat  plausible,  it  doesn’t  hold  up  as  nicely  as  Option  C,  which  provides   the  best  answer.     Let’s  try  with  another  passage…  

Take  your  S   AT  prep  to  the  next  level.  Visit  www.learnerator.com   42  

43     Barbara  Cawdry  is  passionate  about  environmentalism  and  social  protest.  The   fifty-­‐eight-­‐year-­‐old  retired  science  teacher  made  waves  recently  when  she  tied   herself  to  an  ancient  oak  tree  slated  to  be  cut  down  by  the  California  Parks   Department.  Though  interfering  with  the  agenda  and  activities  of  the  state   government  is  considered  a  crime,  Cawdry  remained  tethered  to  the  tree,   refusing  food  or  drink,  until  the  state  police  arrived  to  forcibly  remove  her.  She   is  continuing  her  vigil  in  the  North  County  jail,  where  she  has  threatened  to   stage  a  hunger  strike  if  the  tree  is  cut  down.  At  the  time  of  this  writing,  Cawdry   faces  a  two  thousand  dollar  fine  and  up  to  five  months  in  jail.  When  contacted   for  this  article,  Cawdry’s  message  was  brief:  “All  I’m  doing  is  trying  to  save  the   trees.”     This  is  a  fairly  simple  passage,  so  you  may  not  feel  the  need  to  mark  it  up.  

Take  your  S   AT  prep  to  the  next  level.  Visit  www.learnerator.com   43  

44     2.  Based  on  the  passage,  Cawdry  would  probably  support  which  of  the   following  groups?   A.  Loggers  striking  for  better  pay  and  better  hours   B.  Armed  freedom  fighters  questing  to  resist  an  outside  force  by  whatever   means  necessary   C.  Members  of  the  Animal  Liberation  Front  destroying  labs  where  animal   testing  is  conducted   D.  Anti-­‐war  protestors  disrupting  traffic  during  a  protest     Now,  none  of  those  people  or  scenarios  are  mentioned  in  the  passage.  But  based  on   what  we  know  about  Cawdry,  you  can  easily  set  up  an  analogy  between  her  actions   and  that  of  the  groups  mentioned  in  the  answer  choices.     Cawdry’s  relationship  to  social  protest  does  not  seem  to  be  violent  or  destructive,  so   let’s  see  if  we  can  eliminate  any  of  the  other  choices.  We  don’t  know  anything  about   her  politics  other  than  the  fact  that  she  really,  really  likes  trees,  so  we  can’t   necessarily  set  up  an  analogy  between  her  passions  and  the  passions  of  the  people   in  this  group.  With  that  said,  if  you  look  at  choice  A,  it’s  pretty  obvious  that  Cawdry   would  not  sympathize  much  with  loggers,  as  they  literally  cut  trees  down  for  a   living.  She  may  espouse  a  similar  form  of  protest  –  non-­‐violent  disobedience  –  but   her  politics  are  too  radically  at  odds  with  that  of  the  loggers  to  make  this  a   compelling  analogy.  Strike  answer  A.     The  armed  freedom  fighters  in  Choice  B  who  resort  to  “whatever  means  necessary”   are  probably  too  violent  for  Cawdry’s  tastes;  we  don’t  know  how  she  feels  about   their  grievances,  but  we  can  extrapolate  from  her  choice  of  protest  (tying  herself   peacefully  to  a  tree)  that  she  probably  would  not  endorse  “whatever  means   necessary,”  or  else  she  might  have  resorted  to  violence  to  further  her  aims.   Choice  C  might  look  appealing  at  first;  based  on  Cawdry’s  love  for  trees,  we  might  be   able  to  extrapolate  that  animals  are  near  and  dear  to  her  heart  as  well.  But  unlike   the  members  of  the  Liberation  Front,  she  doesn’t  seem  intent  on  destroying   Take  your  S   AT  prep  to  the  next  level.  Visit  www.learnerator.com   44  

45     property;  her  protest  seems  mainly  relegated  to  doing  harm  to  herself,  not  to  others.     Choice  D  is  the  only  one  left,  and  it  fits  best  because  just  as  Cawdry’s  behaviors  are   non-­‐violent  yet  disruptive,  anti-­‐war  protestors  take  a  similar  tactic.   This  question  asks  you  to  consider  the  different  relationships  between  the  ideas  and   behaviors  mentioned  in  the  article  and  the  ideas  and  behaviors  outlined  in  the   answer  choices;  often,  when  the  SAT  asks  you  to  make  an  analogy,  what  they’re   actually  asking  for  is  some  kind  of  comparison.  Make  a  brief  list  of  the  different   ideas  in  the  passage;  you  will  find  the  correct  answer  when  you  arrive  at  the  best   match  between  the  ideas  in  the  passage  and  the  ideas  in  the  answer  choices.     Try  it  one  more  time,  using  one  of  the  sassiest  passages  in  the  English  language,   “Politics  and  the  English  Language,”  by  George  Orwell:     MOST  PEOPLE  WHO  BOTHER  with  the  matter  at  all  would  admit  that  the   English  language  is  in  a  bad  way,  but  it  is  generally  assumed  that  we  cannot  by   conscious  action  do  anything  about  it.  Our  civilization  is  decadent,  and  our   language-­‐-­‐so  the  argument  runs-­‐-­‐must  inevitably  share  in  the  general  collapse.   It  follows  that  any  struggle  against  the  abuse  of  language  is  a  sentimental   archaism,  like  preferring  candles  to  electric  light  or  hansom  cabs  to  aeroplanes.   Underneath  this  lies  the  half-­‐conscious  belief  that  language  is  a  natural  growth   and  not  an  instrument  which  we  shape  for  our  own  purposes.      Now,  it  is  clear  that  the  decline  of  a  language  must  ultimately  have  political   and  economic  causes:  it  is  not  due  simply  to  the  bad  influence  of  this  or  that   individual  writer.  But  an  effect  can  become  a  cause,  reinforcing  the  original   cause  and  producing  the  same  effect  in  an  intensified  form,  and  so  on   indefinitely.  A  man  may  take  to  drink  because  he  feels  himself  to  be  a  failure,   and  then  fail  all  the  more  completely  because  he  drinks.  It  is  rather  the  same   thing  that  is  happening  to  the  English  language.  It  becomes  ugly  and   inaccurate  because  our  thoughts  are  foolish,  but  the  slovenliness  of  our   Take  your  S   AT  prep  to  the  next  level.  Visit  www.learnerator.com   45  

46     language  makes  it  easier  for  us  to  have  foolish  thoughts.  The  point  is  that  the   process  is  reversible.  Modern  English,  especially  written  English,  is  full  of  bad   habits  which  spread  by  imitation  and  which  can  be  avoided  if  one  is  willing  to   take  the  necessary  trouble.  If  one  gets  rid  of  these  habits  one  can  think  more   clearly,  and  to  think  clearly  is  a  necessary  first  step  towards  political   regeneration:  so  that  the  fight  against  bad  English  is  not  frivolous  and  is  not   the  exclusive  concern  of  professional  writers.  I  will  come  back  to  this  presently,   and  I  hope  that  by  that  time  the  meaning  of  what  I  have  said  here  will  have   become  clearer.     I’ve  annotated  the  passage  like  so,  paying  special  attention  to  the  IDEAS  expressed,   because  remember  that  when  it  comes  to  analogies,  we  want  to  see  if  there’s  an   equivalency  between  one  idea  /  scenario  and  another:     MOST  PEOPLE  WHO  BOTHER  with  the  matter  at  all  would  admit  that  the   English  language  is  in  a  bad  way,  but  it  is  generally  assumed  that  we  cannot  by   conscious  action  do  anything  about  it.  (Our  language  skills  are   deteriorating.)  Our  civilization  is  decadent,  and  our  language-­‐-­‐so  the   argument  runs-­‐-­‐must  inevitably  share  in  the  general  collapse.  (People  think   that  because  our  civilization  is  waning  so  must  our  language).  It  follows   that  any  struggle  against  the  abuse  of  language  is  a  sentimental  archaism,  like   preferring  candles  to  electric  light  or  hansom  cabs  to  aeroplanes.  (Insisting  on   standards  makes  you  seem  really  old-­‐fashioned.)  Underneath  this  lies  the   half-­‐conscious  belief  that  language  is  a  natural  growth  and  not  an  instrument,   which  we  shape  for  our  own  purposes.  (People  believe  that  we  don’t  have   conscious  control  over  the  evolution  of  our  language  –  but  we  do!)     Now,  it  is  clear  that  the  decline  of  a  language  must  ultimately  have  political   and  economic  causes:  it  is  not  due  simply  to  the  bad  influence  of  this  or  that   individual  writer.(It’s  no  one  person’s  fault  that  our  language  is   devolving.)  But  an  effect  can  become  a  cause,  reinforcing  the  original  cause   Take  your  S   AT  prep  to  the  next  level.  Visit  www.learnerator.com   46  

47     and  producing  the  same  effect  in  an  intensified  form,  and  so  on  indefinitely.  A   man  may  take  to  drink  because  he  feels  himself  to  be  a  failure,  and  then  fail  all   the  more  completely  because  he  drinks.  (An  analogy:  this  is  a  vicious  cycle!   Our  language  is  devolving,  so  bad  writers  feel  god  about  writing  bad   stuff,  which  makes  the  language  devolve  further…)  It  is  rather  the  same   thing  that  is  happening  to  the  English  language.  It  becomes  ugly  and   inaccurate  because  our  thoughts  are  foolish,  but  the  slovenliness  of  our   language  makes  it  easier  for  us  to  have  foolish  thoughts.  The  point  is  that  the   process  is  reversible.  (We  can  make  our  language  better)  Modern  English,   especially  written  English,  is  full  of  bad  habits  which  spread  by  imitation  and   which  can  be  avoided  if  one  is  willing  to  take  the  necessary  trouble.  If  one  gets   rid  of  these  habits  one  can  think  more  clearly,  and  to  think  clearly  is  a   necessary  first  step  towards  political  regeneration:  so  that  the  fight  against  bad   English  is  not  frivolous  and  is  not  the  exclusive  concern  of  professional  writers.  I   will  come  back  to  this  presently,  and  I  hope  that  by  that  time  the  meaning  of   what  I  have  said  here  will  have  become  clearer.     Please  note  that  Orwell  himself  makes  use  of  analogies  –  not  once  but  twice!  First  he   says  that  struggling  to  maintain  language  standards  makes  one  seem  old-­‐fashioned,   as  if  campaigning  for  the  return  of  candles  as  opposed  to  electric  lights.  Then  he   compares  the  devolution  of  the  English  language  as  the  kind  of  vicious  cycle  that   alcoholics  suffer.  Orwell  knows:  analogies  are  a  rich  and  complex  way  to  express   ideas.    

Take  your  S   AT  prep  to  the  next  level.  Visit  www.learnerator.com   47  

48     3.  Based  on  the  passage,  which  of  the  following  scenarios  would  Orwell  find   most  objectionable?   A.  Doctors  who  use  outdated  methods  of  medical  observation   B.  Civilizations  that  become  so  decadent  that  their  morals  decay  entirely   C.  Students  whose  spelling  has  been  corrupted  by  social  media   D.  Professional  writers  who  struggle  to  communicate  their  intended  meanings     I  find  it  helpful  to  distill  the  answer  choices  into  a  single  core  idea,  which  may  make   it  easier  to  figure  out  which  answer  is  most  congruous  with  the  ideas  expressed  in   the  passage.     Option  A.  At  the  heart  of  this  option  is  the  idea  of  being  old-­‐fashioned  or  outdated.   While  Orwell  does  talk  about  the  idea  of  being  old-­‐fashioned  (It  follows  that  any   struggle  against  the  abuse  of  language  is  a  sentimental  archaism,  like  preferring   candles  to  electric  light  or  hansom  cabs  to  aeroplanes.  (Insisting  on  standards   makes  you  seem  really  old-­‐fashioned.),  he  isn’t  disapproving  of  this.  In  fact,  he   seems  to  either  endorse  the  notion  of  being  old-­‐fashioned,  or  to  disregard  it  entirely.   Either  way,  he  isn’t  positioning  himself  firmly  as  being  against  old-­‐fashioned   tendencies.  We  can  discard  Option  A.     Option  B.  Distill  this  into  a  single  idea  –  decay.  It’s  true  that  Orwell  seems  pretty   anti-­‐decay  (especially  as  regards  to  language);  though  he  says  little  in  this  passage   about  morals,  there’s  no  evidence  that  he  WOULDN’T  find  this  scenario   objectionable.  Keep  it  for  now.     Option  C.  What  is  at  the  heart  of  this  answer?  The  devolution  of  a  type  of  language.   Orwell  spends  two  paragraphs  railing  against  bad  writers;  there  is  every  indication   that  he  would  find  social-­‐media  spellings  to  be  a  travesty.  In  fact,  this  is  so  directly   parallels  Orwell’s  statement  that  it’s  barely  even  an  analogy  –  it’s  more  a  modern-­‐ day  extension  of  his  opinion.  Clearly  this  is  probably  going  to  be  the  best  option,  but   let’s  look  at  Option  D  as  well.   Take  your  S   AT  prep  to  the  next  level.  Visit  www.learnerator.com   48  

49       Option  D.  Distill  this  answer  choice  into  an  idea:  the  struggle  of  using  language   properly.  Orwell’s  piece  castigates  bad  writers  and  bad  writing  –  but  nowhere  does   he  say  that  writing  well  is  EASY.  In  fact,  he  would  probably  approve  of  a  writer’s   struggle  to  communicate  meaning  clearly  and  cleanly.  Certainly  he  makes  no   statement  that  is  similar  /  in  harmony  with  the  idea  that  writing  should  be  easy.   Of  all  these  choices,  C  is  clearly  the  best  –  it  is  the  MOST  similar  to  what  Orwell  is   saying.   Isolating  the  ideas  in  a  passage  so  that  you  can  answer  questions  about  them  is  a   very  good  strategy  for  mastering  reading  comprehension  questions;  it’s  going  to  be   especially  useful  for  the  next  chapter  of  our  review,  Citing  Textual  Evidence.  

   

 

Take  your  S   AT  prep  to  the  next  level.  Visit  www.learnerator.com   49  

50    

Citing Textual Evidence To  achieve  success  on  the  SAT  and  in  the  classroom,  you  must  be  in  possession  of  all   the  facts.  Aside  from  a  few  math  questions,  the  majority  of  the  questions  are   multiple  choice  –  meaning  that  the  answers  are  already  there.  All  that  remains  is  for   you  to  make  sure  you  are  picking  the  best  one,  and  the  way  to  do  this  is  to  make   sure  that  you’re  paying  attention  to  all  the  evidence.  Every  answer  is  “the  best   answer”  for  a  reason;  you  have  to  make  sure  you  understand  what  that  reason   actually  is.   To  do  this,  you  must  understand  the  passage  thoroughly,  so  you  know  what  actually   counts  as  “evidence”  or  as  “support”  for  a  claim.   Next,  you  must  understand  what  the  question  is  actually  asking  you  so  that  you  are   equipped  to  choose  the  absolute  best  answer  for  it.   Take  this  brief  passage,  for  example:     Upon  my  entrance,  Usher  arose  from  a  sofa  on  which  he  had  been  lying  at  full   length,  and  greeted  me  with  a  vivacious  warmth  which  had  much  in  it,  I  at  first   thought,  of  an  overdone  cordiality  -­‐-­‐  of  the  constrained  effort  of  the  ennuyé   man  of  the  world.  A  glance,  however,  at  his  countenance,  convinced  me  of  his   perfect  sincerity.  We  sat  down;  and  for  some  moments,  while  he  spoke  not,  I   gazed  upon  him  with  a  feeling  half  of  pity,  half  of  awe.  Surely,  man  had  never   before  so  terribly  altered,  in  so  brief  a  period,  as  had  Roderick  Usher.      This  passage  is  a  bit  old-­‐fashioned,  so  let’s  take  our  time  annotating  it:     Upon  my  entrance,  Usher  arose  from  a  sofa  on  which  he  had  been  lying  at  full   length,  (the  man  was  lying  on  a  couch  and  got  up  when  I  entered)  and   greeted  me  with  a  vivacious  warmth  which  had  much  in  it,  I  at  first  thought,  of   an  overdone  cordiality  -­‐-­‐  of  the  constrained  effort  of  the  ennuyé  man  of  the   world.  (his  greeting  seemed  overdone;  he  put  in  too  much  effort).  A  glance,   Take  your  S   AT  prep  to  the  next  level.  Visit  www.learnerator.com   50  

51     however,  at  his  countenance,  convinced  me  of  his  perfect  sincerity.  (But  when  I   looked  at  his  face  I  could  tell  her  was  sincere.)  We  sat  down;  and  for  some   moments,  while  he  spoke  not,  I  gazed  upon  him  with  a  feeling  half  of  pity,  half   of  awe.  (He  didn’t  speak,  and  I  stared  at  him,  half  with  pity  and  half  with   awe.)  Surely,  man  had  never  before  so  terribly  altered,  in  so  brief  a  period,  as   had  Roderick  Usher.  (He  had  changed  very  quickly)      Now  make  a  list  of  all  the  evidence  you  can  cite  from  the  text  to  support  the   following  statement:     There  is  something  wrong  with  Roderick  Usher.     Look  at  the  text.  What  indicates  that  there  might  be  something  wrong?   1.  Well,  for  one,  Usher  is  lying  on  the  sofa  when  our  narrator  walks  in.  Now,  we  don’t   know  what  time  of  day  it  is,  so  by  itself  this  can’t  be  taken  as  indication  that   something  is  wrong,  but  it  could  be  a  telling  detail.   2.  He  has  “an  overdone  cordiality…  constrained  effort  of  the  ennui  man  of  the  world”   –  i.e.  he  seemed  at  first  to  have  the  social  mannerisms  of  a  person  who  was  bored   (“ennui”)  with  the  world.  This  is  a  signal  that  Usher  could  be  depressed  or  at  the   very  least  full  of  existential  boredom  –  not  a  great  state  of  affairs.   3.  “I  gazed  upon  him  with  a  feeling  half  of  pity  and  half  of  awe.”  This  is  telling  –  once   you’re  awed  by  the  amount  of  pity  you  have  for  someone,  you  know  something  is   wrong  with  them.   4.  “Terribly  altered”  –  to  be  changed  is  not  such  a  bad  thing.  But  to  be  “terribly   changed;”  changed  for  the  worst?  Warning  sign.     You  never  know  what  might  be  evidence,  which  is  why  it’s  so  important  to  read   closely  and  to  mark  up  your  text  as  thoroughly  as  possible,  so  when  you’re  passing   back  over  it  you  can  see  at  a  glance  which  ideas  end  up  being  important.    

Take  your  S   AT  prep  to  the  next  level.  Visit  www.learnerator.com   51  

52     Let’s  try  this  again.  Read  the  excerpt,  which  is  from  The  Atlantic,  and  then  see  what   evidence  you  can  find  to  support  the  statement.      “Since  the  release  of  The  Hunger  Games  in  2012,  dystopian  cinema  has  enjoyed   sustained  interest  in  American  culture.  Popular  young  adult  novels  are  being   turned  into  blockbuster  Hollywood  films  every  few  months,  it  seems,  and  with   good  reason:  Beyond  their  built-­‐in  teen  fan  base,  films  like  Divergent,  The  Giver,   and  The  Maze  Runner  draw  on  some  of  adult  society’s  greatest  fears  of  the   moment:  Is  technology  tearing  us  further  apart?  Will  global  warming  destroy   the  planet?  Will  income  inequality  further  create  a  world  of  haves  and  have-­‐ nots?     Critics  have  worried  that  these  particular  films  stoke  an  irrational  fear  of   technology,  or  a  distaste  for  big  government,  but  dystopian  stories  have  long   been  celebrated  (and  used  in  classrooms  across  the  country)  because  of  their   ability  to  push  audiences  to  think  critically  about  their  actions.     Yet  with  the  upcoming  release  of  The  Hunger  Games:  Mockingjay  Part  1,  poised   to  be  the  biggest  film  of  the  year,  it’s  just  as  worthwhile  to  consider  what  these   films  don’t  seem  to  fear.  While  recent  dystopias  warn  youth  about  over-­‐reliance   on  computers,  totalitarian  rule,  class  warfare,  pandemic  panics  and  global   warming,  very  few  ask  audiences  to  think  deeply  about  sexism  and  racism.”       Once  you  have  marked  up  your  passage,  find  all  the  evidence  you  can  to  support  the   following  statement:       The  author  believes  that  most  dystopian  films  and  books  are  not  doing  enough   to  promote  critical  thinking.      Read  carefully,  pulling  out  the  strands  of  the  author’s  argument  in  pieces:  

Take  your  S   AT  prep  to  the  next  level.  Visit  www.learnerator.com   52  

53     1.  “Dystopian  cinema  has  enjoyed  sustained  interest”   OK,  this  argues  for  dystopias’  popularity  –  not  for  their  promotion  of  critical  thinking     2.  “Popular  YA  novels  are  being  turned  into  blockbuster  Hollywood  films  every  few   months…  and  with  good  reason.”   Also  argues  for  dystopias’  popularity  –  not  for  their  promotion  of  critical  thinking     3.  “Draw  on  some  of  adult  society’s  greatest  fears”   Fear  does  not  necessarily  lead  to  critical  thinking,  so  we  can’t  put  this  one  in  the   evidence  column     4.  “Dystopian  stories  have  long  been  celebrated  (and  used  in  classrooms  across  the   country)  because  of  their  ability  to  push  audiences  to  think  critically  about  their   actions”   Finally,  a  mention  of  critical  thinking  à  so  these  films  DO  promote  it.  This  does  not   directly  contradict  the  statement,  however,  that  they  do  not  do  ENOUGH  to  promote   critical  thinking.  Remember,  to  answer  these  types  of  questions  you  must  read   CAREFULLY!     5.  “It’s  just  as  worthwhile  to  consider  what  these  films  don’t  seem  to  fear.”   Ah  ha.  The  author  is  now  asking  you  to  consider  what  these  films  DON’T  do  –  i.e.  to   evaluate  what  is  negative  and/or  missing  about  these  films.  S/he  doesn’t  say  anything   about  critical  thinking,  but  this  might  be  the  turning  point  for  the  article  to  start   discussing  a  specific  failure  of  these  films  –  which  may  include  the  fact  that  they  don’t   do  enough  to  promote  critical  thinking.     6.  “While  recent  dystopias  warn  youth  about  over-­‐reliance  on  computers,   totalitarian  rule,  class  warfare,  pandemic  panics  and  global  warming,  very  few  ask   audiences  to  think  deeply  about  sexism  and  racism.  “We  have  a  winner.  Your  first   clue  is  the  world  “while,”  which  indicates  a  shift  from  a  previously  held/described   opinion.  The  author  spent  the  previous  paragraph  setting  up  a  scenario  (schools  use   Take  your  S   AT  prep  to  the  next  level.  Visit  www.learnerator.com   53  

54     dystopias  to  teach  students  certain  lessons  and  make  them  think  about  things),  and   now  there  is  going  to  be  a  shift:  “Very  few  ask  audiences  to  think  deeply.”       For  an  article  like  this,  it  is  important  to  consider  the  structure  of  a  piece  –  the   author  spends  some  time  setting  up  a  scenario,  in  which  s/he  lays  out  the   foundations  of  an  argument.  A  good  argument  will  always  contain  claims  and   counterclaims,  so  don’t  be  fooled  by  the  parts  of  the  argument  that  give  an  opposite   opinion;  they  should  not  be  taken  as  evidence  against  the  author’s  argument  or   opinion,  but  as  acknowledgments  that  alternate  viewpoints  exist  (usually  prior  to   the  author  demonstrating  why  they  are  invalid).  It’s  not  always  easy  to  tell  which   part  of  a  text  is  the  author’s  argument  and  which  contains  the  claims  that  s/he  will   shortly  dismantle;  that’s  why  transition  words  like  “yet,”  “but,”  “while,”  “however,”   and  “while”  are  so  important.     Let’s  try  one  more  time;  this  time,  you  will  have  to  answer  a  question  with  one  of   four  options.  When  answering,  consider  the  evidence:       This  passage  is  also  from  The  Atlantic:     “This  week,  one  of  the  two  Common  Core  assessment  consortia  announced  its   cut  scores—benchmarks  students  need  to  clear  to  be  considered  proficient  at   the  content  on  which  they’re  tested.  Just  a  third  to  roughly  40  percent  of  K-­‐8   and  high  school  students  are  projected  to  be  proficient  in  math  and  English   Language  Arts…     And  while  tests  before  didn’t  challenge  students  enough,  Minnich  said,  the   conversation  in  states  where  these  Common  Core-­‐aligned  tests  are  imminent   will  be,  ‘Well,  who  decided  this  was  the  right  thing  for  kids?  Who  decided  that   these  test  scores  were  actually  what  kids  need?’     Take  your  S   AT  prep  to  the  next  level.  Visit  www.learnerator.com   54  

55     For  states  to  expand  deeper-­‐learning  techniques,  they’ll  likely  first  need  to   overcome  the  blowback  from  parents  whose  children  are  suddenly  deemed   unprepared  for  the  challenges  of  college.  …It’s  a  recipe  as  simple  as  it  is   complex:  Convince  the  masses  the  Common  Core  is  on  to  something  good,  and   improve  from  there.”     1.  The  author  of  this  article  probably  feels  most  strongly  that   A.  Tests  do  not  challenge  students  enough   B.  Certain  states  are  doing  a  better  job  than  others  of  preparing  students  for   tests   C.  The  Common  Core  needs  to  undergo  change  to  be  truly  meaningful   D.  The  Common  Core  is  not  the  right  thing  for  kids     Now,  you’ll  want  to  pay  attention  to  this  one  because  the  answer  choices  all  echo   sentences  found  in  the  text.  The  trick  is  to  determine  what  the  question  is  actually   asking  you,  so  you  can  understand  the  correct  evidence.  It’s  not  enough  to  simply   quote  from  the  text  –  your  quote  must  support  and  be  relevant  to  an  assertion.     Let’s  look  at  the  answer  choices:     A.  Tests  do  not  challenge  students  enough.   Consider  the  evidence  from  the  text  –  “just  a  third  to  40%  of  students  are  projected   to  be  proficient  in  math  and  English.”  If  only  40%  of  students  are  passing  –  let  alone   doing  well  –  there  is  very  little  compelling  evidence  that  the  tests  are  not  adequately   challenging.  And  look  at  the  next  line  “And  while  tests  before  didn’t  challenge   students  enough…”  So  while  this  may  at  one  point  have  been  an  opinion,  the   question  is  asking  about  tests  NOW.  Lastly,  the  question  is  asking  about   the  author’s  opinion.  At  this  point  in  the  article,  the  author  has  not  yet  stated  an   opinion,  but  is  rather  quoting  statistics  and  other  people’s  opinions.  Pay  attention  to   the  structure  here  –  it’s  important  to  be  able  to  differentiate  the  author’s  opinion,   and  that  of  her  sources.   Take  your  S   AT  prep  to  the  next  level.  Visit  www.learnerator.com   55  

56       B.  Certain  states  are  doing  a  better  job  than  others  of  preparing  students  for   tests.   Look  closely.  Are  any  specific  states  mentioned?  Are  there  any  statistics  for  the   breakdown  of  achievement  by  state?  Is  there  a  single  indication  that  some  states  are   doing  better  than  others?  There’s  absolutely  nothing  in  this  article  to  support  this   opinion;  scrap  this  answer.     C.  The  Common  Core  needs  to  undergo  change  to  be  really  meaningful.   First,  find  all  mentions  of  the  Common  Core:   …  This  week,  one  of  the  two  Common  Core  assessment  consortia  announced  its  cut   scores   OK,  that  has  nothing  to  do  with  changing  to  be  meaningful.     The  conversation  in  states  where  these  Common  Core-­‐aligned  tests  are  imminent  will   be,  ‘Well,  who  decided  this  was  the  right  thing  for  kids?  Who  decided  that  these  test   scores  were  actually  what  kids  need?’   A  quote  from  someone  –  not  the  author  –  predicting  that  certain  states  are  poised  to   criticize  the  Common  Core.  This  could  potentially  be  a  meaningful  piece  of  evidence   as  it  predicts  (the  need  for)  a  possible  change  in  the  Common  Core  standards.     …It’s  a  recipe  as  simple  as  it  is  complex:  Convince  the  masses  the  Common  Core  is  on  to   something  good,  and  improve  from  there.”   OK,  more  possible  evidence  here:  “on  to  something  good  and  improve  from  there.”   Clearly,  the  author  believes  that  the  Common  Core  has  SOME  merit,  and  can  be   improved  on  in  order  to  rectify  or  address  the  issues  mentioned  in  the  article.     Let’s  keep  this  answer  choice  for  now.     D.  The  Common  Core  is  not  the  right  thing  for  kids.   Looking  at  all  the  evidence  we  compiled  for  the  answer  above  –  all  the  mentions  of   Take  your  S   AT  prep  to  the  next  level.  Visit  www.learnerator.com   56  

57     the  Common  Core  –  it’s  clear  that  the  only  people  who  are  asking  “is  this  right  for   kids?”  are  states  and  parents,  NOT  the  author.  Additionally,  questioning  whether   something  is  right  is  NOT  the  same  as  declaring  definitively  that  it  isn’t.  Lastly,  the   author’s  last  words  in  this  excerpt  are  “The  answer  is  simple:  convince  the  masses   the  Common  Core  is  on  to  something  good  and  improve  from  there.”  S/he’s  not   dismissing  the  Common  Core  out  of  hand,  but  saying  instead  that  it  has  the  potential   to  be  meaningful.  It  can’t  be  entirely  the  wrong  thing  for  kids  if  it’s  “on  to  something   good.”     Clearly,  the  evidence  best  supports  choice  C.     Let’s  do  this  one  more  time,  again  with  an  excerpt  from  the  Atlantic.   “Animal  behavior  seems  to  reveal  that  animals  don't  want  to  be  in  enclosures,   either:   A  giraffe  who  freaks  out  about  men  with  large  cameras,  a  brown  bear  whose   cage  door  is  the  subject  of  his  obsessive  compulsive  disorder,  a  5,000-­‐pound   killer  whale  who  shows  her  trainer  who  is  boss  by  dragging  him  underwater  for   just  about  as  long  as  he  can  live,  before  letting  him  go—these  episodes  seem   like  something  more  complicated  than  simple  errors  of  confinement.  It  is  hard   to  avoid  the  conclusion  that  in  some  way  the  animals  understand  that  the   world  around  them  is  an  artificial  one,  that  these  phobias  and  psychotic   episodes  represent  reactions  to  that  artifice,  or  subversions  of  it.”     2.  Based  on  the  passage,  what  is  the  reason  that  animals  in  zoos  can   potentially  exhibit  violent  behavior?   A.  These  animals  have  psychiatric  disorders   B.  These  animals  are  traumatized  by  the  demands  of  their  trainers  and   keepers   C.  These  animals  suffer  from  severe  phobias   D.  These  animals  seek  to  disrupt  what  they  perceive  as  the  unreal  world   around  them   Take  your  S   AT  prep  to  the  next  level.  Visit  www.learnerator.com   57  

58       Look  at  the  evidence  for  Option  A.  While  the  article  mentions  an  animal  “freaking   out”  and  another  with  “obsessive  compulsive  disorder,”  it  does  not  suggest  that  all   of  these  animals  have  psychiatric  disorders.  In  fact,  the  article  tries  to  investigate   the  roots  of  these  behaviors;  to  simply  dismiss  said  violent  behaviors  as  disorders   would  not  get  at  the  heart  of  the  article’s  argument.     Consider  B.  Again,  while  the  article  mentions  “freaking  out  about  men  with  large   cameras,”  the  “obsessive  compulsive”  fixation  on  a  cage,  and  “showing  the  trainer   who  is  boss,”  again,  the  article  is  trying  to  figure  out  why  these  things  happen.  To   use  these  incidents  as  evidence  for  themselves  simply  creates  a  circular  argument.   Check  out  C.  Again,  the  article  does  mention  phobias,  but  there  is  no  explicit  link   drawn  between  the  phobias  and  the  violent  behavior;  additionally,  the  next   sentence  seems  to  suggest  that  (rather  than  one  creating  the  other),  they  stem  from   the  same  source.  To  use  the  phobias  as  evidence  for  why  animals  behave  violently   seems,  again,  to  be  circular,  since  the  ultimate  argument  is  that  these  behaviors  and   attitudes  stem  from  the  same  place.     Last,  look  at  D.  The  last  sentence  of  the  excerpt  says  “in  some  way  the  animals   understand  that  the  world  around  them  is  an  artificial  one,  that  these  phobias  and   psychotic  episodes  represent  reactions  to  that  artifice,  or  subversions  of  it.”  In  other   words,  they  understand  they  should  not  be  in  this  condition,  in  this  enclosure,   penned  in,  and  all  their  behaviors  (their  disorders,  phobias,  feelings  about  trainers   and  keepers,  violent  behaviors,  etc)  all  come  from  the  same  place:  the  desire  to   somehow  disrupt  this  “artificial,”  or  fake,  world  that  they  are  living  in.     In  essence,  the  distractors  for  this  question  are  all  incomplete  –  they  are  all   incomplete  thoughts  that,  taken  together,  create  the  best  and  fullest  answer  to  the   question.  In  the  next  section,  you  will  practice  isolating  the  details  that  together   make  up  the  central  idea  or  theme  of  an  excerpt.  

Take  your  S   AT  prep  to  the  next  level.  Visit  www.learnerator.com   58  

59    

Determining Main Ideas Perhaps  the  most  important  part  of  reading,  and  reading  comprehension,  is   knowing  how  to  determine  the  main  idea  of  a  passage  or  selection.  After  all,  this  is   the  real  test  of  whether  you  understand  what  the  passage  is  trying  to  convey.  You   may  not  understand  every  single  word,  but  the  ability  to  ferret  out  the  general   meaning  of  a  larger  selection  –  whether  it’s  a  paragraph,  a  chapter,  or  even  a  whole   book  –  is  crucial  to  the  process  of  analyzing,  deconstructing,  and  even  enjoying  what   you  read.     But  How?     Determining  a  main  idea  can  be  as  simple  as  looking  at  the  first  and  last  sentences  of   every  paragraph.  In  a  simple  non-­‐fiction  work,  these  sentences  will  often  introduce   or  summarize  the  essential  ideas  in  the  selection.  While  this  is  NOT  a  strategy  that   will  work  with  literary  or  higher-­‐level  texts,  it’s  a  good  stepping-­‐stone  to   understanding  where  you  can  begin  to  look  to  gather  the  main  idea.     Let’s  practice  with  a  brief,  simple  selection  from  The  New  York  Times:   “Thursday  morning  comes  quickly  out  of  the  dark,  and  with  it  the  promise  of   the  Thanksgiving  feast.  Have  you  set  the  table  yet?  You  should.  Set  it  as  if  for  a   sacrament.  The  Thanksgiving  meal  is  America’s  most  holy  secular  ritual.  ‘There   are  three  things  that  people  pick  up  on  the  instant  they  walk  into  your  home  on   Thanksgiving,’  said  Danny  Meyer,  the  New  York  restaurateur,  who  does  not   celebrate  the  holiday  at  Union  Square  Cafe,  Gramercy  Tavern,  North  End  Grill   or  any  of  his  other  considerable  number  of  restaurants,  but  always  with  family   at  home.  ‘They  will  be  able  to  feel  the  human  energy.  They’ll  smell  the  food.  And   they  will  see,  instantly,  the  table.  The  combination  —  if  you’re  in  a  good  mood,   and  you’ve  taken  time  with  the  table,  and  the  turkey  is  cooking  —  sends  a   message  that,  no  matter  what,  everything  is  going  to  be  O.K.’”   Take  your  S   AT  prep  to  the  next  level.  Visit  www.learnerator.com   59  

60       What  is  the  main  idea  of  this  paragraph?       Look  at  the  first  sentence,  which  indicates  that  clearly,  this  passage  will  be  about  the   “Thanksgiving  feast.”  Then  look  at  the  last  “The  combination  (of  good  mood,  good   food,  and  a  set  table)  sends  the  message  that  everything  is  going  to  be  O.K.”     Now,  “Thanksgiving  feast”  is  not  really  an  idea;  it’s  the  subject.  What’s  the   difference?  After  all,  a  subject  is  “what  a  passage  is  about”  and  a  main  idea  is  also   “what  the  passage  is  about.”  Think  of  it  like  this:     A  subject  is  the  broadest,  most  general  way  to  describe  something.   A  main  idea  is  much  more  specific.     A  good  rule  of  thumb  when  trying  to  determine  main  ideas  is  to  put  them  in  a  full   phrase  or  sentence;  after  all,  just  saying  that  the  main  idea  is  the  “Thanksgiving   feast,”  gives  no  indication  of  what  features  or  aspects  of  the  Thanksgiving  feast  are   important.  For  all  we  know,  a  passage  that  is  merely  described  as  being  about  “the   Thanksgiving  feast”  could  end  up  detailing  all  the  ways  in  which  the  Thanksgiving   feast  could  go  wrong!  But  the  words  “no  matter  what,  everything  is  going  to  be   okay”  are  a  pretty  clear  indicator  that  this  isn’t  the  case  here.  So  to  avoid  ambiguity,   get  yourself  in  the  habit  of  scrawling  the  main  idea  (i.e.  a  full  phrase  or  sentence)  in   the  margin  of  each  paragraph.   Thus,  taken  together,  the  first  and  last  sentences  of  each  paragraph  in  a  lower-­‐ complexity  informational  text  should  present  you  with  the  main  idea.  Here  it  might   be  something  like:     “With  enough  preparation,  the  Thanksgiving  feast  will  go  smoothly,”  or  something   similar  to  that.  (There  are,  of  course,  multiple  ways  to  word  this  that  will  still  be   correct!)   Let’s  try  this  once  more  with  a  similarly  simple  text,  also  from  The  New  York  Times:   Take  your  S   AT  prep  to  the  next  level.  Visit  www.learnerator.com   60  

61        “This  season  millions  of  Americans  will  celebrate  with  turkey  on  the  table.  The   turkey  is,  after  all,  the  native  North  American  animal  that  Benjamin  Franklin   considered  “a  much  more  respectable  bird”  than  the  scavenging  bald  eagle.  But   while  the  eagle  landed  on  the  country’s  Great  Seal  and  the  turkey  gets  pride  of   place  at  our  holiday  dinners,  neither  bird  can  claim  to  have  changed  American   culture  more  than  their  lowly  avian  (avian  =  bird)  cousin,  the  chicken.”     First  sentence:  Americans  celebrate  with  turkey.   Last  sentence:  The  chicken  has  changed  American  culture  more  than  any  other  bird.   You  don’t  even  need  the  first  sentence  here  to  determine  the  main  idea  of  the   passage.  Clearly,  it’s  going  to  be  about  how  chickens  have  changed  America.  (Pretty   thrilling  stuff.)   In  isolation,  it’s  pretty  easy  to  determine  the  main  idea  of  a  single  paragraph.  But  the   SAT  will  also  ask  you  to  determine  the  main  ideas  of  PARTS  of  longer  excerpts  –  to   keep  the  longer  excerpt  in  mind  while  choosing  an  answer  that  makes  sense  of  the   different  parts  of  longer  pieces.     Let’s  practice  with  this  1965  speech  on  voting  rights  by  Lyndon  B.  Johnson:     “In  our  time  we  have  come  to  live  with  moments  of  great  crisis.  Our  lives  have   been  marked  with  debate  about  great  issues;  issues  of  war  and  peace,  issues  of   prosperity  and  depression.  But  rarely  in  any  time  does  an  issue  lay  bare  the   secret  heart  of  America  itself.  Rarely  are  we  met  with  a  challenge,  not  to  our   growth  or  abundance,  our  welfare  or  our  security,  but  rather  to  the  values  and   the  purposes  and  the  meaning  of  our  beloved  Nation.     The  issue  of  equal  rights  for  [African-­‐Americans]  is  such  an  issue.  And  should   we  defeat  every  enemy,  should  we  double  our  wealth  and  conquer  the  stars,  and   still  be  unequal  to  this  issue,  then  we  will  have  failed  as  a  people  and  as  a   nation.  For  with  a  country  as  with  a  person,  "What  is  a  man  profited,  if  he  shall   Take  your  S   AT  prep  to  the  next  level.  Visit  www.learnerator.com   61  

62     gain  the  whole  world,  and  lose  his  own  soul  ?"  There  is  no  [African-­‐American]   problem.  There  is  no  Southern  problem.  There  is  no  Northern  problem.  There  is   only  an  American  problem.  And  we  are  met  here  tonight  as  Americans—not  as   Democrats  or  Republicans-­‐-­‐we  are  met  here  as  Americans  to  solve  that   problem.     This  was  the  first  nation  in  the  history  of  the  world  to  be  founded  with  a   purpose.  The  great  phrases  of  that  purpose  still  sound  in  every  American  heart,   North  and  South:  "All  men  are  created  equal"—"government  by  consent  of  the   governed"—"give  me  liberty  or  give  me  death."  Well,  those  are  not  just  clever   words,  or  those  are  not  just  empty  theories.  In  their  name  Americans  have   fought  and  died  for  two  centuries,  and  tonight  around  the  world  they  stand   there  as  guardians  of  our  liberty,  risking  their  lives.     Those  words  are  a  promise  to  every  citizen  that  he  shall  share  in  the  dignity  of   man.  This  dignity  cannot  be  found  in  a  man's  possessions;  it  cannot  be  found  in   his  power,  or  in  his  position.  It  really  rests  on  his  right  to  be  treated  as  a  man   equal  in  opportunity  to  all  others.  It  says  that  he  shall  share  in  freedom,  he  shall   choose  his  leaders,  educate  his  children,  and  provide  for  his  family  according  to   his  ability  and  his  merits  as  a  human  being.”     A  typical  SAT  question  might  ask  you:   What  is  the  main  idea  of  the  second  paragraph?   A.  Having  equal  rights  will  cause  everyone  to  profit   B.  Doubling  national  wealth  and  conquering  the  stars  should  be  among   Americans’  top  priorities   C.  We  will  never  defeat  our  enemies  if  we  do  not  have  equal  rights   D.  For  the  good  of  the  nation,  Americans  must  unite  to  fix  unjust  policies    

Take  your  S   AT  prep  to  the  next  level.  Visit  www.learnerator.com   62  

63     Again,  this  is  a  good  place  to  look  at  the  first  and  the  last  sentences  of  the  paragraph   in  question:  “The  issue  of  equal  rights  is  such  an  issue”  and  “We  are  met  as   Americans  to  solve  that  problem.”     Now,  that’s  not  particularly  helpful  because  without  the  context  of  the  previous   paragraph,  we  don’t  know  what  “such  an  issue”  or  “that  problem”  actually  mean.  But   we  CAN  understand  that  this  speech  deals  with  an  important  issue  that  Americans   must  solve  together.  This  may  even  be  enough  to  help  you  answer  the  question   completely!  But  if  you’re  not  entirely  sure  yet  of  the  answer,  skim  the  first   paragraph  again,  and  find  out  that  “such  an  issue”  (i.e.  equality  for  African-­‐ Americans)  refers  to  “an  issue  that  lays  bare  the  secret  heart  of  America  itself”  –  i.e.   a  very  important  matter.  So  let’s  consider  the  answer  options  now:     A:  Having  equal  rights  will  cause  everyone  to  profit.   While  in  some  (figurative)  sense  this  may  in  fact  be  true,  pay  close  attention  –  is  this   what  Johnson  is  actually  saying  in  the  passage?  Look  at  where  he  mentions  profits:     a.  …should  we  double  our  wealth  and  conquer  the  stars,  and  still  be  unequal   to  this  issue,  then  we  will  have  failed  as  a  people  and  as  a  nation.   b.  For  with  a  country  as  with  a  person,  "What  is  a  man  profited,  if  he  shall   gain  the  whole  world,  and  lose  his  own  soul?"     These  two  sentences  suggest  that  the  current  situation  is  untenable  and  immoral,   and  while  there  is  evidence  that  a  reverse  situation  (equal  rights  may  lead  to  profit)   it’s  not  compelling  enough  to  be  the  main  idea  of  the  paragraph.     Look  at  B:  While  doubling  American  wealth  and  conquering  the  stars  are  good  goals   to  have,  nowhere  does  Johnson  say  they  are  the  most  important;  in  fact,  he  seems  to   suggest  the  opposite  when  he  says  that  achieving  those  goals  without  achieving   equality  will  make  us  “fail  as  a  people  and  a  nation.”  Clearly,  this  isn’t  the  most  

Take  your  S   AT  prep  to  the  next  level.  Visit  www.learnerator.com   63  

64     important  priority  if  there  are  other,  better  goals  to  worry  about.     Check  out  C:  Again,  Johnson  does  mention  “defeating  every  enemy,”  but  in  context  of   an  achievement  that  will  be  meaningless  if  we  do  not  first  achieve  equality  for  all  of   our  citizens.  (“We  will  fail  as  a  people  and  as  a  nation.”)  This  can’t  be  the  right   answer.     D  is  the  only  correct  choice.  Consider  every  part  of  the  sentence:   “For  the  good  of  the  nation”  à  considering  that  other  (extremely  worthwhile)   goals  are  meaningless  without  equality,  it  can  be  inferred  that  achieving   equality  will  indeed  be  for  the  good  of  the  nation.     The  second  part,  “Americans  must  unite”  finds  support  in  the  last  few   sentences:   “There  is  no  Southern  problem.  There  is  no  Northern  problem.  There  is  only  an   American  problem.  And  we  are  met  here  tonight  as  Americans—not  as   Democrats  or  Republicans-­‐-­‐we  are  met  here  as  Americans  to  solve  that   problem.”  Johnson  lists  some  things  that  might  divide  us  –  geographic  region,   political  affiliation  –  before  dismissing  them  as  meaningless,  demonstrating   that  unity  is  key  in  solving  essential  problems.     Thus  far  this  is  all  pretty  straightforward.  But  what  happens  with  a  higher-­‐level  text,   or  a  literary  text  that  doesn’t  do  you  the  very  nice  favor  of  laying  out  main  ideas  in   the  first  and  last  sentences  of  each  paragraph?  When  that  happens,  you  must  notice   the  telling  details  and  any  repetitions  of  words  and  ideas.  Try  it  with  this  (fairly   complex)  excerpt  from  a  David  Foster  Wallace  essay  about  9/11,  written  on   9/13/2001.  I’ve  included  my  annotations  below:     “Everybody has flags out. Homes, businesses. It's odd:

Flags =

You never see anybody putting out a flag, but by Wednesday patriotism?

Take  your  S   AT  prep  to  the  next  level.  Visit  www.learnerator.com   64  

65     morning there they all are. Big flags, small flags, regular flag-size flags. A lot of home-owners here have those special angled flag-holders by their front door, the kind whose brace takes four Phillips screws. And thousands of those little hand-held flags-on-a-stick you normally see at parades – some yards have dozens all over as if they'd somehow sprouted overnight. Rural-road people attach the little flags to their mailboxes out by the street. Some cars have them wedged in their grille or duct-taped to the antenna. Some upscale people have actual poles; their flags are at halfmast. More than a few large homes around Franklin Park or out on the east side even have enormous multistory flags hanging gonfalon-style down over their facades. It's a total mystery where people get flags this big or how they got them up there. My own next-door neighbor, a

Huge flags, hanging like banners (gonfalon— heraldic flag, like knights used to have); making quite a statement

retired CPA and vet whose home- and lawn-care are nothing short of phenomenal, has a regulation-size anodized flagpole secured in 18" of reinforced

-This guy sounds

cement that none of the other neighbors like very much

like he does

because they think it draws lightning. He says there's a very everything by the particular etiquette to having your flag at halfbook: mast: You're supposed to first run it all the way up

"phenomenal

to the top and then bring it halfway down.

homecare,"

Otherwise it's an insult or something. His flag is out

regulation-size

straight and popping smartly in the wind. It's far and away

“official” flagpole”

the biggest flag on our street. You can also hear the wind in

-A good detail –

the cornfields just south; it sounds the way light surf sounds

this guy cares

when you're two dunes back from it. Mr. N–'s flag's halyard

about

has metal elements that clank loudly against the pole when

“procedures” and

it's windy, which is something else the other neighbors don't

“etiquette” and

care for. His driveway and mine are almost side by side, and what is proper he's out here on a stepladder polishing his pole with some -"Or something" kind of ointment and a chamois cloth… and in fairness it's

Take  your  S   AT  prep  to  the  next  level.  Visit  www.learnerator.com   65  

= our narrator is

66     true that his metal pole does shine like God's own

a bit flippant!

wrath. "He[ck] of a nice flag and display apparatus, Mr. N–.""Ought to be. Cost enough." "Seen all the other flags out everywhere this morning? "This gets him to look down and smile, if a bit grimly. "Something isn't it?" Mr. N– is not

Biggest flag

what you'd call the friendliest next-door neighbor. I

because he thinks

really only know him because his church and mine are in the that makes him seem like the same softball league, for which he serves with immense precision as his team's statistician. We are not

biggest

close. He's nevertheless the first one I ask: "Say Mr. N–,

patriot? Clearly

suppose somebody like a foreign person or TV reporter were puts a lot of effort into appearances. to come by and ask you to say what the purpose of all these flags everywhere after the Horror and everything yesterday What’s the was, exactly – what do you think you'd say?" "Why" (after connection a brief interval of giving me the same sort of look he between “God’s wrath” and usually gives my lawn) "to show our support and empathy in terms of what's going on, as Americans. "The

displaying an

point being that on Wednesday here there's a weird

American flag

accretive pressure to have a flag out. If the purpose

right after 9/11?

of a flag is to make a statement, it seems like at a

Think about this

certain point of density of flags you're making more idea… of a statement if you don't have one out. It's not

More details

totally clear what statement this would be. What if

about the

you just don't happen to have a flag? Where has

neighbor. Think

everyone gotten these flags, especially the little ones you can about the put on your mailbox? Are they all from July 4th and people contrast between just save them, like Christmas ornaments? How do they

the two men

know to do this? Even a sort of half-collapsed house down the street that everybody though was unoccupied has a flag in the ground by the driveway. The Yellow Pages have nothing under Flag. There's actual interior tension:

Again, a very

Nobody walks by or stops their car and says, "Hey,

telling detail –

your house doesn't have a flag," but it gets easier

contrast between

Take  your  S   AT  prep  to  the  next  level.  Visit  www.learnerator.com   66  

67     and easier to imagine people thinking it. None of the

the two

grocery stores in town turn out to stock any flags. The novelty shop downtown has nothing but Halloween stuff. Only a few businesses are open, but even the closed ones are displaying some sort of flag. It's almost surreal. The VFW

Accretive =

hall is a good bet, but it can't open til noon if at all (it has a

accumulated,

bar). The lady at Burwell's references a certain hideous Qik-

built up gradually

n-EZ store out by 1-74 at which she was under the impression she'd seen some little plastic flags back in the racks with all the bandannas and Nascar caps, but by the time I get there they turn out to be gone, snapped up by parties unknown. The reality is that there is not a flag to be had in this town. Stealing one out of somebody's yard is clearly out of the question. I'm standing in a Qik-n-EZ afraid to go home. All those people dead, and I'm sent to the edge by a plastic flag. …Until in one more of the Horror's weird twists of fate and circumstance it's the Qik-n-EZ proprietor himself (a Pakistani, by the way) who offers

Pressure! He feels pressure to put out a flag, like he’s not being patriotic “enough” Everyone has a flag but him!

solace and a shoulder and a strange kind of unspoken

Feels very

understanding, and who lets me go back and sit in the

conspicuous

stock room amid every conceivable petty vice and indulgence America has to offer and compose myself, and who only slightly later, over styrofoam cups of a strange kind of tea with a great deal of milk in it, suggests, gently, construction paper and "Magical Markers," which explains my now-beloved homemade flag.

How is the writer internalizing, understanding, and processing the tragedy?

Why would this man offer “unspoken understanding”? In what way are

Take  your  S   AT  prep  to  the  next  level.  Visit  www.learnerator.com   67  

68     they both outsiders?

How does the writer resolve his problem? Do you think that his neighbor would approve of this solution?     Look  back  at  your  notes  –  and  by  the  way,  it  doesn’t  matter  if  there  are  words  whose   meaning  you  don’t  know;  what  matters  is  that  you  get  the  general  gist.   So,  what  are  the  repeated  ideas  in  this  excerpt?   Well,  there’s  flags  –  and,  more  subtly,  what  these  flags  might  mean.  But  certainly  it’s   not  enough  to  say  “the  main  idea  of  this  essay  is  flags.”  Because  again,  that  doesn’t   tell  us  anything  important.   Therefore,  think  about  the  people  mentioned  in  this  essay.  How  do  they  reactto  the   issue  of  flags,  and  why  do  they  react  this  way?  This  might  make  it  easier  to  distill   this  big  essay  into  one  idea.     1.  There’s  our  unnamed  narrator,  who  does  not  have  a  flag  and  feels  bad  /  like  an   outsider  about  it.  (Remember,  he  says  he  feels  “pressure,”  and  like  he  is  “making  a   statement,”  and  he’s  being  “sent  to  the  edge…”  over  this  issue.)   2.  There’s  Mr.  N  from  next  door,  who  has  a  flag  and  clearly  takes  lots  of  pride  in  it.   (It’s  regulation-­‐size,  raised  and  lowered  with  military  precision,  and  as  polished   as  “the  wrath  of  God.”)  

Take  your  S   AT  prep  to  the  next  level.  Visit  www.learnerator.com   68  

69     3.  There’s  the  Pakistani  owner  of  the  shop,  who  listens  to  our  narrator  and   provides  him  with  a  solution  to  his  flag  problem.  (“Offers  solace,”  “gentle   suggestion,”  “unspoken  understanding.”)   Clearly,  the  essay  is  about  more  than  who  has  a  flag.  It’s  clearly  about  what  the  flag   represents  to  each  person  in  this  excerpt.  Remember,  finally,  that  this  essay  was   written  two  days  after  9/11.  The  context  here  is  crucial.  Therefore:     What  is  the  main  idea  of  this  passage  as  a  whole?   A.  The  script  by  which  different  people  process  and  symbolize  complex   emotions,  such  as  patriotism  and  grief   B.  The  proper  procedures  for  showing  off  patriotism  and  reacting  to  a  national   tragedy   C.  The  discrimination  against  Pakistanis  after  9/11   D.  The  pressure  that  comes  with  not  being  adequately  prepared  to  react  to  a   tragedy     Now,  think  about  this  carefully.  Most  of  these  answers  are  pretty  plausible,  because   they  all  have  something  to  do  with  the  passage.  Let’s  start  with  A  –  the  script  by   which  people  process  and  symbolize  emotions  like  patriotism  and  grief.   Now,  Wallace  never  mentions  a  script,  but  he  doesn’t  have  to;  the  majority  of  the   passage  is  about  the  fact  that  everyone  seems  to  have  a  flag  but  him,  as  though  they   are  all  following  a  memo  that  he  didn’t  receive.  (“Where  has  everyone  gotten  these   flags,  especially  the  little  ones  you  can  put  on  your  mailbox?  Are  they  all  from  July  4th   and  people  just  save  them,  like  Christmas  ornaments?  How  do  they  know  to  do  this?   Even  a  sort  of  half-­‐collapsed  house  down  the  street  that  everybody  though  was   unoccupied  has  a  flag  in  the  ground  by  the  driveway.”)  So  the  script  part,  while  subtle,   is  true.  But  what  about  this  “symbolize  complex  emotions”?  At  this  point,  you  should   realize  that  the  flags  here  are  not  just  objects  but  representations  of  a  feeling.  After   all,  Wallace  asks  his  neighbor:     "Say  Mr.  N–,  suppose  somebody  like  a  foreign  person  or  TV  reporter  were  to  come  by   Take  your  S   AT  prep  to  the  next  level.  Visit  www.learnerator.com   69  

70     and  ask  you  to  say  what  the  purpose  of  all  these  flags  everywhere  after  the  Horror  and   everything  yesterday  was,  exactly  –  what  do  you  think  you'd  say?"     Clearly,  this  question  demonstrates  that  there  is  a  larger  purpose  to  these  flags,  that   they  symbolize  and  demonstrate  something  big.  And  that’s  clear  in  Mr.  N’s  answer,   too:  “to  show  our  support  and  empathy  in  terms  of  what's  going  on,  as   Americans.”  Again,  the  flags  are  part  of  a  script.  They  are  the  answer  to  a  tragedy  and   a  means  of  showing  support,  which  is  demonstrated  also  in  the  different  types  of   flags  on  display  –  the  huge  banner-­‐style  ones,  the  regulation-­‐size  ones  on  flagpoles,   the  plastic  ones,  and  even  the  author’s  homemade  Magic  Marker  one.     Now,  let’s  look  at  the  other  answer  choices:   B.  The  proper  procedures  for  showing  off  patriotism  and  reacting  to  a  national   tragedy,  is  a  worthwhile  contender.  However,  remember  that  at  first,  Wallace  does   not  have  a  flag.  Based  on  his  characterization  of  himself  and  his  flag-­‐less-­‐ness  (“If  the   purpose  of  a  flag  is  to  make  a  statement,  it  seems  like  at  a  certain  point  of  density  of   flags  you're  making  more  of  a  statement  if  you  don't  have  one  out.  It's  not  totally  clear   what  statement  this  would  be.  What  if  you  just  don't  happen  to  have  a  flag?”),  is   Wallace  the  sort  of  person  who  would  get  hung  up  on  “proper  procedures”?   Especially  when  you  contrast  him  with  Mr.  N,  who  gives  him  “the  same  sort  of  look   he  usually  gives  my  lawn.”  Though  there  are  points  in  this  essay  that  are  concerned   with  procedure,  these  points  are  more  about  how  other  people  seem  to  be  acting   (again,  following  a  script),  rather  than  an  actual  guide  on  the  best  way  to  act.     C  is  perhaps  the  easiest  answer  to  disregard;  the  Pakistani  storeowner  is  only   mentioned  at  the  very  end  of  the  essay,  and  though  Wallace  does  mention  the  store   owner’s  “unspoken  understanding”  (which  alludes,  in  turn,  to  the  outsider  status  he   probably  feels  in  the  wake  of  the  tragedy),  nowhere  does  he  explicitly  mention   discrimination.  Additionally,  this  “unspoken  understanding”  is  only  mentioned  once,   and  a  main  idea  is  usually  repeated  and  reinforced  in  multiple  ways.  This  is  clearly   not  the  correct  answer.   Take  your  S   AT  prep  to  the  next  level.  Visit  www.learnerator.com   70  

71       D  is  a  contender  –  clearly,  Wallace  feels  pressure!  This  is  an  instance  where  there   are  some  good  answers  that  pale  in  comparison  to  the  “best”  answer.  While  it’s  true   that  there  is  great  pressure  in  feeling  that  you  are  not  reacting  appropriately  to  /   prepared  adequately  for  a  national  tragedy,  these  “reactions”  and  “preparations”  are   in  turn  just  part  of  a  larger  script  that  tells  people  how  they  should  act,  how  they   should  deal  with  their  lack  of  preparedness.  D  is  a  good  answer  because  it’s  really   part  of  answer  A  –  in  many  ways,  a  fragmented  or  incomplete  choice  that  does  not   fully  express  the  main  idea  of  the  essay.     That  was  a  long,  high-­‐level  passage–  not  so  much  because  of  the  words  or  the   structure,  but  because  it  is  full  of  subtly-­‐expressed  and  complex  ideas.  In  the  next   section,  you  will  learn  how  to  break  down  and  summarize  these  kinds  of  ideas.  

Take  your  S   AT  prep  to  the  next  level.  Visit  www.learnerator.com   71  

72    

Summarizing Ever  tried  to  tell  a  friend  a  story?  When  you’re  recounting  an  event  or  a  film,  you   automatically  make  choices  about  which  details  to  emphasize  and  which  to  leave   out.  And  though  your  friend  probably  won’t  mind  if  you  leave  in  irrelevant  details,   one  of  the  most  highly-­‐rated  skills  in  secondary  and  post-­‐secondary  education  is  the   ability  to  summarize  key  information,  which  explains  why  the  College  Board  cares  so   much  about  whether  or  not  you  can  do  it  well.     There  isn’t  a  big  difference  between  identifying  the  main  idea  and  being  able  to   cogently  summarize  the  text;  after  all,  the  main  idea  and  the  summary  are   interrelated  concepts.  In  a  sense,  “summarizing”  just  means  that  you  are  able  to   restate  the  main  idea  of  all  of  or  part  of  a  text  and  marshal  key  pieces  of  evidence  to   support  your  claim.     Learning  how  to  accurately  and  intelligently  summarize  information  requires  three   essential  skills:   1.  Comprehension  of  the  material   2.  The  ability  to  identify  and  isolate  key  supporting  details   3.  The  ability  to  describe,  in  general  terms,  what  those  details  work  together   to  say   Your  ability  to  do  this  may  depend  on  the  level,  complexity,  and  type  of  text  you’re   reading.  Luckily,  every  type  of  text  will  have  certain  “signals”  that  let  you  know   which  pieces  of  information  are  important.  

Take  your  S   AT  prep  to  the  next  level.  Visit  www.learnerator.com   72  

73     Structure     Remember  that  text  structure  is  paramount  in  simpler  non-­‐fiction  texts,  with  each   paragraph  introduced  (or  summarized)  by  a  topic  sentence  that  presents  the  main   idea  of  the  paragraph.  It’s  these  bracketing  sentences  –  the  first  and  last  of  each   paragraph  –  that  will  work  together  to  present  a  useful  summary  of  the  selection  as   a  whole.   For  example,  take  this  excerpt,  from  a  lower-­‐complexity  non-­‐fiction  article  titled   “Traveling  Teaches  Students  in  a  Way  Schools  Can’t,”  recently  published  in  the   Atlantic.  I  have  bolded  parts  of  the  first  and  last  sentences  in  each  paragraph,  which   should  all  help  you  to  summarize  the  text  of  the  passage  as  a  whole.     When  I  turned  15,  my  parents  sent  me  alone  on  a  one-­‐month  trip  to   Ecuador,  the  country  where  my  father  was  born.  This  was  tradition  in  our   family—for  my  parents  to  send  their  first-­‐generation  American  kids  to  the   country  of  their  heritage,  where  we  would  meet  our  extended  family,   immerse  ourselves  in  a  different  culture,  and  learn  some  lessons  on   gratefulness.      My  family’s  plan  worked.  That  month  in  Ecuador  did  more  for  my  character,   education,  and  sense  of  identity  than  any  other  experience  in  my  early  life.  And   five  years  later,  my  experience  in  Ecuador  inspired  me  to  spend  more  time   abroad,  studying  in  South  Africa  at  the  University  of  Cape  Town.  These  two   trips  not  only  made  me  a  lifelong  traveler,  but  also  a  person  who  believes   traveling  to  developing  countries  should  be  a  necessary  rite  of  passage   for  every  young  American  who  has  the  means.     It’s  often  said  that  spending  time  in  less  affluent  countries  teaches   Americans  never  to  take  anything  for  granted.  To  some  extent,  this  is  true.   During  my  time  traveling  in  these  areas,  I  often  traveled  without  access  to  hot   water,  Internet,  air  conditioning,  or  even  basic  electricity.  I  slept  in  rooms  with   Take  your  S   AT  prep  to  the  next  level.  Visit  www.learnerator.com   73  

74     spiders,  mosquitos,  and  bedbugs.  I  rode  on  public  transportation  that  rarely  left   on  time  and  often  broke  down  suddenly  in  remote  areas.  Stripped  of  my  daily   habits  and  expectations,  I  was  forced  to  surrender  the  idea  that  I  have  a  right   to  anything—including  the  luxury  of  convenience,  or  days  when  everything  I’ve   planned  actually  happens.  And  my  minor  travel  hassles  seemed  even  more   petty  when  I  realized  that  they  represented  larger  systemic  problems   that  locals  must  deal  with  every  day.     But  these  trips  didn’t  only  teach  me  to  appreciate  what  I  had;  they  also   moved  me  to  consider  why  I  had  it  in  the  first  place.  I  realized  that  much  of   what  I  thought  was  necessity  was,  in  fact,  luxury  and  began  to  realize  how   easily  I  could  survive  off  of  much  less.  I  didn’t  necessarily  need  hot  water  or  a   timely  bus  or  a  comfortable  bed  to  be  happy  for  the  day.  I  didn’t  necessarily   need  a  jaw-­‐dropping  landscape  or  a  famous  archeological  ruin  or  a  stunning   beach  to  make  my  travels  worth  it.  Instead,  most  of  the  time,  that  fulfillment   came  from  the  people  I  interacted  with—not  the  things  I  had  or  did.  It  came   from  eating  soup  with  locals  at  a  rest  stop  on  a  12-­‐hour  bus  ride,  sharing  a   meal  with  Peruvian  soccer  fans  while  watching  a  match,  or  chatting  with  the   owner  of  my  hostel  during  his  lunch  break.  Discovering  that  my  best  travel   moments  came  from  these  subtle,  personal  moments  instead  of  the  grandiose,   materialistic  ones  made  me  understand  that  living  contently  required   little.  What  I  originally  thought  I  “took  for  granted,”  I  now  rethought   taking  at  all.           1.  What  is  the  most  accurate  summary  of  this  article?   A.  The  author  is  a  lifelong  traveler.   B.  The  author  believes  that  minor  hassles  are  meaningless.   C.  The  author  advocates  travel  abroad  as  an  educational  tool.   D.  The  author  never  takes  anything  for  granted.    

Take  your  S   AT  prep  to  the  next  level.  Visit  www.learnerator.com   74  

75     Now,  all  of  these  options  were  mentioned  somewhere  in  the  text;  maybe  even  in  the   bracketing  sentences.  But  all  the  distractors  are  single  details  that  work  together  to   create  one  cohesive  theme,  which  can  be  best  summarized  in  Option  C.     Remember,  when  thinking  about  summaries,  you  can  and  should  be  as  general  as   possible;  a  summary  isn’t  a  blow-­‐by-­‐blow  account  chock-­‐full  of  individual  details,   but  a  general  “gist”  or  zoomed-­‐out  picture  of  the  main  idea(s).  In  a  simple  non-­‐ fiction  text,  it’s  easy  to  identify  these  ideas  if  you  look  at  the  topic  sentences  of  every   paragraph.   Repetition  of  Ideas     Whenever  the  writer  repeats  a  string  of  words,  a  concept,  or  idea  in  the  text,  it’s  safe   to  say  that  these  repetitions  will  be  key  in  your  general  summary.  Remember,  the   writer  doesn’t  have  to  use  the  exact  same  words  each  time  in  order  to  establish   repetition  –  you  can  repeat  ideas  without  repeating  words.  But  it’s  hard  to  diversify   the  vocabulary  when  you’re  explaining  the  same  thought  over  and  over,  so  there’s  a   good  chance  that  much  of  the  repetitions  will  utilize  at  least  some  of  the  same   words,  making  it  easy  to  skim  the  material  and  find  the  most  heavily-­‐emphasized   ideas.   Take  the  following  example,  from  the  Public  Library  of  Science  Blog.  Here  I  have   bolded  every  instance  of  repetition:   The  American  Society  of  Magazine  Editors  awards  are  the  Academy  Awards  of   magazinedom,  and  this  year  all  the  finalists  in  all  the  long-­‐form,  narrative   categories  went  to  men.  …the  dearth  of  women  among  the  nominees  can   perhaps  be  explained  in  part  by  the  dearth  of  female  bylines  in  the  sort  of   magazines  that  publish  long-­‐form  narrative  journalism.     At  the  New  Yorker,  Harper’s,  The  New  Republic  and  The  Atlantic,  for  instance,   less  than  thirty  percent  of  the  stories  published  in  2011  were  written  by   women,  according  to  this  year’s  VIDA  Count,  which  did  a  gender  breakdown  of   bylines  in  each  magazine.   Take  your  S   AT  prep  to  the  next  level.  Visit  www.learnerator.com   75  

76       I’ve  long  bemoaned  the  fact  that  there  are  so  few  women  represented  among   the  contributing  editors  on  many  of  these  publications.  (Contributing  editors   are  the  writers  whose  work  you’re  most  likely  to  see  in  the  mag;  they  either   have  contracts  for  a  certain  number  of  words  a  year  or  just  enjoy  a  privileged   relationship  with  the  editors.)  It’s  also  true  that  many  of  the  magazines  that   publish  narrative  pieces  are  staffed  largely  by  men.  Part  of  the  reason  is   that  a  lot  of  the  narrative  journalism  is  published  in  magazines  targeted  at   men.     Take  Esquire,  for  instance,  which  often  scoops  up  ASME  nominations  and   awards:  On  the  masthead  of  the  March  issue,  of  the  33  editorial  staffers  listed  – including  the  photo,  art,  and  fashion  people—only  nine  are  women.  And  from   what  I  can  tell,  only  one  or  possibly  two  of  those  are  in  a  position  to  assign   stories.     2.  How  would  you  summarize  this  article?   A.  There  are  too  many  male  journalists.   B.  There  are  not  enough  female  journalists.   C.  Women  are  not  accurately  and  adequately  represented  in  long-­‐form   journalism.   D.  The  magazines  that  publish  long-­‐form  journalism  are  at  fault  for  the   lack  of  female  representation.     Remember,  summarizing  requires  finding  the  general  idea  based  on  what  the  text   actually  says.  Unlike  with  other  skills  on  the  new  SAT,  the  answers  are  already   there;  all  you  need  to  do  is  rephrase  them,  rather  than  making  inferences.     Check  the  bolded  portions  that  repeat  the  main  idea;  none  of  them  say  that  there  are   too  many  male  journalists,  just  that  male  journalists  are  overrepresented.  Similarly,   this  article  doesn’t  say  there  aren’t  enough  female  journalists,  just  that  they’re  not   Take  your  S   AT  prep  to  the  next  level.  Visit  www.learnerator.com   76  

77     recognized  (or  even  hired)  in  the  same  numbers  as  men.  Nor  does  the  piece   explicitly  blame  the  magazines  –  it  just  highlights  the  fact  that  a  discrepancy  exists   between  male  and  female  representation  in  journalism.  Clearly,  all  the  repetitions   serve  to  emphasize  that  female  journalists  exist,  but  are  simply  underrepresented,   as  is  presented  in  answer  C.   Repetition  of  Details     Sometimes  an  author  will  use  an  abundance  of  details  to  continually  demonstrate   one  overarching  idea.  These  details  will  work  together  to  illustrate  a  fact  or  concept   that  the  author  wants  to  emphasize.  For  example,  take  this  non-­‐fiction  piece  about   pupfish.  I’ve  bolded  every  detail  that  the  author  includes  in  order  to  try  and   understand  she  is  trying  to  emphasize  –  which  in  turn  will  lead  me  to  a  cogent   summary  of  this  excerpt.     West  of  Pahrump,  Nevada,  in  a  corner  of  the  Mojave  Desert  a  couple   thousand  feet  above  Death  Valley,  a  warm  aquifer  provides  a  home  for  one  of   the  world’s  rarest  animals.  It’s  a  tiny  silvery-­‐blue  fish,  smaller  than  your   pinkie  toe,  and  in  the  past  50  years  it  has  survived  real-­‐estate  speculators,   death  threats,  congressional  battles,  and  human  screwups.  The  Devil’s  Hole   pupfish—Cyprinodondiabolis—is  nothing  if  not  tenacious.     But  the  biggest  existential  threat  to  the  pupfish  comes  from  its  own  DNA.  Once   upon  a  time,  pupfish  lived  in  a  sprawling  lake.  Around  20,000  years  ago,   water  levels  dropped,  the  landscape  turned  to  desert,  and  the  pupfish  ended  up   in  disconnected  ponds.  Today,  nine  different  species  are  scattered  across   the  Southwest,  and  half  of  them  are  endangered.  Devil’s  Hole  is  the  worst  case;   as  of  September  2012,  there  were  75  fish  left.  Thousands  of  years  of   adaptation  have  left  the  Devils  Hole  pupfish  able  to  live  only  in  one  very   particular  environment:  It  needs  90-­‐degree  water,  low  oxygen,  and  a   shallow  submerged  ledge  on  which  to  spawn.  It’s  hard  enough  being   endangered;  being  endangered  and  picky  is  a  deadly  combination.   Take  your  S   AT  prep  to  the  next  level.  Visit  www.learnerator.com   77  

78       Think  about  what  these  details  are  doing  in  this  piece:  they  are  establishing  a  sense   of  time  and  place  (Mojave  desert,  September  2012),  drawing  a  portrait  of  the  fish   (small,  tenacious),  and  establishing  what  kind  of  threats  the  endangered  fish  face   and  why  (environment,  ability  to  breed  only  in  very  specific  places).  Though  all  of   these  details  are  different,  they  work  together  to  serve  the  same  purpose.  Together,   these  details  will  help  you  answer  the  question:     3.  What  is  the  best  summary  of  this  selection?   A.  Devil’s  Hole  pupfish  are  tenacious  despite  the  odds  they  face.   B.  Due  to  a  combination  of  factors,  Devil’s  Hole  pupfish  face  extinction.   C.  It  is  crucial  that  the  Devil’s  Hole  pupfish  continue  to  survive.   D.  Devil’s  Hole  pupfish  are  extremely  rare.     Now,  many  of  these  details  were  mentioned  in  the  article.  But  think  about  the  “big   picture”  that  these  details  work  together  to  illustrate.  You  need  to  be  able  to  focus   on  one  general  idea  that  doesn’t  extrapolate  from  the  text,  but  simply  gives  a  big-­‐ picture  restatement  of  what  the  text  actually  says.  Option  A  is  mentioned  in  the  text,   but  it  doesn’t  do  a  thorough-­‐enough  job  of  summarizing  the  “big  picture.”  This  detail   is  only  mentioned  once  and  isn’t  explicitly  reinforced  throughout  the  text,  so  it  can’t   be  the  best  summary  of  the  selection.  Option  B  does  a  much  better  job  of  providing  a   summary  –  it  mentions  a  combination  of  factors  (which  is  general  enough  to   encapsulate  many  of  the  details  mentioned  in  the  piece)  and  neatly  summarizes  the   point  of  the  selection  –  that,  as  the  last  sentence  states,  “being  endangered  and  picky   is  a  deadly  combination;”  i.e.,  the  pupfish  face  extinction.  Option  C  is  never  explicitly   mentioned  in  the  text;  while  you  may  be  tempted  to  extrapolate  from  the  text  that  it   is  important  for  biodiversity  that  the  pupfish  survive,  stick  to  what  is  explicitly   stated.  Do  not  make  inferences  while  trying  to  summarize.  Choice  D  is  true,  but  it   isn’t  the  best  summary  –  it’s  just  one  small  detail  that  helps  to  build  the  bigger   picture  of  the  main  idea.     Take  your  S   AT  prep  to  the  next  level.  Visit  www.learnerator.com   78  

79     Even  when  the  details  seem  disparate,  they  often  work  together  to  create  a  cohesive   whole;  when  in  doubt,  think  about  which  answer  option  is  best  SUPPORTED  by  the   details,  and  which  options  merely  help  BUILD  a  larger  idea.  This  will  help  you  better   determine  an  accurate  and  succinct  summary  for  the  piece.      

Take  your  S   AT  prep  to  the  next  level.  Visit  www.learnerator.com   79  

80     Signal  Words  and  Text  Features     When  attempting  to  find  key  ideas,  watch  out  for  words  that  introduce  important   concepts.  Words  like  “important,”  “paramount,”  “clearly,”  and  “obviously,”   “essentially,”  “basically,”  “in  essence,”  “thus,”  and  “ergo”  –among  others—can  signal   that  the  writer  is  introducing  or  summing  up  an  essential  idea.  Words  like  “for   example”  can  help  to  demonstrate  an  essential  concept.  Bolding,  italics,  and  CAPS   can  also  highlight  important  ideas,  so  watch  out  for  those  as  well.  Of  course,  this   isn’t  a  hard-­‐and-­‐fast  rule,  but  it  can  be  helpful  when  you’re  trying  to  decide  which   ideas  to  isolate.   For  example,  take  this  snippet  on  teaching  literacy:       The  concept  of  literacy  is  fundamentally  misunderstood.  There  is  a   misconception  that  literacy  simply  means  “knowing  how  to  read,”  and  while   that  is  partially  true,  it  doesn’t  present  the  whole  picture.  “Knowing  how  to   read”  is  about  much  more  than  just  matching  up  letters  with  the  sounds  that   they  make;  it  is  also  about  understanding  how  information  fits  together  in   context.  Students  who  excel  in  one  context  may  fail  in  another  because  they   lack  the  multiple  types  of  literacy  that  are  necessary  to  succeed.     For  example,  there  is  a  well-­‐known  anecdote  about  a  high-­‐level  student  who   continually  failed  her  math  homework.  Finally,  she  was  sent  to  a  learning   specialist  who  tried  to  diagnose  the  problem.  The  specialist  asked  the  student  to   read  and  explain  one  of  the  math  problems  that  was  giving  her  so  much   trouble.  The  student  read  fluently  “Find  the  product  of  two  integers,”  but  when   it  came  time  to  explain,  she  was  at  a  loss.  She  did  know  how  to  explain  what  a   product  was,  in  the  mathematical  sense,  though  she  could  perfectly  explain  the   concept  of  a  “product”  in  a  historical/social  sciences  setting.     Clearly,  though  she  could  read  perfectly  and  scored  high  in  all  her  English   classes,  she  lacked  basic  math  literacy.   Take  your  S   AT  prep  to  the  next  level.  Visit  www.learnerator.com   80  

81       There  are  a  lot  of  signal  words  here,  and  they  all  work  together  to  demonstrate  a   concept.  Just  as  in  the  last  passage,  there  are  also  details  that  work  together  to   present  a  big  picture,  which  you  will  be  asked  to  summarize.     4.  What  is  the  best  summary  for  this  passage?   A.  The  definition  of  literacy  is  disputed.   B.  Many  people  lack  basic  literacy  skills.   C.  There  are  multiple  types  of  literacy.   D.  Words  can  have  different  meanings,  which  makes  literacy  complicated.     Remember,  think  about  how  the  details  and  the  signal  words  work  together  to   create  one  cohesive  idea  that  is  explicitly  stated  in  the  text.  Nowhere  does  it  say  that   the  meaning  of  literacy  is  “disputed,”  just  that  it  is  “misunderstood.”  You  can  scrap   Option  A.  While  B  may  be  a  tempting  option,  remember  that  it  is  never  explicitly   stated  in  the  text,  and  that  it  is  too  much  of  an  extrapolation  from  the  detail  that   students  may  fail  in  certain  contexts  because  they  lack  a  specific  type  of  literacy.   Option  C  is  supported  in  the  text;  it  is  both  explicitly  stated  in  the  phrase  “multiple   types  of  literacy…  are  necessary  to  succeed”  and  demonstrated  in  the  example  of  the   student  who  had  one  type  of  literacy  but  not  the  other.  Lastly,  Option  D  may  be  true   –  it  is  illustrated  in  the  anecdote  –  but  it  is  not  the  point  of  the  selection;  the   selection  is  there  to  present  a  general  idea  or  main  point  about  literacy,  and  Option   D  is  too  specific.    

Take  your  S   AT  prep  to  the  next  level.  Visit  www.learnerator.com   81  

82     In  Summary     Remember  that  there  are  multiple  ways  to  come  up  with  a  good  summary  of  a  text.   Pay  attention  to  structure,  repetition  of  ideas  and  details,  and  signal  words  and  text   features.  Remember  to  keep  your  summary  general  and  “big  picture”  instead  of   focusing  on  the  multiple  smaller  details  that  work  together  to  create  the  general   idea  of  the  text.  Finally,  don’t  extrapolate  –  when  presenting  a  summary,  stick  to   what  is  explicitly  stated  in  the  text.  Utilizing  these  strategies  will  help  you  score  high   on  any  summary  questions,  on  both  the  SAT  and  in  your  classes.  

   

 

Take  your  S   AT  prep  to  the  next  level.  Visit  www.learnerator.com   82  

83    

Understanding Relationships Part  of  demonstrating  reading  comprehension  skills  is  the  ability  to  understand  the   relationships  between  the  individuals,  ideas,  and  events  mentioned  in  a  text.  This   understanding  will  help  you  better  understand  the  selection.   There  are  multiple  types  of  relationships  that  may  appear  in  a  selection.  Here  are   some  of  the  most  common.   Relationships  between  Individuals     Selections  drawn  from  narrative  pieces  will  usually  feature  characters;  these   characters  will  interact  in  a  variety  of  ways,  which  will  demonstrate  their   relationships.  The  SAT  may  ask  you  about  the  type  of  relationship  that  the   characters  have  to  one  another  –  are  they  siblings?  Coworkers?  Friends?  –  based  on   context,  or  it  may  ask  you  to  qualitatively  describe  the  relationship  –  amicable?   Acrimonious?  –  based  on  the  interactions  that  the  characters  have  and  the  way  that   they  speak  to  one  another.     Take  this  example,  from  Scarlet  Stockings,  a  short  story  by  Louisa  May  Alcott.  As   you  read,  mark  up  the  points  in  the  passage  where  the  characters  reveal  through   their  speech  either  the  NATURE  of  their  relationship  or  a  CHARACTERIZATION  of   their  relationship.  Pay  attention  also  to  characters  who  are  mentioned  but  not   present,  as  well  as  the  characters’  relationship  to  their  setting  (where  it’s   applicable).     "COME  out  for  a  drive,  Harry?"  

-­‐  One  character  spends  a  

"Too  cold."  

long  time  clearly  trying  to  

"Have  a  game  of  billiards?"  

amuse  the  other,  who  

"Too  tired."  

declines  to  be  amused  

"Go  and  call  on  the  Fairchilds?"  

-­‐  This  second  character  is  

Take  your  S   AT  prep  to  the  next  level.  Visit  www.learnerator.com   83  

84     "Having  an  unfortunate  prejudice  against  country  

kind  of  a  snob  

girls,  I  respectfully  decline."  

-­‐  So  these  two  are  sister  

"What  will  you  do  then?"  

and  brother  

"Nothing,  thank  you."   And  settling  himself  more  luxuriously  upon  the  couch,   Lennox  closed  his  eyes,  and  appeared  to  slumber   tranquilly.  Kate  shook  her  head,  and  stood  regarding   her  brother,  despondently,  till  a  sudden  idea  made  her   turn  toward  the  window,  exclaiming  abruptly,   "Scarlet  stockings,  Harry!"       "Where?"  and,  as  if  the  words  were  a  spell  to  break  

-­‐  Kate  knows  how  to  get  

the  deepest  day-­‐dream,  Lennox  hurried  to  the  window,   her  brother’s  interest   with  an  unusual  expression  of  interest  in  his  listless  

-­‐  We  can  reasonably  

face.  

extrapolate  that  Harry  has  

"I  thought  that  would  succeed!  She  isn't  there,  but  I've  

a  little  crush  

got  you  up,  and  you  are  not  to  go  down  again,"  laughed  

-­‐  Harry  is  not  embarrassed  

Kate,  taking  possession  of  the  sofa.  

in  the  least  

"Not  a  bad  maneuver.  I  don't  mind;  it's  about  time  

-­‐  He  clearly  likes  the  red-­‐

for  the  one  interesting  event  of  the  day  to  occur,  so  

stocking  girl,  but  is  pretty  

I'll  watch  for  myself,  thank  you,"  and  Lennox  took  the  

passive  about  it  

easy  chair  by  the  window  with  a  shrug  and  a  yawn.   "I'm  glad  any  thing  does  interest  you,"  said  Kate,   petulantly,  "though  I  don't  think  it  amounts  to  much,   for,  though  you  perch  yourself  at  the  window  every   day  to  see  that  girl  pass,  you  don't  care  enough   about  it  to  ask  her  name."   "I've  been  waiting  to  be  told."     "It's  Belle  Morgan,  the  Doctor's  daughter,  and  my  

Take  your  S   AT  prep  to  the  next  level.  Visit  www.learnerator.com   84  

-­‐  Another  relationship  

85     dearest  friend."  

established  

"Then,  of  course,  she  is  a  blue-­‐belle?"  

-­‐  A  potential  (and  

"Don't  try  to  be  witty  or  sarcastic  with  her,  for  she  

potentially  cantankerous)  

will  beat  you  at  that."  

relationship  established  

"Not  a  dumb-­‐belle  then?"  

-­‐  Harry  asking  if  she  has  a  

"Quite  the  reverse;  she  talks  a  good  deal,  and  very  well  

boyfriend  (ma  belle  =  “my  

too,  when  she  likes."  

beauty”);  character  detail  

"She  is  very  pretty;  has  anybody  the  right  to  call  her  

established  

'Ma  belle'?"  

-­‐  More  relationships  

"Many  would  be  glad  to  do  so,  but  she  won't  have  any  

established  

thing  to  say  to  them."  

-­‐  Kate  clearly  cares  for  and  

"A  Canterbury  belle  in  every  sense  of  the  word  then?"  

worries  about  her  brother  

"She  might  be,  for  all  Canterbury  loves  her,  but  she  isn't   fashionable,  and  has  more  friends  among  the  poor   than  among  the  rich."   "Ah,  I  see,  a  diving-­‐bell,  who  knows  how  to  go  down  into   a  sea  of  troubles,  and  bring  up  the  pearls  worth  having."   "I'll  tell  her  that,  it  will  please  her.  You  are  really   waking  up,  Harry,"  and  Kate  smiled  approvingly   upon  him.   "This  page  of  'Belle's  Life'  is  rather  amusing,  so  read   away,"  said  Lennox,  glancing  up  the  street,  as  if  he   awaited  the  appearance  of  the  next  edition  with   pleasure.     "There  isn't  much  to  tell;  she  is  a  nice,  bright,  

-­‐  Think  of  the  

energetic,  warm-­‐hearted  dear;  the  pride  of  the  

relationships  being  

Doctor's  heart,  and  a  favorite  with  every  one,  though   established  here   she  is  odd."  

-­‐  Establishment  of  a  

"How  odd?"  

possible  relationship  

Take  your  S   AT  prep  to  the  next  level.  Visit  www.learnerator.com   85  

86     "Does  and  says  what  she  likes,  is  very  blunt  and  honest,  

between  two  people  who  

has  ideas  and  principles  of  her  own,  goes  to  parties  in  

haven’t  met  yet  

high  dresses,  won't  dance  round  dances,  and  wears  red   stockings,  though  Mrs.  Plantagenet  says  it's  fast."   "Rather  a  jolly  little  person,  I  fancy.  Why  haven't  we   met  her  at  some  of  the  tea-­‐fights  and  muffin-­‐ worries  we've  been  to  lately?"   "It  may  make  you  angry,  but  it  will  do  you  good,  so  I'll  

-­‐  Clearly,  Harry  doesn’t  

tell.  She  didn't  care  enough  about  seeing  the  

care  much  for  his  society.  

distinguished  stranger  to  come;  that's  the  truth."  

(Dyspepsia  =  indigestion)  

"Sensible  girl,  to  spare  herself  hours  of  mortal  

-­‐  Typical  sibling  

dullness,  gossip,  and  dyspepsia,"  was  the  placid  reply.  

relationship  –  trying  to  get  

"She  has  seen  you,  though,  at  church  and  dawdling  about   a  rise  out  of  her  brother   town,  and  she  called  you  'Sir  Charles  Coldstream'  on  the  

-­‐  Kate  criticizing  her  

spot.  How  does  that  suit?"  asked  Kate,  maliciously.  

brother  (lovingly);  Harry  

"Not  bad,  I  rather  like  that.  Wish  she'd  call  some  day,  and   demonstrating  he  doesn’t   stir  us  up."  

really  care  (“I’m  dying  of  

"She  won't;  I  asked  her,  but  she  said  she  was  very  busy,  

boredom!”)  

and  told  Jessy  Tudor,  she  wasn't  fond  of  peacocks."   "I  don't  exactly  see  the  connection."   "Stupid  boy!  she  meant  you,  of  course."   "Oh,  I'm  peacocks,  am  I?"   "I  don't  wish  to  be  rude,  but  I  really  do  think  you  are  vain   of  your  good  looks,  elegant  accomplishments,  and  the   impression  you  make  wherever  you  go.  When  it's  worth   while  you  exert  yourself,  and  are  altogether  fascinating,   but  the  'I  come  -­‐-­‐  see  -­‐-­‐  and  -­‐-­‐  conquer'  air  you  put  on,   spoils  it  all  for  sensible  people."   "It  strikes  me  that  Miss  Morgan  has  slightly  infected  you  

Take  your  S   AT  prep  to  the  next  level.  Visit  www.learnerator.com   86  

87     with  her  oddity  as  far  as  bluntness  goes.  Fire  away,  it's   rather  amusing  to  be  abused  when  one  is  dying  of   ennui."   "That's  grateful  and  complimentary  to  me,  when  I  have  

-­‐  Kate  reveals  her  

devoted  myself  to  you  ever  since  you  came.  But  every  

annoyance  with  her  

thing  bores  you,  and  the  only  sign  of  interest  you've  

brother  

shown  is  in  those  absurd  red  hose.  I  should  like  to  

-­‐  Harry’s  relationship  to  

know  what  the  charm  is,"  said  Kate,  sharply.  

the  town  revealed  

"Impossible  to  say;  accept  the  fact  calmly  as  I  do,  and  be  

-­‐  Kate’s  relationship  to  the  

grateful  that  there  is  one  glimpse  of  color,  life,  and  

town  revealed,  too  -­‐-­‐  

spirit  in  this  aristocratic  tomb  of  a  town."  

clearly  she  feels  defensive  

"You  are  not  obliged  to  stay  in  it!"  fiercely.    

about  it  

"Begging  your  pardon,  my  dove,  but  I  am.  I  promised  to   -­‐  Circumstances  revealed   give  you  my  enlivening  society  for  a  month,  and  a  

More  relationships  –  dead  

Lennox  keeps  his  word,  even  at  the  cost  of  his  life."  

parents,  a  sibling  

"I'm  sorry  I  asked  such  a  sacrifice;  but  I  innocently  

relationship  neglected  for  

thought  that  after  being  away  for  five  long  years,  you  

five  years  

might  care  to  see  your  orphan  sister,"  and  the  dove   produced  her  handkerchief  with  a  plaintive  sniff.        Now  that  you  have  read  the  annotated  passage,  try  your  hand  at  answering  these   questions.     What  is  the  best  description  of  Kate’s  feelings  towards  her  brother?   A.  Kate  is  disaffected  and  disinterested  in  her  brother   B.  Kate  strives  to  entice  Lennox  to  become  more  social,  with  uneven  results   C.  Kate  is  malicious  towards  Lennox,  while  he  is  bored  with  her   D.  Kate  is  in  awe  of  Lennox’s  formidable  ego    

Take  your  S   AT  prep  to  the  next  level.  Visit  www.learnerator.com   87  

88     When  understanding  relationships  between  characters,  you  will  have  to  do  a  good   deal  of  extrapolating  from  what  is  directly  stated  in  the  text,  but  be  careful  that   every  one  of  your  inferences  is  based  on  actual  evidence  presented  in  the  text.  Look   at  answer  A;  you  should  be  able  to  see  quickly  that  while  Lennox  could  conceivably   be  called  “disaffected  and  disinterested,”  Kate  is  clearly  neither,  as  evidenced  by  her   constant  attempts  to  interest  Lennox  in  the  outside  world.     Option  B  is  certainly  supported  by  the  text;  the  whole  first  paragraph  is  devoted  to   Kate’s  attempts  to  spur  Lennox  towards  some  sort  of  social  activity,  though  he   seems  disinterested  and  criticizes  everyone  in  their  town  as  “dull.”  However,  she   does  manage  to  make  him  to  go  SOME  social  events  (“tea  fights  and  muffin   worries”),  and  to  get  him  talking  about  his  interest  in  Belle  Morgan,  so  clearly  her   efforts  are  not  entirely  futile  –  hence  why  “uneven”  is  a  good  adjective.     Option  C  could  also  conceivably  be  supported  by  the  text  –  the  selection  does  say   that  Kate  “said  maliciously”  and  that  Lennox  complains  of  “ennui,”  but  taken  as  a   whole,  the  selection  does  not  paint  Kate  maliciously.  Instead,  it  is  clear  that  she   cares  about  her  brother  and  simply  wants  to  try  and  enliven  him.  And  while  D  may   also  be  an  option,  as  Kate  does  speak  about  Lennox’s  apparently  impressive  ego,  she   does  not  appear  to  do  so  with  “awe,”  which  is  too  positive  an  adjective  for  her   position.  Therefore,  B  is  the  only  truly  defensible  option.   Which  quote  best  characterizes  the  siblings'  relationship?   A.  “I  promised  to  give  you  my  enlivening  society  for  a  month,  and  a  Lennox   keeps  his  word,  even  at  the  cost  of  his  life."   B.  “When  it's  worthwhile  you  exert  yourself,  and  are  altogether  fascinating,   but  the  'I  come  -­‐-­‐  see  -­‐-­‐  and  -­‐-­‐  conquer'  air  you  put  on,  spoils  it  all  for  sensible   people."   C.  “Be  grateful  that  there  is  one  glimpse  of  color,  life,  and  spirit  in  this   aristocratic  tomb  of  a  town."   D.  "I  thought  that  would  succeed!  She  isn't  there,  but  I've  got  you  up,  and  you   are  not  to  go  down  again,"  laughed  Kate.   Take  your  S   AT  prep  to  the  next  level.  Visit  www.learnerator.com   88  

89       The  use  of  these  quotes  will  force  you  to  consider  and  marshal  the  evidence  for  your   claims.  Let’s  examine  the  first  –  clearly,  Lennox  feels  himself  duty-­‐bound  to  his   sister,  so  while  this  quote  is  directly  relevant  to  the  relationship  between  the  two   siblings,  it  is  in  fact  more  revealing  of  Lennox’s  relationship  to  himself  than  to  his   sister.  He  refers  to  his  own  company  as  “My  enlivening  society”  (perhaps   facetiously),  and  makes  a  claim  about  himself  (“A  Lennox  keeps  his  word.”)  While   we  could  conceivably  use  this  quote  to  understand  Lennox’s  relationship  to  his   sister  (perhaps  one  of  duty),  it  does  not  seem  to  accurately  reflect  the  siblings’   relationship  as  a  whole,  though  if  there  are  no  better  options,  this  one  may  be   suitable.   Next,  try  B;  here,  Kate  is  making  an  observation  about  her  brother’s  personality.   This  quote  clearly  reveals  a  feeling  she  has  about  one  aspect  of  his  personality,   especially  as  he  relates  to  his  society,  but  it  seems  to  be  more  a  comment  about   Lennox’s  “public  face”  than  one  about  the  siblings’  private  relationship.   C  reveals  Lennox’s  feelings  about  his  town,  as  well  as  Lennox’s  feelings  about  Belle,   but  has  nothing  to  do  with  Kate  herself;  it  is  too  much  if  an  extrapolation  to  suggest   that  he  finds  Kate  as  boring  as  the  town  where  they  live.   Finally,  D  reveals  both  how  well  Kate  knows  her  brother,  as  well  as  the  obvious   sense  of  affection  she  feels  for  him.  She  teases  him,  rouses  him  from  his  stupor,  and   makes  him  come  to  the  window  using  a  tactic  only  she  would  know  (since  it  clearly   comes  from  her  observations  of  him),  and  gives  him  a  joking  directive  –  all  signs  that   point  to  an  affectionate  and  close  relationship  between  the  siblings.  Of  all  the   quotes,  this  one  best  encapsulates  the  relationship  between  Kate  and  Lennox;  it  is   not  the  most  obvious  choice,  but  it  subtly  paints  the  complex  relationship  between   cantankerous  Lennox  and  affectionate  Kate.     Relationships  Between  Ideas     The  SAT  will  provide  you  with  several  nonfiction  passages,  many  of  which  will   present  claims  and/or  arguments  that  you  will  have  to  assess.  Questions  may  focus   Take  your  S   AT  prep  to  the  next  level.  Visit  www.learnerator.com   89  

90     on  the  relationship  between  one  idea  and  the  next.  Do  the  ideas  contradict  each   other?  Build  on  one  another?  Influence  one  another?  Work  in  conjunction?   There  are  a  few  ways  to  understand  the  relationship  between  ideas.  The  first  thing   to  do  is  to  try  and  differentiate  between  the  ideas  that  are  being  presented.  This  may   sound  simple,  but  a  high-­‐complexity  text  can  present  confusing  parallels  between   ideas,  so  you  want  to  make  sure  that  understand  the  distinction  between  one  idea   and  the  next.     Chronological  Signal  Words     There  are  some  signal  words  that  should  help  you  make  these  differentiations.  Pay   attention  to  words  that  signal  chronology:   First,  before,  next,  later,  afterwards,  used  to,  recent(ly),  now   For  example,  the  following  sentence  uses  signal  words  to  clearly  differentiate   between  two  ideas:     Though  researchers  used  to  believe  that  there  could  be  no  life  on  Mars,   therecent  discovery  of  rocks  that  appear  to  be  water-­‐weathered  has  changed  this   thinking.     It  may  be  useful  to  label  the  ideas  outlined  here  with  numbers,  so  that  you  can  figure   out  what  scientists  used  to  believe,  what  they  now  believe,  and  what  the   relationship  is  between  the  two  ideas  (clearly,  the  latest  research  now  invalidates   the  earlier  theory.)  Many  relationships  are  often  established,  changed,  or  influenced   by  TIME,  so  any  words  that  signal  a  passage  of  time  are  certainly  worth  paying   attention  to.     Relational  Signal  Words    

Take  your  S   AT  prep  to  the  next  level.  Visit  www.learnerator.com   90  

91     Pay  attention  also  to  words  like:   While,  although,  but,  however,  despite  and   In  addition,  moreover,  also,  due  to,  because,  thanks  to     These  can  also  signal  a  shift  or  a  change  between  two  schools  of  thought,  which  can   be  very  useful  in  determining  the  different  relationships  between  them.  Relational   signal  words  can  signal  a  compare-­‐and-­‐contrast  and  a  cause-­‐and-­‐effect  relationship,   which  is  very  important  to  pay  attention  to.     Compare  and  contrast  the  relationships  outlined  in  the  following  two  sentences:     1.

While  there  was  once  widespread  support  for  the  existence  of  Bigfoot,   recent  research  has  definitively  disproved  this  theory.  The  signal  words   should  indicate  to  you  that  the  relationship  here  is  negative  à  one  theory   has  since  replaced  the  other.  

2.

It  is  thanks  to  Newton’s  understanding  of  gravity  that  Einstein’s  notion  of   relativity  exists.    

Clearly,  this  relationship  is  positive  as  one  theory  builds  on  or  was  inspired  by  the   other.  This  could  in  fact  be  broadly  categorized  as  a  cause  and  effect  relationship.   Now  that  you’ve  practiced  with  simple  samples,  try  your  hand  at  figuring  out  the   relationship  between  the  ideas  presented  in  the  following  blog  post  from  the  Public   Library  of  Science.  This  is  a  relatively  higher-­‐level  text,  so  pay  close  attention  to  the   ideas  that  are  being  discussed  here.   When  I  first  ran  across  Asifa  Majid’s  article  with   -­‐  Olfaction  =  smell   Ewelina  Wnuk  in  Cognition,  about  how  speakers   -­‐  Western  society  =  cares  more   of  Maniq,  a  language  indigenous  to  southern  

about  vision  than  other  senses  

Thailand,  have  a  vocabulary  for  talking  about  

-­‐  Other  cultures  do  not  privilege  

smell,  I  was  taken  aback.  In  anthropology,  

sight  over  all,  neglecting  smell  

especially  since  the  work  of  people  like  David  

Take  your  S   AT  prep  to  the  next  level.  Visit  www.learnerator.com   91  

92     Howes,  Constance  Classen,  and  Andrew  Synott,   we  know  very  well  that  different  cultures   privilege  olfaction  and  other  senses  more  than   Westerners  do.  The  anthropology  of  the  sense   has  made  it  clear  that  the   ideological  privileging  of  vision  in  the   West,  and  relative  underdevelopment  of  sense   of  smell…  is  not  matched  elsewhere.   However,  Wnuk  and  Majid  were  

-­‐  These  two  were  trying  hard  to  

attacking,  with  empirical  observations  and  

disprove  a  theory  that  already  

psychometric  testing,  one  of  the  pillars  of  

existed:  that  evolution  favored  

Western  philosophical  accounts  of  how  

other  senses  over  smell,  since  

human  senses  evolved:  the  idea  that  human  

smell  was  traditionally  

evolution  had  tipped  the  

considered  to  be  “weaker”  and  

balance  decisively  away  from  olfaction.  

less  “precise.”  (This  is  the  OLD  

The  alleged  weakness  and  imprecision  of  

theory).  

olfaction  was  taken  for  granted  in  perceptual   psychology.   Some  of  these  theories  of  sensory  evolution  

-­‐  Old  theory:  our  ancestors  

hold  that  our  ancestors  had,  in  a  way,  paid  for  

evolved  because  they  “gave  up  

our  distinctive  cognitive  and  perceptual  

on”  a  precise  sense  of  smell;  they  

development  by  sacrificing  olfactory  acuity.  

favored  vision,  which  grew  

Vision  increased  precision  at  the  expense  of  

stronger,  as  smell  grew  weaker  

olfaction.  In  fact,  some  theorists  of  brain  

-­‐  Acuity  =  strength  

evolution  go  so  far  as  to  suggest  that  there  

 

was  a  kind  of  neurological  trade-­‐off:  language  

-­‐  Also  part  of  the  old  theory:  it  

use  could  only  grow  as  our  ancestors  lost  a  

was  necessary  for  us  to  sacrifice  

capacity  for  smelling.  The  restraint  and  

this  highly  developed  sense  of  

remove  from  the  immediate  sense-­‐world  

smell  so  we  could  develop  

Take  your  S   AT  prep  to  the  next  level.  Visit  www.learnerator.com   92  

93     necessary  for  logic  and  abstract  thought  was  

language  

opposed  to  the  kind  of  complete  immersion  and     sensory  triggering  of  behavior  that  other  

-­‐  In  other  words,  other  animals  

animals  had  because  of  the  way  aromas  

were  too  distracted  by  smells  to  

dominated  their  perception.  Were  the  senses  in   think  clearly;  we  lost  the  ability  to   a  zero-­‐sum  exchange  where  visual  acuity  and  a  

smell  the  way  animals  do  but  we  

distinctly  human  way  of  life  made  acute  

can  now  think  more  clearly  

olfaction  impossible?  

because  we  aren’t  distracted  by   our  overly-­‐strong  senses  

Research  conducted  by  Asifa  Majid,  together   -­‐  Transition  -­‐-­‐  new  research   with  her  collaborators,  suggests  that  

shows  this  OLD  theory  is  untrue  –  

language  and  olfaction  are  not  at  odds;  the  

the  use  of  language  makes  our  

right  language  can  actually  enhance  the  

senses  STRONGER  

perception  of  aroma,  as  language  has  also  

 

enhanced,  inflected  and  refined  our  other  

-­‐  The  fact  that  our  sense  of  smell  

senses.  Rather  than  a  fact  of  human  being,  the  

is  weak  says  more  about  Western  

neglect  of  olfaction  in  the  West  is  a  result  of  our   culture  than  the  way  that  we   own  cultural  presuppositions  and  sensory  

evolved  physically  

biases:  smell  suffers  from  neglect,  not  an   inescapable  evolutionary  trade-­‐off.     There  are  just  enough  signal  words  here  to  help  you  differentiate  between  the  two   distinct  schools  of  thought.     The  writer  sets  out  a  sequence  of  events  that  STARTS  with  her  coming  across  a   certain  scientist’s  recent  work  (“When  I  first…”).  She  then  speaks  about  what  we   know  from  anthropological  research  (non-­‐Western  cultures  privilege  smell;  the   West  cares  more  about  sight),  segues  into  the  fact  that  Majid  and  Wnuk  are  trying  to   disprove  old  theories  (“However…”),  explains  the  old  theories  (“Some  of  these   theories…”  “In  fact…”),  and  explains  Majid  and  Wnuk’s  new  research.   Take  your  S   AT  prep  to  the  next  level.  Visit  www.learnerator.com   93  

94     The  structure  is  a  little  bit  confusing  because  it  jumps  between  present  (coming   across  this  new  research)  and  past  (explanation  of  older  theories),  but  if  you  read   carefully  and  pay  attention  to  signal  words  like  “First,”  “However,”  “Now,”   “Disprove,”  “Rather,”  you  should  be  able  to  ascertain  the  sequence  of  events  as  well   as  the  relationship  between  the  ideas  explained.     What  was  Majid  and  Wnuk’s  aim  in  conducting  this  new  research?   A.  To  support  the  theory  that  a  degraded  olfactory  sense  was  evolutionarily   beneficial   B.  To  disprove  the  theory  that  a  degraded  olfactory  sense  was  evolutionarily   beneficial   C.  To  demonstrate  that  language  can  enhance  olfaction   D.  To  prove  that  visual  acuity  can  only  increase  as  olfactory  acuity  degrades     Now,  these  distractors  all  use  high-­‐level  language  to  try  and  confuse  you,  but  once   you  differentiate  between  Majid  and  Wnuk’s  aim  and  its  relationship  to  older   theories,  you  will  be  able  to  answer  the  question  easily.  To  begin  with,  remember   that  Majid  and  Wnuk  set  out  to  “attack”  the  old  theory.  So  they  must  be  opposed  to   it,  or  trying  to  disprove  it.  Remember  also  that  the  old  theory  states  that  humans   lost  their  strong,  precise  sense  of  smell  in  order  to  better  develop  the  vision  and  the   language  that  was  more  evolutionarily  advantageous.  Majid  and  Wnuk  instead   found  that  this  WASN’T  true.     Knowing  that,  let’s  look  at  A.  Next  to  “degraded  olfactory  sense  was  evolutionarily   beneficial,”  you  can  write  “Old  Theory.”  So  A  means  that  they  wanted  to  support  the   old  theory,  which  simply  isn’t  true,  as  the  article  says  they  wished  to  “attack”  it.   Scratch  A.     B  is  the  exact  opposite  of  A  –  they  wanted  to  DISPROVE  this  old  theory,  which  is   indeed  true.  Keep  it  for  now.  

Take  your  S   AT  prep  to  the  next  level.  Visit  www.learnerator.com   94  

95     C  is  true,  according  to  their  research,  but  it  wasn’t  explicitly  stated  as  the  aim  for  the   scientists’  research;  instead,  it  was  a  FINDING  of  their  research.  You  can  scratch  C.   D  is  just  a  dressed-­‐up  version  of  the  old  theory  (that  it’s  a  fight  to  the  death  between   the  sense  of  sight  and  the  sense  of  smell,  and  that  one  can  only  grow  at  the  expense   of  the  other).  This  is  the  opposite  of  what  their  research  suggests  as  well  as  the  kind   of  claim  that  they  sought  out  specifically  to  “attack,”  so  you  can  get  rid  of  D.   This  was  a  difficult  question  because  it  asks  you  to  do  a  lot  of  work  simultaneously   (figure  out  the  text  structure,  figure  out  the  relationship  between  two  ideas,  and   figure  out  the  complex  language  in  the  distractors).      

Take  your  S   AT  prep  to  the  next  level.  Visit  www.learnerator.com   95  

96     Relationships  between  Events     Like  individuals  and  ideas,  events  also  have  the  ability  to  influence  each  other.  As   you  no  doubt  learned  in  your  social  science  classes,  events  have  the  ability  to  give   rise  to  other  events;  for  example,  had  the  1963  March  on  Washington  never   happened,  it  is  doubtful  that  the  2008  election  would  have  had  the  same  results.  But   historical  speculation  aside,  the  SAT  will  no  doubt  ask  you  to  identify  and  perhaps   analyze  the  relationships  between  different  events.  Here  are  four  of  the  most   common  kinds:     Cause  and  Effect   -­‐Implies  one  event  caused  the  other   -­‐Signal  words:  due  to,  because,  since,  thus,  therefore,  hence,  since,  thanks  to,  as  a   result,  consequently   -­‐Example:  Due  to  the  rain  on  Monday  (cause),  I  was  unable  to  walk  to  school  (effect).     Chronology   -­‐Indicates  a  sequence  of  events   -­‐Signal  words:  first,  second,  third,  since,  then,  now,  recently,  yesterday,  last  Monday,   for  years   -­‐Example:  For  decades,  scientists  have  believed  the  existence  of  aliens  is  merely  an   overused  sci-­‐fi  trope;  yesterday’s  surprise  visit  by  friendly  extraterrestrials  has  now   proved,  beyond  a  shadow  of  a  doubt,  the  existence  of  intelligent  life  on  other   planets.     Compare  and  Contrast   -­‐Describes  two  (or  more)  similar  or  disparate  events   -­‐Signal  words:  Also,  additionally,  moreover,  in  addition,  furthermore  (compare);   despite,  while,  however,  yet,  but,  whereas  (contrast)   -­‐Example:  The  two  wars  were  remarkably  similar  in  their  carnage  and  vastly  

Take  your  S   AT  prep  to  the  next  level.  Visit  www.learnerator.com   96  

97     inflated  budget,  but  as  one  was  effective  and  the  other  was  not,  they  will  be   remembered  very  differently.     Though  there  are  other  categories,  such  as  Problem  and  Solution,  these  are  the  main   three  that  you  need  to  worry  about;  the  other  categories  could  neatly  fit  in  to   these.  For  more  practice,  read  the  following  short  passages  and  answer  the   questions  about  them:      “For  years,  researchers  have  disagreed  about  whether  Pluto  is  in  fact  a  planet.   Now,  however,  the  diminutive  ball  of  ice  and  methane  might  be  staging  its   comeback.  The  once-­‐planet  was  reclassified  a  “dwarf  planet”  or  “plutoid”  (a   dwarf  planet  further  out  than  Neptune)  in  2003,  leading  to  hundreds  of   nostalgic  protests,  Facebook  groups,  and  even  t-­‐shirts  all  mourning  Pluto’s   demotion.  One  popular  slogan  read  ‘Pluto  will  always  be  a  planet  in  our   hearts.’”     Based  on  the  passage,  which  choice  best  describes  the  relationship  between   Pluto’s  reclassification  and  the  reaction  of  the  public?   A.  The  public  reacted  with  glee  to  Pluto’s  reclassification.   B.  The  public  reacted  with  anger  to  Pluto’s  reclassification.   C.  The  public  reacted  with  confusion  to  Pluto’s  reclassification.   D.  The  public  reacted  with  sadness  to  Pluto’s  reclassification.     This  is  a  fairly  straightforward  cause-­‐and-­‐effect  question:  what  was  the  effect  of   Pluto’s  reclassification  on  the  public?  As  explicitly  stated  in  the  text,  the   reclassification  led  to  “hundreds  of  nostalgic  protests”  and  “mourning;”  these  two   words  should  clearly  establish  that  D  is  the  only  correct  choice.  You  may  not  have   recognized  this  as  a  cause-­‐and-­‐effect  relationship,  but  look  at  the  phrase  “leading   to.”  That  should  clearly  indicate  exactly  what  kind  of  relationship  this  is.    

Take  your  S   AT  prep  to  the  next  level.  Visit  www.learnerator.com   97  

98     “Having  the  essentially  complete  sequence  of  the  human  genome  is  similar  to   having  all  the  pages  of  a  manual  needed  to  make  the  human  body.  The   challenge  to  researchers  and  scientists  now  is  to  determine  how  to  read  the   contents  of  all  these  pages  and  then  understand  how  the  parts  work  together   and  to  discover  the  genetic  basis  for  health  and  the  pathology  of  human   disease.  In  this  respect,  genome-­‐based  research  will  eventually  enable  medical   science  to  develop  highly  effective  diagnostic  tools,  to  better  understand  the   health  needs  of  people  based  on  their  individual  genetic  make-­‐ups,  and  to   design  new  and  highly  effective  treatments  for  disease.     Individualized  analysis  based  on  each  person's  genome  will  lead  to  a  very   powerful  form  of  preventive  medicine.  We'll  be  able  to  learn  about  risks  of   future  illness  based  on  DNA  analysis.  Physicians,  nurses,  genetic  counselors  and   other  health-­‐care  professionals  will  be  able  to  work  with  individuals  to  focus   efforts  on  the  things  that  are  most  likely  to  maintain  health  for  a  particular   individual.  That  might  mean  diet  or  lifestyle  changes,  or  it  might  mean  medical   surveillance.  But  there  will  be  a  personalized  aspect  to  what  we  do  to  keep   ourselves  healthy.  Then,  through  our  understanding  at  the  molecular  level  of   how  things  like  diabetes  or  heart  disease  or  schizophrenia  come  about,  we   should  see  a  whole  new  generation  of  interventions,  many  of  which  will  be   drugs  that  are  much  more  effective  and  precise  than  those  available  today.”       How  will  the  sequencing  of  the  human  genome  affect  scientific  research  in  the   future?   A.  It  will  teach  scientists  how  to  make  the  human  body.   B.  It  will  lead  scientists  to  sequence  other  organisms’  genomes.   C.  It  will  increase  awareness  of  public  health  issues.   D.  It  will  contribute  to  great  gains  in  preventative  medicine.     This  is  a  simple  cause-­‐and-­‐effect  question  that  asks  about  the  effect  that  sequencing   the  genome  will  have  on  scientific  research.  Look  at  all  the  answer  options  to   Take  your  S   AT  prep  to  the  next  level.  Visit  www.learnerator.com   98  

99     determine  which  is  the  best  one.  Though  A  is  clearly  drawn  from  the  text,  it  is  an   obvious  trick;  the  passage  COMPARES  the  sequencing  of  the  human  genome  to  the   creation  of  a  manual  on  how  to  make  a  human  body.  It  does  not  state  or  imply  that   building  a  human  body  will  be  an  immediate  effect  or  priority  after  the  sequencing   occurs.  Option  B  has  no  basis  in  the  text,  as  there  is  no  statement  about  scientists’   plans  for  sequencing  the  genomes  of  other  organisms.  Though  you  may  conceivably   be  able  to  extrapolate  C  from  the  text,  there’s  no  real  evidence  for  this  option.  Option   D  is  explicitly  stated  in  the  text,  and  is  thus  the  best  answer.  

Take  your  S   AT  prep  to  the  next  level.  Visit  www.learnerator.com   99  

100     From  The  Atlantic:     “The  history  of  clip  art  is  in  many  ways  the  history  of  the  World  Wide  Web   itself.  Clip  art  was  made  possible  because  of  the  invention  of  desktop  publishing   in  the  early  1980s.  The  first  library  of  professionally  drawn  clip  art  was   provided  by  VCN  ExecuVision  and  introduced  in  the  IBM  PC  in  1983.  It  offered   images  to  be  used  in  presentations  and  newsletters.  Early  electronic  clip  art   was  simple  line  art  or  bitmap  images.  The  introduction  of  the  Apple  Macintosh   program  MacPaint,  in  particular,  meant  that  consumers  could  use  and  edit  bit-­‐ mapped  clip  art  for  the  first  time.  One  of  the  first  successful  clip-­‐art  producers   was  T/Maker  Company,  which  had  worked  with  Apple  to  develop  an   alternative  word  processor,  WriteNow.  In  1984,  T/Maker  began  publishing   small,  retail  collections  of  those  bitmap  images  under  the  brand  name   "ClickArt."  The  first  version  of  "ClickArt"  offered  images  designed  for  both   professional  and  personal  uses;  the  professional  were  published  in  1984  as   "ClickArt  Publications."  The  widespread  adoption  of  the  CD-­‐ROM  in  the  early   1990s  was  also  a  boon  to  clip  art:  Many  more  companies  began  offering   electronic  clip  art  on  CDs.  As  this  happened,  clip-­‐art  makers  began  emphasizing   quantity  over  quality.  Even  T/Maker,  which  built  its  business  on  sales  of  small,   high-­‐quality  clip-­‐art  packages,  entered  the  volume  clip-­‐art  market.  In  1995,  it   became  the  exclusive  publisher  of  more  than  500,000  copyright-­‐free  images— to  create  what  would  be,  at  the  time,  one  of  the  largest  clip-­‐art  libraries  in  the   world.”     Based  on  the  information  in  the  article,  what  could  be  considered  the  most   important  turning  point  for  clip  art?   A.  The  invention  of  desktop  publishing  in  the  1980s   B.  The  release  of  professional  images  for  public  consumption  in  1984   C.  The  1995  publishing  of  one  of  the  largest  clip-­‐art  libraries  in  the  world   D.  The  adoption  of  the  CD-­‐ROM  in  the  1990s  and  the  subsequent  emphasis  on   quantity,  not  quality   Take  your  S   AT  prep  to  the  next  level.  Visit  www.learnerator.com   100  

101       This  article  is  clearly  structured  sequentially,  with  multiple  dates  and  events   recorded.  Though  the  article  never  explicitly  calls  one  development  more  important   than  the  other,  it  is  possible  to  extrapolate  an  answer  to  this  question.  Look  at  the   words  used  –  “most  important  TURNING  POINT.”  This  should  immediately   disqualify  option  A  –  though  the  existence  of  clipart  is  important,  it  isn’t  a  “turning   point”  because  the  beginning  by  definition  cannot  be  a  turning  point.  Strike  A.  While   B  could  be  a  good  choice,  its  results  are  never  discussed  in  the  article,  so  it  can’t  be  a   turning  point  as  we  are  never  told  what  it  CHANGED.  C  is  highly  tempting,  but  watch   out  –  it  is  the  RESULT  of  a  turning  point,  rather  than  the  turning  point  itself.  We  are   never  told  what  effects  of  the  existence  of  the  largest  clip-­‐art  library  in  the  world,   but  we  ARE  told  that  the  adoption  of  the  CD-­‐ROM  led  to  an  emphasis  on  quantity,   not  quality,  which  directly  leads  to  the  creation  of  one  of  the  largest  clip-­‐art  libraries   in  the  world.  In  order  to  answer  this  question,  you  must  understand  what  it  is   asking  you—to  identify  a  point  in  the  history  of  clip  art  that  led  to  a  change,  or  at   least  to  important  results;  only  Option  D  fits  this  bill.   Understanding  relationships  between  individuals,  ideas,  or  events  can  be  tricky,  but   as  long  as  you  understand  what  the  question  is  asking  you  and  which  signal  words   will  lead  you  to  the  answer,  you  will  achieve  success  on  this  section  of  the  SAT.  

 

Take  your  S   AT  prep  to  the  next  level.  Visit  www.learnerator.com   101  

102    

Interpreting Words and Phrases in Context One  of  the  most  important  features  on  the  new  SAT  is  emphasis  on  vocabulary  in   context.  Gone  are  the  days  of  recondite*  “SAT  Words;”  now  you  will  have  to  figure   out  how  high-­‐level  yet  everyday  words  fit  into  the  context  of  a  piece.  This  isn’t  to  say   that  the  words  will  all  be  familiar  to  you  -­‐-­‐  don’t  confuse  “frequently  used”  with   “easy”  -­‐-­‐  but  all  the  words  on  the  new  SAT  will  be  relevant  to  the  kinds  of  higher-­‐ level  texts  you’ll  be  expected  to  read  beyond  high  school,  so  practicing  this  skill  now   will  help  you  later  on.  

Here are some strategies for determining the meaning of words in context. 1. Pay attention to the context.

Sounds obvious, doesn’t it? But in order to figure out the meaning of a word, you’ll need to pay attention to how it is used. After all, the word “product” means one thing in a math context, and quite another in a discussion of tribal artifacts. So you don’t only need to pay attention to the sentence directly preceding the word in question; pay attention to the setting of the word. Read the introductory blurb that indicates where the reading comes from. A narrative piece will use certain words differently than an economics text or a science article. Read the following excerpt from a social sciences article about class distinctions, and see whether you can assess what the words mean. But it’s not only linguistic differences that make this transition hard. The way a person speaks is just a small part of the social currency that determines who is privileged and who is not. When Lisa Faison, 15, started at her elite private

Take  your  S   AT  prep  to  the  next  level.  Visit  www.learnerator.com   102  

103     school, she quickly learned that it wasn’t “normal” in her school community to have no Internet at home. “We can’t afford it,” she shrugs -- an unfamiliar concept in this world of luxury cars, high-end vacations, and school tuition that costs upwards of $20,000 a year. Based on the context, the word currency most probably means… A. Money B. Exchange C. Community D. Expression

You’ve probably heard the word “currency” before in regard to money. But look at the word before it, “social.” This modifier clearly indicates that this specific use of the word “currency” has something to do with being a part of society. And the sentence before it, about linguistic habits, also isn’t directly relevant to the issue of money. Money is clearly there as an option to try and confuse you, and to assess whether you can differentiate between different uses of the same word. (Plus, “social money” is a very awkward phrase -- when in doubt, try substituting the answer option for the word. In some cases, it will be glaringly obvious that the answer is wrong). According to the article, currency, whatever it is, “determines” whether or not a person is privileged. So currency is something that has a certain power, and it’s something that is decided by an external source (“social” -- clearly, society determines it). Look at the other choices. We have “exchange,” which fits in with the money idea -- after all, money is exchanged for goods and services. Maybe this type of currency is also just an exchange -- when you have certain things, or grow up in a certain community, you are seen in a specific way that others aren’t. This can be thought of as a type of “exchange” -- the things that you HAVE or DO

Take  your  S   AT  prep  to  the  next  level.  Visit  www.learnerator.com   103  

104     determine the way you are THOUGHT OF or SEEN. Plus “social exchange” sounds like it could be an appropriate choice -- the article is about how a girl from a lower-income background fits in with peers of a higher-income background, which is certainly a type of “social exchange.” Before we decide, let’s try C. “Social community” is redundant -- in fact, it’s so redundant that it can’t possibly be right. It could have been a good choice, but in this context it doesn’t make any sense. Lastly, D, expression, is a worthwhile contender. In fact, were it not for “exchange,” D would certainly be the answer. After all, social expression -- the way that people express themselves in society -- goes a very long way in determining who is privileged and who isn’t. But “currency” seems to be about more than just “expression” -- expression suggests that people control the way they are seen, but the article suggests that this isn’t true. Lisa couldn’t control the way her peers saw her; because of one thing beyond her control, they automatically saw her in a completely different way. This suggests that “currency” and “expression” cannot be synonyms. The best answer is B, exchange. Not only does it fit best as a synonym, it is also related to the other meanings of the word “currency.” This will not always be the case with words in context -- they will not always be related in meaning to the same words used in other contexts -- but in this case, it’s a handy bonus that helps to determine the best answer. This was a difficult question because it illustrated a high-level concept; the SAT will ask many questions like this, which ask you to figure out an ordinary word in a complex setting.

Take  your  S   AT  prep  to  the  next  level.  Visit  www.learnerator.com   104  

105     2. Replace, replace, replace.

As mentioned in the previous tip, replacing the word in question with an answer choice can be a very useful way to determine if the word makes sense. For example, try this excerpt from a science article. The experiment had three steps. First, the researchers raised the visibility of the fruit, moving it from its shadowy spot by the juice dispenser to a prominent position near the cash register. Next, they lowered the price of the fruit from one dollar to fifty cents. Last, they plastered images of happy-looking students eating fruit all over the cafeteria walls, hoping to entice more students to share in this activity. “We did see an increase in fruit intake,” said one researcher, “but because of our bundled experiment, we’re not sure why this campaign succeeded.”

Based on the context, bundled most closely means… A. Rushed B. Bungled C. Together D. Multi-step

First determine what this piece is about. Scientists are doing an experiment involving several elements. Clearly, the experiment can’t be “rushed” if it contains multiple processes, and there’s no mention in the article of a time constraint on the research. Rushed is therefore not a good choice. “Bungled” sounds like bundled. The SAT is relying on this fact -- and on the hope that you may not be aware that “bungled” means “messed up” -- to confuse you. But choosing a word solely on basis of its perceived similarity to another word (rather than a shared root) is a terrible strategy, and one that the SAT capitalizes on to trick you. In any event, it is highly unlikely that the scientist admit to messing up the experiment that way, so “bungled” is most likely out.

Take  your  S   AT  prep  to  the  next  level.  Visit  www.learnerator.com   105  

106     “Together” may make sense at first, since there were several elements to the experiment, but read the sentence again: “because of our together experiment” doesn’t make any sense. Pay attention to parts of speech -- you can do an experiment together, but you can’t do a together experiment. At least not one that makes sense. Scratch C. Last, “multi-step” may not SOUND like it’s a synonym for “bundled,” but remember the first sentence, the one that said the experiment had three steps? There’s a dead giveaway for the fact that “multi-step” is the best answer in this context. “Bundled” can mean “packaged together,” which is why “together” could have been an attractive option -- but replacing the word with the answer option will quickly make it clear that sometimes, the most attractive-seeming options are not very feasible at all. 3. Visualize.

Research has demonstrated that visuals help you learn vocabulary. There will be diagrams and charts on the new SAT, but no pictures -- which is why you can be your own illustrator. You don’t need to sketch out a perfect image to understand the meaning of a word -- this is a strategy that will work just fine if you keep it confined to your own head -- but taking the time to visualize what the article is talking about will be very helpful as you answer questions designed to test your comprehension. For example, try visualizing this excerpt, from a social-science article. “But being a consumer and not a producer has its drawbacks. Those with frustrated artistic inclinations often cite a general fatigue with consumption, complaining of an exhausting glut that can only be counterbalanced by the hard work of producing.”

Take  your  S   AT  prep  to  the  next  level.  Visit  www.learnerator.com   106  

107     This is a hard image to visualize in your head, as there isn’t much imagery or a sense of scene or setting. But there are some words that should help -- fatigue, complaining, exhausting, hard work. Imagine a “consumer” -- someone who “consumes,” or “eats,” or “takes in” something. (I am imagining a little Pac-Manlike creature.) Imagine this “over-consumer” getting tired -- consuming too much and slowing down. This exhaustion can only be “counterbalanced” or reversed, by the consumer making something of his or her own (“producing”). Now I am imagining that little Pac-Man creature staying in one place, creating something, losing all that excess weight of “consumption” and getting energized again, thanks to the “hard work of producing.” The visual came to my head -- I didn’t draw it or otherwise mark it down -- but it helped me to better understand the piece by anchoring the abstract (all this talk of producers and consumers and artistic fatigue) to an actual representation of the concepts. Try visualization as you answer this final question about words in context. The following excerpt is from a literary piece. She tensed, expecting the lights to blare and the Matron’s voice to sound over the loudspeakers wired into every room. But all was quiet and still. The night was a neverending black and not a single bird sang or swooped in the trees. She took a deep, quavery breath and stepped out into the uncharted night.

Based on the reading, what is the best meaning for the word “uncharted”? A. Unknown B. Wild C. Frightening D. Shadowy

Take  your  S   AT  prep  to  the  next  level.  Visit  www.learnerator.com   107  

108     From the context, you can probably figure out that there is something a little bit ominous happening here -- our protagonist is “tensing” in fear of a “Matron” who can project her voice into every room (so the protagonist lives in some sort of dystopian communal housing). The “neverending black” of the night and the silence of the birds also contribute to this ominous impression. Based on this reading, you may be tempted to choose “C,” frightening -- but despite this ominous impression, the text does not say that the night is frightening. After all, our protagonist is walking into it willingly. Nor do we have any indication that the night is “wild” (there is no mention of tempestuous weather or wild animals), and “shadowy” is an overly redundant synonym for “neverending black” (plus, shadows need at least some light to exist, and there clearly isn’t any of that here.) The best definition for “uncharted” is “unknown” -- “uncharted territory” is territory that hasn’t been put on a map yet, territory that isn’t familiar. Of all the choices, A and C are the strongest; in this case, the best you can do is use context and common sense to eliminate the answer choices that are clearly wrong and use your vocabulary knowledge (or your best guess) to determine the correct answer. Figuring out words and phrases in context can be tricky, and sometimes there isn't a replacement for simply knowing what a word means. But if you use these tips -- paying attention to every part of the context, replacing the word in question with the answer options to see if it makes sense, and visualizing the context as best you can -- this will be very helpful in figuring out the meaning of vocabulary in context.

  Take  your  S   AT  prep  to  the  next  level.  Visit  www.learnerator.com   108  

109    

 

            Part  II:  Rhetoric  (Reading)  

 

 

Take  your  S   AT  prep  to  the  next  level.  Visit  www.learnerator.com   109  

110    

Analyzing Word Choice “Rhetoric”  is  defined  as  the  art  of  discourse,”  or  (more  informally)  “the  art  of   persuasion.”  The  new  SAT  is  preparing  you  to  read,  analyze,  and  internalize   information  that  you  might  see  in  college  and  the  workplace  -­‐-­‐  mostly  persuasive   writing  that  attempts  to  get  an  important  point  across  succinctly  and  informatively.   In  this  section  of  the  review,  you’ll  learn  some  strategies  for:   1.  Comprehending  the  structure  of  a  persuasive  piece   2.  Analyzing  the  choices  that  a  writer  makes  in  order  for  a  piece  to  become   persuasive   3.  Understanding  why  a  piece  was  written   4.  Understanding  how  each  element  in  the  piece  works  within  the  larger  context     This  particular  article  will  focus  on  that  last  issue;  when  analyzing  rhetoric,  you  can   break  down  the  argument  into  its  smallest  pieces,  which  are  the  words  used  to   effectively  convey  the  idea  in  question.  You  may  be  familiar  with  Common  Core   standards  -­‐-­‐  or  with  English  teachers  -­‐-­‐  that  ask  you  to  determine  how  an  author’s   writing  conveys  or  creates  meaning  and  tone.  This  is  the  exact  same  skill,  only   applied  to  a  broader  swath  of  prose  than  just  literature.   This  skill  could  also  be  called  “paying  close  attention.”  As  you  read,  take  note  of  the   words  that  create  some  sort  of  feeling  or  image.  Does  the  author  use  bland,  boring   words,  or  does  s/he  use  intense,  vivid  language  to  evoke  a  particular  mood  or   scene?  Take  these  two  sentences  as  examples.  Both  say  the  same  thing,  yet  one   evokes  a  mood  much  more  vividly  than  the  other:     I  was  tired  and  hungry  when  I  came  home  the  other  day.   Famished  and  yawning,  I  trudged  up  the  steps  to  my  empty  apartment.     The  latter  sentence  clearly  uses  far  more  vivid  language.  It’s  not  just  the   replacement  of  “famished”  for  “hungry”  or  “yawning”  for  “tired”  (though  “yawning,”   Take  your  S   AT  prep  to  the  next  level.  Visit  www.learnerator.com   110  

111     a  verb,  conveys  a  sense  of  action  that  “tired”  does  not)  that  creates  such  a  strong   sense  of  emotion.  It’s  also  the  inclusion  of  the  word  “empty”  before  “apartment”  -­‐-­‐   the  first  sentence  tells  us  “I  came  home,”  a  rather  neutral  statement,  while  this   sentence  describes  the  home.  It  is  empty  and  isolated.  It  even  gives  us  a  glimpse  at   the  kind  of  home  it  is,  evoking  images  of  stairwells,  the  city,  and  isolation.     Let’s  try  this  one  more  time:   Stephanie  Carmichael  was  a  bully.  There  were  no  two  ways  about  it.   Stephanie  Carmichael  liked  to  hit  people.  Unfortunately,  her  victims  did  not   share  her  enthusiasm.     Now,  both  of  these  sentences  are  equally  vivid,  but  the  tones  are  completely   different.  Take  the  first  sentence  -­‐-­‐  it  is  conversational,  utilizing  informal,  everyday   language  (“no  two  ways  about  it.”)  The  second  sentence  is  more  formal,  almost   sardonic;  it  states  the  obvious  (there  aren’t  many  people  who  enjoy  being  hit),  but   slightly  dramatizes  the  situation  by  calling  Stephanie’s  targets  “victims”  and   referring  to  her  penchant  for  hitting  as  an  “enthusiasm.”   Even  though  the  two  sentences  say  the  same  thing  in  equally  vivid  language,  the   tones  are  vastly  different  because  of  the  words  being  used.  Pay  close  attention  to   distinctions  like  this  and  you  will  do  very  well  on  this  section  of  the  SAT.  It  can  be   very  helpful  to  visualize  everything  that  the  author  describes,  as  this  will  help  to   establish  a  sense  of  tone,  mood,  and  place.  When  I  saw  the  words  “no  two  ways   about  it,”  I  pictured  a  down-­‐home,  folksy  type;  when  I  read,  “her  victims  did  not   share  her  enthusiasm,”  I  pictured  Benedict  Cumberbatch  as  the  narrator  -­‐-­‐  refined,   sardonic,  and  very  British.  Clearly,  the  words  used  can  go  a  long  way  in  evoking  a   very  particular  image.     Extra  Practice     As  you  read,  think  about  how  the  writer  uses  words  to  convey  information.  Take  a   look  at  this  following  example,  paying  attention  to  how  s/he  structures  and  words   Take  your  S   AT  prep  to  the  next  level.  Visit  www.learnerator.com   111  

112     the  argument.  If  possible,  try  to  visualize  the  person  who  wrote  this  piece.  Use  their   words  to  get  a  sense  for  how  they  feel  and  what  they  think  about  the  topic  in   question:     The  recent  exclusion  of  the  Harry  Potter  series  from  your  newspaper’s  Best   Books  Ever  Written  list  is  an  unqualified  travesty.  I  am  outraged  and  confused   that  your  magazine  would  choose  to  exclude  such  a  seminal  work  of  literature   from  the  Western  canon.  The  Harry  Potter  books  rank  among  the  best  ever   written,  inspiring  countless  generations  of  children  and  inculcating  them  with   a  sense  of  wonder  and  delight.  Rest  assured  that  this  snub  will  not  be  ignored;   due  to  your  lack  of  research  and  clearly  unsound  opinions,  I  am  canceling  my   subscription  effective  immediately.     What  is  the  general  tone  of  the  article?   A.  Incensed   B.  Delighted   C.  Unqualified   D.  Distressed   Though  this  question  does  not  directly  ask  you  to  pinpoint  one  word  or  phrase,  it   asks  you  to  consider  the  words  used  in  this  article  as  a  whole.  It  should  be  fairly   easy  to  determine  that  this  writer  is  clearly  not  “delighted”  (B).  Nor  is  s/he   “unqualified”  -­‐-­‐  though  this  word  is  used  in  the  piece,  it  is  an  adjective  (meaning   “total”  or  “complete”)  for  the  “travesty”  of  the  exclusion  of  Harry  Potter  from  a  Best   Of  list.  It  is  clear  that  the  writer  is  D,  distressed  -­‐-­‐  words  like  “outraged”  and   “confused”  make  that  clear  -­‐-­‐  but  a  better  answer  is  A,  “Incensed,”  or  angry.  The   writer  is  clearly  more  than  just  distressed  -­‐-­‐  s/he  is  angry,  threatening  to  cancel   his/her  subscription  to  the  magazine  and  threatening  “this  snub  will  not  be   ignored.”  This  sounds  like  a  lot  more  than  just  “distress.”  As  you  read,  make  sure   that  you’re  paying  attention  to  the  piece  as  a  whole  and  considering  the  very  best   answer.     Take  your  S   AT  prep  to  the  next  level.  Visit  www.learnerator.com   112  

113     Which  quote  from  the  article  best  conveys  the  author’s  tone?   A.  “The  Harry  Potter  books  rank  among  the  best  ever  written”   B.  “Due  to  your  lack  of  research  and  clearly  unsound  opinions”   C.  “I  am  canceling  my  subscription”   D.  “The  exclusion…  is  an  unqualified  travesty”     This  question  is  asking  you  HOW  the  writer  has  conveyed  his/her  tone  of  acute   anger.  Though  each  of  the  quotes  is  lifted  directly  from  the  text,  only  one  best   conveys  a  tone  of  rage.  Clearly,  A,  while  an  important  and  closely  held  opinion,  does   not  convey  the  author’s  rage,  but  the  author’s  opinion  about  the  Harry  Potter  series.   B  gets  closer  to  the  author’s  rage,  as  it  insults  the  writers  in  the  magazines,  but  it   does  not  contain  any  of  the  very  strong  language  that  the  writer  used  in  other  parts   of  the  selection.  C  contains  a  threat,  but  no  strong  language.  Clearly,  D  is  the  best   answer  as  it  most  effectively  conveys  the  author’s  sense  of  rage.       Try  practicing  with  a  slightly  less  colorful  selection  from  a  history  text:   The  senator  had  little  to  say  about  the  barbarism  of  the  crime,  bizarrely  choosing   instead  to  direct  his  vitriol  towards  the  victim  and  her  family  rather  than  the   perpetrators,  who  were  released  on  bail  just  two  days  after  the  attack.  “I  appeal  to  the   American  public  tonight,”  he  stated  in  a  press  conference  last  Monday,  “to  ask  why  this   girl  was  allowed  to  run  amok  after  dark  by  herself.”     What  is  the  author’s  opinion  of  the  senator’s  remarks?   A.  The  author  supports  the  senator’s  appeal  to  the  nation.   B.  The  author  believes  the  senator’s  remarks  are  a  barbaric  crime.   C.  The  author  is  bewildered  and  angered  by  the  senator’s  remarks.   D.  The  author  has  no  opinion  on  the  senator’s  remarks.     In  order  to  correctly  answer  this  question,  you  must  separate  out  the  words  that  the   author  uses  to  refer  to  the  senator,  and  the  words  that  the  author  uses  to  refer  to  the   Take  your  S   AT  prep  to  the  next  level.  Visit  www.learnerator.com   113  

114     crime.  You  must  understand  also  what  the  senator’s  remarks  WERE  -­‐-­‐  refer  to  the   quotes,  which  enclose  the  Senator’s  words.  (Understanding  how  this  text  is   structured  will  come  in  useful  here.)     Be  wary  of  answer  choices  that  quote  directly  from  the  text  -­‐-­‐  they  may  be  trying  to   confuse  you  by  drawing  your  attention  to  an  irrelevant  part  of  the  selection.  For   example,  Answer  A  contains  the  words  “appeal  to  the  nation,”  which  are  lifted   directly  from  the  text.  Yet  there  is  absolutely  no  indication  that  the  author  supports   this  appeal;  in  fact,  it  is  the  senator  who  used  the  word  ‘appeal,’  not  the  author.   Answer  B  also  directly  refers  to  a  quote  from  the  text,  but  again,  the  “barbaric   crime”  has  nothing  to  do  with  the  senator’s  words;  the  senator’s  speech  was  a   response  to  the  crime.  Option  C  is  directly  supported  by  the  text,  as  s/he  says  that   the  senator  “bizarrely”  chose  to  direct  his  “vitriol”  towards  the  victim.  This  use  of   the  word  "bizarre”  in  reference  to  the  Senator’s  choices  showcases  the  author’s   confusion  about  the  politician’s  actions;  the  use  of  the  word  “vitriol”  (cruel  or  bitter   criticism)  also  illustrates  the  author’s  position.  After  all,  the  Senator  is   not  actually  being  vitriolic;  his  language  is  not  harsh  enough  for  that.  Yet  clearly  the   author  feels  that  the  Senator’s  remarks  are  so  unfair  that  they  border  on  abuse  -­‐-­‐  a   position  that  is  clearly  if  subtly  demonstrated  throughout  the  piece.  You  may  be   tempted  to  say  that  D,  the  author  has  no  opinion,  but  the  word  “bizarre,”  again,   demonstrates  that  this  isn’t  true.     Let’s  try  once  more;  pay  careful  attention  to  how  the  author  creates  tone  and  mood   here.  Which  words  are  the  most  useful  in  the  creation  of  this  scene?  Which  words   best  help  you  to  visualize  what  the  author  is  trying  to  convey?   As  the  long  slog  towards  establishing  fair  trade  practices  in  Equatorial  Kundu   continues,  there  is  a  glimmer  of  hope  on  the  horizon.  The  West  African  country  has   long  been  the  focus  of  controversy  for  its  corrupt  practices  regarding  the  mining  of   diamonds  and  the  harvesting  of  coffee,  but  with  the  recent  installation  of  reforming   President  Mbala  Mbame,  new  legislation  could  change  the  way  miners  and  harvesters   are  treated  by  internal  and  international  trading  entities.  “We  will  no  longer  stand  by   Take  your  S   AT  prep  to  the  next  level.  Visit  www.learnerator.com   114  

115     and  watch  the  common  people  suffer,”  President  Mbame  announced  in  a  historic   televised  speech  yesterday.  “Equatorial  Kundu  is  facing  the  dawn  of  a  new  day,  and   anyone  who  stands  in  our  way  will  be  dealt  with  summarily.”  There  is  no  word  yet   from  the  BPA,  the  rebel  group  that  led  riots  in  the  streets  in  response  to  Mbame’s   election,  on  how  these  new  agreements  will  impact  their  flourishing  black  market   economy,  but  Kundu’s  army  is  on  standby  and  the  country  is  both  tense  and  hopeful.     How  does  the  author  paint  a  vivid  image  for  the  reader?   A.  S/he  chose  a  strongly  worded  quote  from  the  president  of  the  country,   which  establishes  a  display  of  strength.   B.  S/he  draws  attention  to  the  plight  of  the  miners  and  harvesters,  helping   the  audience  to  understand  the  dire  economic  conditions  of  Kundu.   C.  S/he  speculates  about  the  reprisals  of  the  BPA,  establishing  a  fearful  tone   for  the  piece.   D.  S/he  uses  multiple  descriptive  words,  such  as  “slog,”  “glimmer,”  “tense,”   and  “hopeful”  to  establish  Kundu’s  general  mood.     Many  of  these  choices  seem  as  though  they  may  be  correct.  For  example,  choice  A  is   at  least  somewhat  true  -­‐-­‐  President  Mbame’s  speech  was  worded  somewhat   threateningly  (“anyone  who  stands  in  our  way  will  be  dealt  with  summarily”).  Yet   the  piece  isn’t  about  the  “display  of  strength”  of  the  new  president;  it’s  about  the   changing  conditions  in  the  country,  and  the  possible  hope  of  a  revitalized  economy   for  the  lower  classes.  While  President  Mbame’s  quote  certainly  evokes  a  mood,  it   does  not  help  establish  the  mood  of  this  piece.     Option  B  is  also  attractive,  but  clearly  not  correct;  the  writer  does  not  detail  the   plight  of  the  harvesters  or  the  miners  at  all.  S/he  mentions  these  low-­‐income   occupations,  but  does  not  use  any  evocative  language  whatsoever  to  describe  their   lifestyles  or  issues.  C  is  also  clearly  incorrect;  there  is  no  speculation  in  the  piece,   only  a  statement  of  facts  (that  the  army  is  on  standby  and  that  there  has  not  yet   been  any  contact  from  the  rebel  group).  Finally,  the  author  DOES  use  descriptive   Take  your  S   AT  prep  to  the  next  level.  Visit  www.learnerator.com   115  

116     words  that  clearly  paint  an  image  of  Kundu  as  being  on  a  long  road  to  recovery;  the   word  “slog”  corresponds  to  the  difficulties  that  the  miners,  harvesters,  and  new   President  face  as  they  stand  up  to  violent  rebel  groups  and  take  back  what  is   rightfully  theirs.  The  word  “glimmer  of  hope”  corresponds  to  the  “hope”  mentioned   at  the  end  of  the  article  but  emphasizes  that  the  battle  hasn’t  been  won  yet.  The   tension  is  palpable,  and  the  writer  reiterates  it  by  detailing  the  struggles  that   President  Mbame  and  his  people  will  face  in  the  future.     As  always  with  these  types  of  questions,  it  is  crucial  that  you  pay  close  attention  to   what  is  stated,  what  can  be  reasonably  inferred,  and  what  is  simply  wild   speculation.  This  will  help  you  determine  which  words  are  most  important  in   context.   Some  types  of  passages  will  make  analyzing  word  choice  easier  than  others.   Different  texts  seek  to  do  different  things.  For  example,  an  objective  piece  that   clearly  lays  out  a  number  of  facts  without  editorializing  will  not  necessarily  have   lots  of  strong  words  you  can  point  to  as  creating  a  tone,  but  a  literary  work  or  an   editorial  will  certainly  use  all  manner  of  rhetorical  devices  from  strong  language  to   evocative  words  in  order  to  create  meaning  or  mood.  Remember  that  “rhetoric”   means  “argument,”  so  when  analyzing  arguments,  be  on  the  lookout  for  strong   language  and  interesting,  unusual,  or  particularly  meaningful  word  choices.  

 

 

Take  your  S   AT  prep  to  the  next  level.  Visit  www.learnerator.com   116  

117    

Analyzing Overall Text Structure It  may  not  seem  like  it,  but  all  texts,  like  houses,  are  constructed  carefully  and  in  a   specific  way.  Even  the  most  stream-­‐of-­‐consciousness  modernist  text  has  a  structure   and  a  plan;  a  lot  of  work  goes  into  polished,  professional  writing,  and  it  is  safe  to   assume  that  there  is  a  strong  relationship  between  a  text’s  aims  (what  it  is  trying  to   do,  say,  convey)  and  the  way  it  is  organized.   It  can  be  helpful  to  think  of  a  text  like  a  house.  Just  as  houses  are  built  to   accommodate  the  specific  needs  of  the  owner  -­‐-­‐  taking  into  consideration  climate,   family  size,  and  personal  taste  -­‐-­‐  texts  are  tailored  to  best  convey  specific   information.  For  example,  a  narrative  text  that  seeks  to  tell  a  story  will  be  structured   very  differently  from  a  persuasive  text  that  seeks  to  inform  or  argue  a  point.  Here   are  some  text  structures  that  may  crop  up  on  the  new  SAT:     Narrative  Texts     There  will  be  one  reading  from  US  and/or  World  Literature  on  the  new  SAT.  It  could   be  structured  in  a  variety  of  ways,  including:   -­‐a  linear  story,  with  a  traditional  plot  arc  (beginning,  middle,  end)   -­‐a  non-­‐linear  story,  told  in  flashbacks  or  switching  between  past,  present,  and  future   events   -­‐stream-­‐of-­‐consciousness  (this  is  unlikely,  as  these  types  of  texts  are  quite  high-­‐level   and  difficult  to  analyze,  but  it’s  best  to  be  prepared!)   Regardless  of  the  structure,  the  resulting  text  will  most  likely  be  heavy  with   description  -­‐-­‐  adjectives,  imagery,  and  other  strategies  intended  to  get  the  reader  to   think,  feel,  imagine,  or  visualize  something.  The  author  might  center  a  piece  directly   around  dialogue,  letting  the  reader  understand  the  character  dynamics  and  predict   the  plot  using  only  the  characters’  words  as  a  guidepost.  For  example,  in  Louisa  May   Alcott’s  story  Scarlet  Stockings,  the  characters’  relationship  and  a  probable  sequence   of  events  emerge  clearly  from  the  following  dialogue:   Take  your  S   AT  prep  to  the  next  level.  Visit  www.learnerator.com   117  

118       "COME  out  for  a  drive,  Harry?"   "Too  cold."   "Have  a  game  of  billiards?"   "Too  tired."   "Go  and  call  on  the  Fairchilds?"   "Having  an  unfortunate  prejudice  against  country  girls,  I  respectfully  decline."   "What  will  you  do  then?"   "Nothing,  thank  you."   And  settling  himself  more  luxuriously  upon  the  couch,  Lennox  closed  his  eyes,   and  appeared  to  slumber  tranquilly.  Kate  shook  her  head,  and  stood  regarding   her  brother,  despondently,  till  a  sudden  idea  made  her  turn  toward  the  window,   exclaiming  abruptly,   "Scarlet  stockings,  Harry!"   "Where?"  and,  as  if  the  words  were  a  spell  to  break  the  deepest  day-­‐dream,   Lennox  hurried  to  the  window,  with  an  unusual  expression  of  interest  in  his   listless  face.   "I  thought  that  would  succeed!  She  isn't  there,  but  I've  got  you  up,  and  you  are   not  to  go  down  again,"  laughed  Kate,  taking  possession  of  the  sofa.   "Not  a  bad  maneuver.  I  don't  mind;  it's  about  time  for  the  one  interesting  event   of  the  day  to  occur,  so  I'll  watch  for  myself,  thank  you,"  and  Lennox  took  the   easy  chair  by  the  window  with  a  shrug  and  a  yawn.   "I'm  glad  any  thing  does  interest  you,"  said  Kate,  petulantly,  "though  I  don't   think  it  amounts  to  much,  for,  though  you  perch  yourself  at  the  window  every   day  to  see  that  girl  pass,  you  don't  care  enough  about  it  to  ask  her  name."   "I've  been  waiting  to  be  told."   "It's  Belle  Morgan,  the  Doctor's  daughter,  and  my  dearest  friend."   "Then,  of  course,  she  is  a  blue-­‐belle?"   "Don't  try  to  be  witty  or  sarcastic  with  her,  for  she  will  beat  you  at  that."   "Not  a  dumb-­‐belle  then?"   "Quite  the  reverse;  she  talks  a  good  deal,  and  very  well  too,  when  she  likes."   Take  your  S   AT  prep  to  the  next  level.  Visit  www.learnerator.com   118  

119     "She  is  very  pretty;  has  anybody  the  right  to  call  her  'Ma  belle'?"   "Many  would  be  glad  to  do  so,  but  she  won't  have  any  thing  to  say  to  them."   "A  Canterbury  belle  in  every  sense  of  the  word  then?"   "She  might  be,  for  all  Canterbury  loves  her,  but  she  isn't  fashionable,  and  has   more  friends  among  the  poor  than  among  the  rich."   "Ah,  I  see,  a  diving-­‐bell,  who  knows  how  to  go  down  into  a  sea  of  troubles,  and   bring  up  the  pearls  worth  having."   "I'll  tell  her  that,  it  will  please  her.  You  are  really  waking  up,  Harry,"  and  Kate   smiled  approvingly  upon  him.   "This  page  of  'Belle's  Life'  is  rather  amusing,  so  read  away,"  said  Lennox,   glancing  up  the  street,  as  if  he  awaited  the  appearance  of  the  next  edition  with   pleasure.   "There  isn't  much  to  tell;  she  is  a  nice,  bright,  energetic,  warm-­‐hearted  dear;   the  pride  of  the  Doctor's  heart,  and  a  favorite  with  every  one,  though  she  is  odd.   "How  odd?"   "Does  and  says  what  she  likes,  is  very  blunt  and  honest,  has  ideas  and  principles   of  her  own,  goes  to  parties  in  high  dresses,  won't  dance  round  dances,  and   wears  red  stockings,  though  Mrs.  Plantagenet  says  it's  fast."   "Rather  a  jolly  little  person,  I  fancy.  Why  haven't  we  met  her  at  some  of  the  tea-­‐ fights  and  muffin-­‐worries  we've  been  to  lately?"   "It  may  make  you  angry,  but  it  will  do  you  good,  so  I'll  tell.  She  didn't  care   enough  about  seeing  the  distinguished  stranger  to  come;  that's  the  truth."   "Sensible  girl,  to  spare  herself  hours  of  mortal  dullness,  gossip,  and  dyspepsia,"   was  the  placid  reply.   "She  has  seen  you,  though,  at  church  and  dawdling  about  town,  and  she  called   you  'Sir  Charles  Coldstream'  on  the  spot.  How  does  that  suit?"  asked  Kate,   maliciously.   "Not  bad,  I  rather  like  that.  Wish  she'd  call  some  day,  and  stir  us  up."   "She  won't;  I  asked  her,  but  she  said  she  was  very  busy,  and  told  Jessy  Tudor,  she   wasn't  fond  of  peacocks."   "I  don't  exactly  see  the  connection."   Take  your  S   AT  prep  to  the  next  level.  Visit  www.learnerator.com   119  

120     "Stupid  boy!  she  meant  you,  of  course."   "Oh,  I'm  peacocks,  am  I?"   "I  don't  wish  to  be  rude,  but  I  really  do  think  you  are  vain  of  your  good  looks,   elegant  accomplishments,  and  the  impression  you  make  wherever  you  go.  When   it's  worth  while  you  exert  yourself,  and  are  altogether  fascinating,  but  the  'I   come  -­‐-­‐  see  -­‐-­‐  and  -­‐-­‐  conquer'  air  you  put  on,  spoils  it  all  for  sensible  people."   "It  strikes  me  that  Miss  Morgan  has  slightly  infected  you  with  her  oddity  as  far   as  bluntness  goes.  Fire  away,  it's  rather  amusing  to  be  abused  when  one  is   dying  of  ennui."   "That's  grateful  and  complimentary  to  me,  when  I  have  devoted  myself  to  you   ever  since  you  came.  But  every  thing  bores  you,  and  the  only  sign  of  interest   you've  shown  is  in  those  absurd  red  hose.  I  should  like  to  know  what  the  charm   is,"  said  Kate,  sharply.   "Impossible  to  say;  accept  the  fact  calmly  as  I  do,  and  be  grateful  that  there  is   one  glimpse  of  color,  life,  and  spirit  in  this  aristocratic  tomb  of  a  town."   "You  are  not  obliged  to  stay  in  it!"  fiercely.   "Begging  your  pardon,  my  dove,  but  I  am.  I  promised  to  give  you  my  enlivening   society  for  a  month,  and  a  Lennox  keeps  his  word,  even  at  the  cost  of  his  life."   "I'm  sorry  I  asked  such  a  sacrifice;  but  I  innocently  thought  that  after  being   away  for  five  long  years,  you  might  care  to  see  your  orphan  sister,"  and  the   dove  produced  her  handkerchief  with  a  plaintive  sniff.   "Now,  my  dear  creature,  don't  be  melodramatic,  I  beg  of  you,"  cried  her   brother,  imploringly.  "I  wished  to  come,  I  pined  to  embrace  you,  and  I  give  you   my  word,  I  don't  blame  you  for  the  stupidity  of  this  confounded  place."     Why  does  Louisa  May  Alcott  choose  to  begin  her  story  this  way?   A.  The  dialogue  helps  establish  the  characters,  setting,  and  plot  without  too   much  exposition   B.  The  dialogue  reveals  the  tragic  pasts  of  her  orphaned  narrators  and  helps   establish  a  sense  of  sympathy   C.  The  dialogue  reveals  what  a  cad  Harry  is,  setting  him  up  for  the  failure  he   Take  your  S   AT  prep  to  the  next  level.  Visit  www.learnerator.com   120  

121     is  likely  to  endure  by  the  end  of  the  story   D.  The  dialogue  helps  establish  a  love  triangle  that  is  likely  to  be  the  focus  of   the  story’s  plot     This  question  may  not  seem  like  it’s  asking  you  about  structure,  but  in  fact  that  is   exactly  what  it  is  doing  -­‐-­‐  trying  to  get  a  sense  for  whether  or  not  you  understand   WHY  an  author  made  a  certain  choice  in  writing  a  story  and  whether  or  not  that   choice  is  effective.  (It  also  asks  you  to  consider  the  author’s  purpose  as  well  as  the   relationship  of  one  part  of  a  story  to  the  whole  of  the  story  -­‐-­‐  three  skills  for  the   price  of  one!)     Consider  the  answer  choices.  Answer  A  is  certainly  correct  -­‐-­‐  we  do  have  a  sense  of   the  characters’  interests,  relationships,  and  personality  (Harry  is  a  grouch  with  a   crush,  Kate  is  a  sweetheart  who  loves  and  worries  about  her  brother,  and  Belle  is   apparently  perfect).  But  the  key  here  is  the  “without  too  much  exposition”  -­‐-­‐  world-­‐ building  is  difficult,  and  it  takes  skill  to  inundate  the  reader  with  information  about   the  characters  and  plot  without  being  boring  or  redundant.  In  effect,  Alcott’s  clever   “in  medias  res”  (in  the  middle  of  things)  dialogue  strategy  plops  the  reader  directly   into  the  Lennoxes’  sitting  room,  eliminating  the  need  for  a  tiresome  introduction   that  would  have  gone  something  like  this:  Harry  and  Kate  were  brother  and  sister,   but  Harry  had  been  away  in  France  for  five  years  and  was  pretty  cranky  about  having   to  be  back  in  a  boring  old  town  that  he  hated.  And  so  forth.     But  let’s  consider  B.  The  specificity  of  the  other  answer  choices  certainly  makes   them  tempting,  but  consider  them  closely:  while  it’s  TRUE  that  Harry  and  Kate  are   orphaned,  this  is  in  no  way  the  purpose  or  entirety  of  their  dialogue.  They  mention   it  once,  towards  the  end  of  the  selection,  and  while  the  background  helps  establish  a   sense  of  context,  it  is  in  no  way  the  focus  of  the  piece.  Nor  does  Harry  come  off  as   being  particularly  sympathetic,  and  Kate  doesn’t  need  the  tragic  orphan  detail  to   become  sympathetic  -­‐-­‐  everything  she  does,  from  striving  to  set  up  her  brother  and   her  best  friend  to  her  strategy  of  caring  for  her  brother,  establishes  her  as  a   Take  your  S   AT  prep  to  the  next  level.  Visit  www.learnerator.com   121  

122     sympathetic  character.  So  beware  of  extreme  specificity  -­‐-­‐  with  questions  like  this,   which  ask  about  a  piece  as  a  whole,  generalized  answers  will  often  work  better  than   specific  ones.     C’s  flaw  is  similar  to  B’s;  it  is  simply  too  specific,  plus  there  isn’t  a  ton  of   support/evidence  for  it.  Yes,  Harry’s  kind  of  a  cad  -­‐-­‐  but  it’s  highly  unlikely  that  his   sister,  who  clearly  loves  and  cares  about  him  and  her  best  friend,  would  be  plotting   to  set  the  two  up  if  she  didn’t  think  they  would  work  well  together.  Plus,  there’s   simply  no  evidence  that  the  two  won’t  hit  it  off  -­‐-­‐  in  fact,  the  evidence  is  to  the   contrary,  since  the  chances  are  that  you’ve  seen  some  variant  of  this  “good  girl   reforms  bad  boy”  trope  before.  Scratch  C.     Ditto  D.  You  may  be  tempted  to  pick  this  one  because  the  dialogue  does  indeed   establish  a  love  story  that  is  integral  to  the  story’s  plot.  But  please  beware  of  this   expression  “love  triangle.”  As  of  now,  there  are  only  two  players  in  this  potential   love  story  -­‐-­‐  Harry  and  Belle.  Kate  certainly  isn’t  part  of  the  love  triangle,  as  she’s   Harry’s  sister  and  playing  matchmaker,  not  striving  for  the  romantic  affection  of   either  party.  Pay  attention  to  small  details  like  this.     Narrative  texts  may  also  be  structured  a-­‐chronologically,  utilizing  literary  devices   such  as  flashbacks,  flash  forwards/foreshadowing,  or  internal  dialogue.  For   example,  look  at  this  following  excerpt,  which  is  heavy  on  internal  dialogue:     “I  hate  this  stupid  place,”  thought  Princess  Bluebell  to  herself  as  she  trudged  up   the  stairs  to  her  new  boarding  school,  The  KAPS  Academy  for  Young  Heroines.   “I  hate  horses,”  she  thought  murderously,  eyeing  the  portraits  of  noble  steeds   that  lined  the  hallways.  “They  smell  terrible.  I  hate  archery,”  she  continued,   looking  askance  at  the  bust  of  Diana  the  Huntress  that  dominated  the  front   entryway.  “It  gives  me  splinters.  And  I  hate,  hate,  hate  that  the  whole  aim  of   this  stupid  school  is  to  teach  me  how  to  be  my  own  hero  instead  of  waiting   around  for  my  handsome  prince.”  At  this,  she  sighed,  pulling  out  her  wallet-­‐size   Take  your  S   AT  prep  to  the  next  level.  Visit  www.learnerator.com   122  

123     photo  of  Charming,  the  handsome  prince  next  door  who  was  no  doubt  going  to   stop  loving  her  when  she  came  back  to  the  castle  over  summer  break,  all   tanned  and  strong  and  able  to  rescue  herself  instead  of  languishing  pale  and   malnourished  in  some  attic  tower  guarded  by  a  dragon.  Everyone  knew  princes   had  no  use  for  strong  girls.  He  would  stop  loving  her  in  an  instant!  “Well,  I  just   won’t  learn  anything,”  she  swore,  “and  Mom  and  Dad  will  let  me  come  home   after  a  semester  and  finally  send  me  off  to  that  witch  who’s  been  agitating  to   keep  me  locked  in  a  tower  until  I  turn  16  and  Charming  can  come  rescue  me.   Everyone  wants  me  to  be  master  of  my  own  fate.  I’ll  beat  them  at  their  game!”   A  wicked  smile  curved  her  lips,  and  she  headed  up  the  stairs  to  her  first  class,   Damsels,  Maidens,  and  Other  Helpless  Twits:  Defying  the  Stereotypes  and   Daring  to  Be  Your  Own  Hero,  determined  not  to  learn  a  single  thing.     What  is  the  primary  effect  of  this  internal  dialogue  on  the  structure  of   the  text  as  a  whole?   A.  The  internal  dialogue  establishes  Bluebell  as  a  sympathetic  character  and   invites  the  reader  to  admire  her  master  plan   B.  The  internal  dialogue  establishes  Bluebell  as  a  foolish  character  deserving   of  mockery  and  invites  the  reader  to  laugh  at  her   C.  The  internal  dialogue  establishes  an  arc  for  plot  and  character   development   D.  The  internal  dialogue  establishes  the  passionate  love  story  sure  to  ensue   between  Bluebell  and  Charming     Again,  remember  that  in  questions  about  text  structure,  general  is  (generally)  better   than  super-­‐specific.  Pay  attention  also  to  the  word  “primary”  -­‐-­‐  what  is  the  MAIN   effect,  or  the  MAIN  purpose  of  this  internal  dialogue?  A  may  be  half-­‐plausible  (it’s   possible  that  the  author  wants  to  portray  Bluebell  as  a  sympathetic  character,   though  from  the  gently  mocking  tone  it’s  fairly  clear  this  isn’t  entirely  the  case)  but   surely  the  second  clause  of  the  sentence  (“invites  the  reader  to  admire  her  master  

Take  your  S   AT  prep  to  the  next  level.  Visit  www.learnerator.com   123  

124     plan”)  will  make  it  easy  to  scratch  A,  as  Bluebell’s  plan  is  fairly  foolish  and  trades   heavily  in  antiquated  stereotypes  about  femininity.     B  is  plausible  too,  but  not  entirely  likely  -­‐-­‐  after  all,  the  question  asks  about  the   PRIMARY  effect  of  the  internal  dialogue.  We  may  be  invited  to  roll  our  eyes  a  little   bit  at  Bluebell,  but  it  is  unlikely  that  the  author  would  have  written  this  piece   specifically  to  make  fun  of  the  main  character  as  this  is  a  fairly  big  waste  of   everyone’s  time  and  makes  for  a  less  than  enjoyable  reading  experience.  So  while   this  could  have  been  a  plausible  choice  had  the  question  not  been  asking  about  the   PRIMARY  effect  of  this  dialogue,  it  isn’t  entirely  likely  to  be  the  main  reason  the   author  uses  this  literary  device.  C,  again,  is  a  nice  general  answer  and  entirely  true  -­‐-­‐   this  sets  Bluebell  up  to  probably  revise  her  thinking,  and  also  gives  us  a  source  of   conflict  /  tension,  which  help  propel  the  plot  forward.  D  is  unlikely;  there  is   certainly  a  love  interest,  but  it’s  hard  to  say  what  will  ensue  between  Bluebell  and   Charming,  especially  considering  that  she  is  about  to  (probably)  go  through  some   very  intense  changes  to  her  worldview.  So  again,  the  general  answer  that  focuses  on   the  CRAFT  -­‐-­‐  the  underlying  structure  -­‐-­‐  of  a  story  rather  than  a  specific  element  of   the  story.     Expository  Texts   Expository  texts  will  make  up  the  bulk  of  the  passages  on  the  new  SAT,  so  it’s   important  to  understand  how  their  structures  will  affect  their  content.  As  discussed   in  a  previous  article,  there  are  multiple  types  of  expository  text  structures,  and   several  ways  to  determine  what  they  are.  Sometimes  texts  will  fall  into  easy-­‐to-­‐spot   structures  (chronological,  cause  and  effect,  compare  and  contrast),  all  of  which  are   briefly  reviewed  here.  But  there  are  multiple  devices  that  authors  use  to  frame   arguments  and  ideas  that  don’t  necessarily  fit  into  these  three  neat  categories,  or   that  utilize  elements  from  one  or  more  of  them  in  order  to  effectively  establish  a   claim.  Those  will  be  reviewed  as  well.    

Take  your  S   AT  prep  to  the  next  level.  Visit  www.learnerator.com   124  

125     Chronology   A  piece  that  is  structured  chronologically  or  sequentially  will  have  all  sorts  of  “time”   words  to  indicate  the  sequence  of  events:  first,  last,  second,  last  Tuesday,  then,   afterwards,  before.  Writers  often  use  this  type  of  structure  to  frame  a  journalistic   piece  whose  purpose  is  to  accurately  convey  information  in  the  order  in  which  it   happened.  This  is  useful  in  pieces  about  current  events  (or  historical  events),  where   dates  and  times  are  very  important  as  they  influence  whatever  will  happen  next.  For   example,  check  out  this  article  on  the  history  of  space  exploration,  from   aerospace.org.  As  you  read,  make  note  of  all  the  chronological  signal  words:     Humans  have  dreamed  about  spaceflight  since  antiquity.  The  Chinese  used   rockets  for  ceremonial  and  military  purposes  centuries  ago,  but  only  in   the  latter  half  of  the  20th  century  were  rockets  developed  that  were  powerful   enough  to  overcome  the  force  of  gravity  to  reach  orbital  velocities  that  could   open  space  to  human  exploration.     As  often  happens  in  science,  the  earliest  practical  work  on  rocket  engines   designed  for  spaceflight  occurred  simultaneously  during  the  early  20th   century  in  three  countries  by  three  key  scientists:  in  Russia,  by  Konstantin   Tsiolkovski;  in  the  United  States,  by  Robert  Goddard;  and  in  Germany,  by   Hermann  Oberth.     In  the  1930s  and  1940s  Nazi  Germany  saw  the  possibilities  of  using  long-­‐ distance  rockets  as  weapons.  Late  in  World  War  II,  London  was  attacked  by   200-­‐mile-­‐range  V-­‐2  missiles,  which  arched  60  miles  high  over  the  English   Channel  at  more  than  3,500  miles  per  hour.     After  World  War  II,  the  United  States  and  the  Soviet  Union  created  their  own   missile  programs.  On  October  4,  1957,  the  Soviets  launched  the  first  artificial   satellite,  Sputnik  1,  into  space.  Four  years  later  on  April  12,  1961,  Russian  Lt.   Yuri  Gagarin  became  the  first  human  to  orbit  Earth  in  Vostok  1.  His  flight   Take  your  S   AT  prep  to  the  next  level.  Visit  www.learnerator.com   125  

126     lasted  108  minutes,  and  Gagarin  reached  an  altitude  of  327  kilometers  (about   202  miles).   The  first  U.S.  satellite,  Explorer  1,  went  into  orbit  on  January  31,  1958.  In   1961  Alan  Shepard  became  the  first  American  to  fly  into  space.  On  February   20,  1962,  John  Glenn’s  historic  flight  made  him  the  first  American  to  orbit   Earth.     “Landing  a  man  on  the  moon  and  returning  him  safely  to  Earth  within  a   decade”  was  a  national  goal  set  by  President  John  F.  Kennedy  in  1961.  On  July   20,  1969,  Astronaut  Neil  Armstrong  took  “a  giant  step  for  mankind”  as  he   stepped  onto  the  moon.  Six  Apollo  missions  were  made  to  explore  the  moon   between  1969  and  1972.     Recognizing  these  signal  words  will  help  you  better  understand  the  structure  of  the   text  -­‐-­‐  and  will  also  help  you  to  answer  questions  that  ask  specifically  about  a   sequence  of  events.  For  example,  a  question  may  ask  you  about  the  first  person  to   enter  space,  the  first  American  to  orbit  Earth,  the  first  person  to  step  on  the  moon  -­‐-­‐   all  distinct  characters.  You  will  have  to  pay  attention  to  the  signal  words  and  the   dates  in  order  to  answer  those  questions  accurately.  A  question  about  the  structure   of  the  text  may  ask  you  something  like:     Why  does  the  author  choose  to  introduce  the  article  with  the  example  of   ancient  Chinese  ceremonial  rockets?   A,  These  were  the  first  rockets  into  space,  and  as  such  they  are  notable  and   relevant  to  an  article  about  space  exploration   B,  The  article  is  about  Chinese  space  exploration,  and  so  these  rockets  are  a   valuable  precursor  to  the  article  as  they  help  to  better  understand  the   timeline   C,  These  rockets  are  a  successful  framing  device  that  help  to  introduce  the   history  of  space  exploration,  from  rockets  that  tried  and  failed  to  reach  space  

Take  your  S   AT  prep  to  the  next  level.  Visit  www.learnerator.com   126  

127     to  rockets  that  landed  human  beings  on  the  moon   D,  The  use  of  the  Chinese  rockets  example  is  irrelevant  to  the  text  as  a  whole     Of  course,  using  what  you  already  know  about  text  structure  -­‐-­‐  go  general,  think   about  craft  and  purpose,  and  pay  close  attention  -­‐-­‐  it  is  pretty  clear  that  C  is  the  best   answer.  A  is  incorrect  as  the  article  clearly  states  that  these  ancient  rockets  didn’t   reach  space;  B  is  clearly  incorrect  because  the  Chinese  are  not  mentioned  as  being   pioneers  of  space  exploration;  and  D  simply  isn’t  true  because  the  example  is  indeed   relevant  in  kicking  off  the  article.    

Take  your  S   AT  prep  to  the  next  level.  Visit  www.learnerator.com   127  

128     The  new  SAT  may  also  focus  on  questions  such  as:     What  is  the  effect  of  using  multiple  dates  in  this  article  to  introduce   different  events?   A.  The  use  of  dates  imparts  information  in  the  simplest,  most  objective,  and   most  historically  accurate  way   B.  The  use  of  dates  is  meant  to  demonstrate  how  many  years  it  took  for   Americans  to  achieve  space  flight   C.  The  use  of  dates  is  meant  to  highlight  Soviet  achievement  at  the  expense  of   American  achievement   D.  The  use  of  dates  is  meant  to  demonstrate  that  it  has  been  a  long  time  since   the  American  economy  could  support  exploratory  space  flight     This  question  asks  you  to  analyze  why  an  author  chose  a  specific  structure,  and   potentially  to  evaluate  whether  this  choice  was  the  most  effective  one.  Again,  using   our  strategy  of  being  GENERAL  and  focusing  on  CRAFT  and  STRUCTURE  rather  than   specific  pieces  of  content,  it  should  be  fairly  clear  that  A  is  the  only  correct  answer.   The  objective  tone  of  the  article  (it  uses  no  laudatory  adjectives  and  offers  no   opinions,  just  lays  out  facts)  makes  it  obvious  that  the  author  is  not  interested  in   criticizing  or  lauding  either  the  Americans  or  the  Soviets,  eliminating  B  and  C.  And   there  is  simply  no  support  in  the  article  for  D,  as  there  is  no  mention  of  the  economy.    

Take  your  S   AT  prep  to  the  next  level.  Visit  www.learnerator.com   128  

129     Cause  and  Effect   Use  of  the  cause  and  effect  structure  is  especially  prevalent  in  science  or  history   articles  that  seek  to  explain  or  understand  specific  phenomena.  This  structure   quickly  becomes  evident  with  words  like  “due  to,”  “thanks  to,”  “because  of,”  “leading   to,”  “resulting  in”  and  the  like.  This  article  about  diversity  on  television  uses  a  very   subtle  cause  and  effect  structure;  as  you  read,  pay  attention  to  words  that  signal  a   cause  and  an  effect.     Though  the  demographics  represented  on  television  have  shifted  -­‐-­‐  with  greater   representation  of  blended,  mixed,  or  minority  families  -­‐-­‐  there  is  still  work  to  be   done.  Blonda  Thimes,  the  creative  mastermind  behind  such  long-­‐running   blockbuster  hits  as  Shay’s  Anatomy,  Disaster,  and  How  to  Get  Away  With  A   Terrible  Crime,  has  often  been  credited  for  bringing  minority  actors  to  the   forefront  of  American  television.  Lauded  for  her  “color-­‐blind”  casting,  Thimes   had  a  lot  to  say  when  we  contacted  her  on  Thursday.  “It’s  pretty  sad,”  Blonda   says,  “that  in  2014  it’s  still  somehow  notable  or  strange  that  a  black  woman  is  a   heroine  or  a  main  character  instead  of  a  sassy  friend  or  sidekick.”  Thimes’   casting  has  also  led  to  a  minor  revolution  in  the  representation  of  gay  and   lesbian  characters  on  television,  as  her  shows  focus  on  diverse  representations   that  stray  quite  far  from  stereotypical  depictions  of  this  minority  group.  Thimes   called  these  stereotypes  “lazy,  pathetic  writing”  and  claimed  that  they  are   “responsible  for  mass  social  ignorance.”     Now,  there  are  multiple  causes  and  effects  here,  so  pay  close  attention  to  how  the   author  chooses  to  present  the  multiple  factors  that  went  into  this  article.     According  to  the  article,  what  is  the  main  effect  of  Thimes’  “color-­‐blind”   casting?   A.  Her  work  has  received  acclaim  and  success   B.  She  has  been  lauded  by  critics  

Take  your  S   AT  prep  to  the  next  level.  Visit  www.learnerator.com   129  

130     C.  American  television  is  more  diverse  than  ever  before   D.  Stereotypical  depictions  of  the  GLBT  community  have  lessened     Now,  pay  close  attention  to  the  word  “main  effect,”  because  while  A  is  true,  it  clearly   isn’t  the  focus  of  the  article.  B  is  also  true  -­‐-­‐  in  fact,  it  is  nearly  identical  to  A  -­‐-­‐  but  it   also  isn’t  the  focus  of  the  article.  C  is  clearly  true  (the  article  states  she  has  “led  to”  a   minor  revolution  and  she  has  been  “credited  with”  -­‐-­‐  i.e.  she  has  caused  -­‐-­‐  greater   representation  of  minority  actors  on  television),  so  this  is  clearly  the  best  answer.  D   is  tricky,  because  while  Thimes  has  been  credited  with  positively  representing  the   gay  community,  this  does  not  mean  that  other  shows  have  suddenly  stopped   utilizing  reductive  stereotypes.  An  increase  in  positive  representation  does  NOT   equal  a  decrease  in  negative  representation,  so  be  careful  of  false  effects  like  this   and  take  care  to  pay  attention  to  the  underlying  structure  of  the  article.     Compare  and  Contrast   Many  articles  will  utilize  comparison  and  contrast  in  order  to  effectively  convey  a   point.  Compare  and  contrast  can  be  quite  subtle;  while  signal  words  like  “worst,”   “best,”  “like,”  “dissimilar,”  and  others  can  set  up  parallels,  authors  can  signal  a   compare  and  contrast  structure  by  using  disparate  examples  that  ask  the  reader  to   think  about  the  elements  that  are  implicitly  being  compared.  Take  a  look  at  this   article  from  the  New  York  Times  and  note  the  multiple  instances  of  compare  and   contrast:     “Teenagers…  might  not  be  needing  their  social  networks  in  the  way  that   you  or  I  as  adults  might  really  need  our  social  networks,”  Emily  White,   author  of  the  book  “Lonely:  Learning  to  Live  With  Solitude,”  told  Op-­‐ Talk.    When  she  and  her  spouse  separated,  she  said,  “I  needed  help  in  all  sorts  of   ways,  and  if  that  help  hadn’t  been  there  I  would’ve  felt  lonely,  whereas  if  I  was   16  and  you  asked  me  after  drama  class  whether  my  social  support  network   mattered  a  lot  to  me,  my  answer  might  have  been  no.”     Take  your  S   AT  prep  to  the  next  level.  Visit  www.learnerator.com   130  

131     She  also  floated  a  potential  explanation  for  the  finding  that  students’  self-­‐ reported  loneliness  dropped  even  as  social  network  isolation  increased.   Environmentalists,  she  said,  sometimes  use  the  term  “environmental  amnesia.”   “What  it  means  is  that  every  generation  is  born  into  a  more  degraded   ecosystem,”  she  explained,  “and  they  take  that  degraded  ecosystem  as  their   baseline,  and  they  adjust  to  it.”  Maybe  teenagers  today  have  a  sort  of  “social   amnesia”  —  “I’m  wondering  if  these  high-­‐school  students  have   had  weaker  support  networks  their  whole  lives,  and  so  they’re  reporting  less   loneliness  because  that’s  what  they’re  used  to.”     The  first  paragraph  contains  a  fairly  obvious  type  of  comparison  and  contrast;   teenagers  and  adults  clearly  process  loneliness  in  different  ways  and  rely  on  social   networks  in  different  amounts  at  different  points  in  their  lives.  But  it’s  the  second   paragraph  that  interestingly  and  implicitly  compares  teenagers  and  adults.  The   word  “weaker”  should  have  you  thinking  “Weaker  than  WHAT?”  and  striving  to   answer  that  question  -­‐-­‐  what  are  the  elements  being  compared  here?  Relational   words  like  this  are  always  a  signal  that  SOMETHING  is  being  compared  and   contrasted.  The  “degraded  ecosystem”  should  also  be  a  clue  -­‐-­‐  clearly,  this   researcher  is  highlighting  a  difference  in  the  “Social  ecosystem”  a  teenager  in  2014   and  a  teenager  in,  say,  1994.  Understanding  that  this  text  has  a  cause  and  effect   structure  will  help  you  to  answer  questions  about  it  with  greater  accuracy.     How  does  Emily  White  explain  the  fact  that  teenagers  face  increased   social  isolation  yet  report  less  loneliness?   A.  She  argues  that  there  is  no  connection  between  social  isolation  and   loneliness   B.  She  gives  the  example  of  separating  from  her  spouse  to  demonstrate  that   teens  don’t  feel  loneliness  the  way  adults  do   C.  She  claims  students  do  not  need  social  networks  to  get  over  loneliness   D.  She  uses  a  cross-­‐disciplinary  example  to  demonstrate  that  teens  don’t   understand  loneliness  and  support  the  way  older  generations  do.   Take  your  S   AT  prep  to  the  next  level.  Visit  www.learnerator.com   131  

132       In  essence,  this  question  asks  you  to  figure  out  WHY  this  argument  was  structured   this  way  -­‐-­‐  why  did  White  first  lead  with  her  example  about  being  16,  and  then  go   into  an  example?  What,  in  essence,  is  she  trying  to  establish  by  presenting  her   argument  in  this  fashion?     A  clearly  cannot  be  correct  as  White  is  obviously  not  claiming  that  there  is  no   connection  between  social  isolation  and  loneliness;  in  fact,  she  strives  to  explain   their  relationship  by  giving  the  example  of  the  degraded  ecosystem.  If  there  were  no   relationship  between  the  two,  why  would  White  try  to  explain  it?  Clearly,  A  is   incorrect.   B  could  be  plausible,  but  read  carefully  -­‐-­‐  she  didn’t  say  teens  don’t  feel  lonely  like   adults  do.  She  said  teens  don’t  need  social  networks  like  adults  do.  Those  are  two   separate  issues,  according  to  the  article.     C  is  also  incorrect  -­‐-­‐  there  is  no  support  for  this  in  the  article.  She  never  once  says   that  teens  do  not  need  social  networks  to  get  over  loneliness,  only  that  they  view   social  networks  differently  than  adults  do.     D  is  the  best  answer  by  process  of  elimination,  but  also  because  it  is  true  -­‐-­‐  she  DOES   use  a  cross-­‐disciplinary  example  that  illustrates  that  the  older  generation  has  a   “better”  ecosystem  in  terms  of  relationships,  loneliness,  and  social  networks,  which   helps  to  explain  why  teens  view  these  differently  from  adults  and  why  their   loneliness  has  dropped  as  their  isolation  has  increased.  The  way  those  two  factors   interact  is  different  today  (she  claims)  than  it  was  twenty  years  ago,  before  the  rise   of  the  internet.   The  comparison  and  contrast  is  subtle  here,  and  often  the  new  SAT  will  rely  on  this   subtlety  in  order  to  potentially  confuse  you.  Be  on  the  lookout  for  words  that  subtly   signal  relationships  (weak  ER,  strong  ER,  etc.)  as  they  should  lead  you  to  look  for   and  try  to  understand  the  elements  that  are  being  compared.     Take  your  S   AT  prep  to  the  next  level.  Visit  www.learnerator.com   132  

133     Use  of  Quotes  and  Dialogue   Chronological  and  relational  signal  words  are  simple  to  spot,  but  not  every  piece  of   journalism  will  come  with  a  helpful  string  of  dates,  times,  causes,  effects,  and   comparisons.  To  make  a  piece  interesting  and  informative,  writers  will  often   interview  experts,  eyewitnesses,  and  other  people  of  interest  who  will  bolster  (or   contradict)  whatever  claims  the  article  is  making.  Articles  like  this  are  certainly   more  interesting  than  a  dry  string  of  dates  and  facts,  but  they  can  be  confusing  to   follow  as  different  experts  can  have  different  opinions  and  you  need  to  differentiate   between  who  is  saying  what,  and  in  what  context.     For  example,  look  at  the  following  article  from  the  Atlantic,  on  the  importance  of   field  trips:   What  if  those  field  trips  actually  had  a  proven,  tangible  benefit  to  student   learning?  That’s  the  premise  set  out  in  a  new  study  by  Jay  Greene,  a  professor  of   education  reform  at  the  University  of  Arkansas.   Greene  evaluated  670  students,  who  were  divided  into  two  groups.  The  first   group  of  students  was  chosen  at  random  to  see  a  live  theater  performance  of   either  Hamlet  or  A  Christmas  Carol.  The  second  group  either  read  the  texts  of   the  plays  or  watched  film  versions.     When  compared  with  their  peers  in  the  second  group,  the  students  who   attended  live  theater  scored  significantly  higher  on  a  vocabulary  test  that   incorporated  language  from  plays,  and  they  were  also  better  able  to  answer   questions  about  the  plot  and  characters,  according  to  Greene's  findings.     The  live  theater  group  also  scored  higher  on  tests  that  measured  their   tolerance  of  diverse  points  of  view  and  ability  to  detect  emotions  in  other   people.  Those  gains  were  still  measurable  six  weeks  after  students  attended  the   live  theater  performance,  Greene  said.     "Schools  are  increasingly  focused  on  the  things  that  we’ve  told  them  to  be   Take  your  S   AT  prep  to  the  next  level.  Visit  www.learnerator.com   133  

134     focused  on,  namely  improving  math  and  reading  test  scores,"  Greene  told  me.   "Anything  that  isn’t  directly  related  to  that  doesn’t  attract  as  much  of  their   attention,  their  resources,  or  their  time."     In  metro  Atlanta,  field  trips  are  being  used  as  both  a  means  of  reinforcing   classroom  instruction  and  providing  students  with  new  experiences.   "It’s  important  for  [students]  to  learn  the  standards  and  perform  well  on  these   standardized  tests,"  Jason  Marshall,  a  principal  at  an  elementary  school  in  the   Atlanta  area,  told  the  Atlanta  Journal-­‐Constitution.  "But  I  think  the  way  we’ve   always  approached  it  is  the  day  they  take  a  test  is  really  just  a  snapshot  of  what   they  learn.  We’re  interested  in  them  learning  much  more  about  their   community,  …  their  state,  country,  world  and  how  all  those  things  are   interconnected."     A  question  about  text  structure  for  an  article  like  this  may  ask  you  what  a  specific   expert  says,  forcing  you  to  ensure  that  you  understand  and  can  differentiate   between  one  expert’s  opinion  and  another’s  (or  the  expert’s  opinion  and  the  opinion   of  the  writer).  Keep  your  eyes  peeled  for  quotes  and  make  sure  that  you  mark  down   who  is  saying  what,  as  the  use  of  multiple  quotes  from  multiple  sources  can  often   confuse  students  about  who  is  saying  what  and  why  they  are  saying  it.  For  example,   try  this  question:     Based  on  the  quotes  from  educators  in  this  article,  how  would  a  high-­‐ school  principal  approach  field  trips  (leaving  aside  concerns  about   cost)?   A.  There  is  not  enough  information  in  this  article  to  answer  this  question   B.  A  high  school  principal  would  most  likely  be  opposed  to  a  field  trip,  as  it   would  distract  students  and  take  away  instructional  time   C.  A  high  school  principal  would  only  support  field  trips  to  go  see   Shakespearean  plays  

Take  your  S   AT  prep  to  the  next  level.  Visit  www.learnerator.com   134  

135     D.  A  high  school  principal  would  be  invested  in  the  concept  of  an  educational   field  trip  as  it  encourages  different  forms  of  learning.     The  first  thing  you  must  do  when  answering  this  question  is  find  the  quotes  from   educators  so  that  you  can  make  a  reasonable  inference  about  how  a  principal  might   feel.  (Yes,  questions  about  text  structure  often  require  some  inferences  -­‐-­‐  authors   generally  do  not  spell  out  WHY  they  make  certain  choices  in  writing,  leaving  it  to  the   reader  to  figure  it  out  for  themselves.)     Leave  A  for  now  -­‐-­‐  let’s  consider  the  other  answer  choices  and  whether  or  not  they   are  supported  by  the  article  before  we  decide  whether  or  not  there  is  enough   information  in  the  article  to  answer  the  question.  There  is  support  for  B  in  the  quote   “Schools  are  focused  on  the  things  that  we’ve  told  them  to  be  focused  on…  anything   that  isn’t  directly  related…  doesn’t  attract  as  much  of  their  attention.”  But  if  you   continue  reading  (and  paying  attention  to  the  way  this  article  is  structured,  with   quotes  introducing  and  FRAMING  an  issue),  then  it  becomes  clear  that  this  isn’t  the   be-­‐all,  end-­‐all  of  quotes  when  it  comes  to  how  principals  feel  about  field  trips.  Look   at  the  quote  from  the  principal  in  the  last  paragraph:  “The  day  they  take  the  test  is   just  a  snapshot;”  “we’re  interested  in  them  learning  much  more  about  their   community…”  This  doesn’t  sound  like  the  perspective  of  a  person  opposed  to   educational  field  trips,  plus  it  goes  hand  in  hand  with  the  previous  statements  about   field  trips  being  educationally  useful.  So  while  B  is  somewhat  plausible  if  you  refer   only  to  one  quote  from  the  article,  the  end  quote  from  an  actual  school  principal,   coupled  with  the  statements  about  the  positive  effects  of  field  trips,  work  together   to  make  it  clear  that  B  cannot  be  the  answer.     C  is  too  specific;  while  the  research  focused  on  plays,  the  principal  quoted  in  the  last   paragraph  makes  no  distinction  between  the  types  of  field  trips  and  the  specific   content  knowledge  that  his  school  cares  most  about  imparting.  D  is  best  supported   by  the  principal’s  statement  in  the  last  paragraph  -­‐-­‐  if  his  school  cares  most  about   supporting  different  types  of  learning,  then  a  field  trip  with  multiple  educational   Take  your  S   AT  prep  to  the  next  level.  Visit  www.learnerator.com   135  

136     outcomes  is  actually  the  ideal.  Clearly,  there  was  enough  information  in  the  article  to   answer  the  question,  so  A  cannot  be  correct.     The  Structure  of  an  Argument   Finally,  many  of  the  texts  on  the  new  SAT  will  focus  on  rhetoric:  claims,   counterclaims,  and  arguments.  Later  articles  will  focus  with  greater  specificity  and   detail  on  these  issues,  but  as  they  are  closely  related  to  structure,  it  is  important  to   be  able  to  identify  them.     Writers  will  often  set  up  an  argument  by:   1.  Establishing  a  claim   2.  Defending  that  claim   3.  Potentially  addressing  other  claims  /  opinions   4.  Demonstrating  why  those  other  opinions  are  incorrect     Steps  1  and  2  are  essential.  Steps  3  and  4  are  optional  and  you  will  not  always  see   them.  But  it  is  essential  that  you  learn  to  identify  the  structure  of  an  argument  and   understand  how  a  claim  is  established  and  defended.  Look  at  this  article  and  try  to   understand  what  this  writer  is  agitating  for,  and  whether  the  structure  of  this   argument  is  effective:     “But  if  the  University  wants  us  to  take  its  strictures  seriously,  the  least  it  can  do   is  pretend  to  listen  to  our  demands.  We  are  being  asked  to  fork  over   increasingly  high  tuitions  each  year,  to  partake  in  the  mandatory  (and   expensive)  dining  and  boarding  plan  for  the  sake  of  “communal  class  bonding,”   and  to  participate  in  community-­‐building  exercises  that  benefit  no  one  and   alienate  a  significant  percentage  of  the  student  body.  The  University  would  do   well  to  consider  that  it  endangers  the  future  contributions  of  its  alumni  base   with  its  attitude  towards  its  students,  which  is  one  of  shameless  financial   wrangling  and  gross  neglect.”     Take  your  S   AT  prep  to  the  next  level.  Visit  www.learnerator.com   136  

137      How  does  the  structure  of  this  argument  help  to  create  a  tone  of   indignation?   A.  The  author  clearly  establishes  her  indignation  by  ending  the  letter  with  a   subtle  threat  to  the  university   B.  The  author  clearly  establishes  her  indignation  with  the  use  of  informal   language  such  as  “fork  over”   C.  The  author  clearly  establishes  her  indignation  by  placing  certain  words  in   quotations  to  indicate  disdain   D.  The  author  clearly  establishes  her  indignation  by  making  a  specific   complaint  and  enumerating  the  reasons  for  it     This  is  an  interesting  question  because  it  doesn’t  ask  you  to  evaluate  the  claim  or   even  to  evaluate  whether  the  structure  helps  establish  the  claim;  instead  it  asks  you   to  assess  how  the  structure  helps  to  create  a  tone.  This  is  a  “double  whammy”   question  that  gets  at  skills  like  analyzing  word  choice  as  well  as  analyzing  structure.   Have  a  look  at  the  answer  choices.     A  is  tempting  because  it  is  partially  true  -­‐-­‐  the  author  DOES  end  the  letter  with  a   subtle  threat  -­‐-­‐  but  it  is  clear  well  before  this  threat  in  the  last  sentence  that  she  is   unhappy.  Therefore  she  doesn’t  ESTABLISH  her  indignation  in  the  last  sentence,  just   CEMENTS  it.  Pay  attention  to  distinctions  like  this  because  they  could  be  the   difference  between  choosing  the  right  answer  and  choosing  a  plausible  distractor.   B  is  implausible;  use  of  words  like  “fork  over”  makes  the  author  sound  a  little  bit   folksy,  but  doesn’t  necessarily  establish  a  sense  of  indignation.  It’s  not  like  “fork   over”  is  the  exclusive  phrase  of  people  who  are  unhappy  with  a  specific  financial   situation.     C  is  interesting;  it’s  clear  from  the  quotes  that  the  author  doesn’t  think  much  of  this   so-­‐called  “communal  class  bonding,”  but  she  doesn’t  spend  much  time  critiquing  it,   nor  does  she  establish  her  indignation  by  using  this  phrase  -­‐-­‐  her  indignation  was  

Take  your  S   AT  prep  to  the  next  level.  Visit  www.learnerator.com   137  

138     established  well  before,  in  the  first  sentence.  So  C  can’t  be  correct.     D  is  the  only  obvious  choice  -­‐-­‐  not  only  is  it  the  most  all-­‐encompassing/general,  it   also  best  reflects  the  structure  of  this  article  and  argument.      There  are  multiple  types  of  text  structures;  this  is  just  a  sampling.  But  if  you  take   away  anything  from  this  article,  it  should  be  to  pay  attention  to  the  subtleties  and   signal  words.  They  are  often  the  key  to  unlocking  the  entire  piece.    

 

Take  your  S   AT  prep  to  the  next  level.  Visit  www.learnerator.com   138  

139    

Analyzing Part-Whole Relationships As  discussed  in  the  previous  article,  rhetoric  relies  heavily  on  a  well-­‐constructed   argument.  It’s  important  that  you  can  analyze  the  structure  of  the  text  as  a  whole,   but  it’s  equally  important  to  be  able  to  pull  out  specific  parts  of  the  text  and  analyze   how  they  relate  or  fit  in  with  the  rest  of  the  structure.     Think  of  texts  like  tapestries,  or  Jenga  towers:  every  element  needs  to  be  balanced   and  well-­‐considered,  or  else  the  finished  whole  can  unravel  completely.  There  are   essential  functions  that  each  small  part  of  the  text  must  fulfill:     1.  Logic  /  setup  →  every  element  of  the  text  must  fit  together  logically  and   clearly.   2.  Focus  →  every  element  of  the  text  should  stay  on  point;  this  is  especially   important  in  an  expository  text.   3.  Support  →  every  element  of  the  text  should  be  doing  something:   introducing    a  topic,  supporting  a  claim,  countering  a  claim,  establishing  evidence,   describing  a  feature,  outlining…   4.  Inform  →Is  this  sentence  informing  you  about  something?  A  setting,  a   character,  a  piece  of  research,  an  opinion?  If  it  isn’t,  consider  its  purpose.  It   may  not  belong  there.     The  new  SAT  may  ask  you  about  the  function  of  a  word  within  a  sentence  (as   discussed  here),  a  sentence  within  a  paragraph,  or  a  paragraph  within  a  larger  text,   so  it’s  important  to  understand  the  function  of  the  larger  element  as  well  as  the   function  of  the  smaller  element.  Here  are  some  common  types  of  part/whole   questions  the  new  SAT  will  focus  on:    

Take  your  S   AT  prep  to  the  next  level.  Visit  www.learnerator.com   139  

140     Sentence-­‐to-­‐Paragraph  Relationships     Logic  /  Set-­‐up   In  order  to  ensure  that  you  fully  comprehend  structure,  the  new  SAT  will  often  ask   you  what  a  sentence  is  “doing”  in  the  middle  of  a  paragraph.  You’ll  have  to  analyze   the  surrounding  context  in  order  to  correctly  answer  these,  as  not  every  sentence   functions  in  the  same  way  within  a  paragraph.  For  example,  an  expository  essay   may  start  with  an  introductory  or  topic  sentence  that  reveals  the  purpose  or  subject   of  the  paragraph,  while  a  narrative  selection  will  kick  off  with  a  first  sentence  that   helps  to  characterize  or  describe  someone.  These  bits  of  text  are  doing  the  hard   work  of  “set-­‐up,”  and  often  they  set  the  tone  for  the  rest  of  the  piece.  For  example,   look  at  the  first  sentences  of  these  two  separate  paragraphs  and  try  to  figure  out   their  functions  and  relationship  to  the  paragraph  as  a  whole:     Lupita  didn’t  much  care  for  bees,  but  she  wasn’t  going  to  sit  idly  by  and  watch   as  Bobby  Valente  -­‐-­‐  by  all  accounts  the  biggest  jerk  in  the  entire  seventh  grade  -­‐ -­‐  smashed  this  one  with  the  edge  of  his  Understanding  New  Horizons!  science   textbook.  “You  can’t  just  kill  something  just  because  you  don’t  like  it,”  she   informed  him,  edging  close  and  blocking  the  bee  from  his  reach.  Bobby  eyed  her   strangely.  “But  it’s  a  bee,”  he  said.  “It  might  hurt  us  first.”   -­‐-­‐-­‐-­‐   (from  Wikipedia)  Bees  may  be  solitary  or  may  live  in  various  types  of   communities.  The  most  advanced  of  these  are  eusocial  colonies  found  among   the  honey  bees,  bumblebees,  and  stingless  bees.  Sociality,  of  several  different   types,  is  believed  to  have  evolved  separately  many  times  within  the  bees.  In   some  species,  groups  of  cohabiting  females  may  be  sisters,  and  if  there  is   a  division  of  labor  within  the  group,  then  they  are  considered  semi  social.     It  should  be  pretty  obvious  that  these  two  paragraphs  do  NOT  have  the  same   purpose.  Their  tones  are  different,  their  subject  matter  is  different,  and  their   organization  is  different.  Therefore,  if  we  break  down  these  paragraphs  into  their   Take  your  S   AT  prep  to  the  next  level.  Visit  www.learnerator.com   140  

141     component  parts,  we  won’t  find  that  every  sentence  corresponds  in  function  and   type.  Therefore,  beware  of  falling  into  traps  like  “the  first  sentence  always   introduces  a  topic.”  This  may  be  true  SOMETIMES,  or  for  CERTAIN  types  of   passages,  but  it  all  depends  on  the  surrounding  context.     What  is  the  purpose  of  the  first  sentence  in  the  first  selection?   A.  To  provide  a  thesis  statement  for  a  larger  argument   B.  To  demonstrate  Lupita’s  feelings  about  bees   C.  To  establish  a  conflict  between  bees  and  humans   D.  To  flesh  out  a  character     It  should  be  fairly  clear  from  the  selection  that  this  selection  is  not  making  an   argument,  and  as  such  does  not  require  a  thesis  statement  to  establish  a  claim.  You   can  get  rid  of  answer  A.  While  B  is  semi-­‐plausible,  this  does  not  seem  to  be  the  focus   of  the  selection  -­‐-­‐  the  sentence  states  that  Lupita  “didn’t  care  for”  bees,  yet  she   campaigned  for  the  bee’s  rights/safety  anyway.  This  says  more  about  Lupita  than   about  Lupita’s  actual  feelings  about  bees.  Keep  B  until  you  can  eliminate  C  and  D.  C   is  clearly  incorrect  -­‐-­‐  there  isn’t  actually  a  conflict  here  between  bees  and  humans;  if   there  were,  Lupita  wouldn’t  be  agitating  to  save  the  bee.  D  is  the  best  answer,   because  what  actually  matters  in  this  paragraph  (and  this  sentence)  is  the   establishment  of  a  character  who  cares  about  the  helpless  and  isn’t  afraid  to  stand   up  to  the  “biggest  jerk  in  the  entire  seventh  grade”  in  order  to  defend  her  beliefs,   even  though  these  beliefs  might  be  unpopular.  You  get  more  information  about   Lupita  here  than  about  Lupita’s  feelings  about  bees  or  the  conflict  between  bees  and   humans,  which  is  a  pretty  good  indicator  that  D  is  the  best  answer.  When  in  doubt,   consider  the  answer  choices  in  terms  of  the  information  they  impart;  the  one  that   seems  like  it  imparts  the  most  relevant  information  will  probably  be  the  right   answer.     What  is  the  function  of  the  first  sentence  in  the  second  paragraph?   A.  To  introduce  various  specialized  terms  and  vocabulary  that  will  be  used  in   Take  your  S   AT  prep  to  the  next  level.  Visit  www.learnerator.com   141  

142     the  article   B.  To  lend  support  for  an  argument   C.  To  set  up  the  purpose  and  topic  of  the  paragraph.   D.  To  counter  a  claim     If  you  read  the  first  sentence  of  the  second  article  carefully,  you’ll  see  that  there  is   no  specialized  vocabulary  until  the  second  sentence;  here,  the  question-­‐writers  are   hoping  you’ll  see  the  word  “introduce,”  stop  reading,  and  pick  A.  This  is  a  bad   strategy.  Read  the  whole  question  and  the  whole  answer.  B  can’t  be  right  since  there   is  no  preceding  argument,  so  the  first  sentence  can’t  support  an  argument  that   doesn’t  exist.  C  is  obviously  true  -­‐-­‐  the  second  paragraph  is  all  about  the  different   types  of  communities  in  which  bees  live,  and  the  first  sentence  clearly  indicates  this.   D,  like  B,  can’t  be  correct  since  there  is  no  preceding  argument  or  claim  to  support   or  refute.     Focus   Every  text  has  (or  should  have)  a  focus  -­‐-­‐  in  essence,  a  “point.”  And  every  component   of  the  text  (sentences,  paragraphs,  etc)  should  reinforce  the  central  point  -­‐-­‐  or   introduce  new  points  for  consideration.  Think  of  texts  almost  like  interlocking   puzzle  pieces  -­‐-­‐  every  sentence  should  “fit”  in  some  way  with  the  one  before  it,  so   even  if  a  sentence  is  introducing  a  new  point,  it  shouldn’t  come  out  of  nowhere.  For   example,  here  are  two  examples  of  excerpts,  one  of  which  keeps  its  focus  and  one  of   which  doesn’t:     1. The  books  were  organized  alphabetically.    There  was  a  gap  on  the  shelf   between  Melville  and  Milton,  where  Mencken  had  been  pulled  out.   2. The  books  were  organized  alphabetically.  Cheese  is  delicious.     It’s  not  too  difficult  to  see  that  retaining  focus  is  important,  or  else  the  reader  will   give  up.  The  excerpt  below  contains  a  good  example  of  an  excerpt  that  shifts   focus  subtly,  without  descending  into  incoherence.   Take  your  S   AT  prep  to  the  next  level.  Visit  www.learnerator.com   142  

143       Vampires  -­‐-­‐  in  all  their  various  incarnations  -­‐-­‐  are  dead.  The  pale,  the  tortured,   the  lonely,  the  en-­‐souled,  the  menacing,  even  the  sparkling:  as  a  romantic  trope   or  an  object  of  nightmares,  vampires  have  had  their  moment  in  the  sun,  and   their  long  dark  night  has  finally  come.  They  are  dead,  but  our  obsession  with   the  supernatural  isn’t.  In  short  order,  hopeful  Young  Adult  writers  and  cynical   movie  producers  have  foisted  upon  us  every  half-­‐baked  example  of  the  liminal,   undead,  or  unknown:  zombies,  ghosts,  half-­‐demons,  fallen  angels,  werewolves,   rockstars  masquerading  as  college  students…  What  these  varyingly  successful   attempts  to  breathe  new  life  into  the  “creature  story”  fail  to  understand,   however,  is  that  it  is  the  human  monster  that  is  the  scariest,  the  darkest,  the   most  intense…  and  ultimately  the  most  interesting.     How  does  the  first  sentence  establish  the  tone  of  the  rest  of  the  article?   (logic/set-­‐up)   A.  By  dispassionately  stating  a  fact,  therefore  creating  an  objective,   journalistic  tone   B.  By  passionately  declaiming,  therefore  creating  a  fiery  tone   C.  By  including  a  pun,  therefore  creating  a  dry  tone   D.  By  utilizing  a  dependent  clause  within  the  two  hyphens,  therefore  creating   a  digressive  tone     This  question  asks  you  to  consider  the  first  sentence  in  context  of  the  rest  of  the   article;  you  must  pick  out  specific  words  and  phrases  that  establish  a  tone  in  order   to  accurately  answer  the  question.  Consider  Option  A.  The  sentence  does,  in  some   way,  include  a  fact  -­‐-­‐  vampires  are  indeed  dead.  But  the  tone  in  the  remainder  of  the   article  is  hardly  objective;  the  author  uses  a  lot  of  strong  language  (“foisted  upon   us,”  “half  baked,”  “varyingly  successful,”  “fail  to  understand,”)  to  establish  a  clear   and  obvious  opinion.  Answer  A  can  therefore  not  be  correct.  (This  is  why  it  pays  to   read  the  entire  answer  choice!)  B  is  clearly  incorrect:  the  author  does  not   passionately  declaim  in  the  first  sentence,  even  though  parts  of  the  selection  do   Take  your  S   AT  prep  to  the  next  level.  Visit  www.learnerator.com   143  

144     indeed  utilize  fiery  language.  C  is  correct  -­‐-­‐  there  is  a  pun  here  (vampires  are  dead  in   the  literal  sense,  but  they  are  also  “dead”  in  the  sense  that  they  are  culturally   irrelevant),  and  the  tone  of  the  article  is  indeed  dry  (the  quote  “vampires  have  had   their  moment  in  the  sun”  and  the  inclusion  of  “rockstars  masquerading  as  college   students”  in  a  list  of  undead/unknown  creatures  both  indicate  that  the  writer  has  a   somewhat  wry  sense  of  humor  about  this  topic.)  D  is  half-­‐correct  as  well  -­‐-­‐  there  is  a   dependent  clause  hidden  within  those  two  hyphens  -­‐-­‐  but  it  does  not  necessarily   create  a  digressive  tone,  as  the  article  stays  on  focus  the  whole  time,  demonstrating   that  vampires  were  once  culturally  relevant  and  now  no  longer  are,  or  should  be.   Don’t  be  impressed  by  D’s  “grammar-­‐speak;”  this  is  the  SAT’s  transparent  attempt  to   try  and  trick  you  into  choosing  an  answer  just  because  it  sounds  like  it  might  be   right.     Which  sentence  in  the  selection  introduces  a  shift  in  focus?   A.  “The  pale,  the  tortured,  the  lonely...”  because  it  shifts  the  focus  of  the   article  back  to  when  vampires  were  culturally  relevant.   B.  “They  are  dead…”  because  it  shifts  its  focus  from  vampires  to  other   creatures.   C.  “In  short  order…”  because  it  shifts  the  focus  of  the  article  to  writers  and   movie  producers   D.  “What  these  varyingly  successful  attempts…”  because  it  introduces  the   author’s  premise     This  is  a  difficult  question  because  it  asks  you  both  to  understand  what  the  focus  of   the  article  is  as  well  as  how  the  author  creates  a  subtle  shift  in  focus.  From  reading   the  paragraph,  it  should  be  clear  that  the  focus  of  the  article  isn’t  necessarily  going   to  remain  on  vampires,  so  you  need  to  be  able  to  make  a  logical  inference  about   where  the  focus  of  the  article  will  later  rest.     Look  at  A.  The  second  sentence  of  the  article  is  still  about  vampires,  and  in  fact   categorizes  all  these  different  types  of  vampires  in  order  to  demonstrate  that  yes,  all   Take  your  S   AT  prep  to  the  next  level.  Visit  www.learnerator.com   144  

145     of  them  are  now  culturally  irrelevant  (“their  long  dark  night  has  finally  come”),  so  A   can’t  be  correct.     B  reinforces  the  idea  that  vampires  are  dead,  and  introduces  creatures  who,  like   vampires,  command  our  attention  as  a  culture  -­‐-­‐  perhaps  signaling  a  shift  from   vampires  to  other  creatures.  B  is  actually  a  good  answer,  so  let’s  keep  it  in  our  back   pocket  until  we  read  the  other  answer  choices.  C  is  an  example  of  a  thought  that  B   introduces,  so  it  can’t  be  the  “shift,”  because  it’s  dependent  on  another  sentence.  D,   however,  is  the  best  answer  -­‐-­‐  it  shifts  away  from  vampires  and  all  the  other   supernatural  creatures  that  our  culture  is  obsessed  with,  and  establishes  a  premise  -­‐ -­‐  that  all  of  these  books  and  films  about  supernatural  creatures  are  missing  a   treasure  trove  of  literary  inspiration  (from  humans).  D  indicates  (correctly)  that  the   preceding  example  of  vampires  just  serves  to  introduce  or  set  up  the  premise  -­‐-­‐  that   the  human  psyche  could  be  a  darker/better  inspiration  for  literature  than  made-­‐up   creatures.  Though  B  is  a  good  answer,  D  is  better.     Support   Expository  texts  will  make  claims;  a  good  expository  text  will  support  those  claims.   The  SAT  will  test  your  understanding  of  how  these  supports  /  evidence  work.  For   example,  it  is  pretty  clear  from  the  following  two  examples  that  the  second  sentence   supports  the  first  by  giving  some  sort  of  reason  or  explanation  for  it:     •

I  hate  animals.  They  smell  bad.    

Don’t  be  fooled  by  sentences  like  this,  though,  which  look  like  they  are  making  a   well-­‐supported  claim:   •

The  person  who  wrote  the  above  sentence  is  clearly  a  sad,  sad  human  being.   Animals  are  awesome.    

While  both  sentences  are  true,  the  writer  doesn’t  take  the  time  to  demonstrate  the   relationship  between  them,  leaving  the  reader  to  make  the  connection.  But  in  truth,   Take  your  S   AT  prep  to  the  next  level.  Visit  www.learnerator.com   145  

146     these  are  two  disparate  clauses  that  have  nothing  to  do  with  one  another  -­‐-­‐  so  make   sure  that  the  sentences  actually  do  support  each  other.     Test  your  understanding  of  supports  with  the  following  excerpt:     Team  Voldemort  is  not  exactly  what  it  sounds  like.  This  group  of  12  fifteen-­‐ year-­‐old  girls  from  British  Columbia  has  no  interest  in  magic,  spells,  or  world   domination  -­‐-­‐  though  they  do  often  enter  the  field  with  two  red  slashes  painted   under  their  eyes.  “It  symbolizes  the  blood  of  our  enemies,”  says  Katie  Canuckie,   the  cheerful,  pigtailed  tenth-­‐grade  captain  of  the  team.  “We’re  preparing  for   victory  and  we  will  win.”  She  flashes  a  smile  that  glints  with  heavy  orthodontia.       The  award-­‐winning  girls’  lacrosse  team,  which  has  broken  three  international   records  with  its  15-­‐year  winning  streak,  was  renamed  in  2002  to  symbolize   “that  we  should  be  taken  seriously,”  quoth  Brenda  Ballyhoo,  the  team’s  coach   and  a  former  Olympic  hopeful,  who  was  behind  the  name  change.     “Look,”  she  says,  speaking  frankly,  “people  don’t  take  girls’  sports  seriously.   That  is  a  documented  fact.  How  much  attention  does  the  WNBA  receive  as   compared  to  the  NBA?  How  about  women’s  soccer  in  comparison  to  football,  or   to  the  World  Cup  nationwide?  Women’s  hockey  as  opposed  to  men’s?  Face  it:   people  don’t  care  about  girls’  sports.  Well,  we’re  here  to  change  that.  Our  name   might  be  cutesy,  but  we’re  dead  serious  about  our  mission.”  When  asked   whether  she  was  concerned  about  the  message  that  the  name  Team  Voldemort   sends,  Brenda  laughed  darkly.  “We’ll  see  who’s  concerned  when  we’re  done   with  our  season.”     Which  sentence  provides  the  best  support  for  Brenda’s  claim  that   “people  don’t  take  girls’  sports  seriously”?   A.  “Our  name  might  be  cutesy,  but  we’re  dead  serious  about  our  mission.”   B.  “We’ll  see  who’s  concerned  when  we’re  done  with  our  season.”   Take  your  S   AT  prep  to  the  next  level.  Visit  www.learnerator.com   146  

147     C.  “That’s  a  documented  fact.”   D.  “How  much  attention  does  the  WNBA  receive  as  compared  to  the  NBA?”     Ballyhoo’s  assertion  is  that  no  one  cares  about  girls’  sports.  Claims  made  in  A  and  B   about  the  “dead  serious-­‐ness”  of  Team  Voldemort  are  a  response  to  the  idea  that  “no   one  cares  about  girls’  sports,”  NOT  a  refutation  of  it.  C  is  a  plausible  choice  -­‐-­‐  but  it   doesn’t  actually  support  the  claim  because  without  actual  statistics  or  verification,   it’s  an  easy  claim  to  make,  and  it  doesn’t  need  to  be  proven  in  any  way.  The  best   answer  for  this  question  is  D,  because  it  provides  an  example  of  what  Ballyhoo  is   talking  about  -­‐-­‐  she  says  that  no  one  cares  about  girls’  sports  and  gives  a  clear   example  of  a  type  of  sport  in  which  women  are  underrepresented  or  neglected  by   the  public.     How  does  the  writer  of  the  article  support  the  assertion  that  “Team   Voldemort”  is  “not  what  it  sounds  like”?   A.  By  asking  the  coach  whether  she  is  concerned  about  the  message  implied   by  the  team’s  name   B.By  giving  statistics  about  the  team’s  victories  and  awards   C.  By  subtly  contrasting  the  aims  and  attitudes  of  the  team  with  Voldemort’s   aims   D.  By  describing  Katie  Canuckie’s  orthodontia     Often  with  “support”  questions,  the  support  for  the  assertion  will  be  directly   after  the  claim  itself  (which  gets  back  to  text  organization/structure  and   focus  -­‐-­‐  having  the  support  placed  too  far  from  the  claim  would  lead  to   confusion!)  Look  at  A  -­‐-­‐  at  the  end  of  the  article  (far  away  from  the  actual   claim  itself,  which  is  made  at  the  very  beginning  of  the  article),  the  writer   asks  the  coach  if  she  is  concerned  about  the  team’s  name.  But  this  question   has  little  to  do  with  the  fact  that  the  team  “isn’t  what  it  sounds  like.”  Giving   statistics  about  the  team’s  victories  and  awards  does  paint  the  team   positively  (in  contrast  to  the  team  name,  which  could  be  construed   Take  your  S   AT  prep  to  the  next  level.  Visit  www.learnerator.com   147  

148     negatively),  but  this  contrast  isn’t  strong  or  explicit  enough  to  provide   support  for  the  fact  that  Team  Voldemort  “isn’t  what  it  sounds  like.”  C  is   plausible,  however,  as  the  author  does  contrast  the  attitudes  of  the  team  with   Voldemort’s;  the  author  states:  “This  group…  has  no  interest  in  magic,  spells,   or  world  domination;”  s/he  may  as  well  say  “unlike  Voldemort.”  Describing   Katie’s  orthodontia  does  somewhat  subtly  set  up  a  contrast  between  this   innocent  teenage  girl  and  the  menacing  Voldemort,  but  again,  as  in  B,  this   contrast  isn’t  strong  or  explicit  enough  to  really  be  construed  as  “support”  for   this  claim.     Paragraph  to  Whole-­‐Text  Relationships   The  previous  questions  asked  about  the  relationship  between  a  sentence  and   a  paragraph.  Just  as  each  sentence  does  its  “work”  within  a  paragraph,  each   paragraph  has  a  specific  function  within  the  text  as  a  whole.  Obviously,   however,  paragraphs  within  literary  texts  will  perform  very  different   functions  than  paragraphs  within  expository  texts.  Still,  each  paragraph   should  be  doing  one  or  more  of  the  following:     •

Setting  up  a  situation  or  claim  



Supporting  OR  refuting  a  claim  



Maintaining  Focus  



Informing    

Each  function  will  come  with  signal  words.  Some  words  that  might  be  associated   with  greater  support  or  refutations  would  be:     because,  in  support  of,  therefore,  thus,  ergo,  hence,  due  to  this,  moreover,  additionally,   as  well,  also,  and,  further,  an  example  of  this  is…     Pay  attention  to  these  words;  they  will  most  likely  show  up  in  non-­‐fiction  expository   texts,  as  opposed  to  narrative  works,  but  even  narrative  works  have  signal  words   Take  your  S   AT  prep  to  the  next  level.  Visit  www.learnerator.com   148  

149     that  indicate  a  paragraph’s  function(s).  As  you  read  below,  underline  or  annotate   the  words  that  seem  to  indicate  the  function  of  the  paragraph.  The  following  excerpt   is  a  narrative  piece.   1.  I  never  considered  myself  an  immigrant,  though  I  was  with  my  parents  on  the   ten-­‐hour  Iberia  flight  from  Casablanca  to  Newark  that  took  us  farther  from   home  than  any  of  us  had  ever  been  before.  But  a  five-­‐year-­‐old  is  already  an   immigrant  anyway,  confronting  confusing  new  rules  and  codes  every  time  she   leaves  the  house;  maybe  this  was  why  I  took  to  America  so  much  more  quickly   than  either  of  my  parents.  Or  maybe  I  was  always  going  to  be  “like  an   American,”  trying  desperately  to  find  my  way  home.  My  parents’  wanderlust  -­‐-­‐   not  to  mention  the  lack  of  economic  opportunity  in  the  Old  Country  -­‐-­‐  just  took   the  guesswork  out  of  it.     2.  By  the  age  of  ten,  my  accent  had  melted  away,  my  French  was  rusty  enough   to  make  my  aunts  click  their  tongues  at  my  mother  whenever  I  attempted   transatlantic  conversation,  and  though  twice-­‐weekly  “Arabic  school”  had  kept   me  functionally  literate,  my  spelling  was  a  disaster.     3.  On  the  other  hand,  I  excelled  at  my  role  as  a  mediator  between  my  parents   and  the  phalanx  of  bureaucrats  who  controlled  seemingly  every  aspect  of   American  life.  From  parent-­‐teacher  conferences  (“Maman,  my  teacher  wants  to   speak  with  you  on  Tuesday.”  “Why?  What  have  you  done?!”  “Nothing,  I  swear!   They’re  speaking  to  everyone’s  parents!”)  to  doctor’s  visits  (“Baba,  he  says  I   need  another  shot.”  “What,  only  one?  Surely  you’re  at  risk  for  more  than  one   disease”),  I  became  an  expert  navigator  of  the  foibles,  interests,  and  various   idiocies  of  the  adult  world.     4.  By  the  time  I  was  sixteen,  my  parents’  English  was,  if  not  flawless,  then  at   least  less  fractured  than  before.  Certainly  it  was  sufficient  to  embarrass  me  at   every  college  fair  we  attended  (naturally,  my  parents  were  obsessed  with   getting  me  into  a  “good  college,”  which  to  them  meant  Harvard  and  Harvard   Take  your  S   AT  prep  to  the  next  level.  Visit  www.learnerator.com   149  

150     only,  watching  me  graduate  from  medical  school,  and  then  marrying  me  off  to   another  doctor  who  had  also  graduated  from  Harvard;  naturally,  I  was   obsessed  with  piercing  as  much  of  my  body  as  I  could  and  studying   photography.  Thus  far  I  had  not  made  these  desires  known  to  either  of  them,   but  as  we  had  learned  in  English  class,  “things  fall  apart.”  If  I  were  forced  into   another  AP  Biology  or  Chemistry  class,  the  center  would  not  only  no  longer   hold,  but  would  implode  completely.)     What  is  the  relationship  between  the  first  and  the  fourth  paragraph  of   this  text?   A.  The  fourth  paragraph  provides  a  counterexample  to  the  first   B.  The  fourth  paragraph  gives  a  thorough  analysis  of  principles  outlined  in   the  first   C.  The  fourth  paragraph  refutes  the  first  paragraph   D.  The  fourth  paragraph  expands  in  detail  on  the  general  principles  outlined   in  the  first  paragraph     Literary  or  narrative  texts  don’t  often  utilize  the  same  rhetorical  devices  and   strategies  as  other  non-­‐fiction  texts,  which  may  make  your  job  of  figuring  out  these   questions  much  easier.  After  all,  it’s  pretty  clear  that  this  writer  isn’t  trying  to   persuade  the  reader,  but  instead  to  tell  a  story.  Therefore,  words  like  “thorough   analysis,”  “counterexample,”  and  “refute”  won’t  necessarily  apply  to  works  like  this.   You  can  automatically  scratch  A  (there  is  no  example,  let  alone  a  counterexample,  to   analyze  here),  B  (there  is  no  thorough  analysis  here)  and  C  (this  kind  of  narrative   wouldn’t  be  interested  in  refuting  itself  -­‐-­‐  the  author  is  telling  a  story,  not  trying  to   make  a  claim.  It  would  be  counterintuitive  for  the  writer  to  try  and  refute   herself!)Though  D  may  not  seem  immediately  obvious,  it’s  certainly  the  best  answer   -­‐-­‐  the  general  principle  outlined  in  paragraph  one  is  the  idea  of  immigration,  of   being  a  foreigner  in  some  sense,  and  of  the  narrator  being  “an  American”  by   disposition.  By  the  fourth  paragraph,  in  which  the  narrator  is  disillusioned  with  her   parents’  ideals  and  interested  in  expressing  her  individuality,  it  should  be  clear  that   Take  your  S   AT  prep  to  the  next  level.  Visit  www.learnerator.com   150  

151     this  is  just  a  further  example  of  the  general  idea  expressed  in  paragraph  one.     Taken  in  context,  what  is  the  purpose  of  Paragraph  3?   A.  Paragraph  3  provides  examples  that  vividly  illustrate  the  narrator’s  point   B.  Paragraph  3  further  reinforces  the  points  made  in  Paragraph  2   C.  Paragraph  3  provides  an  analysis  of  the  various  ways  that  being  an   immigrant  has  changed  the  narrator   D.  Paragraph  3  provides  a  set-­‐up  for  Paragraph  4     Look  closely  at  these  answer  options  and  read  carefully.  As  per  Option  A,   paragraph  3  certainly  is  full  of  examples,  as  evidenced  by  the  multiple   parenthetical  asides  that  humorously  illustrate  the  author’s  interactions  with   her  parents  as  well  as  the  other  adults  in  her  world.  Keep  answer  A  for  now.   Look  at  B.  B  requires  that  you  also  be  familiar  with  paragraph  2,  but   Paragraph  2  simply  states  that  the  narrator  no  longer  speaks  French  and   Arabic  with  native  proficiency.  This  does  not  mean,  however,  that  the   narrator  would  automatically  be  better  equipped  to  deal  with  the  adult   American  world  than  her  parents;  it  simply  means  that  she  no  longer  speaks   good  French.  In  fact,  Paragraph  3  goes  in  a  completely  different  direction   from  Paragraph  2  -­‐-­‐  it  gives  examples  about  an  entirely  different  part  of  the   narrator’s  life,  so  it  can’t  be  taken  as  “reinforcement”  of  anything  stated  in   Paragraph  2.     C  could  be  a  good  answer,  but  again  -­‐-­‐  this  isn’t  an  analysis  so  much  as  a  list   of  examples.  Additionally,  we  don’t  know  if  this  is  a  “change”  in  the  narrator’s   personality  -­‐-­‐  all  she  has  ever  been  is  an  immigrant  -­‐-­‐  so  C  is  in  fact  a  fairly   worthless  answer  to  begin  with.     Lastly,  look  at  D;  while  it’s  tempting  to  assume  that  all  paragraphs  will  be   linearly  linked,  as  they  often  are  in  expository  texts,  this  is  obviously  not   always  the  case,  especially  with  literary  narratives.  Additionally,  Paragraph  3   Take  your  S   AT  prep  to  the  next  level.  Visit  www.learnerator.com   151  

152     does  not  set  up  Paragraph  4,  as  3  documents  how  helpful  the  narrator  is  to   her  parents,  and  how  she  acted  as  a  go-­‐between,  while  Paragraph  4  discusses   the  fact  that  she  now  wishes  to  be  her  own  person  instead  of  playing  a  pre-­‐ ordained  part.  Moreover,  Paragraph  4  starts  off  with  a  transition  (“by  the   time  I  was  16…”),  which  indicates  that  there’s  a  shift  in  perspective.  A  is  the   best  answer.     When  answering  questions  like  this,  you  must  think  critically  about  what  each   paragraph  strives  to  do,  not  just  what  each  paragraph  says.  Pay  attention  to  the   words  that  the  writer  uses;  transition  words  that  indicate  the  passing  of  time  (as  in   Paragraph  4)  or  a  change  of  opinion  or  topic  (“on  the  other  hand,”  in  Paragraph  3)   will  help  you  figure  out  whether  a  paragraph  is  setting  up,  expanding,  extending,   refuting,  or  shifting  focus  completely.     Expository  Texts   There  are  multiple  signal  words  that  help  indicate  the  purpose  of  each  paragraph.   As  you  may  remember  from  the  review  of  text  structure,  signal  and  relational  words   can  indicate  the  type  of  text  as  a  whole,  which  will  help  you  understand  what  each   paragraph  is  doing  in  service  of  the  larger  whole.  For  example,  look  at  this   chronologically  structured  text  from  the  Public  Library  of  Science  Blogs  Network,   and  figure  out  from  the  signal  words  what  purpose  each  paragraph  holds  within  the   text  as  a  whole:     1.  Within  5  years…  I  think  people  will  know  what  an  exome  is,  because   analyzing  it  will  be  as  common  as  a  CBC  or  blood  lipid  profile  is  today  before   visiting  the  doc.  As  costs  decrease  and  gene  discoveries  increase,  we’ve  reached   a  tipping  point,  by  definition  when  “a  series  of  small  changes  or  incidents   becomes  significant  enough  to  cause  a  larger,  more  important  change.”  Until   “exome”  becomes  a  household  world,  clever  studies  are  illuminating  pioneering   applications  of  the  technology.     Take  your  S   AT  prep  to  the  next  level.  Visit  www.learnerator.com   152  

153     2.  The  exome,  the  part  of  the  genome  that  encodes  protein,  harbors  85%  of   disease-­‐causing  gene  variants  (we’re  not  supposed  to  say  “mutation”  anymore,   but  that’s  what  I  mean).  Results  from  several  large  studies  have  been  published   over  the  past  3  years,  but  a  paper  in  last  week’s  Science  Translational   Medicine  from  Stephen  Kingsmore’s  group  at  Children’s  Mercy–Kansas  City   offers  the  most  promising  results  yet.     3.  “It  heralds  the  dawning  of  the  new  age  of  clinical  genetics.  We’ve  been   waiting  for  this  to  come  around  for  10  to  15  years,  and  it’s  finally  here,”  says   Robert  Marion,  MD,  chief  of  the  division  of  genetics  at  The  Children’s  Hospital   at  Montefiore  and  a  developmental  pediatrician  at  the  Albert  Einstein  College   of  Medicine,  about  the  paper  (he’s  not  part  of  the  team).  I  devoured  his  book   “Genetic-­‐Rounds:  A  Doctor’s  Encounters  in  the  Field  that  Revolutionized-­‐ Medicine.”     4.    Last  month,  the  Journal  of  the  American  Medical  Association  published   findings  of  two  ongoing  prospective  exome  sequencing  studies  of  individuals   with  symptoms  suggesting  an  inherited  condition.  A  group   from  UCLA  diagnosed  213  of  814  (26%)  cases  that  hadn’t  been  diagnosed   clinically  or  with  single-­‐gene  tests  or  panels.  The  26%  rose  to  31%  if  parents   had  their  exomes  sequenced  too.  The  second  report,  from  Baylor  College  of   Medicine,  diagnosed  504  of  2000  (25.2%)  patients.  Both  studies  weren’t  just   children.     As  you  may  have  noticed,  there  are  multiple  words  here  that  signal  that  this  is  a   chronological  text  (within  5  years,  over  the  past  3  years,  heralds  the  dawning  of  a   new  age,  last  month,  etc.);  knowing  that  this  is  a  chronological  text  may  help  you   understand  the  author’s  purpose  and  therefore  why  she  chose  to  structure  this   essay  the  way  she  did.   What  is  the  function  of  paragraph  2  within  the  larger  text?   A.  Paragraph  2  illustrates  the  author’s  larger  point   Take  your  S   AT  prep  to  the  next  level.  Visit  www.learnerator.com   153  

154     B.  Paragraph  2  provides  a  thesis  for  the  rest  of  the  text   C.  Paragraph  2  introduces  a  change  in  focus  from  Paragraph  1   D.    Paragraph  2  summarizes  the  rest  of  the  article     Look  at  option  A.  To  rule  it  out,  you  need  to  know  what  the  author’s  larger  point  is.   Because  this  is  an  expository  text,  there  is  a  good  chance  that  this  thesis  statement   resides  somewhere  in  the  first  paragraph;  sure  enough,  the  first  paragraph  is  home   to  the  author’s  claim  that  “exome”  will  soon  become  a  household  word  thanks  to  its   importance  to  medical  science.  Therefore,  we  now  need  to  see  whether  Paragraph  2   illustrates  this  claim.  Paragraph  2  gives  a  definition  for  the  word  and  highlights  its   importance  to  medical  science  based  on  the  past  few  studies  on  exomes.  This  does   help  illustrate  that  exome  may  become  a  household  word  (there  have  been  many   studies  and  apparently  many  medically  important  findings),  but  before  we  settle  on   this  answer,  let’s  examine  the  other  options.     B  is  clearly  untrue;  the  thesis  is  found  in  Paragraph  1.  Paragraph  2  provides   illustration  for  a  thesis.  Scratch  B.   You  can  scratch  C,  too.  There  is  no  shift  in  focus  -­‐-­‐  it’s  a  continuation  of  the  focus.   Nor  can  D  be  true;  the  second  paragraph  in  a  four-­‐paragraph  article  certainly  can’t   summarize  the  rest  of  the  article,  since  the  article  has  barely  begun  by  Paragraph  2.   Clearly,  A  is  the  best  answer.     Whether  or  not  the  chronological  signal  words  helped  you  assess  the  function  of   each  paragraph,  it’s  still  a  good  idea  to  make  sure  you  understand  text  structure  as   this  can  potentially  influence  the  rest  of  the  text.     The  new  SAT  may  not  necessarily  word  its  questions  so  bluntly;  rather  than  ask  you   the  “purpose”  or  “function”  of  each  paragraph  in  a  text,  the  test  may  ask  about  the   way  the  author  “creates  an  argument,”  “structures  his/her  claim,”  “reinforces   his/her  position,”  “builds  on  the  argument,”  “refutes  the  argument,”  or  similar.   These  are  really  just  forms  of  asking  you  “What  is  the  purpose  of  this  paragraph”   Take  your  S   AT  prep  to  the  next  level.  Visit  www.learnerator.com   154  

155     since  it  is  through  the  organization  and  structure  of  each  individual  paragraph  that   writers  most  effectively  structure  and  word  their  arguments.  

   

Take  your  S   AT  prep  to  the  next  level.  Visit  www.learnerator.com   155  

156    

Analyzing Point of View A  text’s  “point  of  view”  is  the  perspective  from  which  it  is  written.  Tone,  stance,   opinion:  all  of  these  are  part  and  parcel  of  the  concept  of  “point  of  view.”  Not  every   piece  takes  a  perspective  or  point  of  view;  think  about  articles  that  strive  for   journalistic  objectivity,  whose  strength  is  in  the  fact  that  they  present  ideas  and   events  without  extraneous  commentary  or  obvious  emotion.     Perhaps  in  middle  school  you  learned  that  every  text  is  written  from  a  specific   “point  of  view”  (first  person,  second  person,  third  person).  This  is  an  extension  of   that  concept,  focusing  on  the  idea  that  HOW  a  piece  is  written  (i.e.  its  form  -­‐-­‐  the   perspective  it  takes,  the  point  of  view  it  espouses)  will  clearly  influence  its  content.     Each  point  of  view  can  be  classified  into  three  broad  categories:     1. Positive   2. Negative   3. Neutral     Of  course,  these  aren’t  particularly  nuanced  designations;  there  are  a  multitude  of   ways  in  which  a  point  of  view  can  be  “positive”  or  “negative.”  However,   understanding  the  broad  category  into  which  a  point  of  view  fits  will  help  you   understand  it  on  a  more  nuanced  and  detailed  level.     As  discussed  in  this  article,  signal  words  will  help  you  understand  what  an  author  is   trying  to  impart.  Perspective  is  closely  related  to  tone,  so  pay  attention  to  signal   words  that  illustrate  how  an  author  feels  about  the  topic.  Try  to  assess  the  following   piece,  from  Vox.com,  based  on  specific  signal  words  that  demonstrate  how  the   author  feels  about  the  topic  at  hand.     Take  your  S   AT  prep  to  the  next  level.  Visit  www.learnerator.com   156  

157     Every  day  seems  to  turn  up  opportunities  to  abuse  science  in  new  and  perverse   ways,  especially  when  it  comes  to  health.  You  open  a  newspaper  or  news  site,   and  you  read  about  a  health  claim  making  the  rounds:  a  diet  that  will  give  you   the  energy  of  a  teenager,  an  exercise  routine  that  will  elongate  your  legs,  a   policy  that  will  protect  Americans  from  scary  viruses.  Many  of  these  claims  —   even  the  ones  that  come  from  the  lips  of  the  most  esteemed  doctors  and  public   officials  —  aren't  backed  by  any  good  evidence.  Some  even  run  in  the  opposite   direction  of  what  the  best-­‐available  evidence  tells  us.  In  the  interest  of  the   correcting  the  record,  we  rounded  up  the  most  egregious  abuses  of  health   science  in  2014.     What  are  some  words  that  signal  the  author’s  perspective  on  these  health  claims?   Well,  we  have  the  sarcastic  phrases  “a  diet  that  will  give  you  the  energy  of  a   teenager,"  "an  exercise  routine  that  will  elongate  your  legs,"  "a  policy  that  will   protect  Americans  from  scary  viruses.”  All  of  these  are  semi-­‐sarcastic  indications   that  the  author  thinks  that  these  claims  are  silly  -­‐-­‐  clearly,  if  you  use  logic,  it  is  easy   to  see  that  no  diet  can  actually  elongate  your  legs.  But  perhaps  more  obvious  are  the   following  phrases  “aren’t  backed  by  any  good  evidence”  “run  in  the  opposite   direction  of  what  the  best-­‐available  evidence  tells  us.”  And  finally,  “most  egregious   abuses  of  health  science”  should  indicate  pretty  clearly  that  what  is  about  to  follow   is,  in  the  author’s  opinion,  arrant  nonsense.  All  of  this  is  established  by  certain  signal   words,  which  establish  the  tone,  which  make  it  pretty  clear  what  the  author’s   perspective  is.  Broadly  categorized,  it’s  obvious  that  the  perspective  is  negative.  But   within  that  negativity  lie  shades  and  nuances,  as  indicated  in  the  following  question:     The  author’s  perspective  in  this  piece  is  that  of…   A.  A  social  reformer  on  a  crusade   B.A  medical  professional  dispensing  advice   C.A  scholar  exposing  bedrocks  of  hypocrisy   D.  An  observer  interested  in  scientific  integrity     Take  your  S   AT  prep  to  the  next  level.  Visit  www.learnerator.com   157  

158     Now,  it  becomes  quite  clear  from  these  answer  choices  that  these  answer  choices   reach  far  beyond  the  “negative,”  “positive,”  and  “neutral”  designations.  So  examine   them  closely.  Option  A  is  tempting  -­‐-­‐  after  all,  this  writer  is  very  unimpressed  with   the  “egregious  abuses  of  health  science”  committed  in  2014.  And  one  could  argue   that  correcting  these  misimpressions  is  a  type  of  “social  reform.”  So  though  this  isn’t   a  perfect  analogy,  keep  it  for  now.  B  is  tempting  too;  however,  there  is  no  evidence   that  this  writer  is  a  medical  professional,  nor  is  there  any  advice  being  dispensed  -­‐-­‐   only  facts  (presumably).  Be  careful  of  choices  like  these  and  again,  pay  attention  to   the  nuance.  The  third  option  is  also  tempting  -­‐-­‐  however,  there  isn’t  any  “hypocrisy”   here,  just  misinformation.  And  again,  there’s  no  indication  that  this  person  is  any   sort  of  scholar.  Finally,  D  is  tempting  -­‐-­‐  “observer”  is  general  enough  to  be  true   (clearly,  this  person  did  observe  what  s/he  feels  to  be  “egregious  abuses  of  health   science”)  without  inserting  any  background  or  credential  that  isn’t  stated  in  the  text   (i.e.  “medical  professional”  or  “scholar”),  and  the  second  part  works  as  well  -­‐-­‐   correcting  this  misinformation  is  indeed  a  form  of  pursuing  “scientific  integrity.”  So   between  A  and  D,  think  about  which  is  more  accurate  -­‐-­‐  there’s  more  evidence  for   the  pursuit  of  scientific  integrity  than  for  the  “crusade”  for  various  social  reformers.     The  SAT  will  ask  you  about  the  perspectives  of  multiple  kinds  of  texts.  Try  this  one,   from  a  New  York  Times  entitled  “Should  You  Have  Things:”     “...the  loss  of  possessions,  ones  deeply  associated  with  the  self,  can  cause  real   grief.”   At  The  New  Yorker,  Allen  Kurzweil  writes  about  one  such  possession  —  an   Omega  Seamaster  watch  that  had  belonged  to  his  father,  who  died  when  he   was  5.  The  watch  fell  victim,  he  believes,  to  the  influence  of  his  boarding-­‐school   bully,  Cesar  Augusto  Viana:   “Within  the  week,  his  henchman  admitted  that  he’d  hurled  my  watch  off  a   balcony  on  a  dare.  I  ran  down  the  stairs,  dashed  outside,  and  dug  through  knee-­‐ deep  snow  until  my  fingers  turned  white  and  tingly.  The  watch  never  surfaced.   The  loss  left  me  more  than  bereft.  I  felt  annihilated.”   Take  your  S   AT  prep  to  the  next  level.  Visit  www.learnerator.com   158  

159     Much  later,  after  he’s  confronted  Mr.  Viana  as  an  adult,  Mr.  Kurzweil  realizes:   “My  father’s  Omega  turned  out  to  be  more  than  a  talisman.  It  was  a  time   machine  that  had  transported  me  back  to  a  moment  when  my  family  was  intact   and  I  was  happy.”  And,  he  writes:  “When  I  told  my  wife  and  son  that  I  was   banishing  Cesar  from  our  lives,  they  celebrated  his  eviction  by  giving  me  an   extravagant  gift.  I  am  wearing  it  on  my  wrist.”   Experiences,  good  or  bad,  may  become  stories,  but  Mr.  Kurzweil’s  account…   suggests  that  things  may  have  a  narrative  power,  too.  They  may  help  us  tell  the   stories  of  our  lives,  of  what  we’ve  lost  and  what  we’ve  gained.     This  is  a  slightly  more  challenging  text  to  parse,  as  the  author  does  not  necessarily   offer  his  or  her  own  perspective  -­‐-­‐  at  least  not  a  strongly-­‐worded  one.  But  if  you   read  carefully,  paying  attention  to  text  structure  (i.e.  differentiating  between  the   quotes  from  outside  sources  and  the  perspective  of  the  writer  his  or  herself),  and   highlighting  important  signal  words,  you’ll  be  able  to  accurately  answer  this   question:     The  writer’s  perspective  on  material  objects  is  that…   A.  Attachment  to  material  objects  signals  an  underdeveloped  sense  of  self   B.  Attachment  to  material  objects  encourages  a  sense  of  victimhood   C.  We  must  be  careful  of  the  power  of  our  attachment  to  material  objects   D.  Attachment  to  material  objects  is  psychologically  powerful  for  a  reason     This  will  be  easy  to  answer  if  you  pay  attention  to  the  fact  that  the  writer  is  careful   not  to  express  a  strongly  worded  opinion;  instead,  s/he  allows  the  sources  and  the   quotes  to  speak  for  themselves.  Nowhere  does  the  writer  indicate  that  s/he  believes   that  A  is  correct,  and  while  B  clearly  plays  on  the  story  of  bullying  described  in  the   third  paragraph,  this  story  of  bullying  itself  was  not  motivated  by  the  material   object,  nor  was  the  attachment  to  the  object  the  reason  for  Mr.  Kurzweil’s  sense  of   victimhood.  C  is  somewhat  plausible,  as  the  author  does  state  in  the  last  paragraph   that  material  objects  have  a  “narrative  power,”  but  the  author  doesn’t  indicate  that   Take  your  S   AT  prep  to  the  next  level.  Visit  www.learnerator.com   159  

160     this  is  a  power  that  should  cause  us  to  be  wary  or  careful.  Instead,  D  is  clearly  the   best  answer,  as  it  indicates  that  these  material  objects  do  have  power  and  that  this   power  does  have  a  reason  (“they  may  help  us  tell  the  stories  of  our  lives.”)  As  you   can  see,  perspective  isn’t  always  indicated  in  strongly  worded  rants;  it  is  just  one  of   the  ways  that  writers  can  frame  or  illustrate  their  ideas.     Understanding  point  of  view  requires  a  fair  amount  of  reading  comprehension.  You   must  be  comfortable  with  analyzing  word  choice  and  text  structure,  with  isolating   details  from  the  text,  and  with  citing  textual  evidence  for  the  correct  answer.   Additionally,  you  must  read  the  text  carefully  in  order  to  differentiate  between  the   author’s  perspective  and  that  of  the  figures  involved  in  the  narrative.  Finally,  the   SAT  will  present  a  multitude  of  seemingly  plausible  choices  when  it  asks  you  to   parse  the  particular  perspective  of  a  piece.  Try  assessing  the  perspectives  found  in   the  following  article,  excerpted  from  The  Atlantic,  which  presents  multiple   perspectives:     And  yet,  on  Tuesday  morning,  the  nuns  and  priests  who  presented  the  findings   of  [a  report  about  American  nuns]  seemed  hopeful—for  the  future  of  American   nuns,  but  also  for  the  relationship  between  U.S.  sisters  and  the  Vatican,  which   has  been  strained.  The  report  recognizes  some  women's  "perception  of  not   having  enough  input  into  pastoral  decisions  which  affect  them  or  about  which   they  have  considerable  experience  and  expertise."  And  the  Vatican  let  [a  nun]   fully  design  and  execute  the  report,  which  other  [nuns]  appreciated:  When   representatives  visited  her  congregation,  Holland  said,  "it  was  evident  that   these  were  sisters  like  us,  to  whom  we  could  speak  openly  and  honestly."     Holland's  reaction  is  particularly  revealing,  because  she  speaks  for  an   organization  of  American  sisters  that  has  long  been  criticized  by  the   Vatican.  [Nuns]  in  the  United  States  are  represented  by  two  organizations:  The   Leadership  Conference  of  Women  Religious,  or  LCWR,  an  association  of   congregation  leaders  that  represents  about  80  percent  of  American  sisters;  and   Take  your  S   AT  prep  to  the  next  level.  Visit  www.learnerator.com   160  

161     the  Council  of  Major  Superiors  of  Women  Religious,  or  CMSWR,  which   represents  the  other  20  percent.  The  Council  is  typically  characterized  as  more   "conservative,"  or  traditional:  Most  of  the  women  it  represents  wear  habits,  and   some  live  in  cloisters.  The  LCWR,  on  the  other  hand,  represents  many   congregations  whose  members  wear  street  clothes  and  work  in  their  local   communities.     The  chairwoman  of  the  more  conservative  CMSWR,  Mother  Agnes  Mary   Donovan,  was  very  positive  about  the  report,  saying  the  information-­‐gathering   process  "was  overwhelmingly  a  beautiful  experience."  She  noted  that  the   average  age  of  women  religious  represented  by  the  CMSWR  is  53,  which  is  "well   below  the  average  trend."  Nearly  1,000  women  are  preparing  to  take  their  final   vows  into  CMSWR  organizations,  which  she  said  was  a  good  sign  for   recruitment.     But  the  report  warned  that  the  less  traditional  lifestyles  of  the  majority  of  U.S.   congregations  might  be  a  turn-­‐off  for  women  who  are  interested  in  religious   life.  "Candidates  often  desire  the  experience  of  living  in  formative  communities   and  many  wish  to  be  externally  recognizable  as  consecrated  women  [by   wearing  habits],"  it  says.  "This  is  a  particular  challenge  in  institutes  whose   current  lifestyle  does  not  emphasize  these  aspects  of  religious  life."     Among  the  women  who  live  in  those  less  traditional  congregations,  the  report   will  likely  be  met  with  mixed  reactions.  As  Holland  said  during  the  press   conference,  "The  visitation  was  met  by  some  ...  with  suspicion  and   apprehension."  Not  all  congregations  agreed  to  participate  in  the  study,  which   representatives  of  the  Vatican  called  "a  painful  disappointment."  And  this   report  says  nothing  about  the  ongoing  investigation  into  the  activities  and   teachings  of  the  LCWR,  which  "often  contradict  or  ignore  magisterial   teaching,"  as  a  2012  Church  document  put  it.     Take  your  S   AT  prep  to  the  next  level.  Visit  www.learnerator.com   161  

162     Although  there  may  be  hope  for  healing  between  American  nuns  and  the   Vatican,  and  although  there's  clearly  a  strong  will  to  address  the  issues  facing   women  religious,  this  report  mostly  surfaces  issues,  rather  than  resolving  them.     "I'm  concerned  about  those  who  may  still  be  angry,"  Holland  said."     Now,  this  piece  presents  multiple  perspectives  -­‐-­‐  as  well  as  a  shifting  timeline  -­‐-­‐  on   one  report.  Because  this  is  the  excerpt  to  a  larger  piece,  and  because  this  text  details   the  lifestyles  of  a  particular  subculture  that  may  not  be  known  to  you,  this  may  be  a   difficult  text  to  parse.  But  if  you  annotate  the  text,  keeping  the  different  players   straight,  then  you  should  be  able  to  answer  the  following  questions  without  an  issue.   First,  read  the  questions  and  see  whose  perspectives  the  question  is  asking  about;   then,  go  back  to  the  text  and  underline,  star,  or  color-­‐code  the  particular  players  that   the  article  is  asking  you  about.  This  should  make  it  easier  to  keep  track  of  who  is   speaking,  and  what  their  particular  perspective  is.     Based  on  the  article,  the  majority  of  American  nuns  would  take  which   perspective  on  the  results  of  this  report?   A.  Most  American  nuns  would  react  with  “painful  disappointment”   B.  Most  American  nuns  would  have  no  unified  perspective  on  this  report   C.  Most  American  nuns  would  be  angry  about  this  report   D.  Most  American  nuns  would  be  very  positive  about  this  report     In  order  to  answer  this  question,  you  must  understand  who  this  “majority”  of   American  nuns  is.  After  all,  there  are  two  major  groups  mentioned:  "[Nuns]  in  the   United  States  are  represented  by  two  organizations:  The  Leadership  Conference  of   Women  Religious,  or  LCWR,  an  association  of  congregation  leaders  that  represents   about  80  percent  of  American  sisters;  and  the  Council  of  Major  Superiors  of  Women   Religious,  or  CMSWR,  which  represents  the  other  20  percent.  The  Council  is  typically   characterized  as  more  "conservative,"  or  traditional:  Most  of  the  women  it  represents   wear  habits,  and  some  live  in  cloisters.  The  LCWR,  on  the  other  hand,  represents  many   Take  your  S   AT  prep  to  the  next  level.  Visit  www.learnerator.com   162  

163     congregations  whose  members  wear  street  clothes  and  work  in  their  local   communities."     Clearly,  the  article  is  asking  you  about  that  80%,  i.e.  the  slightly  “less  traditional”   group  of  sisters  who  wear  street  clothes  (as  opposed  to  habits)  and  work  in  their   local  communities.  This  automatically  eliminates  any  answer  choice  that  gives  the   perspective  of  that  20%  of  “more  traditional”  nuns.  Now  that  you  recognize  this,   read  answer  A.  The  article  does  mention  the  words  “painful  disappointment,”  but   this  isn’t  part  of  the  perspective  of  the  nuns  -­‐-­‐  instead,  it’s  the  reaction  of  the  Vatican   to  some  congregations’  unwillingness  to  participate  in  this  survey.  Scratch  A.     B  claims  that  most  nuns  would  have  no  unified  perspective.  If  you  read  the  article   closely,  you’ll  see  that  Holland,  who  seems  to  speak  for  that  80%  of  “less  traditional”   sisters,  at  times  is  hopeful  (“other  nuns  appreciated;”  “it  was  obvious  these  were   sisters  like  us  and  we  could  speak  honestly  and  openly;”  “the  visit  was  met  by  some   with  suspicion  and  apprehension;”  “I’m  concerned  about  those  who  may  still  be   angry.”)  Most  tellingly,  the  article  states  that  these  “less  traditional”  nuns  -­‐-­‐  i.e.,   eighty  percent  of  nuns  in  America,  i.e.  the  majority  that  this  question  is  asking  about   -­‐-­‐  will  have  “mixed  reactions”  to  this  report.  Taking  all  of  these  factors  into   consideration,  it  is  clearly  difficult  to  find  a  unified  perspective  here  that  all   American  nuns  would  espouse.  B  is  clearly  the  correct  answer.     C  claims  that  “most  American  nuns  would  be  angry,”  but  there’s  no  evidence  for  that   from  the  text  -­‐-­‐  the  text  does  claim  that  some  might  be  angry,  but  this  isn’t  the   majority  of  American  sisters.  D  claims  that  these  nuns  would  feel  very  positive  about   the  report,  but  again,  there  is  no  evidence  for  this  in  the  text;  it  is  the  minority  of   American  nuns  (that  20%  of  “more  conservative”  nuns)  who  would  probably  feel   “very  positive”  about  the  report,  considering  that  their  spokeswoman  called  the   experience  of  gathering  information  for  the  report  “overwhelmingly  a  beautiful   experience.”     Take  your  S   AT  prep  to  the  next  level.  Visit  www.learnerator.com   163  

164     This  was  a  hard  question  to  answer,  because  it  asks  you  to  understand  each  part  of   the  text  as  well  as  to  rely  on  inference  and  nuance  rather  than  the  more  obvious   triggers  of  strong  language,  description,  and  tone.     Based  on  the  article,  most  American  nuns  take  which  of  the  following   perspectives  on  the  Vatican?   A.  They  are  above  all  devoted  to  the  Vatican’s  strictures   B.  They  have  entirely  broken  from  the  Vatican’s  decrees   C.  They  have  an  uneasy  relationship  to  the  Vatican’s  authority   D.  They  remain  objectively  neutral  to  the  Vatican     Remember,  this  question  asks  you  about  “most”  American  nuns,  so  be  sure  that  you   are  paying  attention  to  that  80%  of  nuns  instead  of  the  20%.  If  you  read  the  article   carefully,  you’ll  see  that  “the  relationship  [between  the  two]  has  been  strained,”  and   that  many  American  sisters  have  been  “criticized  by  the  Vatican”  for  “contradicting   and  ignoring  magisterial  teachings.”  Moreover,  some  congregations  refused  to   participate  in  this  Vatican-­‐sponsored  fact-­‐finding  mission,  clearly  indicating  that   they  have  little  interest  in  helping  the  Vatican.  All  of  this  helps  to  get  rid  of  Option  A.   Option  B  clearly  can’t  be  true  since  many  of  these  women  DID  participate  in  this   Vatican  survey,  and  had  “mixed  feelings”  towards  it  -­‐-­‐  if  they  have  entirely  broken   from  the  Vatican,  surely  they  wouldn’t  have  cared  about  “healing”  their  relationship   to  the  institution.  C  surely  seems  to  be  true,  whereas  D  is  clearly  false.  Clearly,  C  is   the  only  correct  answer.     Determining  point  of  view  is  about  more  than  just  figuring  out  if  the  article  is   written  in  first,  second,  or  third  person.  Signal  words,  adjectives,  and  other   descriptions  can  help  you  figure  out  the  perspective  espoused  by  the  writer  or  by   one  of  his/her  sources,  but  it’s  also  important  to  remember  that  you  must  bear  in   mind  text  structure  and  organization  when  reading  to  discover  perspectives,  as   these  will  help  you  follow  along  and  understand  the  multiple  perspectives  that  may   be  contained  in  a  single  text. Take  your  S   AT  prep  to  the  next  level.  Visit  www.learnerator.com   164  

165    

Analyzing Text Purpose   Every  text  you’ll  find  on  the  SAT  has  been  written  for  a  purpose.  Perhaps  you   learned  in  middle  school  that  all  texts  were  written  for  one  of  three  purposes:     1. To  inform   2. To  persuade   3. To  entertain     This  is  true,  broadly  speaking,  but  again,  it  isn’t  a  very  nuanced  way  of  thinking   about  text  purpose.  Within  “to  inform”  or  “to  entertain”  lie  a  multitude  of  other,   more  precise  reasons  why  a  writer  may  have  chosen  to  write  a  text.  Since  sadly   most  texts  don’t  come  with  a  disclaimer  stating  “THIS  IS  MY  PURPOSE,”  it  is  up  to   the  reader  to  determine  the  author’s  purpose  in  writing  a  text  or  including  a  specific   detail  in  said  text.  In  fact,  you  will  be  using  the  same  skills  in  parsing  an  author’s   purpose  as  you  did  while  analyzing  part-­‐whole  relationships  and  assessing  the   purpose  of  a  particular  sentence  or  paragraph  within  a  larger  text.     As  always,  signal  words  that  demonstrate  a  personal  connection  to  the  text  are   especially  important.  Additionally,  pay  attention  to  contextual  information,  either   included  in  the  text  of  directly  preceding  it  that  may  reveal  the  text’s  purpose.  For   example,  just  the  fact  that  this  particular  article  is  hosted  on  an  education  blog,   under  a  page  called  SAT  Review,  should  tell  you  that  the  purpose  of  this  article  is  to   help  you  study  for  the  SATs.  Using  contextual  information  as  well  as  tone,  assess  the   following  text’s  purpose,  paying  attention  to  the  tone  as  well  as  to  any  contextual   information:     “We  Shall  Fight  on  the  Beaches,”  by  Winston  Churchill,  June  1940:   I  have,  myself,  full  confidence  that  if  all  do  their  duty,  if  nothing  is  neglected,   and  if  the  best  arrangements  are  made,  as  they  are  being  made,  we  shall  prove   ourselves  once  again  able  to  defend  our  Island  home,  to  ride  out  the  storm  of   Take  your  S   AT  prep  to  the  next  level.  Visit  www.learnerator.com   165  

166     war,  and  to  outlive  the  menace  of  tyranny,  if  necessary  for  years,  if  necessary   alone.  At  any  rate,  that  is  what  we  are  going  to  try  to  do.  That  is  the  resolve  of   His  Majesty’s  Government-­‐every  man  of  them.  That  is  the  will  of  Parliament  and   the  nation.  The  British  Empire  and  the  French  Republic,  linked  together  in  their   cause  and  in  their  need,  will  defend  to  the  death  their  native  soil,  aiding  each   other  like  good  comrades  to  the  utmost  of  their  strength.  Even  though  large   tracts  of  Europe  and  many  old  and  famous  States  have  fallen  or  may  fall  into   the  grip  of  the  Gestapo  and  all  the  odious  apparatus  of  Nazi  rule,  we  shall  not   flag  or  fail.  We  shall  go  on  to  the  end,  we  shall  fight  in  France,  we  shall  fight  on   the  seas  and  oceans,  we  shall  fight  with  growing  confidence  and  growing   strength  in  the  air,  we  shall  defend  our  Island,  whatever  the  cost  may  be,  we   shall  fight  on  the  beaches,  we  shall  fight  on  the  landing  grounds,  we  shall  fight   in  the  fields  and  in  the  streets,  we  shall  fight  in  the  hills;  we  shall  never   surrender,  and  even  if,  which  I  do  not  for  a  moment  believe,  this  Island  or  a   large  part  of  it  were  subjugated  and  starving,  then  our  Empire  beyond  the  seas,   armed  and  guarded  by  the  British  Fleet,  would  carry  on  the  struggle,  until,  in   God’s  good  time,  the  New  World,  with  all  its  power  and  might,  steps  forth  to  the   rescue  and  the  liberation  of  the  old.     The  purpose  of  this  speech  was  most  likely:   A.A  declaration  of  independence   B.  The  setting  of  a  new  agenda  for  the  nation   C.  An  inspirational  battle  cry   D.  The  pillorying  of  a  public  enemy     From  context,  you  should  have  figured  out  that  this  speech  was  given  sometime   during  WWII.  Knowing  this,  you  can  easily  scratch  A  -­‐-­‐  this  isn’t  a  declaration  of   independence  so  much  as  a  call  to  war.  B  is  also  uninspiring  -­‐-­‐  while  Churchill  is   setting  an  agenda  of  war  for  the  nation  with  his  insistence  that  “we  shall  not  flag  or   fail….  we  shall  fight  on  the  seas  and  oceans…”  his  passionate  declamation  is  at  odds   with  the  dry,  corporate  tone  of  B.  C  is  certainly  correct,  as  evidenced  by  Churchill’s   Take  your  S   AT  prep  to  the  next  level.  Visit  www.learnerator.com   166  

167     obvious  passion  and  fiery  language  (“we  shall  never  surrender…  even  if…  this   Island…  were  subjugated…  then  our  empire  beyond  the  seas  would  carry  on  the   struggle…”)  Clearly,  he  is  attempting  to  inspire  his  people  and  assure  them  of  their   victory  over  the  Nazis.  And  while  he  clearly  has  no  love  for  the  Nazi  regime,  this   speech  is  less  concerned  with  pillorying  (criticizing)  this  public  enemy  than   inspiring  the  British  population.     Every  text,  even  the  driest  one,  has  some  kind  of  purpose;  sometimes,  as  with  this   text,  paying  attention  to  the  intended  audience  will  also  help  you  to  understand  the   purpose  of  a  text.  This  speech  was  given  at  the  House  of  Commons  but  likely   broadcast  on  the  wireless,  so  it’s  safe  to  assume  that  the  audience  was  the  entire   population  of  England.  When  the  audience  is  that  large,  the  writer’s  purpose  will   clearly  be  different  from  the  his/her  purpose  when  addressing  a  small,  intimate   gathering.     For  example,  try  to  assess  the  audience  and  the  purpose  of  the  following  excerpt,   given  by  First  Lady  Michelle  Obama  at  the  Brookings  Institute  in  December  2014:     And  then  there  are  the  issues  of  quality  and  value,  which  are  similarly  complex.   As  you  know,  when  deciding  whether  it’s  worth  sending  their  girls  to  school,   parents  aren’t  just  asking  themselves,  will  this  be  a  good  experience  for  my   daughter,  they’re  calculating  what  those  school  fees  will  mean  for  their  family’s   food  budget,  they’re  contemplating  the  loss  of  household  help  that  is  critical  to   the  survival  of  that  family.  So  they  want  to  see  real  evidence  that  their   daughter  is  learning  real,  marketable  skills  –-­‐  things  like  literacy,  numeracy,   vocational  skills  that  will  help  her  provide  for  herself  and,  ultimately,  her   family.     That’s  the  kind  of  bar  that  we  need  to  clear  as  we  move  forward.  Because  in  our   work  to  educate  girls,  especially  adolescent  girls,  we’re  often  asking  families  to   do  what  seems  to  be  in  the  exact  opposite  of  their  daughters’  and  their  families’   Take  your  S   AT  prep  to  the  next  level.  Visit  www.learnerator.com   167  

168     best  interests.  Often,  we’re  asking  them  to  change  or  disregard  some  of  their   most  strongly  held  values  and  traditions.     So,  yes,  we  need  more  infrastructure.  We  need  more  resources.  And,  yes,  we   need  more  good  laws  and  policies  -­‐–  those  are  absolutely  the  necessary  building   blocks  for  change.  But  we  also  need  buy-­‐in  from  those  families  and  those   communities.  We  need  parents  to  actually  believe  that  their  daughters  are  as   worthy  of  an  education  as  their  sons,  and  that  sending  girls  to  school  is  a  good   investment  for  their  future.     And  that  might  take  some  real  effort  on  the  ground  to  actually  understand   people’s  concerns,  to  gain  their  trust,  to  determine  what  resources  they  need  to   make  the  sacrifice  of  educating  their  daughters.   What  is  First  Lady  Obama’s  purpose  in  giving  this  talk?   A.  To  inform  the  nation  about  a  major  problem  facing  girls  and  women   worldwide.   B.  To  criticize  the  mishandling  of  an  international  crisis.   C.  To  persuade  the  members  of  this  institute  to  contribute  funds  for  the   amelioration  of  this  issue.   D.  To  spur  greater  involvement  among  key  players  poised  to  redress  the   issue  of  girls’  neglected  educations  worldwide     Remember,  the  context  of  this  speech  is  given  to  you:  First  Lady  Obama  is  speaking   at  the  Brookings  Institute.  Even  if  you  don’t  know  that  the  Institute  is  a  think  tank   based  in  Washington  D.C.,  it  should  be  clear  that  this  isn’t  a  State  of  the  Union   address  aimed  at  the  entirety  of  the  nation,  but  a  talk  given  to  a  limited  number  of   members  of  the  American  public.     Therefore  you  can  scratch  A  immediately  -­‐-­‐  the  First  Lady  is  not  informing  “the   nation”  but  instead  addressing  a  group  of  people  who  are  likely  already  acquainted   with  the  depths  of  this  issue.  Nor  does  B  have  any  basis  in  the  text  -­‐-­‐  there  is  no   Take  your  S   AT  prep  to  the  next  level.  Visit  www.learnerator.com   168  

169     criticism,  nor  any  indication  that  the  First  Lady  believes  that  there  has  been  a   “mishandling”  of  the  crisis.  C  isn’t  particularly  convincing  either  -­‐-­‐  though  the  First   Lady  does  detail  some  of  the  challenges  that  girls  worldwide  face,  nowhere  does  she   ask  for  more  funds  to  address  these  issues.  Clearly,  D  is  the  only  correct  answer  -­‐-­‐   the  First  Lady  succinctly  highlights  some  of  the  issues  that  girls  worldwide  face,  as   well  as  possible  avenues  to  take  when  addressing  those  problems.  And  the  fact  that   she  is  giving  this  speech  to  a  small  number  of  people  at  a  think  tank  clearly  indicates   that  these  are  the  people  who  have  some  sort  of  influence  and  ways  to  deal  with  this   problem.  So  while  this  certainly  falls  under  the  broad  umbrella  of  “informing,”  with   a  slight  tinge  of  “persuading,”  this  speech  is  clearly  far  more  nuanced  than  either  of   those  designations  can  adequately  capture.     The  SAT  will  also  test  your  understanding  of  the  purpose  of  literary  narratives.   Obviously,  a  literary  narrative’s  purpose  will  always  be,  on  some  level,  to   “entertain.”  But  when  reading,  try  to  go  beyond  the  idea  of  literature  as   entertainment.  Delve  into  the  context  and  pay  attention  to  signal  words  that  will   help  you  to  understand  the  true  purpose  of  the  author’s  writing.  With  questions   about  purpose,  the  answer  choices  are  your  friend  -­‐-­‐  you’re  not  being  asked  to  get   into  the  author’s  mind,  necessarily,  just  to  pick  the  most  plausible  out  of  a  range  of   options.  In  essence,  the  work  has  already  been  done  for  you;  you  just  need  to   recognize  the  right  answer.     Try  to  figure  out  the  purpose  of  this  brief  literary  passage,  Snow,  by  Julia  Alvarez:     Our  first  year  in  New  York  we  rented  a  small  apartment  with  a  Catholic  school   nearby,  taught  by  the  Sisters  of  Charity,  hefty  women  in  long  black  gowns  and   bonnets  that  made  them  look  peculiar,  like  dolls  in  mourning.  I  liked  them  a  lot,   especially  my  grandmotherly  fourth  grade  teacher,  Sister  Zoe.  I  had  a  lovely   name,  she  said,  and  she  had  me  teach  the  whole  class  how  to  pronounce  it.  Yo-­‐ lan-­‐da.  As  the  only  immigrant  in  my  class,  I  was  put  in  a  special  seat  in  the  first   row  by  the  window,  apart  from  the  other  children  so  that  Sister  Zoe  could  tutor   Take  your  S   AT  prep  to  the  next  level.  Visit  www.learnerator.com   169  

170     me  without  disturbing  them.  Slowly,  she  enunciated  the  new  words  I  was  to   repeat:  laundromat,  cornflakes,  subway,  snow.     Soon  I  picked  up  enough  English  to  understand  holocaust  was  in  the  air.  Sister   Zoe  explained  to  a  wide  eyed  classroom  what  was  happening  in  Cuba.  Russian   missiles  were  being  assembled,  trained  supposedly  on  New  York  City.  President   Kennedy,  looking  worried  too,  was  on  the  television  at  home,  explaining  we   might  have  to  go  to  war  against  the  Communists.  At  school,  we  had  air  raid   drills:  an  ominous  bell  would  go  off  and  we'd  file  into  the  hall,  fall  to  the  floor,   cover  our  heads  with  our  coats,  and  imagine  our  hair  falling  out,  the  bones  in   our  arms  going  soft.  At  home,  Mami  and  my  sisters  and  I  said  a  rosary  for  world   peace.  I  heard  new  vocabulary:  nuclear  bomb,  radioactive  fallout,  bomb  shelter.   Sister  Zoe  explained  how  it  would  happen.  She  drew  a  picture  of  a  mushroom   on  the  blackboard  and  dotted  a  flurry  of  chalk  marks  for  the  dusty  fallout  that   would  kill  us  all.     The  months  grew  cold,  November,  December.  It  was  dark  when  I  got  up  in  the   morning,  frosty  when  I  followed  my  breath  to  school.  One  morning  as  I  sat  at   my  desk  daydreaming  out  the  window,  I  saw  dots  in  the  air  like  the  ones  Sister   Zoe  had  drawn  random  at  first,  then  lots  and  lots.  I  shrieked,  "Bomb!  Bomb!"   Sister  Zoe  jerked  around,  her  full  black  skirt  ballooning  as  she  hurried  to  my   side.  A  few  girls  began  to  cry.     But  then  Sister  Zoe's  shocked  look  faded.  "Why,  Yolanda  dear,  that's  snow!"  She   laughed.  "Snow."   "Snow,"  I  repeated.  I  looked  out  the  window  warily.  All  my  life  I  had  heard   about  the  white  crystals  that  fell  out  of  American  skies  in  the  winter.  From  my   desk  I  watched  the  fine  powder  dust  the  sidewalk  and  parked  cars  below.  Each   flake  was  different,  Sister  Zoe  had  said,  like  a  person,  irreplaceable  and   beautiful.     Take  your  S   AT  prep  to  the  next  level.  Visit  www.learnerator.com   170  

171     What  is  the  author’s  purpose  in  recounting  this  story?   A.  To  illustrate  the  hardships  that  new  immigrants  face  when  assimilating  to   their  new  country   B.  To  provide  a  searing  indictment  of  America’s  Cold  War  policy   C.  To  give  a  poignant  snapshot  of  the  various  joys  and  challenges  of  learning   to  be  an  American   D.  To  demonstrate  the  fear  that  many  children  felt  during  the  Cold  War     Though  the  excerpt  is  brief,  you  can  easily  isolate  its  main  subject,  which  is   immigration  and  assimilation  into  American  society.  Yet  Option  A  is  clearly   incorrect  -­‐-­‐  though  immigrating  to  the  United  States  is  no  doubt  a  difficult   enterprise,  especially  without  an  adequate  command  of  English,  this  brief  excerpt   details  none  of  the  hardships  of  assimilation.  You  may  have  highlighted  the   “holocaust  was  in  the  air”  line  to  support  this  answer,  yet  remember  that  any   hardships  that  came  with  being  in  America  during  the  early  1960s  were  shared  by   all  Americans,  not  just  immigrants.    (We  know  that  the  story  takes  place  in  the  early   1960s  because  of  the  mention  of  President  Kennedy,  who  served  as  President  from   1961  -­‐  1963.)     B  is  also  incorrect  -­‐-­‐  this  is  clearly  not  a  polemic  against  American  policies  since  no   policies  are  even  mentioned  here.  The  author  presents  the  “holocaust  in  the  air”  as  a   fact  of  life,  not  something  to  rail  against.  C  seems  to  be  the  best  answer,  as  the  piece   as  a  whole  is  largely  about  the  different  aspects  of  learning  to  be  an  American  -­‐-­‐   everything  from  learning  a  language  to  experiencing  snow  for  the  first  time  is  part   and  parcel  of  these  joys  and  hardships.  D  isn’t  a  terrible  answer,  as  this  passage  does   clearly  demonstrate  fear,  yet  considering  the  upbeat  ending  and  the  matter-­‐of-­‐fact   way  in  which  the  author  discusses  the  fear  that  she  and  her  family  and  classmates   felt,  this  is  a  less  convincing  answer  than  C,  which  more  fully  encompasses  every   aspect  of  the  passage.    

Take  your  S   AT  prep  to  the  next  level.  Visit  www.learnerator.com   171  

172     In  learning  to  parse  purpose,  you  must  consider  the  text  as  a  whole.  What  is  its  main   focus?  What  most  concerns  the  author?  Are  there  ideas  or  themes  that  are   consistently  repeated?  Who  is  the  intended  audience?  All  of  these  considerations   will  help  you  to  better  understand  purpose  and  to  accurately  answer  questions   about  an  author’s  purpose  in  writing  and  publishing  a  text.  

   

Take  your  S   AT  prep  to  the  next  level.  Visit  www.learnerator.com   172  

173    

Analyzing Claims and Counterclaims One  of  the  most  important  features  of  the  kinds  of  texts  you’ll  see  on  the  SAT  (as   well  as  in  any  academic  setting)  is  “argument.”  This  isn’t  to  say  that  all  texts  contain   “fightin’  words;”  instead,  most  expository  and  some  narrative  texts  contain  a  central   thesis  that  they  try  to  prove  with  details,  explanations,  and  other  forms  of  evidence   and  support.  The  ability  to  parse  an  argument  is  absolutely  crucial  to  your   understanding  of  the  text.  But  understanding  the  thesis  isn’t  the  only  aspect  of   understanding  the  argument;  you  must  also  be  able  to  break  down  an  argument  into   its  component  claims  and  counterclaims.     Claims:   A  claim  is  the  central  argument  of  the  text.  It  can  also  be  called  a  thesis,  a   proposition,  or  -­‐-­‐  if  there  is  only  one  -­‐-­‐  simply  “the  argument.”  Assess  the  claim  of   the  following  brief  excerpt:     One  of  the  ways  in  which  ordinary  people  can  prevent  gingivitis  is  by  gargling  twice   daily  with  a  dentist-­‐approved  mouthwash.     Broadly  stated,  the  claim  of  this  “passage”  is  that  mouthwash  will  prevent  gingivitis.   Presumably,  the  rest  of  the  passage  will  go  on  to  explain  why  mouthwash  works,  in   an  effort  to  back  up  its  central  claim.  Unfortunately,  because  the  nature  of  claims  is   so  variable  -­‐-­‐  all  texts  will  look  different,  and  all  texts  will  make  different  types  of   claims  -­‐-­‐  there  is  no  immediately  obvious  signal  word  that  will  help  you  to  ascertain   the  nature  of  a  claim.  In  order  to  figure  out  what  a  claim  is,  you  must  simply  read   carefully.  Does  the  writer  provide  lots  of  supporting  details?  Are  the  paragraphs   organized  effectively,  with  topic  sentences  that  introduce  the  central  focus  of  each   paragraph?  If  so,  the  claim  is  often  found  in  the  last  sentence  of  the  first  paragraph.   If  you’re  in  a  pinch  and  can’t  find  the  claim,  skimming  the  first  and  second   paragraphs  of  an  expository  text  may  help.  However,  it  is  not  enough  to  simply   Take  your  S   AT  prep  to  the  next  level.  Visit  www.learnerator.com   173  

174     identify  the  claim,  as  a  sophisticated  argument  will  also  include  what  is  called  a   “counterclaim.”     Counterclaims:   Counterclaims  provide  an  opposing  viewpoint  to  the  central  claim.  Including  a   counterclaim  in  your  writing  helps  you  to  prove  that  yes,  your  opinion  is  the  best-­‐ substantiated  /  most  widely-­‐supported  one.  By  giving  the  opposition  a  voice  and   then  knocking  it  down,  the  writer  demonstrates  a  deep  competence  and  familiarity   with  the  topic  -­‐-­‐  provided  that  s/he  can  effectively  address  the  counterclaim.  Look  at   the  way  this  author  structured  his  counterclaim:     One  of  the  ways  in  which  ordinary  people  can  prevent  gingivitis  is  by  gargling  twice   daily  with  a  dentist-­‐approved  mouthwash.  However,  in  a  recent  survey  of  American   dentists,  some  questioned  the  use  of  mouthwash  as  a  tactic  to  prevent  gingivitis.     As  you  can  see,  the  second  sentence  indicates  that  the  author  is  presenting  a   “counterclaim”  -­‐-­‐  a  refutation  to  the  central  claim  that  using  mouthwash  is  an   effective  tactic  for  preventing  gingivitis.  One  easy  way  to  spot  a  counterclaim  is  to   look  for  signal  words  like  “however,”  “but,”  “on  the  other  hand,”  “yet,”  “in  contrast,”   “otherwise,”  “notwithstanding,”  “nevertheless,”  “on  the  contrary”  -­‐-­‐  any  words  that   signal  a  contrast  will  help  you  find  the  counterclaim.  In  fact,  if  you’re  having  a  hard   time  finding  the  actual  argument  in  the  essay,  work  backwards.  Look  for  the   transitional  words  that  may  signal  a  counterclaim,  and  assess  what  the  counterclaim   says.  Then  figure  out  the  claim  it  is  attempting  to  refute.     Countering  the  Counterclaim:   Sophisticated  essays  counter  their  counterclaims  in  order  to  illustrate  how   thoroughly  they  have  considered  the  topic  and  how  effectively  they  can  argue.   Writers  include  counterclaims  either  to  knock  them  down,  as  in  a  persuasive  essay,   or  to  maintain  objectivity  by  showing  the  “other  side  of  the  story,”  as  in  a  piece  of   journalism.  What  is  the  aim  in  this  piece?   Take  your  S   AT  prep  to  the  next  level.  Visit  www.learnerator.com   174  

175       One  of  the  ways  in  which  ordinary  people  can  prevent  gingivitis  is  by  gargling  twice   daily  with  a  dentist-­‐approved  mouthwash.  However,  in  a  recent  survey  of  American   dentists,  some  questioned  the  use  of  mouthwash  as  a  tactic  to  prevent  gingivitis.  The   survey,  which  was  released  on  Tuesday,  apparently  polled  a  total  of  three  dentists,   none  of  whom  had  completed  their  dental  studies.     Clearly,  this  writer  used  the  counterclaim  in  order  to  further  bolster  his  own   argument  -­‐-­‐  by  discrediting  the  opposition.     Read  the  following  passage  -­‐-­‐  a  commencement  address  by  Ursula  K.  LeGuin,   delivered  in  May  of  1983  -­‐-­‐  and  determine  how  LeGuin  establishes  claims  and   counterclaims.  As  you  read,  evaluate  the  effectiveness  of  the  writer’s  strategy,  and   determine  the  purpose  of  setting  up  a  claim,  counterclaim,  and  refutation.     Intellectual  tradition  is  male.  Public  speaking  is  done  in  the  public  tongue,  the   national  or  tribal  language;  and  the  language  of  our  tribe  is  the  men's   language.  Of  course  women  learn  it.  We're  not  dumb.  If  you  can  tell  Margaret   Thatcher  from  Ronald  Reagan,  or  Indira  Gandhi  from  General  Somoza,  by   anything  they  say,  tell  me  how.  This  is  a  man’s  world,  so  it  talks  a  man’s   language.  The  words  are  all  words  of  power.  You’ve  come  a  long  way,  baby,  but   no  way  is  long  enough.  You  can’t  even  get  there  by  selling  yourself  out:  because   there  is  theirs,  not  yours.     Maybe  we’ve  had  enough  words  of  power  and  talk  about  the  battle  of  life.   Maybe  we  need  some  words  of  weakness.  Instead  of  saying  now  that  I  hope  you   will  all  go  forth  from  this  ivory  tower  of  college  into  the  Real  World  and  forge  a   triumphant  career  or  at  least  help  your  husband  to  and  keep  our  country   strong  and  be  a  success  in  everything  -­‐-­‐  instead  of  talking  about  power,  what  if   I  talked  like  a  woman  right  here  in  public?  It  won’t  sound  right.  It’s  going  to   sound  terrible.  What  if  I  said  what  I  hope  for  you  is  first,  if  -­‐-­‐  only  if  -­‐-­‐  you  want   Take  your  S   AT  prep  to  the  next  level.  Visit  www.learnerator.com   175  

176     kids,  I  hope  you  have  them.  Not  hordes  of  them.  A  couple,  enough.  I  hope  they’re   beautiful.  I  hope  you  and  they  have  enough  to  eat,  and  a  place  to  be  warm  and   clean  in,  and  friends,  and  work  you  like  doing.     Well,  is  that  what  you  went  to  college  for?  Is  that  all?  What  about  success?     Success  is  somebody  else’s  failure.  Success  is  the  American  Dream  we  can  keep   dreaming  because  most  people  in  most  places,  including  thirty  million  of   ourselves,  live  wide  awake  in  the  terrible  reality  of  poverty.  No,  I  do  not  wish   you  success.  I  don’t  even  want  to  talk  about  it.  I  want  to  talk  about  failure.     Because  you  are  human  beings  you  are  going  to  meet  failure.  You  are  going  to   meet  disappointment,  injustice,  betrayal,  and  irreparable  loss.  You  will  find   you’re  weak  where  you  thought  yourself  strong.  You’ll  work  for  possessions  and   then  find  they  possess  you.  You  will  find  yourself  -­‐-­‐  as  I  know  you  already  have  -­‐-­‐   in  dark  places,  alone,  and  afraid.     ...So  what  I  hope  for  you  is  that  you  live  there  not  as  prisoners,  ashamed  of   being  women,  consenting  captives  of  a  psychopathic  social  system,  but  as   natives.  That  you  will  be  at  home  there,  keep  house  there,  be  your  own  mistress,   with  a  room  of  your  own.  That  you  will  do  your  work  there,  whatever  you’re   good  at,  art  or  science  or  tech  or  running  a  company  or  sweeping  under  the   beds,  and  when  they  tell  you  that  it’s  second-­‐class  work  because  a  woman  is   doing  it,  I  hope  you  tell  them  to  go  to  hell  and  while  they’re  going  to  give  you   equal  pay  for  equal  time.  I  hope  you  live  without  the  need  to  dominate,  and   without  the  need  to  be  dominated.  I  hope  you  are  never  victims,  but  I  hope  you   have  no  power  over  other  people.  And  when  you  fail,  and  are  defeated,  and  in   pain,  and  in  the  dark,  then  I  hope  you  will  remember  that  darkness  is  your   country,  where  you  live,  where  no  wars  are  fought  and  no  wars  are  won,  but   where  the  future  is.    

 

Take  your  S   AT  prep  to  the  next  level.  Visit  www.learnerator.com   176  

177     What  is  LeGuin’s  central  claim?   A.  That  men  abuse  their  power   B.  That  victimhood  is  a  physically  and  emotionally  untenable  situation   C.  That  every  person  should  strive  to  define  themselves  as  they  want   D.  That  to  be  human  is  to  be  doomed  to  fail     LeGuin  makes  a  few  claims  over  the  course  of  this  argument,  and  they  are  all  subtly   constructed.  Though  she  would  probably  not  disagree  with  choice  A  -­‐-­‐  “the  language   of  our  tribe  is  the  man’s  language”  -­‐-­‐  this  does  not  appear  to  be  her  central  claim;   remember,  a  central  claim  is  the  most  important  or  main  one  that  an  author  makes.   Though  LeGuin  discusses  “the  language  of  the  tribe”  she  does  not  focus  on  male   abuse  of  power.  Nor  does  she  focus  on  victimhood  -­‐-­‐  she  mentions  it  once  in  the  last   line  of  her  speech.  C  is  a  well-­‐supported  claim,  as  she  devotes  the  majority  of  the   speech  to  “talking  like  a  woman”  -­‐-­‐  refuting  traditional  (ie  male)  ideas  of  success  by   encouraging  women  to  do  or  be  what  they  want  (including  mothers,  their  “own   mistress”es,  “doing  whatever  [they’re]  good  at.”  And  though  D  seems  correct  -­‐-­‐  she   does  talk  about  failure  quite  extensively  -­‐-­‐  the  word  “doomed”  suggests  that  LeGuin   sees  failure  as  something  negative,  to  be  avoided,  instead  of  natural  and  to   embraced.  Clearly,  C  is  the  best  answer.     Which  line  from  the  text  provides  the  best  counterclaim  to  LeGuin’s   central  assertion?   A.  “No,  I  do  not  wish  you  success.  I  don’t  even  want  to  talk  about  it.”   B.  “You  will  find  yourself…  in  dark  places,  alone  and  afraid.”   C.  “I  hope  you  have  no  power  over  other  people.”   D.  “Well,  is  that  what  you  went  to  college  for?  Is  that  all?”     Remember,  LeGuin’s  central  assertion  is  that  people  should  define  themselves  how   they  want,  without  recourse  to  external  benchmarks  about  what  constitutes   happiness.  We’re  looking  for  an  effective  counter  to  that.  A  is  simply  an  extension  of   her  argument,  not  in  any  way  a  counter  of  it.  B  is  also  an  extension  of  her  argument   Take  your  S   AT  prep  to  the  next  level.  Visit  www.learnerator.com   177  

178     (that  no  one  should  be  afraid  of  ending  up  “in  dark  places.”)  C,  too,  is  an  extension  of   her  argument.  Only  D  is  “another  voice,”  providing  another  perspective  on  her   argument.  D  asks  “well,  what  about  success?  What  about  why  you  even  went  to  this   college  in  the  first  place?”  D  makes  a  counterclaim  to  LeGuin’s  assertion  that  it’s  just   fine  to  make  motherhood  your  only  ambition.  This  question  “what  about  success  /   why  did  you  go  to  college”  is  only  important  to  LeGuin’s  argument  because  she   knocks  it  down  immediately,  saying  it  is  irrelevant.  Clearly,  she  only  includes  a   counterclaim  in  order  to  demonstrate  that  her  argument  takes  the  best  perspective.     What  is  LeGuin’s  tone  in  this  commencement  address?   A.  The  tone  of  an  optimist  who  believes  that  everything  will  turn  out  the  way   it  should   B.  The  tone  of  a  pessimist  who  believes  that  everyone  is  destined  to  fail   C.  The  tone  of  a  realist  who  acknowledges  the  various  possibilities  and   opportunities  that  exist  in  this  world   D.  The  tone  of  an  idealist  who  places  her  trust  in  a  higher  power     This  is  a  review  question  designed  to  jog  your  memory  about  strategies  used  to   analyze  tone.  It  should  be  clear  from  reviewing  LeGuin’s  claim  and  counterclaim  that   she  acknowledges  the  realities  of  the  world  -­‐-­‐  she  talks  about  failure  and  darkness   too  much  to  be  considered  an  optimist.  But  nor  is  she  a  pessimist.  Clearly,  she   believes  in  the  power  of  possibility  as  well  as  the  strength  of  the  human  spirit.  There   is  no  evidence  that  she  places  her  trust  in  a  “higher  power,”  which  leaves  only  C  as   the  best  option  -­‐-­‐  she  does  acknowledge  that  various  opportunities  and  possibilities   exist  in  this  world,  as  evidenced  by  her  obvious  hope  that  each  woman  is  “her  own   mistress”  and  her  insistence  that  “darkness  is  your  country.”     The  structure  and  content  of  this  argument  is  somewhat  unusual.  Try  your  hand  at   another  narrative  text,  and  this  time  consider  the  author's  purpose  in  including  the   various  claims  and  counterclaims  found  in  the  article.     Take  your  S   AT  prep  to  the  next  level.  Visit  www.learnerator.com   178  

179     “We  sent  you  to  college  to  learn  THIS?”  That  was  the  reaction  that  Matilda   Flanagan,  19,  received  from  her  incensed  mother  after  she  came  home  during   winter  break  of  her  sophomore  year  of  college,  flushed  with  pride  and  relief  at   finally  having  chosen  a  major.  Flanagan’s  mother,  Brunhilde,  a  nurse,  had   hoped  her  daughter  -­‐-­‐  a  straight  A  student  who  won  a  national  science  fair  in   the  eleventh  grade  for  her  rather  gory  project  on  the  life  cycle  of  a  tapeworm  -­‐-­‐   would  go  on  to  major  in  chemical  engineering  or  biology,  “something  really   practical  and  lucrative,”  as  Brunhilde  bluntly  puts  it.  But  instead,  the  19  year   old  Flanagan,  who  describes  herself  as  a  “huge  nerd,”  had  chosen  to  take   another  path:  Fandom  Studies.  More  specifically,  her  concentration  was  on  “the   collective  fascination  with  cultural  artifacts  of  the  1990s,”  i.e.  Buffy  the   Vampire  Slayer  and  My  So  Called  Life.     “She’s  majoring  in  television!”  Brunhilde  Flanagan  raged  during  a  recent   interview.  “I  mean,  why  are  we  paying  45k  a  year  for  her  to  write  some  essays   on  a  TV  show?  She  could  have  easily  done  that  while  living  at  home  and   working  at  Walgreens!”  (Matilda  worked  15  hours  a  week  at  Walgreens  during   high  school.)     The  elder  Flanagan  isn’t  alone  in  her  ire.  Increasingly,  the  parents  of  students   at  expensive  private  liberal  arts  colleges  are  throwing  up  their  hands  and   asking:  “Where  exactly  is  my  money  going?”     Bob  Litchfield,  a  litigator  from  Denver,  echoed  Brunhilde  Flanagan’s   frustration.  “Why  exactly  is  Yale  charging  my  son  -­‐-­‐  or  more  accurately,  me  -­‐-­‐   over  fifty  thousand  dollars  a  year  so  that  he  can  study  Pokemon?  Please,  explain   this  to  me,”  he  said  in  a  phone  interview,  the  anger  palpable  in  his  voice.   Litchfield  had  hoped  that  his  son,  a  20-­‐year-­‐old  classically  trained  violinist,   would  follow  his  footsteps  and  enter  the  rarefied  (not  to  mention   remunerative)  world  of  corporate  litigation.  But  Humperdinck  Litchfield  is  far   more  interested  in  demonstrating  how  Charmander  represents  the  shattered   Take  your  S   AT  prep  to  the  next  level.  Visit  www.learnerator.com   179  

180     dreams  of  a  nation.  Since  matriculating  at  Yale,  he  has  become  fluent  in   Japanese,  and  has  committed  the  contents  of  every  Pokemon  card  ever  released   to  memory.     “This  is  an  outrage,”  says  Brandywine  Buckley,  a  stay-­‐at-­‐home  mother  of   quadruplets  who  attend  the  University  of  Florida.  “Why  exactly  are  my  four   daughters  studying  pop  music?  One  of  them  recently  wrote  a  term  paper  on   what  Britney  Spears  ‘means’  for  our  culture.  I’ll  tell  you  what  she  means:  she   means  you’ll  never  get  a  job  like  this!  We  wanted  our  daughter  to  become  a   statistician.  Why  is  she  wasting  her  time?”     Money  is  certainly  a  central  concern  for  these  apoplectic  parents,  but  these   students  have  a  quick  response.  “In  a  way,  this  is  the  definition  of  academics,”   Matilda  said,  animated.  “I  mean,  what  could  be  more  academic  than  pursuing   knowledge  that  is  by  definition  completely  and  entirely  useless  in  the  context  of   the  real  world?  I  don’t  see  anybody  questioning  history  nerds  who  choose  to   spend  7  years  of  their  lives  in  the  library,  pursuing  pointless  degrees  in   medieval  history.  Why  is  that  allowed    -­‐-­‐  heck,  even  celebrated  -­‐-­‐  when  a  study   of  our  own  pop  culture  is  demonized?”  To  which  her  mother  responded  drily,   “medieval  historians  don’t  exactly  make  any  money,  either.”     The  battle  between  practical  parents  footing  impressively  (some  might  say   frighteningly)  high  tuition  bills  and  passionate  students  who  have  chosen  to   pursue  their  own  untraditional  paths  won’t  end  anytime  soon,  as  more  and   more  universities  add  Cultural  Studies  classes  to  their  course  offerings.  It  is   impossible  to  say  whether  these  “impractical”  courses  of  study  will  outlive  the   current  decade,  as  more  and  more  graduates  of  cultural  studies  programs   graduate  with  serious  debt  and  no  immediately  applicable  job  skills  -­‐-­‐  unless   you  count  the  ability  to  name  every  single  episode  of  Buffy  the  Vampire  Slayer   in  a  single  breath.     Take  your  S   AT  prep  to  the  next  level.  Visit  www.learnerator.com   180  

181     This  is  an  interesting  piece  to  parse,  because  -­‐-­‐  unlike  the  previous  straightforward   text  -­‐-­‐  this  article  offers  multiple  perspectives  and  has  a  somewhat  untraditional   structure  with  seemingly  no  “central  argument.”  But  if  you  read  closely,  you’ll  see   two  sides  of  an  argument  emerge,  with  the  author  taking  a  position  of  her  own.  It’s   important  that  you  are  able  to  apply  the  ability  to  assess  claims  and  counterclaims   even  to  texts  with  an  untraditional  structure,  as  you  will  be  learning  to  assess  all   forms  of  informative  texts  is  an  essential  skill  on  the  SAT,  in  college,  and  later  in  life.     What  is  the  central  claim  of  the  text  as  a  whole?   A.  That  cultural  studies  are  a  pointless  waste  of  time   B.  That  cultural  studies  are  the  purest  form  of  academic  inquiry   C.  That  cultural  studies  are  a  recent  phenomenon  that  has  sparked  strong   feelings   D.  That  cultural  studies  are  an  important  part  of  college     If  you  read  the  question  closely,  you’ll  see  that  you  are  being  asked  to  assess  the   central  claim  of  the  whole  text,  NOT  any  individual  claim  espoused  by  one  person.   This  question  is  basically  asking  you  what  the  text  is  ABOUT.  While  answers  A  and  B   are  opinions  held  by  the  various  players  in  this  article,  neither  is  the  “point”  of  the   text,  or  the  central  claim  that  the  author  makes.  All  the  author  is  trying  to  do  is   illustrate  a  recent  trend,  making  C  the  best  answer;  there  is  no  evidence  for  D.     Based  on  the  article,  what  is  the  primary  underlying  motivation  for   these  irate  parents’  anger?   A.  Parents  are  concerned  that  cultural  studies  will  interfere  with  their   children’s  social  skills   B.  Parents  are  concerned  that  cultural  studies  will  not  adequately  prepare   their  children  for  the  job  market   C.  Parents  are  concerned  that  cultural  studies  are  shallow  and  unproductive   D.  Parents  are  concerned  that  cultural  studies  are  extremely  expensive     Take  your  S   AT  prep  to  the  next  level.  Visit  www.learnerator.com   181  

182     This  question  asks  you  to  assess  the  claims  made  by  the  parents  interviewed  in  the   article  and  evaluate  the  reasoning  behind  them.  It  is  fairly  obvious  that  parents   think  that  cultural  studies  are  a  waste  of  time,  but  though  obsessive  fandom  may   interfere  with  students’  social  skills,  there  is  no  real  evidence  for  this  in  the  article   or  in  any  of  the  parents’  arguments,  so  you  can  scratch  A.  B  is  a  strong  contender,   especially  considering  that  every  single  parent  interviewed  expressed  hopes  for   their  children’s  future  in  the  job  market  (see:  the  father  who  wanted  his  son  to  be  a   lawyer,  the  mother  who  hoped  her  daughter  would  be  a  statistician).  And  though  C   is  a  reasonable  inference,  it  doesn’t  seem  to  be  the  primary  motivator  for  these   parents’  anger.  D  is  also  a  reasonable  contender,  as  parents  mention  money  often  -­‐-­‐   but  if  you  read  very  closely,  it’s  clear  that  they  are  willing  to  spend  a  lot  of  money  on   their  children’s  education  provided  their  children  actually  acquire  usable  skills  in   the  job  market.  So  it’s  not  necessarily  that  parents  are  reticent  about  spending   money  -­‐-­‐  the  argument  has  more  to  do  with  disappointment  that  they  will  not   recoup  this  investment.  Clearly,  B  is  the  best  choice.     How  does  the  author  subtly  demonstrate  her  own  opinion  in  this   article?   A.  By  quoting  more  irate  parents  than  fandom-­‐loving  students   B.  By  calling  Matilda  Flanagan  a  “huge  nerd”   C.  By  quoting  Matilda  Flanagan  extensively,  demonstrating  an  acceptance  of   Matilda’s  opinion   D.  By  subtly  insinuating  that  cultural  studies  are  impractical     Again,  this  really  is  a  question  about  claims  -­‐-­‐  what  is  the  author  subtly  claiming,  and   how?  A  may  be  tempting  -­‐-­‐  it  is  true  that  the  author  quotes  three  irate  parents  and   only  one  cultural  studies  student.  But  this  is  not  necessarily  indicative  of  an  obvious   bias;  it’s  possible  that  the  writer  did  not  have  the  same  access  to  students  as  she  did   to  parents.  Keep  this  answer  for  now,  but  consider  the  other  options.  B  is  clearly   incorrect,  because  Matilda  calls  herself  a  “huge  nerd”  (seemingly  as  a  point  of  pride).   And  C  can’t  be  true  because  quoting  someone  extensively  does  not  imply  an   Take  your  S   AT  prep  to  the  next  level.  Visit  www.learnerator.com   182  

183     endorsement  of  their  opinion  -­‐-­‐  if  that  were  the  case,  both  A  and  C  would  be  correct.   But  the  last  paragraph  of  the  text  provides  evidence  for  D  -­‐-­‐  the  author  calls  into   question  the  continued  existence  of  these  cultural  studies  program,  claiming  that   they  leave  students  with  “no  applicable  job  skills”  and  subtly  denigrating  the  skills   they  do  impart  (the  ability  to  name  all  the  episodes  of  Buffy  the  Vampire  Slayer  in  a   single  breath).  Though  this  is  supposedly  a  neutral  text  (the  author  sets  up  multiple   claims),  the  inclusion  of  this  last  sentence  is  a  not-­‐so-­‐subtle  hint  that  the  author  also   has  some  feelings  about  this  issue.  When  evaluating  a  claim  and/or  a  bias,  consider   any  reason  the  author  may  have  for  including  information.  If  it  doesn’t  serve  the   obvious  purpose  of  bolstering  the  central  claim  (as  the  quotes  from  parents  and   students  do),  then  there  is  a  very  good  chance  that  it  is  being  included  for  the  sole   purpose  of  demonstrating  the  author’s  opinion.     Claims  and  counterclaims  are  an  important  part  of  many  forms  of  writing,  whether   narrative,  expository,  or  persuasive.  If  you  can  spot  a  claim  when  it  is  being  made   and  analyze  its  purpose,  you  are  already  poised  for  success  on  the  SAT  as  well  as  in   any  later  academic  context.  

 

Take  your  S   AT  prep  to  the  next  level.  Visit  www.learnerator.com   183  

184    

Assessing Reasoning As  demonstrated  in  previous  articles,  the  new  SAT  is  very  concerned  with  testing   your  ability  to  understand  arguments  -­‐-­‐  in  structure,  tone,  style,  and  reasoning.  But   it  is  not  enough  to  simply  identify  the  claims  and  counterclaims  that  an  author   makes  over  the  course  of  an  argument.  It  is  crucial  that  you  are  able  to  assess  those   claims  and  arguments,  and  evaluate  whether  or  not  they  are  sound.   When  assessing  a  claim,  you  must  consider  both  its  soundness  and  its  validity.  A   valid  argument  is  one  that  follows  logically  (even  if  it  isn’t  true).  For  example:     Premise:   All  people  can  fly.   Beyonce  is  a  person.   Therefore,  Beyonce  can  fly.     This  is  a  perfectly  valid  argument.  It  happens  to  be  complete  nonsense,  but  it  follows   logically.  When  analyzing  an  author’s  argument,  you  must  consider  whether  the   argument  follows  logically  -­‐-­‐  whether  each  premise  builds  logically  on  the  next.  This   is  one  very  important  part  of  assessing  reasoning.     The  SAT  isn’t  only  asking  you  to  assess  whether  a  text  is  valid,  however;  it  also  asks   you  to  assess  whether  an  author’s  reasoning  is  sound  -­‐-­‐  i.e.,  whether  the  argument  is   not  only  logical  but  true.  You  may  not  have  deep  familiarity  with  every  topic  that  the   SAT  Reading  section  will  cover;  often,  you  will  be  presented  with  persuasive  or   expository  texts  that  contain  information  that  you  will  not  be  able  to  assess  because   you  don’t  have  the  content  knowledge  to  do  so.  For  readings  like  that,  assume  that   the  content  is  actually  true  -­‐-­‐  the  SAT  will  not  trip  you  up  or  try  to  “trick”  you  by   presenting  you  with  false  information.  Therefore,  as  you  read,  assume  the  argument   is  true  (unless  presented  with  evidence  otherwise,  i.e.  if  the  piece  is  intended  to  be  

Take  your  S   AT  prep  to  the  next  level.  Visit  www.learnerator.com   184  

185     absurdist  or  satiric)  and  assess  instead  its  logic  and  the  flow  or  structure  of  its   premises.     Assess  the  following  excerpt’s  validity  and  soundness;  as  you  read,  it  may  be  helpful   to  annotate,  underline,  or  number  key  portions  of  the  text  that  make  up  the   argument.  See  if  you  can  follow  the  argument’s  trajectory.  This  speech,  “The  Perils  of   Indifference,”  was  given  by  Elie  Weisel  in  1999.     In  a  way,  to  be  indifferent  to...  suffering  is  what  makes  the  human  being   inhuman.  Indifference,  after  all,  is  more  dangerous  than  anger  and  hatred.   Anger  can  at  times  be  creative.  One  writes  a  great  poem,  a  great  symphony.   One  does  something  special  for  the  sake  of  humanity  because  one  is  angry  at   the  injustice  that  one  witnesses.  But  indifference  is  never  creative.  Even  hatred   at  times  may  elicit  a  response.  You  fight  it.  You  denounce  it.  You  disarm  it.     Indifference  elicits  no  response.  Indifference  is  not  a  response.  Indifference  is   not  a  beginning;  it  is  an  end.  And,  therefore,  indifference  is  always  the  friend  of   the  enemy,  for  it  benefits  the  aggressor  -­‐-­‐  never  his  victim,  whose  pain  is   magnified  when  he  or  she  feels  forgotten.  The  political  prisoner  in  his  cell,  the   hungry  children,  the  homeless  refugees  -­‐-­‐  not  to  respond  to  their  plight,  not  to   relieve  their  solitude  by  offering  them  a  spark  of  hope  is  to  exile  them  from   human  memory.  And  in  denying  their  humanity,  we  betray  our  own.   Indifference,  then,  is  not  only  a  sin,  it  is  a  punishment.  And  this  is  one  of  the   most  important  lessons  of  this  outgoing  century's  wide-­‐ranging  experiments  in   good  and  evil.     What  is  the  reasoning  behind  Elie  Weisel’s  claim  that  indifference  is  the   greatest  evil?   A.  Indifference  is  a  sin  and  a  punishment   B.  Indifference  is  more  dangerous  than  anger  and  hatred  

Take  your  S   AT  prep  to  the  next  level.  Visit  www.learnerator.com   185  

186     C.  Indifference  benefits  the  aggressor   D.  Indifference  denies  the  humanity  of  others     In  order  to  answer  this  question,  you  must  consider  how  Weisel  structures  his   argument.  His  central  claim  is  that  indifference  is  the  greatest  evil,  but  how  does   he  justify  this  claim?  Read  answer  A  closely  -­‐-­‐  while  he  does  say  that  indifference  is  a   sin  and  a  punishment,  this  does  not  reveal  the  reasoning  behind  his  argument.  In   fact,  this  answer  option  simply  repeats  or  extends  his  claim  without  actually   providing  the  reasoning  behind  it.  The  same  is  true  for  answer  B  and  partially  for   answer  C  -­‐-­‐  none  actually  explain  Weisel’s  reasoning,  or  the  way  he  configures  the   argument  behind  the  central  premise.  Only  answer  D  logically   explains  why  indifference  is  the  greatest  evil.  When  you  think  about  the  way   reasoning  works,  ask  yourself:  does  this  explain  WHY  the  author  has  made  this   central  claim?  If  you  can’t  answer  that  question,  then  the  answer  option  is  probably   incorrect.     One  way  to  tackle  this  problem  is  to  map  out  the  argument  itself  and  see  which   pieces  of  it  lead  logically  to  others.     1.  In  a  way,  to  be  indifferent  to...  suffering  is  what  makes  the  human  being  inhuman.       This  is  the  central  claim.  The  rest  of  the  argument  should  prove  this,  and  answer  the   question  “why  is  this  claim  true?”     2.  Indifference,  after  all,  is  more  dangerous  than  anger  and  hatred.     This  does  not  answer  the  question  of  “why  is  this  claim  true;  it  simply  extends  the   premise  that  indifference  to  suffering  is  evil.     3.    Anger  can  at  times  be  creative.  One  writes  a  great  poem,  a  great  symphony.  One   does  something  special  for  the  sake  of  humanity  because  one  is  angry  at  the  injustice   Take  your  S   AT  prep  to  the  next  level.  Visit  www.learnerator.com   186  

187     that  one  witnesses.  But  indifference  is  never  creative.  Even  hatred  at  times  may  elicit  a   response.  You  fight  it.  You  denounce  it.  You  disarm  it.     This  begins  to  answer  the  question  of  “why:”  indifference  is  the  greatest  evil   because,  unlike  anger  or  hatred,  it  is  not  creative.     4.  Indifference  elicits  no  response.  Indifference  is  not  a  response.  Indifference  is  not  a   beginning;  it  is  an  end.  And,  therefore,  indifference  is  always  the  friend  of  the  enemy,   for  it  benefits  the  aggressor  -­‐-­‐  never  his  victim,  whose  pain  is  magnified  when  he  or  she   feels  forgotten.  The  political  prisoner  in  his  cell,  the  hungry  children,  the  homeless   refugees  -­‐-­‐  not  to  respond  to  their  plight,  not  to  relieve  their  solitude  by  offering  them   a  spark  of  hope  is  to  exile  them  from  human  memory.  And  in  denying  their  humanity,   we  betray  our  own.     Again,  this  reinforces  the  premise  by  answering  the  question  of  “why:”  why  is   indifference  the  greatest  evil?  Because  it  isn’t  creative  and  has  no  response,  and   ignores  and  forgets  the  people  who  need  the  most  help.  In  so  doing  it  betrays  the   humanity  we  all  share.     5.    Indifference,  then,  is  not  only  a  sin,  it  is  a  punishment.  And  this  is  one  of  the  most   important  lessons  of  this  outgoing  century's  wide-­‐ranging  experiments  in  good  and   evil.     This  extends  the  argument  -­‐-­‐  BECAUSE  indifference  is  the  greatest  evil  (the  “then”   indicates  the  idea  of  “because”),  it  is  both  a  sin  and  a  punishment.  This  subtly  moves   on  from  the  central  premise  and  extends  it  so  that  it  is  even  more  damning.  When   you’re  stuck,  use  this  numbering  /  why  strategy,  and  see  if  it  helps  you  better  assess   and  evaluate  the  reasoning  behind  an  argument.    

Take  your  S   AT  prep  to  the  next  level.  Visit  www.learnerator.com   187  

188     Try  this  strategy  one  more  time  with  the  following  text  from  the  Atlantic,  entitled   North  Korea  Is  Not  Funny.  As  you  read,  ask  yourself:     What  is  the  author’s  claim?   What  is  the  reasoning  behind  it?   Is  the  reasoning  behind  it  logical,  valid,  and  sound?   If  you're  not  sure  how  to  do  this,  number  the  claims  and  make  sure  that   all  of  them  answer  the  question  of  “why.”     This  film  (The  Interview,  about  the  assassination  of  Kim  Jong  Un)  is  not  an  act   of  courage.  It  is  not  a  stand  against  totalitarianism,  concentration  camps,  mass   starvation,  or  state-­‐sponsored  terror.  It  is,  based  on  what  we  know  of  the  movie   so  far,  simply  a  comedy,  made  by  a  group  of  talented  actors,  writers,  and   directors,  and  intended,  like  most  comedies,  to  make  money  and  earn  laughs.   The  movie  would  perhaps  have  been  better  off  with  a  fictitious  dictator  and   regime;  instead,  it  appears  to  serve  up  the  latest  in  a  long  line  of  cheap  and   sometimes  racism-­‐tinged  jokes,  stretching  from  Team  America:  World  Police  to   ongoing  sketches  on  Saturday  Night  Live.     Humor  can  be  a  powerful  tool  for  surviving  in  a  closed  society,  and  lampooning   dictators  can  lend  latent  popular  movements  the  confidence  they  need  to   challenge  their  oppressors.  In  Libya,  dissidents  heaped  mockery  on  the  Qaddafi   family  in  the  early  stages  of  their  Arab  Spring  revolution.  In  the  Soviet  Union,   activists  like  Natan  Sharansky  employed  dark  humor  to  weather  persecution   and  labor  camps.  In  a  “confrontation  with  evil,”  Sharansky  once  observed,  it  is   important  “to  take  yourself  and  everything  that’s  happening  very  seriously,  to   understand  that  you  are  part  of  a  very  important  historical  process,  and  that’s   why  everything  [that]  you’ll  say  and  do  has  tremendous  importance  for  the   future.”  Nevertheless,  he  added,  “it’s  very  important  not  to  take  anything   seriously,  to  be  able  to  laugh  at  everything,  at  the  absurdity  of  this  regime,  at   this  KGB  prison,  and  even  at  yourself.”   Take  your  S   AT  prep  to  the  next  level.  Visit  www.learnerator.com   188  

189       Yes,  North  Korea  has  long  been  ruled  by  an  eccentric  dynasty  of  portly  dictators   with  bad  haircuts.  Yes,  the  propaganda  the  regime  regularly  trumpets  to  shore   up  its  cult  of  personality  is  largely  ridiculous.  And  yes,  we  on  the  outside  know   better,  and  can  take  comfort  in  pointing  fingers  and  chuckling  at  the  regime’s   foibles.     But  it  takes  no  valor  and  costs  precious  little  to  joke  about  these  things  safely   oceans  away  from  North  Korea’s  reach.  When  a  North  Korean  inmate  in  a   political  prison  camp  or  a  closely  monitored  Pyongyang  apparatchik  pokes  fun   at  Kim  Jong  Un  and  the  system  he  represents—that  is  an  act  of  audacity.  It  very   literally  can  cost  the  person’s  life,  and  those  of  his  or  her  family  members.  To   pretend  that  punch  lines  from  afar,  even  in  the  face  of  hollow  North  Korean   threats,  are  righteous  acts  is  nonsense.     What’s  more,  crowding  the  North  Korea  “story”  with  anecdotes  of  nutty   behavior  and  amusing  delusions  may  ironically  benefit  those  in  charge  in   Pyongyang.  It  serves  to  buffer  and  obscure  the  sheer  evil  of  a  regime  that   enslaves  children  and  sentences  entire  families  to  death  for  crimes  of  thought,   while  building  ski  resorts,  dolphinariums,  and  other  luxury  escapes  for  elites   with  funds  that  could  feed  its  malnourished  people  for  several  years.  How  many   people  would  have  watched  The  Interview  and  concluded  that  they  should  do   something  to  help  change  this  odious  regime  and  bring  about  human  rights  for   North  Koreans?     In  Charlie  Chaplin’s  1964  autobiography,  the  star  discussed  the  backlash  that   he  faced  from  Hollywood  and  the  German  and  British  governments  when  plans   for  The  Great  Dictator’s  release  were  announced.  He  moved  forward  with  the   project  despite  these  concerns,  but  years  later  suggested  that  he  regretted  that   decision:  “Had  I  known  of  the  actual  horrors  of  the  German  concentration  

Take  your  S   AT  prep  to  the  next  level.  Visit  www.learnerator.com   189  

190     camps,  I  could  not  have  made  The  Great  Dictator;  I  could  not  have  made  fun  of   the  homicidal  insanity  of  the  Nazis.”     Kim  Jong  Un…  and  his  barons  are  also  representative  of  a  singularly  horrific   system,  one  in  which  the  scale  and  scope  of  suffering  among  25  million  North   Koreans  does  not,  as  a  recent  United  Nations  inquiry  noted,  “have  any  parallel   in  the  contemporary  world.”  North  Korea  is  not  funny.  It  is  hard  to  imagine  a   comparable  comedy  emerging  about  quirky  Islamic  State  slavers  or  amusing   and  “complicated”  genocidaires  in  the  Central  African  Republic.  The  suffering   in  question  is  happening  now,  as  I  write.  The  day  will  soon  come  when  North   Koreans  are  finally  free,  and  liberated  concentration  camp  survivors  will  have   to  learn  that  the  world  was  more  interested  in  the  oddities  of  the  oppressors   than  the  torment  of  the  oppressed.     What  is  the  central  claim  of  this  text?   A.  That  films  can  never  adequately  capture  the  horror  of  man’s  evils   B.  That,  like  Charlie  Chaplin,  the  makers  of  this  film  will  regret  their  decision   to  parody  an  evil  regime   C.  That  comedy  in  the  face  of  evil  is  a  bold  and  audacious  response   D.  That  The  Interview  is  a  poor  vehicle  for  adequately  expressing  the  evils  of   the  North  Korean  regime     Consider  the  title  of  this  text:  North  Korea  Isn’t  Funny.  From  there  it  should  be  fairly   simple  to  isolate  the  central  claim.  Answer  A  is  too  much  of  a  stretch;  in  fact,  the   writer  says  The  movie  would  perhaps  have  been  better  off  with  a  fictitious  dictator   and  regime,  suggesting  that  there  is  a  certain  value  -­‐-­‐  even  if  it  is  just  entertainment  -­‐ -­‐  to  depicting  evil  on  film.  Nor  is  there  evidence  for  B.  C  is  certainly  an  assertion,  as   evidenced  by  this  quote:  “Humor  can  be  a  powerful  tool  for  surviving  in  a  closed   society,  and  lampooning  dictators  can  lend  latent  popular  movements  the  confidence   they  need  to  challenge  their  oppressors.”  But  if  you  follow  the  structure  of  the   argument,  that  is  a  counterclaim,  not  a  central  claim,  and  it  does  not  apply  to  the   Take  your  S   AT  prep  to  the  next  level.  Visit  www.learnerator.com   190  

191     main  premise,  which  is  D  -­‐-­‐  that  The  Interview  is  a  poor  vehicle  for  this  kind  of   commentary,  and  that  “North  Korea  is  not  funny,”  certainly  not  when  those  of  us   poking  fun  at  it  live  thousands  of  miles  away  from  this  regime.  As  the  writer  says:   “But  it  takes  no  valor  and  costs  precious  little  to  joke  about  these  things  safely  oceans   away  from  North  Korea’s  reach.  When  a  North  Korean  inmate  in  a  political  prison   camp  or  a  closely  monitored  Pyongyang  apparatchik  pokes  fun  at  Kim  Jong  Un  and  the   system  he  represents—that  is  an  act  of  audacity.  It  very  literally  can  cost  the  person’s   life,  and  those  of  his  or  her  family  members.  To  pretend  that  punch  lines  from  afar,   even  in  the  face  of  hollow  North  Korean  threats,  are  righteous  acts  is  nonsense.”     What  is  the  writer’s  reasoning  behind  the  claim  that  The  Interview  is  a   poor  vehicle  through  which  to  express  outrage  at  North  Korea’s  various   abuses  of  human  rights?   A.  The  film  is  slightly  racist,  and  thus  inappropriate   B.  The  film  is  not  funny,  and  thus  it  is  inappropriate   C.  The  film  is  a  comedy  and  not  a  meaningful  stand   D.  The  film  will  be  overly  sympathetic  to  the  North  Korean  regime     This  question  asks  you  to  trace  the  writer’s  reasoning.  If  the  central  claim  is  that  the   film  is  inappropriate,  you  must  ask  yourself  as  you  read  each  answer  choice  “Does   this  answer  the  question  of  WHY  the  film  is  an  inappropriate  vehicle?”  A  is  partially   true  -­‐-­‐  the  writer  does  say  that  the  portrayal  is  “cheap  and  sometimes  racism-­‐ tinged.”  Keep  it  for  now.  B  is  not  supported  by  the  text;  the  writer  has  not  seen  the   film  and  makes  no  mention  of  its  quality,  just  its  subject  matter.  C  is  supported   repeatedly  throughout  the  text,  as  the  writer  continually  makes  clear  that  making  a   comedy  about  North  Korea  is  deeply  insensitive  (“The  day  will  soon  come  when   North  Koreans  are  finally  free,  and  liberated  concentration  camp  survivors  will  have  to   learn  that  the  world  was  more  interested  in  the  oddities  of  the  oppressors  than  the   torment  of  the  oppressed”)  as  well  as  lacking  in  any  actual  meaning:  (“But  it  takes  no   valor  and  costs  precious  little  to  joke  about  these  things  safely  oceans  away  from   North  Korea’s  reach.”  “What’s  more,  crowding  the  North  Korea  “story”  with  anecdotes   Take  your  S   AT  prep  to  the  next  level.  Visit  www.learnerator.com   191  

192     of  nutty  behavior  and  amusing  delusions  may  ironically  benefit  those  in  charge  in   Pyongyang.  It  serves  to  buffer  and  obscure  the  sheer  evil  of  [the]  regime.”  “How  many   people  would  have  watched  The  Interview  and  concluded  that  they  should  do   something  to  help  change  this  odious  regime  and  bring  about  human  rights  for  North   Koreans?”)  All  of  these  arguments  demonstrate  the  writer’s  line  of  reasoning  when   s/he  claims  that  the  film  is  inappropriate.  So  while  A  may  be  partially  true,  it  is  in  no   way  the  best  or  most  widely-­‐supported  answer.     As  you  can  see  from  this  article,  in  order  to  assess  reasoning,  you  must  also  be  able   to  cite  evidence  accurately  to  justify  your  assessment;  the  easiest  and  best  way  to  do   this  is  by  asking  yourself  “why?”  or  "Does  this  support  the  central  claim?"  You  will   learn  more  about  this  skill  in  the  next  article.  

Take  your  S   AT  prep  to  the  next  level.  Visit  www.learnerator.com   192  

193    

Analyzing Evidence   Analyzing  Evidence   As  discussed  in  the  last  article,  part  of  being  able  to  assess  an  author’s  argument  is   the  ability  to  analyze  the  evidence  that  s/he  brings  to  justify  the  central  claims  of  the   argument  s/he  makes.  You  must  be  able  point  to  the  claim,  prove  how  the  author   uses  reasoning  to  support  it,  and  analyze  whether  or  not  this  reasoning  (i.e.  the   evidence  presented)  is  convincing.     As  always,  you  must  be  able  to  ask  yourself  “why?  Why  does  the  author  make  this   claim?  How  does  the  author  justify  this  claim?”  If  pieces  of  evidence  thoroughly  and   convincingly  answer  the  central  question  of  “why,”  then  they  work;  if  not,  then  they   don’t.     Try  with  this  scathing  review  of  Love  Actually  from  the  Atlantic.  As  you  read,  assess   what  the  central  claim  is,  and  how  the  writer  supports  the  claim.  Are  his  reasoning   and  his  evidence  convincing?  (Can  you  think  of  a  counter  to  any  of  them?)     ...So  take  the  film  on  its  own  titular  terms.  What  does  Love  Actually  tell  us  about   love,  actually?  Well,  I  think  it  tells  us  a  number  of  things,  most  of  them  wrong   and  a  few  of  them  appalling.  Now,  anyone  who  goes  to  the  cineplex  with  any   regularity  knows  that  the  last  decade  has  seen  more  than  its  share  of  bad   romantic  comedies.  But  Love  Actually  is  exceptional  in  that  it  is  not  merely,  like   so  many  other  entries  in  the  genre,  unromantic.  Rather,  it  is  emphatically,   almost  shockingly,  anti-­‐romantic.     ...The  bulk  of  the  film…  offers  up  at  least  three  disturbing  lessons  about  love.   First,  that  love  is  overwhelmingly  a  product  of  physical  attraction  and  requires   virtually  no  verbal  communication  or  intellectual/emotional  affinity  of  any   kind.  Second,  that  the  principal  barrier  to  consummating  a  relationship  is   Take  your  S   AT  prep  to  the  next  level.  Visit  www.learnerator.com   193  

194     mustering  the  nerve  to  say  “I  love  you”—preferably  with  some  grand  gesture— and  that  once  you  manage  that,  you’re  basically  on  the  fast  track  to  nuptial   bliss.  And  third,  that  any  actual  obstacle  to  romantic  fulfillment,  however   surmountable,  is  not  worth  the  effort  it  would  require  to  overcome.     Begin  with  the  elevation  of  physical  attraction  over  any  of  the  other  factors   typically  associated  with  romantic  compatibility:  similar  likes  and  dislikes,   overlapping  senses  of  humor,  shared  values,  what  have  you.  [One  character]   falls  in  love  with  [another]  the  first  time  he  speaks  with  her—“Get  a  grip,”  he   chides  himself  moments  afterward—when  essentially  the  only  thing  he  knows   about  her  is  that  she  accidentally  uses  profanity  a  lot.  (Charming?  Sure.   Evidence  of  a  soul  mate?  Unlikely.)    [Two  characters]  meanwhile,  fall  in  love   despite  not  sharing  a  word  of  language  in  common.  Moreover,  the  movie   telegraphs  very  clearly  that  the  moment  when  Firth  really  falls  for  Moniz  is   when  he  watches  her  strip  down  to  her  underwear.     ...The  pattern  is  repeated  throughout  the  film.  [One  character]  is  in  love  with  a   beautiful,  popular  girl  at  school  with  whom  he’s  never  spoken.  [One  character]   recognizes  that  a  ray  of  sunshine  may  enter  his  entombed  love  life  the  instant   he  meets  a  mom  who  looks  exactly  like  (i.e.,  is  played  by)  Claudia  Schiffer.  We   can  assume,  I  suppose,  that  [two  characters]  have  had  some  conversations— they  do  work  in  the  same  office,  after  all—but  the  film  doesn’t  bother  to  show   them  having  any.  All  we  know  about  him  is  that  she  thinks  he’s  “too  good  for   her”  and,  later,  that  he  has  washboard  abs.  The  storyline  regarding  [one   character]’s  quest  for  American  babes  is  played  as  a  gag,  of  course:  dorky   British  guy  is  convinced  that  his  accent  will  prove  irresistible  to  super-­‐hotties  in   Wisconsin—and,  lo  and  behold,  he’s  right!  But  the  plotline’s  comically   exaggerated  infatuation  with  physical  attraction  is  actually  not  very  far  out  of   keeping  with  the  rest  of  the  film.    

Take  your  S   AT  prep  to  the  next  level.  Visit  www.learnerator.com   194  

195     Creepiest  of  all  is  the  storyline  involving  [two  characters].  Why  is  he  so   desperately  in  love  with  his  best  friend’s  bride?  Well,  it’s  not  the  result  of  any   conversation  they’ve  had  or  experience  they’ve  shared,  because  the  movie  is  at   pains  to  note  that  he’s  barely  spoken  to  her  and  he  goes  out  of  his  way  to  avoid   her  company.  Indeed,  the  video  tribute  to  her  bridal  radiance  that  he  records  at   her  wedding  makes  pretty  clear  what  it  is  about  her  that  so  captivates  him.   (Hint:  not  her  mind.)  And  he,  too,  like  [another  character]  ultimately  suggests   that  the  only  way  he  will  ever  get  over  this  love  of  his  life  is  by  hooking  up  with   a  supermodel.  I’m  barely  scratching  the  surface  of  what’s  wrong  with  this   subplot—the  movie’s  worst—which  somehow  manages  to  present  the  idea  that   it’s  romantic  to  go  behind  a  friend’s  back  to  ostentatiously  declare  your   everlasting  love  for  his  wife.  But  let’s  not  get  off  track.     This  is  the  point  at  which  defenders  of  the  film  will  reply,  reasonably  enough:  So   what?  In  movies  beautiful  people  always  fall  in  love  with  other  beautiful   people!  What’s  wrong  with  love  at  first  sight,  anyway?  Which  are  both  fair   responses,  as  far  as  they  go.  But  Love  Actually  is  a  considerable  outlier  among   romantic  comedies  in  its  rigorous  conviction  not  only  that  people  fall  in  love   without  really  knowing  one  another,  but  that  they  don’t  even  need  to  learn   anything  about  each  other  to  confirm  their  initial  attraction.     This  is  not  some  abstruse  or  esoteric  component  of  high-­‐end  cinema.  The  core   of  most  romantic  comedies—the  core,  for  that  matter,  of  most  romantic   comedies  written  and/or  directed  by  Richard  Curtis—is  one  form  or  another  of   mutual  exploration  between  potential  lovers.  Some  movies  do  it  well  and  some   do  it  poorly,  but  almost  all  at  least  make  an  effort  to  do  it.  The  protagonists   bicker  their  way  into  love  (27  Dresses,  Sweet  Home  Alabama,  Something's   Gotta  Give...).  The  guy  gradually  persuades  the  gal  that  he’s  worthy,  or  vice   versa  (Groundhog  Day,  Knocked  Up,  Working  Girl  ...).  One  helps  the  other   overcome  a  foolish  obsession  with  a  Mr.  (or  Mrs.)  Wrong  (The  Wedding  Singer,   Forgetting  Sarah  Marshall,  While  You  Were  Sleeping  ...).  The  free  spirit  teaches   Take  your  S   AT  prep  to  the  next  level.  Visit  www.learnerator.com   195  

196     the  control  freak  to  let  go  and  embrace  life  (Along  Came  Polly,  Pretty  Woman,   The  Ugly  Truth  ...).  Opposites  discover  that  they  are  attracted  (Two  Weeks   Notice,  Notting  Hill,  Maid  in  Manhattan  ...).  Etc.,  etc.  My  point  is  in  no  way  to   suggest  that  these  are  all  good  movies.  (They’re  emphatically  not.)  Rather  it  is   to  point  out  just  how  far  outside  the  ordinary  it  is  that  none  of  Love  Actually’s   fated  couples  spends  any  meaningful  time  getting  to  know  one  another  at  all.       What  is  the  author’s  main  source  of  support  for  his  central  claim  that   the  film  Love  Actually  is  “anti-­‐romantic”?   A.  Unlike  other  films,  it  doesn’t  accurately  portray  relationships   B.  The  film  offers  up  three  disturbing  lessons  about  love   C.  The  film  is  about  love  at  first  sight,  which  is  an  unrealistic  concept   D.  The  film  emphasizes  physical  attraction  over  everything  else     The  author  is  clearly  building  a  case  against  Love  Actually.  While  he  does  compare  it   to  other  films,  he  does  not  suggest  that  these  other  films  are  “accurate,”  which  leaves   A  in  the  dust.  B  is  true,  and  a  direct  quote  from  the  text,  but  it  is  not  support  for  the   idea  that  Love  Actually  is  anti-­‐romantic;  it’s  more  an  extension  of  the  argument  as  it   does  not  adequately  answer  the  question  of  “why.”  It  just  makes  a  claim  but  doesn’t   back  it  up.  C  is  also  true-­‐ish,  but  it  isn’t  the  main  source  from  where  the  author   draws  his  argument.  Instead,  the  author  spends  nearly  the  entirety  of  this  article   proving  that  D,  the  film  emphasizes  physical  attraction  over  everything  else,  and  this   is  the  real  reason  that  the  film  is  anti-­‐romantic.     How  does  the  author  use  evidence  to  support  his  argument?   A.  The  author  gives  multiple  examples  from  the  film  that  all  function  as   support  for  his  central  claim   B.  The  author  compares  Love  Actually  to  other  films  to  support  his  central   claim   C.  The  author  effectively  addresses  a  counterclaim  in  order  to  support  his  

Take  your  S   AT  prep  to  the  next  level.  Visit  www.learnerator.com   196  

197     argument   D.  All  of  the  above     This  question  doesn’t  ask  you  what  the  author’s  evidence  IS,  but  rather  asks  you  to   evaluate  his  strategy.  A  is  clearly  true;  the  majority  of  the  article  is  given  over  to   discussing  the  various  characters  and  their  relationships.  The  author  does  compare   the  film  to  others  in  its  genre,  all  to  demonstrate  that  its  conception  of  relationships   is  “far  outside  the  ordinary.”  The  author  does  address  the  counterclaim  (“beautiful   people  fall  in  love  all  the  time”)  by  saying  that  Love  Actually’s  idea  of  “love  at  first   sight”  does  not  fit  any  sort  of  accepted  “love  at  first  sight”  model  because  the   characters  do  not  even  speak  to  each  other  once  they  have  fallen  in  love.  Clearly,  D  is   the  best  answer  -­‐-­‐  the  author  uses  multiple  tactics  in  order  to  demonstrate  his  main   point.     Try  once  again  to  analyze  the  evidence  that  an  author  uses  to  structure  and  support   the  following  argument.  As  you  read,  make  sure  you  are  paying  attention  to  the   central  claim  as  well  as  to  HOW  the  author  uses  evidence  to  prove  his  point,  as  well   as  the  actual  content  of  the  evidence,  and  whether  or  not  it  effectively  supports  the   central  claim.  This  article  is  also  from  The  Atlantic.     One  of  the  most  celebrated  pieces  of  advice  to  writers  is  “Write  what  you  know.”   Unfortunately,  it  shows.     The  demographics  of  published  writers  in  the  West  are  largely  homogeneous,   and  as  a  result,  our  literature  is  also  largely  homogeneous.  Growing  up,  for   example,  my  heroes  were  Atreju,  Frodo,  and  Paul  Atreides.  All  I  ever  really   wanted  to  do  was  go  on  adventures  like  them.  I  readily  identified  with  them,   and  their  trials  became  my  scripture:  the  loss  of  Artax,  the  recovery  at   Lothlórien,  the  knife  fight  with  Feyd-­‐Rautha.  Despite  a  liberal  upbringing  and   an  education  at  a  women’s  college,  it  didn’t  occur  to  me  that  my  identification   with  male  heroes  had  damaged  me  in  any  way—that  is,  until  I  became  a  writer,   Take  your  S   AT  prep  to  the  next  level.  Visit  www.learnerator.com   197  

198     and  found  myself  weirdly  reluctant  to  write  a  woman  hero.  This  wasn’t  an   accident.     As  Vanessa  Veselka  wrote  in  The  American  Reader,  there  is  a  profound  relative   lack  of  female  road  narratives  in  the  Western  literary  tradition.  This  absence   hurt  her  in  much  more  concrete  ways.  When  recounting  her  years  as  a  teenage   hitchhiker,  Veselka  writes,  “my  survival  depended  on  other  people’s  ability  to   envision  a  possible  future  for  me…[but]  there  was  no  cultural  narrative  for  [us]   beyond  rape  and  death.”  Male  hitchhikers  had  Jack  Kerouac,  Walt  Whitman,   and  dozens  of  others.  Veselka  had  bodies  in  dumpsters  on  the  six  o’clock  news.     Meanwhile,  in  Joseph  Campbell’s  The  Hero  With  a  Thousand  Faces,  a  work  that   compares  mythologies  from  cultures  around  the  globe,  the  hero  pretty  much   just  has  one  face:  that  of  a  white  (or  white-­‐washed)  man.  Women  are  usually   guiding  spirits  or  goddesses  encountered  along  the  way,  not  the  heroes   themselves.  This  has  troubling  implications  when  we  view  writing  stories  as  an   act  of  creation:  not  just  of  a  narrative,  but  of  the  society  in  which  we  live,  and   the  possibilities  prescribed  for  the  people  who  live  in  it.  Literature  is  our   collective  creation  myth.     ...Western  writers  still  make  up  the  majority  of  published  English  language   authors,  and  English  is  one  of  the  global  lingua  franca.  Western  literature   already  has  extraordinary  women  heroes  created  by  extraordinary  writers:   Toni  Morrison’s  Sethe,  Ursula  Le  Guin’s  Tenar,  Chimamanda  Ngozi  Adichie’s   Ifemelu.  But  they’re  a  tiny  proportion  of  the  whole.  We  need  more.  Writing   characters  different  from  us—for  all  creators,  in  all  directions—is  integral  to   creating  a  literature  in  which  all  phenotypes  are  heroic,  and  therefore,  all  are   humanized.     I  won’t  go  so  far  as  to  say  that  creators  have  a  responsibility  to  do  so.  But  I  do   submit  that  our  increasingly  global  society  offers  an  unprecedented   Take  your  S   AT  prep  to  the  next  level.  Visit  www.learnerator.com   198  

199     opportunity  for  all  creators  to  write  what  we  don’t  know.  The  defining  heroic   journey  of  the  20th  century  was  to  conquer  evil:  the  Nothing,  Sauron,  the   Harkonnens.  But  the  defining  heroic  journey  of  the  21st  century  will  be  to   reconcile  the  Other  with  the  Self.     What  is  the  writer’s  central  premise  in  this  text?   A.  That  the  lack  of  diversity  in  Western  literature  has  concrete  negative   effects  on  our  society   B.  That  writers  have  a  responsibility  to  the  reader  to  create  more  diverse   characters   C.  That  the  advice  “write  what  you  know”  is  mistaken  and  should  be   rethought   D.  That  identification  with  male  heroes  is  damaging     The  writer  does  spend  the  bulk  of  the  text  lamenting  the  lack  of  diversity  in  Western   literature,  first  citing  her  own  experiences  reading  exclusively  male  protagonists,   then  citing  other  cases  of  women  who  have  found  themselves  stymied  and   frustrated  by  the  lack  of  protagonists  and  narratives  that  accurately  represent  the   vast  spectrum  of  humanity.  Keep  A  for  now.  B  is  clearly  wrong,  as  the  writer  says  in   the  last  paragraph  “I  won’t  go  so  far  as  to  say  that  creators  have  a  responsibility  [to   write  characters  different  from  themselves].”  She  does  start  off  with  C  -­‐-­‐  that  “write   what  you  know”  is  popular  advice  -­‐-­‐  but  she  doesn’t  advocate  letting  go  of  this   advice  entirely.  And  while  she  does  claim  that  her  identification  with  male  heroes   was  personally  damaging,  this  is  not  the  central  claim  of  her  article  -­‐-­‐  rather,  it  is  a   piece  of  evidence  that  she  uses  to  bolster  her  claim.  (It  answers  the  question   “why?”)     Which  is  the  most  effective  piece  of  evidence  that  the  writer  uses,  and   why?   A.  Her  assertion  that  her  own  identification  with  male  heroes  was  damaging,   because  it  adds  a  personal  touch  to  this  persuasive  text  that  helps  the  reader   Take  your  S   AT  prep  to  the  next  level.  Visit  www.learnerator.com   199  

200     connect  better  with  its  content   B.  Her  example  of  Vanessa  Veselka’s  lack  of  female  role  models  in  the  road   narrative,  because  it  graphically  emphasizes  the  gulf  between  what  is  viewed   as  possible  for  men  and  what  is  viewed  as  possible  for  women   C.  Her  example  of  the  different  evils  our  literature  has  had  to  conquer,   because  they  give  a  real  face  to  the  problem  that  our  literature  and  our   society  face   D.  Her  assertion  that  literature  is  our  “collective  creation  myth,”  because  this   illustrates  just  how  deep  this  problem  runs.     Remember,  the  writer  uses  evidence  to  support  her  central  claim  that  the  lack  of   diversity  in  modern  literature  is  deeply  problematic.  To  be  effective,  these  pieces  of   evidence  must  answer  the  question  of  WHY  this  assertion  is  true.  Read  A;  while  this   is  certainly  evidence  that  she  used,  and  while  it  does  add  a  personal  touch,  consider   whether  it  indeed  helps  the  reader  connect  better  with  the  content.  Does  this   accurately  demonstrate  the  extent  or  seriousness  of  the  damage  done  to  our   society?  Keep  A  for  now,  as  it  does  reflect  a  form  of  damage,  but  consider  whether   it’s  the  best  answer.  B  is  certainly  true;  it  is  extremely  damaging  for  a  young  woman   to  see  only  two  (very  dark)  options  for  herself  as  she  embarks  on  a  road  trip.  The   fact  that  women  are  missing  entirely  from  the  road  narrative  certainly  constitutes  a   compelling  piece  of  evidence  from  the  assertion  that  the  lack  of  diversity  in   literature  is  indeed  damaging  to  our  society.  C  is  a  clever  answer  that  in  fact  has   nothing  to  do  with  the  argument;  she  gives  an  example  of  these  evils  (the  Nothing,   Sauron,  etc)  only  to  contrast  these  villains  with  the  villains  that  we  find  within   ourselves  (“the  Self  and  the  Other”).  And  D  is  also  a  clever  answer,  but  it  doesn’t   prove  her  point  since  there  is  no  tangible  evidence  for  it;  it  is  just  another  piece  or   extension  of  her  central  argument.  Between  A  and  B,  B  is  the  better  answer;  while   both  provide  examples  of  the  kind  of  damage  caused  by  the  lack  of  diversity  in   literature,  B’s  example  is  far  more  graphic,  pervasive,  and  violent,  and  therefore   more  convincing  as  an  example  of  "damage."     Take  your  S   AT  prep  to  the  next  level.  Visit  www.learnerator.com   200  

201     As  you  read  and  analyze  evidence  and  claims,  make  sure  that  you  understand  the   distinction  between  evidence  /  support  and  the  actual  argument  itself.  The  new  SAT   will  attempt  to  distract  you  by  offering  you  answer  choices  about  pieces  of   “evidence”  that  are  simply  extensions  of  the  author’s  argument.  Watch  out  for  this,   and  make  sure  you  are  always  asking  yourself  “does  this  answer  the  question  of   ‘why’?”    

 

 

Take  your  S   AT  prep  to  the  next  level.  Visit  www.learnerator.com   201  

202    

 

            Part  III:  Synthesis  (Reading)  

 

 

 

Take  your  S   AT  prep  to  the  next  level.  Visit  www.learnerator.com   202  

203    

Take  your  S   AT  prep  to  the  next  level.  Visit  www.learnerator.com   203  

204     Of  the  various  skills  required  for  success  on  the  SAT  (as  well  as  other  academic   pursuits),  one  of  the  most  important  (and  sometimes  the  most  difficult  for  students)   is  synthesis,  or  the  ability  to  understand  and  integrate  the  information  found  in   multiple  texts.  A  truly  high-­‐level  student  can  extract  relevant  information  from   multiple  texts  and  weave  together  a  coherent  argument  that  takes  into  account   these  various  sources  of  information.     Synthesis  is  just  a  combination  of  the  skills  you  have  reinforced  over  the  course  of   the  previous  articles,  but  in  the  context  of  multiple  texts.  This  skill  requires  you  to   read  carefully,  think  critically,  make  inferences,  asses  reasoning,  analyze  evidence,   and  form  justifiable  opinions  -­‐-­‐  on  basis  of  more  than  one  passage.  Sometimes  the   texts  will  present  two  sides  of  the  same  argument  -­‐-­‐  in  some  ways  making  your  job   of  analysis  easier  -­‐-­‐  and  sometimes  they  will  not.  Try  your  hand  at  analyzing  the   following  two  texts;  as  you  read,  pay  careful  attention  to  similarities  and  differences   between  the  two.  It  may  be  helpful  to  make  a  chart  of  the  various  ways  in  which   these  texts  discuss  the  same  main  ideas:     Text  One,  from  The  University  of  Florida  Institute  of  Food  and  Agricultural   Sciences     A  species  is  endangered  when  it  is  threatened  with  extinction.  Since  time  began,   countless  species  have  gone  extinct  from  natural  processes.  The  extinction  of   dinosaurs  is  the  best  known  example.     Why  Save  Endangered  Species?   If  extinction  is  a  natural  process,  why  should  we  make  an  effort  to  save   endangered  species?  Because  we  can  no  longer  attribute  the  accelerating   extinction  of  plants  and  animals  to  natural  causes.  Today  most  species  of  plants   and  animals  become  extinct  because  of  habitat  destruction  (loss  of  living  space   to  development  or  pollution),  introduction  of  non-­‐native  organisms,  and  direct   killing  (over-­‐harvesting,  poisoning)....   Take  your  S   AT  prep  to  the  next  level.  Visit  www.learnerator.com   204  

205       Our  understanding  of  the  value  of  endangered  species  to  humans  has  increased   together  with  the  recognition  that  human  activities  cause  extinction.  In   general,  benefits  of  species  can  be  classified  as  ecological,  economic,  and  social.   Different  combinations  of  benefits  occur  for  any  particular  species,  and  some   species  are  obviously  more  "valuable"  than  others.  More  important  than   knowing  why  a  particular  species  is  valuable  is  understanding  why  so  many   kinds  of  plants  and  animals  are  valuable.     Biological  Diversity   The  assemblage  of  populations  of  plants  and  animals  in  an  area  is  termed  its   "biological  diversity."  The  term  biological  diversity  is  often  used   interchangeably  (sometimes  confusingly)  with  two  other  terms,  "genetic   diversity"  and  "ecological  diversity."  Genetic  diversity  (amount  of  genetic   variability  among  individuals  of  the  same  species)  and  ecological  diversity   (number  and  relative  abundance  of  species)  are  both  components  of  biological   diversity.     Genetic  diversity  is  directly  related  to  a  species'  ability  to  survive  environmental   change.  For  example,  plants  and  animals  can  be  characterized  by  their  ability   to  exist  under  different  climatic  (moisture  and  temperature)  conditions.     However,  within  different  species  there  is  a  certain  amount  of  variability  in  the   tolerance  of  individuals  to  climatic  conditions.  The  ability  of  different  species  to   cope  with  environmental-­‐-­‐in  this  example  climatic-­‐-­‐  change  depends  on  this   variability.  When  genetic  variability  is  reduced,  as  with  the  Florida  panther,  the   risk  of  extinction  increases.     The  loss  of  a  single  species  can  set  off  a  chain  reaction  affecting  many  other   species.  The  total  impact  of  extinction  is  not  always  apparent,  and  is  difficult  to  

Take  your  S   AT  prep  to  the  next  level.  Visit  www.learnerator.com   205  

206     predict,  but  it  is  clear  that  conserving  biological  diversity  is  essential  for   maintaining  intact  ecosystems.     Benefits  of  Biological  Diversity   How  does  maintaining  biological  diversity  benefit  humanity?  It  only  takes  a   moment  to  realize  that  throughout  history  plants  and  animals  have  provided   humans  with  food,  clothing,  energy,  medicines,  and  structural  materials.  Today,   solutions  to  problems  in  agricultural  production  in  tropical  countries,  reliance   on  petrochemicals,  and  the  cures  for  cancers  may  lie  in  organisms  not  yet   discovered.  It  would  be  a  shame  to  lose  these  benefits  without  even  knowing  we   had  them.     Text  Two,  Public  Library  Of  Science  Blogs  Network  -­‐  Plants  with  Personality     If  you  had  to  be  an  endangered  animal,  you’d  be  better  off  as  a  tiger  than  a   toad.  If  you  were  a  tiger,  filmmakers  might  cast  you  in  wildlife  documentaries   and  journalists  might  write  heart-­‐rending  stories  about  the  disappearance  of   your  kind.  Your  furry  mug  might  appear  on  magazine  covers  and  postage   stamps.  And  conservation  organizations  just  might  make  you  their  flagship   species,  a  stand-­‐in  for  all  the  critters  whose  survival  is  threatened.  In  other   words,  if  you  were  a  tiger,  you  might  have  a  fighting  chance  of  at  least  making   humans  care  about  your  predicament.     That’s  a  taller  order  if  you’re  a  toad,  an  animal  that  wins  over  few  human   hearts.  Instead,  we  prefer  the  so-­‐called  “charismatic  mega  fauna,”  funneling   our  emotional  and  conservational  energies  into  species  like  tigers,  lions,   elephants,  dolphins,  pandas,  and  the  like.   A  number  of  psychologists  and  biologists  have  begun  to  uncover  why  some   species  appeal  to  us  more  than  others,  identifying  a  number  of  factors  that   make  certain  kinds  of  critters  especially  attractive.  For  instance,  we  have  a  soft   spot  for  our  fellow  mammals,  and  we    prefer  big  beasts  to  smaller  ones.  We’re   Take  your  S   AT  prep  to  the  next  level.  Visit  www.learnerator.com   206  

207     also  strongly  attracted  to  “neotenic,”  or  juvenile-­‐looking,  features.  The   youngsters  of  many  species  have  large  heads,  large  eyes,  big  foreheads,  and   snub  noses.  Human  infants  have  these  characteristics,  as  do  puppies,  kittens,   and  all  sorts  of  other  critters  that  we  find  cute.  In  some  species,  adult  animals   retain  features  associated  with  youth–such  as  oversized  eyes–and  we’re   naturally  drawn  to  these  neotenic  faces.     ...Our  preferences  for  certain  species  over  others  have  serious  implications  for   conservation.    Studies  have  shown  that  charismatic  mega  fauna  attract  more   than  their  fair  share  of  conservation  attention  and  funding.  As  Stokes  put  it  in   his  2006  paper:  “Much  of  the  world’s  biodiversity  will  survive  only  if  humans   choose  to  protect  it.  Given  that  people  are  likely  to  protect  what  is  important  to   them,  human  preferences  will  be  important  determinants  of  many  species’   prospects  for  survival…”     As  you  read,  take  note  of  the  fact  that  these  two  articles  clearly  discuss  a  similar   theme  -­‐-­‐  but  that  they  take  very  different  angles  on  this  theme.  Make  sure  that  you   pay  attention  to  similarities  and  differences  in  context,  tone,  style,  and  structure  -­‐-­‐   and,  of  course,  to  the  content  of  the  passages  themselves.     What  is  the  best  way  to  characterize  the  difference  between  Text  One   and  Text  Two?   A.  Text  One  is  persuasive,  while  Text  Two  is  purely  informative   B.  Text  One  contains  much  scientific  jargon,  whereas  Text  Two  is  highly   descriptive   C.  Text  One  provides  general  information  about  a  scientific  context  while   Text  Two  details  a  phenomenon  specific  to  that  context   D.  Text  One  is  a  call  to  action  whereas  Text  Two  is  clearly  written  increase   awareness    

Take  your  S   AT  prep  to  the  next  level.  Visit  www.learnerator.com   207  

208     These  two  texts,  while  somewhat  related  in  their  subject  matter,  are  in  fact  very   different  -­‐-­‐  partially  because  of  their  structure,  partially  because  of  their  tone,  and   partially  because  of  their  content.  However,  A  claims  that  Text  One  is  persuasive,   which  -­‐-­‐  if  you  read  it  closely  -­‐-­‐  it  isn’t.  It  doesn’t  contain  much  of  a  central  claim,   other  than  the  fact  that  extinction  is  a  bad  thing;  nor  does  it  spend  too  much  time   trying  to  convince  you  that  extinction  is  a  bad  thing.  Instead,  it  simply  and  clearly   lays  out  the  facts  about  extinction  and  genetic,  ecological,  and  biological  diversity.   For  this  reason  alone  answer  A  is  wrong.  B  is  incorrect  as  well  -­‐-­‐  while  it’s  true  that   Text  One  contains  many  scientific  definitions,  Text  Two  contains  at  least  two   unfamiliar  scientific  words  as  well  (“charismatic  megafauna”  and  “neotenic.”)   Additionally,  Text  Two  hardly  contains  multiple  adjectives  or  long  descriptions;  it  is   somewhat  informal,  rather  than  “highly  descriptive.”  Answer  C  is  true;  Text  One   DOES  provide  general  information  about  why  endangerment  and  extinction  are   highly  negative  phenomena,  and  Text  Two  DOES  detail  a  specific  phenomenon   within  the  context  of  endangerment  and  extinction,  as  it  describes  the  animals  that   are  most  likely  to  be  helped  or  saved  by  concerned  humans.  D  is  incorrect;  Text  One,   objective  as  it  is,  is  hardly  a  call  to  action.  There  are  no  strong  words  and  there  is  no   direct  address  of  the  reader;  simply  detailing  why  extinction  and  endangerment  are   negative  things  surely  does  not  constitute  a  call  to  action.  And  while  Text  Two  DOES   inform  the  reader  of  a  specific  phenomenon,  it’s  hard  to  get  away  with  saying  that   its  primary  aim  is  to  “raise  awareness,”  as  that  has  a  political  connotation  that  is   absent  here.  All  in  all,  C  is  the  best  answer.     In  what  way  do  the  structures  of  these  two  texts  help  to  shape  the  way   they  convey  information?   A.  Text  One  provides  a  sequence  of  events  that  allows  readers  to  better   understand  the  process  of  extinction;  Text  Two  offers  no  such  sequence   B.  Both  texts  provide  in-­‐depth,  objective  explanations  of  scientific   phenomena,  allowing  the  reader  to  better  understand  the  material   C.  Both  texts  lay  out  a  problem  and  suggest  possible  solutions,  offering  the   reader  an  insight  into  some  scientists’  approaches  to  mass  extinctions   Take  your  S   AT  prep  to  the  next  level.  Visit  www.learnerator.com   208  

209     D.  Text  One  defines  important  terms  while  Text  Two  outlines  cause  and   effect,  so  that  readers  are  better  able  to  understand  the  immediacy  and   importance  of  the  phenomena  described     There  is  a  definite  text  structure  to  Text  One;  the  multiple  subheadings  should  alert   you  to  that  immediately.  However,  this  text  structure  is  not  at  all  a  sequence  of   events;  you’ll  be  hard  pressed  to  find  even  one  chronologically  significant  signal   word  (e.g.  “first,”  “second,”  “third,”  etc.)  For  this  alone  you  can  get  rid  of  answer  A.  B   is  certainly  true  of  Text  One,  but  it’s  hard  to  make  that  same  case  for  Text  Two,  as   the  majority  of  the  content  is  taken  up  with  a  catchy  introduction  and  a  less-­‐than-­‐ objective-­‐sounding  tone.  Keep  B  for  now  but  consider  better  options.  The  first  part   of  C  is  true  -­‐-­‐  both  texts  do  lay  out  a  problem  -­‐-­‐  but  neither  proposes  a  possible   solution.  D  is  true  -­‐-­‐  Text  One  is  full  of  important  terms  that  help  the  reader  better   understand  the  phenomena  described,  while  Text  Two  does  provide  a  cause   (humans  are  predisposed  to  like  “neotenic”  animals)  and  effect  (non-­‐neotenic   endangered  animals  are  in  greater  danger  than  neotenic  ones).  Of  all  the  answers,  D   is  the  most  true.     Both  texts  can  be  said  to  have  the  aim  of…   A.  Galvanizing  the  reader  towards  change   B.  Frightening  the  reader  with  grim  tidings   C.  Educating  the  reader  with  accessible  language   D.  Propelling  the  reader  towards  an  unavoidable  conclusion     This  question  asks  you  to  make  inferences  regarding  why  both  of  these  texts  were   written.  Assess  the  tone  of  these  texts  -­‐-­‐  one  is  neutral  and  objective,  laying  out  facts,   while  the  other  is  informal  and  engages  more  with  the  reader.  But  both  contain   important  scientific  information  that  sheds  light  on  certain  aspects  of  the  same   phenomenon.  A  may  be  correct  -­‐-­‐  it’s  possible  that  the  writers  of  both  these  texts   wished  to  spur  the  reader  towards  some  kind  of  action  -­‐-­‐  but  it’s  also  quite  doubtful   considering  that  neither  of  these  can  really  be  considered  a  call  to  action,  as  neither   Take  your  S   AT  prep  to  the  next  level.  Visit  www.learnerator.com   209  

210     uses  fiery  language  or  impels  the  reader  towards  any  real  kind  of  action.   Additionally,  B  is  also  possible  -­‐-­‐  but  it  is  unlikely  considering  the  notable  absence  of   any  strong  language  or  sobering  statistics.  C  is  certainly  true  -­‐-­‐  though  these  articles   do  contain  some  “jargon,”  they  both  take  care  to  either  define  these  terms  or  to  give   the  reader  enough  context  to  understand  them,  and  both  pieces  are  written  in  a   style  that  is  easy  to  follow;  additionally,  the  textbook-­‐like  tone  of  the  first  passage  is   a  strong  indicator  that  the  author  wished  to  educate  and  inform  the  reader.  D  is   interesting,  but  not  very  compelling  -­‐-­‐  which  conclusion,  exactly,  does  the  second   article  try  to  propel  the  reader  towards?  What  words  or  sentences  can  be  said  to  be   “propelling”  the  reader?  Though  you  may  indeed  reach  an  “unavoidable”  conclusion   after  reading  these  pieces,  the  neutral,  subdued  tone  of  the  first  piece  and  the  lack  of   any  conclusion  in  the  second  make  it  very  hard  to  find  evidence  for  D  being  the  right   answer.  C  is  the  best  answer.     Both  Text  One  and  Text  Two  contain  unfamiliar  words  and  phrases.   What  is  the  approach  these  texts  take  to  defining  these  phrases?   A.  Both  texts  consistently  define  every  unfamiliar  phrase  they  use   B.  Text  One  consistently  defines  phrases  whereas  Text  Two  defines  some   phrases  and  lets  the  reader  infer  the  meaning  of  others   C.  Text  One  does  not  define  relevant  phrases  whereas  Text  Two  does   D.  Neither  text  adequately  explains  the  scientific  phrases  it  uses     As  explained  above,  these  texts  have  similar  aims  in  that  they  both  seek  to  make   scientific  phenomena  clear  and  easy  for  the  layperson  to  understand.  Therefore   their  approach  to  phrasing  and  terminology  is  very  important;  if  you  look  closely  at   Text  One,  it  certainly  defines  all  the  terms  it  uses,  from  “endangered”  to  “genetic   diversity.”  Yet  Text  Two  does  not  ever  define  the  phrase  “charismatic  mega  fauna,”   leaving  it  to  the  reader  to  figure  it  out.  This  invalidates  Answer  A  and  makes  Answer   B  the  only  correct  choice.    

Take  your  S   AT  prep  to  the  next  level.  Visit  www.learnerator.com   210  

211     Based  on  Text  Two,  “charismatic  mega  fauna”  most  closely  means…   A.  Those  with  great  personalities   B.  Humans  invested  in  saving  animals   C.  Animals  that  humans  feel  compelled  to  save   D.  Plants  with  personality     You  may  be  tempted  to  choose  “A”  based  on  your  understanding  of  the  word   “charisma”  (magnetism),  but  remember,  this  is  a  scientific  article  about  animals,  so   it’s  unlikely  that  the  writer  would  devote  much  time  to  considering  animals’  “great   personalities.”  B  is  clearly  wrong;  the  article  uses  the  phrase  in  the  following   context:     That’s  a  taller  order  if  you’re  a  toad,  an  animal  that  wins  over  few  human  hearts.   Instead,  we  prefer  the  so-­‐called  “charismatic  megafauna,”  funneling  our  emotional   and  conservational  energies  into  species  like  tigers,  lions,  elephants,  dolphins,  pandas,   and  the  like.     Replace  the  word  “charismatic  megafauna”  with  “humans  invested  in  saving   animals”  and  it  quickly  becomes  clear  how  ridiculous  this  answer  is.  C  is  a  much   better  choice.  D  references  the  title  of  the  piece,  but  as  there  is  no  mention  of  plants   yet,  it  makes  little  sense  in  this  context.  Lastly,  if  you  know  the  meaning  of  “flora”   (plants)  and  “fauna”  (animals),  the  meaning  of  this  should  be  clear,  as  only  one  of   the  choices  is  “animals.”     How  do  the  varying  tones  of  these  works  influence  the  relationship   between  the  reader  and  the  texts?   A.  The  objective  tone  of  Text  One  introduces  a  problem  to  the  reader,   whereas  the  informal  tone  of  Text  Two  allows  the  reader  to  feel  satisfied  that   the  problem  is  being  addressed   B.  The  second-­‐person  tone  of  Text  Two  contains  a  subtle  indictment  of  the   reader,  whereas  the  detached  tone  of  Text  One  distances  the  reader  from  the   Take  your  S   AT  prep  to  the  next  level.  Visit  www.learnerator.com   211  

212     immediate  problems  of  endangered  species   C.  The  jargon  in  the  first  text  gives  the  reader  an  unbiased,  in-­‐depth   perspective,  while  the  informal  tone  of  the  second  text  clearly  reveals  the   author’s  perspective   D.  The  textbook-­‐like  tone  of  the  first  text,  contrasted  with  the  informal  tone   of  the  second,  make  it  clear  that  these  articles  would  be  read  in  very  different   contexts.     This  question  is  in  essence  asking  you  about  the  tone  of  the  text.  Don’t  be  fooled  by   this  jargon-­‐y  idea  of  “the  relationship  between  the  reader  and  the  texts;”  simply   focus  on  the  fact  that  the  question  is  asking  you  about  the  tone.  Assess  the  answer   choices  and  see  which  makes  the  most  sense  -­‐-­‐  even  if  you  don’t  understand  the   question,  you  should  be  able  to  eliminate  answer  choices  because  some  of  them  will   contain  blatantly  inaccurate  information…  such  as  answer  A.  While  the  first  part  is   true,  the  informal  tone  of  Text  Two  does  nothing  to  address  the  problem.  In  fact,  all   the  text  does  is  present  a  problem,  without  even  a  mention  of  a  solution.  This   answer  is  wrong.  B  is  also  wrong;  there  is  absolutely  no  “subtle  indictment”  of  the   reader  here.  Or  rather,  if  there  is  a  subtle  indictment  (criticism)  of  the  reader,  it  is  so   subtle  as  to  be  nonexistent.  There  are  simply  no  words  in  this  text  that  contain  any   sort  of  criticism  of  the  reader,  or  even  imply  it.  This  alone  makes  the  answer  wrong.   C  is  also  wrong;  we  have  no  idea  what  the  author’s  perspective  is  for  the  second   article.  Just  because  an  article  is  informal  does  not  mean  it  is  biased;  this  is   important  to  remember.  The  second  article  still  clearly  lays  out  all  of  the  facts,  and   quotes  experts  in  the  field  -­‐-­‐  while  at  the  same  time  not  even  hinting  at  the  author’s   perspective.  The  only  answer  left  is  D,  and  it  is  clearly  true  -­‐-­‐  these  clearly  WERE   written  for  very  different  contexts,  such  as  a  textbook  or  a  magazine.  In  fact,  you   don’t  need  much  beyond  the  little  blurb  at  the  top  of  each  text  to  tell  you  this  -­‐-­‐  Text   One  comes  from  a  college  website  while  Text  Two  comes  from  the  Public  Library  of   Science’s  Blogs  network.  Even  if  you’re  not  totally  sure  what  a  question  is  asking   you,  simply  paying  attention  to  the  context  can  sometimes  be  enough  to  propel  you  

Take  your  S   AT  prep  to  the  next  level.  Visit  www.learnerator.com   212  

213     towards  the  right  answer.     These  two  passages  are  best  read  together  because…   A.  Text  Two  defines  and  explains  Text  One   B.  Text  Two  gives  a  reason  for  a  phenomenon  found  in  Text  One   C.  They  give  two  oppositional  sides  of  one  story   D.  Text  One  contains  a  general  picture  of  a  phenomenon  while  Text  Two   details  a  specific  feature  of  this  phenomenon   This  question  asks  you  to  consider  why  the  evil  geniuses  behind  the  SAT  paired   these  texts.  Clearly,  all  paired  texts  will  have  some  sort  of  relationship.  Clearly,  A  is   incorrect;  considering  its  definition-­‐heavy  tone,Text  One  needs  no  additional   definition  or  explanation.  B  is  tempting,  but  please  consider  that  Text  Two  does  not   actually  give  a  REASON  that  certain  species  are  endangered  -­‐-­‐  it  simply  gives  a   reason  for  why  certain  endangered  species  are  more  “cared  about”  or  impacted  by   conservation  efforts.  Nor  is  C  correct;  these  two  articles  do  not  give  opposing  sides   or  differing  accounts;  they  both  simply  describe  certain  features  of  endangerment.  D   is  clearly  the  best  answer  -­‐-­‐  while  Text  One  is  about  a  general  phenomenon   (endangerment  and  extinction),  Text  Two  addresses  another,  smaller  phenomenon   that  is  only  possible  in  context  of  the  phenomenon  described  in  Text  One.  Clearly,  D   is  the  best  answer.     As  you  can  see,  synthesis  will  mostly  center  on  asking  you  to  compare  and  contrast   the  various  tones  and  approaches  that  two  texts  take.  Remember,  the  SAT  pairs   texts  for  a  reason,  and  it  won’t  always  be  as  simple  as  “one  is  pro,  one  is  con.”  For   example,  the  texts  above  are  clearly  related  yet  not  at  all  the  same;  as  soon  as  you’ve   assessed  the  similarities  (thematic  or  otherwise)  between  two  texts,  it  will  become   much  easier  to  assess  how  they  diverge  and  how  they  complement  one  another.   This  will  make  it  easier  to  understand  how  to  answer  Synthesis  questions.    

Take  your  S   AT  prep  to  the  next  level.  Visit  www.learnerator.com   213  

214    

Analyzing Quantitative Information The  new  SAT  will  have  a  strong  emphasis  on  the  role  that  quantitative  information   plays  within  a  larger  text.  In  a  world  that  is  increasingly  governed  by  data,  the   ability  to  identify,  understand,  summarize,  and  effectively  use  data  to  come  to   conclusions  is  paramount.  Data  will  make  an  appearance  on  all  sections  of  the  SAT  -­‐-­‐   not  just  math,  but  Reading  and  Writing,  too.  Luckily,  data  analysis  is  just  another   form  of  reading  comprehension,  so  all  of  the  work  you  have  been  doing  in  the  past   few  articles  will  help  you  form  data-­‐based  conclusions.   Perhaps  the  most  important  part  of  analyzing  quantitative  information  is  simply  to   pay  attention.  Look  at  what  information  the  data  is  presenting.  Read  all  parts  of  the   graph  or  chart,  including  the  labeling  of  the  horizontal  and  vertical  axes,  and  make   sure  that  you  pay  attention  also  to  the  way  information  and  numbers  are   categorized.  Remember  also  that  charts  and  data  are  not  there  to  confuse  you;   instead,  they  are  there  to  help  you  better  understand  the  text  of  the  article,  so  think   of  them  as  an  added  benefit  to  your  reading.   Let’s  start  with  the  following  article,  linked  here.  As  you  can  see,  the  map  is  clearly   here  to  illustrate  an  essential  theme  of  the  article.  The  article  asserts  that  North   Korea  is  very  poor  and  gives  a  map  as  evidence,  then  explains  how  the  map   functions.     What  does  the  inclusion  of  this  map  show  about  poverty  in  North   Korea?   A.  That  it  is  restricted  to  only  a  few  small  sections  of  the  country   B.  That  economic  sanctions  against  North  Korea  have  increased  in  recent   years   C.  That  the  poverty  in  North  Korea  is  a  heinous  crime   D.  That,  compared  to  neighboring  countries,  North  Korea  is  very  poor     It  is  important  to  understand  what  the  question  is  really  asking  -­‐-­‐  it  is  asking  how   the  map  changes  or  deepens  your  understanding  of  poverty  in  North  Korea   Take  your  S   AT  prep  to  the  next  level.  Visit  www.learnerator.com   214  

215     specifically.  But  read  carefully  -­‐-­‐  while  A  looks  like  it  may  be  true  (after  all,  the  lights   are  restricted  to  one  part  of  the  country)  if  you  read  closely  you’ll  see  that  the   clustering  of  lights  is  in  fact  an  indication  that  resources,  not  poverty,  are  restricted   to  just  one  area.  The  emptiness  and  lack  of  light  in  the  surrounding  areas  denote   poverty.  Clearly,  considering  how  little  light  there  actually  is,  the  poverty  isn’t   restricted  -­‐-­‐  the  resources  are.  A  is  wrong.  B  is  true  -­‐-­‐  economic  sanctions  against   North  Korea  HAVE  increased  in  recent  years  -­‐-­‐  but  is  completely  unrelated  to  the   data  you  see  in  front  of  you.  (While  the  article  does  say  that  electricity  use  increases   in  cities  but  dims  in  the  countryside  after  new  sanctions  are  imposed,  we’d  have  to   see  a  much  more  detailed  breakdown  of  the  light  situation  after  a  few  sanctions  in   order  for  this  claim  to  be  relevant  to  the  data.)  C  is  also  true,  and  also  supported  by   the  article,  but  has  little  to  do  with  the  data.  Maps  and  charts  reveal  facts  and   statistics;  they  do  not  make  judgments.  D  is  the  best  answer  because  remember  that   the  map  compares  two  different  but  neighboring  countries  -­‐-­‐  North  and  South   Korea.  Without  the  comparison  to  South  Korea,  this  map  of  North  Korea’s  light   situation  (or  lack  thereof)  would  be  completely  meaningless,  as  we’d  have  nothing   against  which  to  measure  it,  no  real  rubric  for  understanding  this  map.  As  this   question  proves,  you  must  read  carefully  and  make  sure  you  understand  and  are   answering  the  question,  or  else  your  choice  will  be  inaccurate.     This  article,  too,  contains  data  that  will  help  you  to  better  understand  what  the   article  itself  is  trying  to  say?     Based  on  the  chart,  what  inference  can  you  make  about  the  number  of   MFAs  that  will  be  earned  in  2012-­‐2013?   A.  The  number  will  most  likely  continue  to  go  up   B.  The  number  will  most  likely  stay  the  same   C.  The  number  will  most  likely  decrease   D.  There  is  no  prediction  that  can  reasonably  be  made  about  this  chart    

Take  your  S   AT  prep  to  the  next  level.  Visit  www.learnerator.com   215  

216     If  you  look  at  the  chart  -­‐-­‐  and  read  the  article  -­‐-­‐  it’s  pretty  clear  that  the  number  of   people  earning  MFAs  has  gone  up  significantly  in  the  past  few  years,  and  that  this   trend  shows  no  sign  of  stopping  despite  the  high  costs  and  low  pay  of  being  an   artist.  Based  on  the  trends  depicted  in  the  graph  -­‐-­‐  as  well  as  the  fact  that  artists   consider  their  profession  to  be  an  honor,  and  tend  to  have  wealthier  parents  than   the  rest  of  the  population  -­‐-­‐  it  is  pretty  easy  to  predict  that  A  is  the  only  correct   answer.   This  article  contains  a  more  challenging  graph.  Read  the  article,  and  look  at  the   graphs,  then  answer  the  questions.  Some  will  ask  you  merely  to  point  to  data,   whereas  others  will  ask  you  to  analyze  it:     Based  on  the  first  graph,  what  percentage  of  five-­‐year-­‐olds  believe  in   Santa?   A.  Nearly  70%   B.  Nearly  83%   C.  Nearly  62%   D.  Nearly  65%     This  is  the  kind  of  question  designed  to  see  if  you  are  paying  attention.  A  refers  to   the  percentage  of  three  year  olds  who  believe  in  Santa,  B  is  the  correct  answer,  C   refers  to  number  of  three-­‐year-­‐olds  who  believe  in  the  Tooth  Fairy,  and  D  refers  to   the  number  of  five-­‐year-­‐olds  who  believe  in  the  Tooth  Fairy.  When  answering   questions  like  this,  all  you  need  to  do  is  pay  attention  to  what  the  graph  actually   says.     Based  on  both  the  article  and  the  first  graph,  what  conclusion  can  you   draw  about  the  number  of  eight-­‐year-­‐olds  who  believe  in  Santa?   A.  Twice  as  many  eight-­‐year-­‐olds  than  nine-­‐year-­‐olds  believe  in  Santa   B.  Eight-­‐year-­‐olds  on  the  whole  no  longer  believe  in  Santa   C.  Eight-­‐year-­‐olds  who  no  longer  believe  in  Santa  most  likely  also  no  longer  

Take  your  S   AT  prep  to  the  next  level.  Visit  www.learnerator.com   216  

217     believe  in  the  Tooth  Fairy   D.  Eight-­‐year-­‐olds  are  the  most  skeptical  age  group   The  question  asks  about  both  the  article  and  the  first  graph,  so  make  sure  that   you’re  using  information  from  the  article  to  answer  this  question.  The  article  does   mention  that  a  1978  study  found  that  only  a  quarter  of  eight-­‐year-­‐olds  believe  in   Santa  -­‐-­‐  though  the  graph  would  seem  to  contradict  this  finding  because  more  than   25%  of  nine-­‐year-­‐olds  believe  in  Santa,  and  it  doesn’t  make  sense  for  the  belief  in   Santa  to  go  up  and  then  down  again  once  children  reach  a  certain  age.  (Though  it’s   true  that  belief  in  Santa  goes  up  between  the  ages  of  three  and  five,  this  can  be   explained  by  the  fact  that  three-­‐year-­‐olds  just  might  not  be  mature  enough  to  grasp   the  concept  of  Santa  fully).  Based  on  the  trend  in  the  chart  (a  decreasing  belief  in   Santa  correlated  with  age)  and  the  anecdotes  in  the  article,  it  doesn’t  stand  to  reason   that  “twice  as  many”  eight  year  olds  believe  in  Santa;  not  even  twice  as  many  seven-­‐ year-­‐olds  believe  in  Santa  than  nine-­‐year-­‐olds,  and  based  on  the  graph  we  can  safely   assume  that  a  belief  in  Santa  will  only  decrease,  not  increase,  with  age.  There  is  also   no  evidence  for  B,  considering  that  nearly  a  third  of  nine-­‐year-­‐olds  believe  in  Santa,   according  to  the  chart,  so  it  makes  little  sense  for  no  eight  year  olds  to  believe  in   Santa.  C  is  supported  by  the  text  as  well  as  the  chart;  belief  in  Santa  and  the  Tooth   Fairy  appear  to  be  related,  as  each  time  a  belief  in  Santa  grows  or  shrinks,  belief  in   the  Tooth  Fairy  does  the  same.  We  can  assume  that  eight-­‐year-­‐olds  skeptical  of   Santa  will  also  be  skeptical  of  the  Tooth  Fairy,  but  we  cannot  assume  that  eight-­‐ year-­‐olds  are  the  most  skeptical  age  group,  especially  given  the  data  and  the   evidence  about  Wooley’s  daughter  (though  the  article  does  not  say  how  old  she   was).     Based  on  the  second  graph,  adults  of  which  religious  group  are  most   likely  to  still  believe  in  Santa?   A.  No  adults  believe  in  Santa   B.  Catholic  adults  are  most  likely  to  believe  in  Santa   C.  Adults  of  “another  religion”  are  most  likely  to  believe  in  Santa   D.  Mainline  Protestant  adults  are  most  likely  to  believe  in  Santa   Take  your  S   AT  prep  to  the  next  level.  Visit  www.learnerator.com   217  

218       Again,  this  question  is  just  trying  to  see  whether  or  not  you  are  paying  attention.  All   you  need  to  do  is  pay  attention  to  the  key  -­‐-­‐  the  graph  is  color  coded.  The  green  bar   will  tell  you  what  percentage  of  which  group  of  adults  still  believes  in  Santa;  looking   at  the  green  bar,  it  is  clear  that  adults  of  “Other  Religion”  are  most  likely  to  still   believe  in  Santa.  All  this  question  is  asking  you  to  do  is  to  look  at  information  that  is   already  in  the  text.     What  is  the  relationship  between  the  two  graphs?   A.  Both  graphs  depict  the  same  trend:  that  belief  in  Santa  peaks  between  the   ages  of  5-­‐8   B.  Both  graphs  depict  the  same  trend:  that  children  under  five  are  more   likely  to  believe  in  Santa   C.  Both  graphs  depict  the  same  trend:  that  belief  in  Santa  is  correlated  with   belief  in  the  Tooth  Fairy   D.  Both  graphs  depict  the  same  trend:  that  belief  in  Santa  is  correlated  with   religion     Remember,  this  question  is  asking  you  about  the  relationship  of  the  two  graphs  to   one  another  -­‐-­‐  not  about  one  graph  in  isolation.  Consider  the  options  carefully;  the   first  graph  does  indeed  show  that  five-­‐year-­‐olds  are  the  most  likely  of  any  age  group   to  believe  in  Santa,  while  the  second  graph  is  a  little  bit  more  generous  and  says  that   five  to  eight  year  olds  are  the  most  likely  to  believe  in  Santa.  In  essence  both  graphs   say  the  same  thing,  though  the  parameters  given  by  the  second  graph  are  slightly   larger  than  the  parameters  given  in  the  first.  Neither  graph  depicts  the  trend   described  in  option  B,  and  only  one  graph  depicts  the  trend  found  in  C.  Only  the  last   graph  depicts  the  trend  found  in  D.  Clearly,  there  is  only  one  right  answer,  and  it  is   A.     What  is  one  surprising  question  that  the  data  in  the  second  graph   presents  but  does  not  explain?   Take  your  S   AT  prep  to  the  next  level.  Visit  www.learnerator.com   218  

219     A.  Why  five-­‐to-­‐eight-­‐year-­‐old  children  of  “other  religions”  are  least  likely  to   believe  in  Santa  while  nine-­‐to-­‐twelve  year  old  children  and  adults  of  “other   religions”  are  most  likely  to  believe  in  Santa   B.  Why  5-­‐8  year-­‐old  children  of  “no  religion”  are  most  likely  to  believe  in   Santa   C.  Why  adults  believe  in  Santa   D.  Why  more  kids  don’t  believe  in  Santa     This  is  a  bit  of  a  tricky  question,  because  the  only  role  of  this  data  is  to  present   information.  You’ll  have  to  look  at  the  data  critically  and  in  isolation  -­‐-­‐  meaning  that   you  can’t  use  too  much  outsider  information,  like  the  idea  that  Christmas  is   technically  a  Christian  holiday,  to  drive  your  question.  Instead,  look  only  at  the  data.   The  first  option,  A,  is  based  on  data  and  only  on  data  -­‐-­‐  it  has  nothing  to  do  with   other  religions.  It  simply  depicts  the  most  interesting  and  unexpected  trend.  Where   the  rest  of  the  religions  are  all  more  or  less  the  same  (belief  peaks  between  5-­‐8  and   steadily  declines),  the  Other  Religion  bar  shows  belief  peaking  at  9-­‐12,  declining  at   older  than  twelve,  and  then  peaking  again  during  adulthood.  Why  would  this   happen?  The  data  does  not  behave  like  the  data  for  the  other  religions,  which  is  why   A  is  the  best  answer.  B  is  also  an  interesting  question,  but  it  isn’t  surprising  -­‐-­‐  after   all,  the  findings  there  are  consistent  with  the  findings  for  all  five-­‐to-­‐eight-­‐year-­‐olds;   and  the  percent  of  5-­‐8-­‐year-­‐olds  of  No  Religion  who  believe  in  Santa  is  not   significantly  higher  than  the  percentage  of  most  5-­‐8-­‐year-­‐olds  who  believe  in  Santa.   C  has  nothing  to  do  with  the  data;  personal  beliefs  are  not  the  concern  of  data  and   charts.  Our  only  interest  here  is  why  the  data  is  fluctuating  -­‐-­‐  hence  why  D  is  also   incorrect.     As  demonstrated  by  the  questions  above,  data  analysis  and  the  use  of  quantitative   information  can  enrich  your  reading  experience  tremendously.  Pay  attention  to   detail  and  remember  to  think  objectively  about  data,  and  you  will  see  much  success   on  this  portion  of  the  SAT.     Take  your  S   AT  prep  to  the  next  level.  Visit  www.learnerator.com   219  

220    

 

            Part  IV:  Expression  of  Ideas  (Writing)  

Take  your  S   AT  prep  to  the  next  level.  Visit  www.learnerator.com   220  

221    

Development Text  Development   Text  development  refers  to  the  rhetorical  structure  and  purpose  of  a  text.  That  is,  it  is   a  way  to  describe  the  components  of  a  text  that  make  that  text  successful  in   communicating  ideas.     Analyzing  text  development  entails  looking  at  four  components:   •

Proposition  –  the  main  claim  or  controlling  idea  of  the  text  



Support  –  the  details,  facts,  and  statistics  used  to  back  up  the  proposition  



Focus  –  the  relevance  of  all  of  the  details  to  the  main  claim  



Quantitative  Information  –  information  in  graphs,  charts,  or  tables  that  can   be  related  to  information  in  the  text.    

Review  the  following  text  and  chart  before  continuing  the  explanation  that  follows     Sample  Text  -­‐  Linking  Poverty  and  Obesity   By  now  it  has  become  common  knowledge  that  America  is  suffering  from  an  obesity   epidemic.  The  United  States  is  home  to  the  greatest  number  of  overweight  or  obese   people  in  the  world.  In  fact,  66%  of  all  Americans  are  overweight.  The  statistical   frequency  of  obesity  in  the  U.S.  is  a  complicated  number.  Although  America  is  one  of   the  wealthiest  nations  in  the  world,  obesity  is  most  often  associated  with  its  poorest   citizens.  Researchers  have  identified  a  direct  correlation  between  BMI,  or  body  mass   index,  a  measurement  of  obesity,  and  income.  However,  the  connection  between   obesity  and  poverty  is  not  a  result  of  uneducated  choices.  Rather,  those  living  in   poverty  are  more  likely  to  become  obese  because  of  the  unique  challenges  in   accessing  nutritious  foods  one  faces  when  living  in  a  high  poverty  area.     In  low-­‐income  neighborhoods,  residents  are  less  likely  to  find  large  grocery  stores,   farmers’  markets,  or  other  places  where  they  can  purchase  healthy  foods  including   fruits,  vegetables,  whole  grains,  or  low-­‐fat  dairy  products.  As  their  food  shopping   Take  your  S   AT  prep  to  the  next  level.  Visit  www.learnerator.com   221  

222     choices  are  often  limited  to  convenience  stores  or  corner  stores,  the  poor  often  find   that  their  food  choices  are  consequently  limited  to  convenience  foods,  such  as   frozen  meals,  canned  foods,  and  other  choices  that  are  high  in  fat  and  salt,  and  often   contain  highly  refined  grains  and  added  sugar.     When  high  quality  nutritious  foods  are  available  to  those  living  in  poverty,  these   food  items  are  often  much  more  expensive  than  less  nutritious,  but  more  filling   foods,  such  as  boxed  pasta  mixes.  With  limited  funds  to  spend  on  food,  poor   shoppers  seek  to  maximize  the  calories  per  dollar  ratio.  For  example,  while  a   healthy  lunch  choice,  such  as  a  chicken  Caesar  salad,  provides  about  60  calories  for   each  dollar  spent,  a  frozen  beef  burrito  provides  112  calories  for  the  same  dollar,   making  it  the  more  economical  choice.  Unfortunately,  these  inexpensive,  high   calorie  foods  are  not  nutrient  rich.  Combined  with  the  uniquely  ample  portion  sizes,   most  Americans  have  become  accustomed  to,  these  cheap,  filling  foods  lead  to   overconsumption  and  weight  gain.     In  addition,  those  living  in  low-­‐income  neighborhoods  often  have  easy  access  to  fast   food  restaurants.  In  fact,  in  many  neighborhoods,  fast  food  restaurants  outnumber   stores  selling  nutritious  foods  nearly  four  to  one.  Fast  food  restaurants  offer  high   calorie  meals  for  relatively  low  prices.  A  fast  food  taco,  for  example,  provides  100   calories  per  dollar.  Unfortunately,  the  fast  food  taco  also  offers  high  fat  and  sugar   content,  and  very  little  nutritious  value.  Consuming  fast  food  on  a  regular  basis  can   lead  to  rapid  weight  gain  and  obesity.  Some  people  believe  fast  food  restaurants   should  be  held  responsible  for  obesity  rates  nationwide.  However,  these  businesses   often  counter  than  they  provide  affordable  food  options  to  Americans  at  all  income   levels  and  that  healthy  choices  are  available  on  their  menus.     Finally,  while  a  simplistic  analysis  of  obesity  rates  among  the  poor  could  lead  to  the   conclusion  that  lack  of  education,  depression,  or  other  factors  such  as  substance   abuse  could  cause  the  higher  rates  of  obesity  among  that  population,  a  close  analysis   demonstrates  that  the  opportunity  to  make  different  food  choices  may  not  exist.   Take  your  S   AT  prep  to  the  next  level.  Visit  www.learnerator.com   222  

223     Often  the  poor  live  in  what  have  been  called  “food  deserts,”  areas  where  affordable,   nutritious  food  is  difficult  or  impossible  to  obtain.    

  Proposition   The  proposition  is  the  main  claim  a  text  sets  out  to  prove.  One  can  usually  find  the   proposition  near  the  end  of  the  first  paragraph.  In  an  argumentative  essay,  the   proposition  is  also  called  the  thesis  statement.     In  the  article  above,  which  sentence  is  most  likely  the  proposition?   1. Rather,  those  living  in  poverty  are  more  likely  to  become  obese  because  of  the   unique  challenges  in  accessing  nutritious  foods  one  faces  when  living  in  a  high   poverty  area.     2. Although  America  is  one  of  the  wealthiest  nations  in  the  world,  obesity  is  most   often  associated  with  its  poorest  citizens.   3. Researchers  have  identified  a  direct  correlation  between  BMI,  or  body  mass   index,  a  measurement  of  obesity,  and  income.     The  answer  is  A.     While  all  three  sentences  make  a  connection  between  poverty  and  obesity,  only  A   includes  the  argument  the  author  is  about  to  make,  that  the  connection  between   obesity  and  poverty  is  caused  by  a  lack  of  access  to  nutritious  foods.    

Take  your  S   AT  prep  to  the  next  level.  Visit  www.learnerator.com   223  

224     In  the  sample  text,  the  proposition  is  introduced  specifically  in  the  introduction,  and   confirmed  in  the  conclusion.  Note  that  both  sentences  of  the  conclusion  make   reference  to  the  availability  of  nutritious  foods:   •

“…a  close  analysis  demonstrates  that  the  opportunity  to  make  different  food   choices  may  not  exist.”  



“Often  the  poor  live  in  what  have  been  called  “food  deserts,”  areas  where   affordable,  nutritious  food  is  difficult  or  impossible  to  obtain.”    

When  analyzing  a  text  to  determine  the  proposition,  it  is  crucial  that  the  reader   consider  the  entire  text.  Finding  the  proposition  is  not  as  simple  as  choosing  the  last   sentence  in  the  introduction.     Sample  Question:  How  does  the  author  introduce  the  proposition?   A.  By  lamenting  America's  obesity  epidemic   B.  By  railing  against  the  many  factors  that  have  led  to  the  obesity  epidemic   C.  By  leveling  blame  on  the  poor   D.  By  dispassionately  outlining  facts     Your  clues  for  answering  these  questions  can  be  found  in  the  verbs  that  make  up  the   answers.  There  is  simply  no  textual  evidence  to  support  Option  A,  which  claims  that   the  author  "laments"  the  obesity  academic.  This  strong  language  suggests  that  the   author  has  a  strong,  regretful  opinion  of  the  epidemic  -­‐-­‐  but  there  is  no  evidence  of   this  kind  of  passion  in  the  article.  The  same  is  true  of  B  and  C  -­‐-­‐  there  is  no  "railing   against"  or  "blame."  In  fact,  if  you  look  at  how  the  author  introduces  the  proposition,   all  you  will  see  are  facts  outlined  without  real  comment.  In  order  to  answer  this   question,  you  must  be  familiar  with  both  the  terms  -­‐-­‐  i.e.,  to  know  what  a   proposition  actually  is  -­‐-­‐  as  well  as  the  vocabulary  used  in  the  answer  options.  You   must  also  understand  the  difference  between  an  objective  tone  (which  is  used  here,   in  this  expository  text)  and  a  subjective  one  (which  makes  use  of  many  adjectives,   descriptions,  and  clearly  illustrates  the  author's  opinion.)     Take  your  S   AT  prep  to  the  next  level.  Visit  www.learnerator.com   224  

225     Support   The  support  consists  of  specific  facts,  examples,  and  details  used  to  provide   evidence  for  the  claim.  Support  can  be  in  the  form  of  statistic  data,  but  may  also   include  elaboration  or  explanation  that  strengthens  the  proposition.   In  the  sample  text,  which  information  is  not  support  for  the  proposition?   1. “…68%  of  all  Americans  are  overweight…”   2. “…chicken  Caesar  salad  provides  about  60  calories  for  each  dollar  spent…”   3. “…in  many  neighborhoods,  fast  food  restaurants  outnumber  stores  selling   nutritious  foods  nearly  four  to  one.”     The  answer  is  A.       The  percentage  of  Americans  who  are  overweight  is  certainly  a  statistic,  which   might  lead  one  to  conclude  that  it  supports  the  proposition.  However,  here  that  is   not  the  case.  The  use  of  this  statistic  provides  introductory  context,  and  lets  the   reader  know  why  this  topic  is  important,  but  the  percentage  of  all  Americans   suffering  from  obesity  does  not  support  the  proposition’s  connection  between   obesity  and  poverty.     Choice  B,  however,  provides  information  that  explains  why  high  calorie  foods  may   be  chosen  more  frequently  than  healthier,  lower  calorie  foods.     Choice  C  elaborates  on  the  imbalance  of  food  options  available  in  high-­‐poverty   neighborhoods,  supporting  the  author’s  proposition  that  obesity  is  a  result  of  a  lack   of  access  to  nutritious  foods.     When  analyzing  a  text  to  determine  what  information  supports  the  proposition,  it  is   important  that  the  reader  does  not  assume  all  statistics  or  numbers  are  relevant   support.  Understanding  the  text  holistically  is  necessary  in  order  to  identify  relevant   support.     Take  your  S   AT  prep  to  the  next  level.  Visit  www.learnerator.com   225  

226     Sample  question:  What  is  the  best  support  from  the  article  for  the  claim   that  the  poor  have  unique  challenges  when  it  comes  to  making  healthy   choices?   A.  The  poor  have  many  issues  with  substance  abuse  in  their  communities   that  make  it  hard  to  eat  well.   B.    Fast  food  restaurants  do  not  offer  healthy  food  options  at  all.   C.  Lower-­‐income  neighborhoods  are  "food  deserts"  that  do  not  offer   affordable  high-­‐quality  foods.   D.  High-­‐calorie  foods  are  often  not  nutrient-­‐rich.     This  question  asks  you  to  consider  information  that  supports  a  claim.  Often,  answer   choices  for  questions  like  this  will  include  information  that  was  actually  featured  in   the  article,  but  be  careful  to  pay  attention  and  make  sure  that  this  information   actually  supports  the  claim.  For  example,  while  Option  A  isn't  incorrect  -­‐-­‐  the  poor   can  have  issues  with  substance  abuse,  which  are  mentioned  in  the  article  -­‐-­‐  you   should  not  use  outside  knowledge,  only  what  is  explicitly  stated  in  the  text,  to   demonstrate  your  understanding  of  the  way  the  text  is  structured.  In  fact,  the  text   explicitly  states  that  this  explanation  is  "simplistic"  -­‐-­‐  so  clearly  it's  not  the  best   support  for  this  claim.  B  is  incorrect  as  well,  as  the  text  states  that  "fast-­‐food   restaurants  provide  affordable  food  options  to  Americans  at  all  income  levels  and   that  healthy  choices  are  available  on  their  menu."  C  is  the  best  answer,  as  the  writer   mentions  in  the  last  paragraph  of  the  text  that  this  is  a  challenge  that  the  poor  face.   D  is  also  true,  but  has  nothing  to  do  with  the  claim  statement  in  question.     Focus   When  evaluating  the  focus  of  a  text  the  reader  should  consider  how  and  if  each   paragraph  clearly  relates  to  the  proposition.  In  the  sample  text,  the  proposition   makes  a  connection  between  poverty  and  obesity.  Therefore,  all  of  the  information   in  the  body  paragraphs  should  provide  support  for  that  claim.     Which  of  the  following  details  is  not  relevant  to  the  proposition?   Take  your  S   AT  prep  to  the  next  level.  Visit  www.learnerator.com   226  

227     1. “Some  people  believe  fast  food  restaurants  should  be  held  responsible  for   obesity  rates  nationwide.”   2. “Combined  with  the  uniquely  ample  portion  sizes  most  Americans  have   become  accustomed  to,  these  cheap,  filling  foods  lead  to  overconsumption   and  weight  gain.”   3. “…poor  shoppers  seek  to  maximize  the  calories  per  dollar  ratio…”     The  answer  is  A.       Remember,  the  purpose  of  this  essay  is  to  support  the  proposition.  The  two   sentences  at  the  end  of  the  fourth  paragraph  that  refer  to  the  debate  about  whether   or  not  fast  food  restaurants  are  responsible  for  obesity  are  not  related  to  this   proposition.   The  writer  argues  that  lack  of  access  to  affordable,  high  quality  foods  leads  to   obesity.  There  is  no  claim  that  there  is  a  connection  directly  to  fast  food  restaurants.   And,  the  author  also  mentions  convenience  and  corner  stores  as  other  sources  of   low  quality,  low  price  food.     When  analyzing  the  focus  of  a  text  it  is  important  to  keep  the  specific  proposition  in   mind.  Information  that  seems  to  be  related  may  only  refer  to  the  same  topic,  but  not   the  same  controlling  idea,  as  in  this  example.  Again,  it  is  impossible  to  analyze  focus   without  understanding  the  entire  text.     Where  is  the  most  logical  place  to  insert  the  following  statement?  "In   fact,  many  have  claimed  that  it  is  this  factor  that  differentiates   American  consumers  from  consumers  in  other  countries;  no  one  knows   how  these  over-­‐large  servings  developed,  but  experts  argue  that   without  proper  portion  control  there  can  be  no  answer  to  this   epidemic."   A.  Right  before  this  statement:  "A  fast  food  taco,  for  example,  provides  100   calories  per  dollar.  Unfortunately,  the  fast  food  taco  also  offers  high  fat  and   Take  your  S   AT  prep  to  the  next  level.  Visit  www.learnerator.com   227  

228     sugar  content,  and  very  little  nutritious  value."   B.  Right  after  this  sentence:  "However,  the  connection  between  obesity  and   poverty  is  not  a  result  of  uneducated  choices."   C.  Right  after  this  sentence:  "Combined  with  the  uniquely  ample  portion   sizes,  most  Americans  have  become  accustomed  to,  these  cheap,  filling  foods   lead  to  overconsumption  and  weight  gain."   D.  Right  after  this  sentence:  "Finally,  while  a  simplistic  analysis  of  obesity   rates  among  the  poor  could  lead  to  the  conclusion  that  lack  of  education,   depression,  or  other  factors  such  as  substance  abuse  could  cause  the  higher   rates  of  obesity  among  that  population,  a  close  analysis  demonstrates  that   the  opportunity  to  make  different  food  choices  may  not  exist."     This  question  tests  your  understanding  of  how  to  best  organize  a  piece  of  writing  so   that  it  retains  its  focus.  Pay  attention  to  the  sentence  itself  "it  is  this  factor  that   differentiates  American  consumers..."  So  you  know  that  this  sentence  must  have  to   go  after  another  sentence  that  introduces  some  new  factor.  Look  at  A.  The  sentence   before  "A  fast  food  taco,  for  example"  reads  "Fast  food  restaurants  offer  high  calorie   meals  for  relatively  low  prices."  Perhaps  this  phenomenon  could  be  a  differentiating   factor  -­‐-­‐  but  look  at  the  rest  of  the  sentence,  which  is  clearly  about  overlarge   servings.  The  preceding  sentence  makes  no  mention  of  overlarge  servings,  so  this   cannot  be  the  right  place  for  a  sentence  about  portion  control.     If  you  look  at  the  rest  of  the  choices,  the  only  one  that  mentions  portion  control  is  C  -­‐ -­‐  so  C  must  be  the  best  answer,  as  it  is  the  only  one  that  introduces  the  "unique   differentiating  factor"  of  portion  control,  and  is  thus  the  best  place  for  a  sentence   about  portion  size.     Quantitative  Information   Often  a  graph  or  chart  will  accompany  a  text  and  provide  additional  information   related  to  the  text.  When  analyzing  quantitative  information,  it  is  necessary  to   consider  the  information  in  that  graphic  as  it  relates  to  the  proposition  in  the  text.   Take  your  S   AT  prep  to  the  next  level.  Visit  www.learnerator.com   228  

229     Many  conclusions  can  likely  be  drawn  from  a  graphic  representation,  but  only  those   related  to  the  proposition  are  relevant.     What  related  conclusion  can  the  reader  reasonably  draw  based  on  the  information   in  the  graph  “Obesity  Rate  by  Income?”     1. Those  earning  more  than  $50,000  a  year  are  more  likely  to  have  access  to   farmers’  markets  than  those  earning  $30,000  a  year.   2. Those  earning  $35,000  a  year  are  less  likely  to  be  obese  than  those  earning   $20,000  a  year.   3. To  avoid  becoming  obese  Americans  should  try  to  earn  at  least  $50,000  a   year.     Answer:  A   According  to  the  text  one  reason  those  living  in  poverty  are  more  likely  to  be  obese   is  that  they  don’t  have  access  to  large  supermarkets  or  farmers’  markets  where  they   can  purchase  nutritious  foods.  Based  on  the  claim  in  support  of  the  text’s   proposition,  the  reader  can  conclude  that  it  is  likely  that  wealthier  Americans  have   greater  access  to  these  sources  of  food.     Choice  B  is  accurate,  and  it  is  obvious  that  more  income  is  correlated  to  lower   obesity  rates.  However,  there  is  no  direct  connection  between  this  observation  and   the  proposition  of  the  text.  That  is,  this  statement  doesn’t  make  the  connection  to   the  availability  of  nutritious  foods.     Choice  C  is  not  the  correct  choice  because  the  proposition  of  the  text  is  not  to   advocate  for  higher  wages  or  to  suggest  that  those  living  in  poverty  simply  try  to   earn  more  money  in  order  to  improve  their  health.  This  observation  is  not  related  to   the  text.    

Take  your  S   AT  prep  to  the  next  level.  Visit  www.learnerator.com   229  

230     Conclusion   Before  responding  to  questions  about  the  development  of  a  text  it  is  crucial  that  the   reader  consider  the  entire  text  holistically  and  be  particularly  focused  on  the   proposition  or  main  claim  the  text  makes.  Begin  by  underlining  or  highlighting  the   proposition,  and  consider  other  components,  including  support,  focus,  and   quantitative  information  as  they  apply  to  that  proposition.  

Take  your  S   AT  prep  to  the  next  level.  Visit  www.learnerator.com   230  

231    

Organization Text  Organization   The  organization  of  a  text  refers  to  all  of  the  elements  that  make  it  logical  and   cohesive.  All  levels  of  a  text,  from  sentences  to  paragraphs,  should  be  organized  and   ordered  logically  and  cohesively.   To  analyze  the  organization  of  a  text,  it  is  necessary  to  consider  four  different   elements:     •

Sequence  –  the  order  in  which  information  is  presented  



Introductions  –  how  a  text  begins,  and  what  information  is  provided  to   establish  context  and  purpose  of  the  text.  



Conclusions  –  how  a  text  ends,  and  the  synthesis  of  ideas  to  leave  the  reader   with  an  understanding  of  the  purpose  and  content  



Transitions  –  words  and  sentences  used  to  move  from  one  concept  to   another  within  a  text    

Sequence   The  sequence  of  a  text  is  the  order  in  which  details  are  presented  in  support  of  the   main  idea.  Often,  the  thesis  or  proposition  will  establish  the  order  of  information.   There  are  several  ways  to  organize  details  in  a  text:     •

Chronologically  –  in  time  order.  This  strategy  is  particularly  effective  for   narrative  fiction,  biographies,  or  texts  that  recount  events  in  history.  



Description  –  focusing  on  sensory  details.  This  order  divides  details  into   what  can  be  seen,  heard,  smelled,  tasted,  and  felt.  



Process  –  describes  a  sequence  of  steps  necessary  to  complete  a  task  



Division  and  classification  –  sorts  details  into  categories  



Compare  and  contrast  –  identifies  similarities  and  differences  between  two   entities  

Take  your  S   AT  prep  to  the  next  level.  Visit  www.learnerator.com   231  

232     •

Causal  (cause  and  effect)  –  examines  either  a  single  case/effect  relationship   or  a  causal  chain,  or  series  of  components  leading  to  a  phenomenon,  event,  or   condition.    

Examples  of  Organizational  Strategies:   Strategy  

Example  

Chronologically  

George  Washington  was  born  on  February  22,  1732  in   Westmoreland,  Virginia.  When  he  was  just  11  years  old,  in  1743,   Washington’s  father  passed  way,  and  little  George’s  half-­‐brother   took  him  into  his  home  and  raised  him  with  his  own  children.  In   1748,  when  Washington  was  16,  he  went  on  his  first  surveying   job,  mapping  the  western  sections  of  Virginia.  

Description  

The  sand  beneath  my  feet  is  damp,  cool  and  soft.  A  gentle  breeze   wafts  in  from  the  water,  bringing  with  it  the  smell  of  seaweed   and  cotton  candy  from  a  carnival  just  a  half  a  mile  away.  As  I  walk   closer  to  the  carnival  the  sounds  of  people  screaming  in  joyful   fear  from  the  top  of  the  roller  coaster  reaches  my  ears.  I  can  still   taste  the  salt  on  my  lips  from  my  swim  in  the  water  earlier  today.  

Process  Analysis   Before  mixing  the  ingredients,  preheat  the  oven  to  350  degrees.   Slowly  mix  the  butter,  flower,  and  cream  together,  being  careful   to  remove  all  lumps  from  the  batter.  Then,  slowly  add  cocoa,   vanilla,  and  chopped  nuts.   Division  and  

Developing  a  strong  heart  and  healthy  muscles  requires  a  

classification  

regimen  of  both  aerobic  and  anaerobic  exercises.  Aerobic   exercises  include  swimming,  walking,  running  and  biking.   Anaerobic  exercises  include  weight  lifting  and  yoga.  

Compare  and  

Both  cats  and  dogs  are  considered  valued  pets  and  even  

contrast  

members  of  American  families.  There  are  differences  between   these  two  most  common  pets  that,  if  analyzed,  can  help  a   potential  pet  owner  understand  which  pet  might  be  a  better  

Take  your  S   AT  prep  to  the  next  level.  Visit  www.learnerator.com   232  

233     choice.  For  example,  cats  are  often  considered  more  difficult  to   train  than  dogs,  while  dogs  tend  to  require  more  maintenance,   including  frequent  exercising  and  grooming.   Causal  

Poor  lifestyle  choices  in  one’s  youth  can  lead  to  serious  health   issues  that  don’t  emerge  until  middle  age.  For  example,  people   who  smoke  between  10-­‐20  cigarettes  per  day  for  five  years  or   more  are  70%  more  likely  to  develop  emphysema  by  age  sixty   than  those  who  smoke  fewer  cigarettes  or  don’t  smoke  at  all.  

  Use  the  text  below  to  answer  the  questions  about  these  concepts.   1.  The  Hindi-­‐language  film  industry,  based  out  of  Mumbai,  India,  is  one  of  the   most  prolific  producers  of  feature  films  in  the  world,  producing  up  to  1000  films   per  year.  Nicknamed  “Bollywood,”  a  combination  of  the  name  of  the  American   film-­‐hub,  Hollywood,  and  the  colonial  name  of  Mumbai,  Bombay,  the  network  of   studios  produces  hundreds  of  films  per  year,  many  of  which  reach  worldwide   audiences.  Unlike  Hollywood,  which  is  a  geographical  location,  Bollywood  is  a   concept  that  includes  production  companies  located  throughout  India  and  in   other  parts  of  the  world  where  Hindi-­‐language  films  are  produced.     2.  Bollywood  films  have  enjoyed  a  long  and  profitable  history,  beginning  with   the  first  film,  a  silent  picture  produced  in  1913,  around  the  time  when  the   American  movie  industry  was  relocating  from  New  York  to  California,  and   producing  the  first  Hollywood  films.  The  first  Bollywood  film  with  sound   appeared  in  1931,  just  four  years  after  Hollywood’s  breakthrough  “talking   picture”  The  Jazz  Singer  By  the  1940’s  Bollywood  films  were  garnering   worldwide  attention,  and  in  1946  Neecha  Nagar,  a  Bollywood  film,  won  the   best  picture  award,  called  the  Palme  d’Or,  or  Golden  Palm,  at  the  Cannes  Film   Festival  in  France.     3.  While  the  development  of  Bollywood  films  may  have  paralleled  the  growth  of  

Take  your  S   AT  prep  to  the  next  level.  Visit  www.learnerator.com   233  

234     the  American  film  industry  in  California,  Hindi-­‐language  films  are  unique  in   several  ways,  and  hardly  an  imitation  of  American  film  genres.  For  example,   Bollywood  films  tend  to  be  considerably  longer  than  American  films,  stretching   up  to  three  hours,  as  compared  to  two  hours  or  less  for  American  films.  In   addition,  almost  every  Bollywood  film,  regardless  of  genre,  includes  at  least  one   elaborate  singing  and  dancing  segment,  which  is  often  cast  as  dream  within  the   plot.  These  musical  numbers  are  also  a  response  to  strict  censorship  codes  that   were  once  imposed  on  films  produced  in  India.      Because  characters  could  not   kiss,  hold  each  other,  or  engage  in  more  intimate  contact,  the  music  and   dancing  came  to  represent  romance  within  a  love  story.  Eventually,  the  musical   numbers,  and  in  fact,  the  films  themselves  developed  unique  aesthetic   signatures,  including  the  bumping  of  shoulders  between  characters  who  were   romantically  involved,  exaggerated  expressions  of  love  or  passion,  and   elaborate  dances  that  led  to  the  romantic  characters  being  almost,  but  not   quite,  cheek  to  cheek.     4.  The  Bollywood  film  industry  has  experienced  three  distinct  eras  in  its  history.   Like  the  American  film  industry,  Bollywood  found  its  roots  in  a  studio-­‐based   model,  or  the  studio  era.  During  this  time,  from  the  dawn  of  Bollywood  films  to   about  1950,  studios  and  the  executives  who  ran  them  maintained  almost  all   control  of  film  production,  and  garnered  the  most  profits  from  those  films.   Actors,  directors,  musicians,  and  all  of  the  other  creative  people  engaged  in  film   production  were  merely  employees  of  the  studios,  often  paid  flat  salaries,  and   required  to  work  on  the  films  to  which  they  were  assigned  for  the  duration  of   their  employment  contract.     5.  During  World  War  II,  it  became  difficult  for  Bollywood  studios  to  procure   raw  film  stock.  In  fact,  the  only  film  stock  available  was  on  the  black  market,   and  filmmaking  became  a  moderately  criminal  activity.  Producers  engaged  in  a   wide  range  of  crimes  saw  film  production  as  a  means  to  launder  money,  or  turn  

Take  your  S   AT  prep  to  the  next  level.  Visit  www.learnerator.com   234  

235     illegally  obtained  profits  into  legal  funds  that  could  be  banked  or  used  for   purchases  in  general  market.  In  the  midst  of  this  cycle,  actors  began  to   command  and  receive  very  high  salaries  for  their  work,  as  the  black  market   studios  saw  actor  salaries  as  a  means  of  converting  illegal  profit  to  legal   business  expenses.     6.  Following  World  War  II,  and  the  demise  of  the  black  market  studios,  a  few   film  directors  emerged  as  the  most  sought-­‐after.  These  directors  partnered   with  the  most  successful  and  popular  music  directors  and  a  few  styles  of   filmmaking  and  musical  production  dominated  the  industry  from  the  1950s   until  the  1990s.  As  the  celebrity  of  specific  directors’  styles  continued  to   increase,  so  too  did  their  salaries.  Like  the  exorbitant  salaries  paid  actors   during  the  war  years,  these  expenditures  were  not  sustainable.  With  the  onset   of  digital  music  sources  and  music  pirating,  the  profits  studios  once  saw  from   selling  the  soundtracks  to  films  all  but  evaporated,  and  it  became  increasingly   difficult  to  meet  the  salary  demands  of  the  music  directors.  In  the  1990s  several   studios  were  bankrupted  and  forced  to  close.  This  period  in  Bollywood  history  is   referred  to  as  the  Directors’  Era.     7.  Some  critics  refer  to  the  period  from  about  1998  to  the  present  as  a  time  of   transition,  or  the  Transition  Era.  There  are  still  some  film  and  music  directors   who  command  higher  salaries  than  others,  based  on  their  past  success  and   unique  approaches  to  film.  However,  they  now  accept  package  salaries,  from   which  they  are  required  to  pay  many  other  employees,  including  musicians,   camera  operators,  etc.      These  directors  are,  in  fact,  contractors  within  the   industry.  The  new  payment  scheme  encourages  cost  savings  so  that,  for   example,  instead  of  hiring  a  full  orchestra  for  days  at  a  time,  the  music  director   might  record  all  of  the  music  in  a  short  period  of  time  and  use  digital  music  for   some  sequences.    

Take  your  S   AT  prep  to  the  next  level.  Visit  www.learnerator.com   235  

236     8.  Some  actors,  similarly,  are  paid  more  than  others,  and  there  are  a  few,   mostly  male,  actors  who  command  salaries  much  higher  than  any  other   performer.  These  actors,  however,  receive  payment  through  a  combination  of   acting  salaries,  musical  royalties,  and  payments  for  personal  appearances  and   endorsements.     9.  Despite  these  hurdles  and  recent  changes  in  the  Bollywood  film  industry,  it  is   a  vibrant  and  growing  business.  Today,  Bollywood  films  are  screened  in  ninety   different  countries,  and  in  India  alone,  about  fourteen  million  people  see  a   Bollywood  or  Hindi-­‐language  movie  every  day.   The  essay  above  uses  compare  and  contrast  to  provide  context  for  the  details  about   Bollywood  films.  Aspects  of  Bollywood  films  are  compared  to  aspects  of  American  films   in  order  to  clarify  points  for  an  American  reader.     What  other  organizational  strategy  is  applied  in  the  passage  above?   A.  Chronological   B.  Process   C.  Causal     Answer:  A   The  essay  above  is  organized  chronologically,  which  is  a  strategy  well  suited  to   explain  the  history  of  a  phenomenon.  The  essay  starts  with  the  beginning  of   Bollywood  films,  in  1913,  and  ends  with  a  description  of  current  trends.     Introductions   A  successful  introduction  must  accomplish  two  goals:   •

Provide  context  or  background  for  the  information  in  the  essay.  



Reveal  the  controlling  idea  or  proposition  of  the  essay    

Take  your  S   AT  prep  to  the  next  level.  Visit  www.learnerator.com   236  

237     There  are  several  ways  to  introduce  a  text,  each  of  which  is  particularly  applicable   to  specific  types  of  essays.  This  list  is  not  exhaustive,  but  does  provide  some  of  the   most  common  strategies:     •

Describe  a  scene  or  tell  an  anecdote  –  with  a  description  or  anecdote  the   reader  is  brought  into  the  world  of  the  subject,  either  through  empathy  with   the  subject,  or  with  an  understanding  of  the  setting  necessary  to  comprehend   the  details.  



Provide  relevant  background  information  –  this  is  historical  or  technical   information  necessary  to  understand  the  purpose  and  relevance  of  the  text.  



Use  a  startling  statistic  or  remark  from  a  cited  source  –  this  strategy  is   particularly  effective  when  writing  about  a  topic  that  readers  may  not   consider  a  serious  or  important  issue.  A  startling  statistic  or  quote  will   highlight  the  relevance  of  the  subject.  



Define  an  important  term  –  providing  a  definition  can  reveal  rarely   considered  context  of  a  subject.  Usually  this  is  not  a  dictionary  definition,  but   one  that  relies  on  scientific  research  or  discovery  or  a  philosophical  point  of   view.    

Take  your  S   AT  prep  to  the  next  level.  Visit  www.learnerator.com   237  

238     Examples  of  Introductions   Type  of  Introduction  

Example  

Description/anecdote  

The  first  time  I  saw  a  Bollywood  film  I  was  amused,   confused,  and  utterly  thrilled.  I  can  still  remember  how   surprised  I  was  when  a  seemingly  predictable  romantic   comedy  suddenly  morphed  into  an  elaborate  musical   dance  number.  When  I  left  the  theater  that  afternoon,  I  was   hooked,  and  wanted  to  know  more  about  the  roots  and   history  of  Bollywood.  

Relevant  background  

There  are  more  than  1,000  Bollywood  films  produced  each  

information  

year,  and  these  films  are  shown  on  almost  every  continent   every  day.  The  first  Bollywood  film  was  produced  in  1913.  

Startling  statistic  or  

“I  would  love  to  work  in  a  Bollywood  film,  as  there  is  so  

remark  

much  drama  and  color,  in  the  films  there”  (Pitt,  2013).   Brad  Pitt,  the  seminal  American  film  star  is  not  alone  in  his   admiration  for  Bollywood  cinema.  These  unique  films   continue  to  read  a  broader,  global  audience  of  people  who   are  wowed  by  the  unique  spectacle  only  Bollywood  can   deliver.  

Define  an  important  

The  term  “Bollywood”  does  not  refer  to  a  specific  place.  

term  

Rather  Bollywood  is  a  term  coined  to  refer  to  Hindi-­‐ language  films  produced  in  Mumbai,  India.  

 

  Review  the  text  above.  Which  is  most  likely  the  introduction  strategy   this  author  applied?   A.  Provide  relevant  background  information   B.  Define  an  important  term   C.  Use  a  startling  statistic    

Take  your  S   AT  prep  to  the  next  level.  Visit  www.learnerator.com   238  

239     Answer:  A     In  this  essay  the  author  provided  relevant  background  information  about  the  Hindi-­‐ language  film  industry  and  the  origin  of  the  term  “Bollywood.”  This  information   provides  relevant  context  for  the  reader  who  is  not  familiar  with  these  types  of   films.     If  the  writer  was  to  revise  this  introduction  and  focus  on  defining  an   important  term,  which  would  be  the  most  relevant  term  that  would   benefit  from  a  clearly  articulated  definition?   A.  Bollywood   B.  Hindi   C.  Film  concept     Answer:  A     “Bollywood”  may  be  an  unfamiliar  term  to  most  readers,  and  it  is  a  sort  of   “nickname”  applied  to  Hindi-­‐language  films.  Since  it  is  not  a  commonly  understood   term,  a  thorough  definition  would  benefit  the  reader  by  providing  information   necessary  to  understand  the  rest  of  the  essay.     Transitions   Transitions  are  words  or  phrases  used  to  connect  ideas  between  sentences  or   between  paragraphs.  Effective  transitions  build  coherence  in  an  essay  and  ensure   the  reader  is  able  to  follow  a  logical  line  of  reasoning  from  the  beginning  to  the  end   of  a  text.   Transitional  words  and  phrases  fall  into  many  categories.  Review  the  categories  and   examples  below.     To  show  agreement  or  

In  addition  

Likewise  

Take  your  S   AT  prep  to  the  next  level.  Visit  www.learnerator.com   239  

Similarly  

240     similarity:   To  show  contrast  or  

Although  

contradiction:  

On  the  other  

Nevertheless  

hand…  

To  provide  examples  or  

In  other  

Notably  

In  fact…  

emphasize  a  point:  

words…  

To  show  cause/effect:  

As  a  result…  

Because…  

Therefore…  

Sequence/time:  

In  the  first  

Prior  to…  

Next  

place…   To  summarize  or  conclude:  

In  conclusion…   Summing  up…  

On  the   whole…  

 

   Review  the  text  above.  Which  type  of  transition  would  most   successfully  improve  the  logical  development  from  paragraph  (4)  to   paragraph  (5)?   A.  Cause/effect   B.  Sequence/time   C.  Summarize/conclude    

Answer:  B     Paragraph  (4)  discusses  the  challenges  faced  by  the  film  industry  during  WWII.       Paragraph  (5)  begins  to  describe  a  new  era  in  Bollywood  film,  following  WWII.  This   is  clearly  a  change  in  time  and  a  sequence  or  time  transition,  such  as  “following,   after,”  or  “later”  would  be  appropriate.     Review  the  text  above.  Which  type  of  transition  would  most  successfully   improve  the  logical  development  from  paragraph  (8)  to  (9)?   A.  Cause/effect   Take  your  S   AT  prep  to  the  next  level.  Visit  www.learnerator.com   240  

241     B.  Sequence/time   C.  Summarize/conclude     Answer:  C     Paragraph  (9)  is  the  concluding  paragraph  of  the  essay.  To  signal  the  end  of  the  text   and  to  alert  the  reader  that  summary  or  implication  will  follow,  a  summarizing  or   concluding  transition  is  appropriate.     Conclusion:   The  conclusion  of  an  essay  pulls  together  all  of  the  information  presented  and   comes  to  some  final  concept  that  expresses  the  relevance  of  the  text.  There  are   several  strategies  that  can  be  applied  to  the  conclusion,  including:     •

Brief  summary  of  the  essay’s  main  points  –  the  conclusion  pulls  the  most   important  information  from  the  essay  and  synthesizes  it  into  an  objective   summary.  



Provocative  question  –  challenges  the  reader  to  reconsider  previously  held   opinions  on  the  topic  



Call  for  action  –  challenges  the  reader  to  do  something  about  the  topic.  



Explain  the  significance  or  implications  of  the  findings  in  the  text  –  how  does   this  information  change  the  body  of  knowledge  on  the  topic,  and  change  how   the  topic  will  be  considered  in  the  future?    

Which  of  the  strategies  listed  is  most  likely  the  one  used  by  the  author  of   the  text  above?   A.  Brief  summary  of  main  points   B.  Provocative  question   C.  Explain  significance  or  implications    

Take  your  S   AT  prep  to  the  next  level.  Visit  www.learnerator.com   241  

242     Answer:  A     The  conclusion  of  this  essay  explains  the  significance  and  implications,  by  explaining   that  the  Bollywood  film  industry  will  likely  continue  to  prosper,  despite  challenges   faced  over  its  long  history.  The  inclusion  of  the  statistic  about  the  number  of  people   who  view  a  Bollywood  movie  daily  suggests  a  large  audience  for  these  films,  further   solidifying  the  point.     Which  other  strategy  could  be  successfully  applied  to  this  essay?   A.  Brief  summary  of  main  points   B.  Provocative  question   C.  Call  to  action     Answer:  A     A  brief  summary  of  the  essay’s  main  points  would  offer  another  effective  conclusion   to  this  essay.  Since  the  essay  is  expository,  or  informative,  rather  than   argumentative  or  persuasive,  a  call  to  action,  such  as  “See  a  Bollywood  movie  soon,”   would  not  effectively  synthesize  the  author’s  main  points.  The  essay  does  not  offer  a   contentious  issue  or  debate,  so  a  provocative  question  would  not  be  an  effective  way   to  end  the  essay.    

Take  your  S   AT  prep  to  the  next  level.  Visit  www.learnerator.com   242  

243    

Precision and Concision Effective Language Use

Ideas are only communicated effectively if a text uses language effectively. There are many considerations that impact how effective a text is in achieving the writer’s rhetorical purpose. Among these are: •

Precision  –  The  exactness  of  word  choice  



Concision  –  The  economy  of  word  choice  

Precision

Precise language uses the most accurate, descriptive term. Nouns are specific, rather than general, and modifying words call on imagery to draw connections a reader can relate to in order to develop a mental image of a situation that closely matches the intent of the writer. Some characteristics of precise writing include: •

Use of the active voice in which the noun or subject performs the action of the verb. o Active: Marta moved the couch to the center of the room. o Passive: The couch was moved to the center of the room.



Use of precise words, rather than their definitions. o

Precise:  Marta  rode  a  commuter  train  to  work  yesterday.  

o

Imprecise:  Yesterday,  Marta  utilized  a  mode  of  rail  transportation   frequented  by  those  who  work  some  distance  from  their  homes.  



Convert nouns to verbs o

Precise:  Marta  worked  with  other  teachers  to  develop  a  new   curriculum.  

o

Imprecise:  Marta  worked  with  other  teachers  in  the  development  of  a   new  curriculum.  

Take  your  S   AT  prep  to  the  next  level.  Visit  www.learnerator.com   243  

244     •

Replace vague words with specific words o

Precise:  The  band  featured  a  talented  singer  with  a  soulful  voice  who   reminded  me  of  Aretha  Franklin.  

o

Imprecise:  The  band  was  good.  

Read the text below and continue on to the content that follows: California’s  San  Joaquin  Valley  is  home  to  more  than  ten  percent  of   California’s  agricultural  production,  which  means  one  of  every  ten  farms  in   the  state  is  in  the  Valley.1  The  United  States  relies  on  this  narrow  strip  of   farmland,  located  in  central  California,  for  food  and  cotton2.  In  recent  years,   however,  an  extended  drought,  which  has  gone  on  for  a  long  time3,  has   threatened  crop  production.  To  combat  this  drought,  farmers  have  turned  to   deep  wells,  which  pump  ground  water  from  far  below  the  earth’s   surface.    What  seems  like  a  reasonable  solution  to  a  bad4  agricultural  problem   may  have  significant  unforeseen  consequences.  Recent  research  suggests  that   the  removal  of  ground  water  in  the  Valley  has  elevated  mountains,  caused   several  small  shifts  of  the  earth’s  crust,  resulting  in  vibrations5,  and  has  the   potential  to  trigger  more  frequent  earthquakes.  The  earthquakes  could  also   become  more  intense.6   Review the excerpt (2) in the passage above. What would be the best way to make this sentence more precise?

A. …for growing food and cotton B. …for artichokes, oranges, lettuce, and cotton C. …for many kinds of food and cotton D. …for many plants used for food and manufacturing Answer: B This sentence already includes a specific crop, cotton. To make the entire Take  your  S   AT  prep  to  the  next  level.  Visit  www.learnerator.com   244  

245     sentence more precise, the best strategy is to list specific foods that are grown in the region.

Review the excerpt (4) in the passage above. What would be the best way to make this sentence more precise?

A. …reasonable solution to a serious… B. …reasonable solution to a worsening… C. …reasonable solution to a growing… D. …more reasonable solution to an increasing problem… Answer: B The main idea of this passage is that the problems caused by deep well pumping are becoming more serious. While “growing” suggests some increase, it is not as precise as “worsening,” which clearly indicates the problem will only become more severe as time goes on.

Concision

Concise writing is direct and to the point. It avoids restating ideas, and is clear and specific. There are several strategies to evaluate the concision of a text, including: •

Identifying  unnecessary  words  or  phrases  



Identifying  passive  sentences  



Identifying  negative  sentences,  which  use  more  words  than  positive   sentences  to  express  the  same  idea.  



Using  short,  related  sentences,  rather  than  combining  ideas  into  fewer,   concise  sentences.  

Read excerpt (5) in the passage above. What problem interferes with the concision of this sentence?

A. Unnecessary words or phrases Take  your  S   AT  prep  to  the  next  level.  Visit  www.learnerator.com   245  

246     B. Passive sentence C. Negative sentence D. Improper word use Answer: A This sentence includes unnecessary words and phrases. Since the sentence already uses the word “earthquake,” the author attempted to vary the sentences by using the definition of an earthquake in this section. The result, however, is a sentence that is overly wordy and potentially confusing. The sentence should be revised. Which is the most effective revision of excerpt 5 in the passage above?

A. …caused small earthquakes… B. …caused vibrations from shifts in the earth’s crust… C. …caused several small shifts of the earth’s crust, called earthquakes… D. …led to vibrations or earthquakes… Answer: A Small vibrations caused by shifts in the earth’s crust are called earthquakes. Avoid defining a term rather than naming it, especially when the term is commonly understood by the average reader. If the term were highly technical or specific to a certain discipline, it might be necessary to elaborate, but that is not the case in this excerpt.

Review excerpt (6) in the passage above. What problem interferes with the concision of the text?

A. Passive sentence B. Negative sentence C. Lack of concision in sentences

Take  your  S   AT  prep  to  the  next  level.  Visit  www.learnerator.com   246  

247     D. Lack of precision in word choice Answer: C The last sentence in the text provides additional information about the predictions for future earthquakes. The result is a passage that is overly wordy, and the suggestion that new, more detailed information will be presented. The passage should be revised.

What is the best revision of excerpt (6) in the passage above?

A. …and has the potential to trigger more frequent and intense earthquakes. B. …and has the potential to trigger more frequent earthquakes that could be more intense. C. …and has the potential to trigger earthquakes. The earthquakes could be more frequent and more intense. D. …and has the potential to trigger more intense earthquakes that happen more frequently. Answer: A Option A combines the important details from both sentences into a single, concise sentence that successfully communicates that future earthquakes may be both more frequent and intense.

 

 

Take  your  S   AT  prep  to  the  next  level.  Visit  www.learnerator.com   247  

248    

Style and Tone Effective Language Use

Ideas are only communicated effectively if a text uses language effectively. There are many considerations that impact how effective a text is in achieving the writer’s rhetorical purpose. Among these are: •

Style  – most often refers to the diction of the writing, which can be formal, informal, or very informal. The style is the created through the author’s word choice and organization. An author’s style is also influenced by his or her intended audience. For example, someone writing for an audience of children will maintain a different style than an author writing for graduate students.



Tone  - the writer’s attitude toward the subject, which must be appropriate to the purpose.

Style

Style refers to how something is written to achieve a specific purpose. The style of the writing affects the reader’s impression of what he or she is reading, including assessments of the credibility and persuasiveness of an argument. The style of a text must remain consistent in order to achieve its purpose. Review the passage below and assess the writer's style, paying careful attention to the words and sentence structure that s/he uses.

Take  your  S   AT  prep  to  the  next  level.  Visit  www.learnerator.com   248  

249     Venice, Italy is a city composed of 117 small islands, connected by bridges and canals. There is no wheeled traffic in the streets of Venice. In other words, there are no cars, no bicycles, no scooters, and no horse carts in the entire city. There is, however, a public transportation system, consisting of vaporetti, or water buses, which provides service to the city’s main canals, the lagoon surrounding the city, and the train station. Visitors travel to Venice either by waterbus, from nearby Marco Polo Airport, or by train from many cities in Europe. The train station, Venezia Santa Lucia, is located on the banks of the Grand Canal, Venice’s main thoroughfare. Think about how this passage is written. Is it informative? Does it give directions? Is it poetic, with a lyrical style? Does it directly address the reader or give the writer's opinion? Once you assess the passage and consider these factors, take a stab at the following question:

Which sentence most effectively matches the style of this passage and could be added to the end of the paragraph?

A. On my last trip to Venice, the lack of wheeled traffic drove me crazy! B. Those arriving by train can transfer to local waterbuses, which offer stops near most hotels and tourist attractions, or hire private water-taxis to take them directly to the their destination. C. Though many of the ancient canals are choked by trash and grime, the water retains a smoky, mystical quality when lit by the soft glow of streetlights that wink D. If arriving by train, you can either take one of the waterbuses, which stop at most of the hotels, or get a private water taxi to get right to the doorstep of your own hotel. Answer: B

Take  your  S   AT  prep  to  the  next  level.  Visit  www.learnerator.com   249  

250     This sentence matches the style of the passage for several reasons. First, the sentence is informative -- rather than narrative or persuasive -- matching the high level of detail and elaboration in the rest of the passage. Second, it maintains focus on visitors to Venice, rather than residents, and, third, it does not express an opinion or address the reader directly (you). While this is a passage intended for visitors, it is more informative than directive. Taking these ideas into consideration, review the passage below: Motivated by concern for the environment, as well as a desire to avoid large financial commitments, fans of tiny houses, or microhomes, have rejected the typical 2600 square foot American house in favor of homes measuring just 100 to 400 square feet. Tiny homes offer many benefits. Residents pay less to heat and cool their homes, and property taxes are much lower than those on typical houses. Tiny homes consume less resources in construction and use less energy overall. There are, however, some drawbacks to micro-living. Residents often find it difficult to entertain, because there is very little space for guests, or even for additional seating. Finally, living in a microhome requires discipline. Since a small space can quickly become cluttered, residents must discard anything they don’t use regularly. Which of the following sentences is aligned with the style of this passage and could be added as the first, or topic sentence of the paragraph?

A. In the last decade a growing number of Americans have embraced the tiny house movement. B. Micro is a Latin prefix that means very small. C. Tiny homes have pros and cons, but mostly they are more affordable than traditional homes and much better for the environment. D. Microhomes are also called tiny homes and they are a new fad that is really growing in popularity.

Take  your  S   AT  prep  to  the  next  level.  Visit  www.learnerator.com   250  

251     Answer: A Begin by analyzing the overall style of the passage. Notice the word choices the author makes, “large financial commitment,” rather than “big expense,” and “motivated by concern for the environment,” rather than “because they care about the environment.” This analysis tells the reader that the writer anticipates a specific audience – adults familiar with home ownership and familiar with trends in environmental activism. From this analysis, we can assume the diction, or word choice, will be elevated, and the author will not define terms (as in Option B) or use informal language (as in D -- "really growing in popularity" is too colloquial, or conversational, to fit in with this style). Nor will the author express an opinion (as in C) -- instead, s/he simply lays out the facts.

Tone

The style of a piece is often closely linked to the tone. The tone of a text expresses the writer’s attitude toward the subject. The most effective way to analyze tone is to establish if the text is objective or subjective. •

Objective  tone §

Impartial,  neutral,  unbiased  

§

Does  not  show  an  opinion  or  preference  for  a  topic  

§

Uses  words  with  neutral/objective  connotations  ("economical"  rather   than  "cheap,"  for  example).  



§

Avoids  first  and  second  person  pronouns  

§

Generally  more  formal  diction  or  style  

Subjective  tone §

Personal  point  of  view  

§

May  use  first  or  second  person  pronouns  

§

Often  relies  on  emotions  and  personal  experiences  or  impressions  

Take  your  S   AT  prep  to  the  next  level.  Visit  www.learnerator.com   251  

252     §

Uses  evaluative  words  (clever,  thoughtful,  lazy,  argumentative)  

§

Biased  or  clearly  opinionated  

§

Generally  informal  diction  or  style  

In both objective and subjective texts, the reader can further analyze the tone by evaluating the connotations, or implications, of the words the author chooses. The author’s attitude toward the subject is revealed in the evaluative or connotative words he or she uses. Review the following excerpt from Wuthering Heights: This time, I remembered I was lying in the oak closet, and I heard distinctly the gusty wind, and the driving of the snow; I heard, also, the fir bough repeat its teasing sound, and ascribed it to the right cause: but it annoyed me so much, that I resolved to silence it, if possible; and, I thought, I rose and endeavoured to unhasp the casement. The hook was soldered into the staple: a circumstance observed by me when awake, but forgotten. ‘I must stop it, nevertheless!’ I muttered, knocking my knuckles through the glass, and stretching an arm out to seize the importunate branch; instead of which, my fingers closed on the fingers of a little, ice-cold hand! The intense horror of nightmare came over me: I tried to draw back my arm, but the hand clung to it, and a most melancholy voice sobbed, ‘Let me in—let me in!’ ‘Who are you?’ I asked, struggling, meanwhile, to disengage myself. ‘Catherine Linton,’ it replied, shiveringly (why did I think of Linton? I had read Earnshaw twenty times for Linton)—‘I’m come home: I’d lost my way on the moor!’ As it spoke, I discerned, obscurely, a child’s face looking through the window. Terror made me cruel; and, finding it useless to attempt shaking the creature off, I pulled its wrist on to the broken pane, and rubbed it to and fro till the blood ran down and soaked the bedclothes: still it wailed, ‘Let me in!’ and maintained its tenacious gripe, almost maddening me with fear. ‘How can I!’ I said at length. ‘Let me go, if you want me to let you in!’ The fingers relaxed, I snatched mine through the hole, hurriedly piled the books up in a pyramid against it, and stopped my ears to exclude the lamentable prayer. I seemed to keep them closed above a quarter of an hour; yet, the instant I listened

Take  your  S   AT  prep  to  the  next  level.  Visit  www.learnerator.com   252  

253     again, there was the doleful cry moaning on! ‘Begone!’ I shouted. ‘I’ll never let you in, not if you beg for twenty years.’ ‘It is twenty years,’ mourned the voice: ‘twenty years. I’ve been a waif for twenty years!’ Thereat began a feeble scratching outside, and the pile of books moved as if thrust forward. I tried to jump up; but could not stir a limb; and so yelled aloud, in a frenzy of fright. To my confusion, I discovered the yell was not ideal: hasty footsteps approached my chamber door; somebody pushed it open, with a vigorous hand, and a light glimmered through the squares at the top of the bed. I sat shuddering yet, and wiping the perspiration from my forehead: the intruder appeared to hesitate, and muttered to himself. At last, he said, in a half-whisper, plainly not expecting an answer, ‘Is any one here?’ I considered it best to confess my presence; for I knew Heathcliff’s accents, and feared he might search further, if I kept quiet. With this intention, I turned and opened the panels. I shall not soon forget the effect my action produced. Which words best described the author’s tone in the passage?

A. Frightened and embarrassed B. Apprehensive and angry C. Panicked and spiteful D. Alarmed and callous Answer: D The narrator in this passage is alarmed by the noise outside the window, and again by the clasp of the ghostly hands on his own. He states, “Terror made me cruel,” and goes on to explain how he tricked the ghost into releasing him, only to taunt her by telling her he will never let her in. Review the following sentence from the passage above: “I  heard,  also,  the  fir  bough  repeat  its  teasing  sound,  and  ascribed  it  to  the  right   cause:  but  it  annoyed  me  so  much,  that  I  resolved  to  silence  it,  if  possible;  and,  I  

Take  your  S   AT  prep  to  the  next  level.  Visit  www.learnerator.com   253  

254     thought,  I  rose  and  endeavored  to  unhasp  the  casement.”   What tone does the author employ in this sentence to best express the narrator’s emotional state?

A. Fear B. Anxiety C. Frustration D. Apprehension Answer: C In this sentence the author chose words such as “teasing” and “endeavored” to communicate a frustrated tone, as the narrator complains about an annoying sound outside his window and struggles to open the window to resolve the problem. (This question has a very obvious answer if you read the text: "It annoyed me so much...") Sometimes a writer will let his/her feelings creep into an otherwise neutral piece. Use context clues, such as words that demonstrate emotion, to ascertain which parts of a text display some sort of bias. Review the excerpt below before going on to the questions that follow: The Dark Romantic authors are a favorite among middle school and high school readers. The works of Poe, and to a lesser extent Hawthorne and Melville, touch on the depraved, the grotesque, and the horrific, all topics that seem elicit, yet appealing to the adolescent mind.1 Many Americans, in fact, report that the most memorable reading experiences of their secondary academic careers involved reading the works of one or more of these authors. In contrast, the Transcendentalist works of Emerson and Thoreau, when they are taught, tend to leave student-readers feeling confused and disconnected from the text. Although the Dark Romantics emerged directly as a response to Transcendentalism, the

Take  your  S   AT  prep  to  the  next  level.  Visit  www.learnerator.com   254  

255     connection between the two genres is seldom explored in secondary classrooms, thus leaving students with an incomplete understanding of the works they so enjoy. 2Unlike the Transcendentalists, who celebrated the inherent perfection of the human condition and the notion that the spirit of each individual, if it could be reached, would connect all people and nature, the Dark Romantics asserted that mankind was inherently flawed, fallible, and prone to sin and self destruction.3 That is, while the heroes of Transcendental literature sought their more perfect selves, the protagonists of Dark Romantic literature struggled to avoid their natural states of depravity, and while the Transcendentalists saw the supreme divinity personified in the natural world, accessible to anyone who could transcend the

selfishness of the individual, the Dark Romantics perceived the world

around them to be populated with anthropomorphic representations of the devil and his demon-minions.4 Almost every character in a Dark Romantic text has the potential to be Satan himself. Edgar Allen Poe, perhaps the most well-known of the Dark Romantics, rejected Transcendentalism to the point of publicly mocking the movement for what he considered an overtly literary approach that lacked meaning. He decried the use of “metaphor for metaphor’s sake,” and thought many of the self-proclaimed Transcendentalists were pretenders to the movement who produced falsely mystical stories which included impossibly spiritual outcomes.5 Poe’s own works were populated by criminals and madmen. As the inventor of American detective fiction, Poe created a genre dedicated to identifying and flushing out evil in society. His focus on motive, on what made criminals commit crime, frequently led his detectives to identify deep-seeded madness within a criminal, and to suggest that all humans are capable of such depravity, but that some are more successful than others at suppressing it.6

Take  your  S   AT  prep  to  the  next  level.  Visit  www.learnerator.com   255  

256     While Poe’s reaction to Transcendentalism was primary based on what he’d read and those he encountered who were engaged in the movement, Nathaniel Hawthorne found himself drawn away from Transcendentalism to Dark Romanticism because of his own experience and family history. Which sentence in the passage above most demonstrates a biased or subjective point of view?

A. Sentence (1) B. Sentence (2) C. Sentence (4) D. Sentence (5) Answer: B In the following sentence, the author demonstrates a subjective opinion of the teaching of Transcendentalism and Dark Romanticism in secondary classrooms. The author notes, “Although the Dark Romantics emerged directly as a response to Transcendentalism, the connection between the two genres is seldom explored in secondary classrooms, thus leaving students incomplete understanding of the works they so enjoy.” This sentence suggests that teachers ignore the connection between the two schools of literature and thus neglect making connections necessary for students to best understand literature they already enjoy.

What revision of sentence (2) best results in an objective tone?

A. While 90% of secondary teachers report teaching the works of the Dark Romantics, only 40% report that they consistently teach the works of the Transcendentalists, whose works inspired the Dark Romantics. B. Most teachers cover the Dark Romantics in great detail but don’t bother with the Transcendentalists because their work is too challenging for high

Take  your  S   AT  prep  to  the  next  level.  Visit  www.learnerator.com   256  

257     school students. C. Teachers should cover both the Dark Romantics and the Transcendentalists, since it is difficult to understand the works of authors like Poe if one hasn’t read authors like Emerson. D. Dark Romantic literature is very popular with students, so it is taught more often than Transcendentalism, which students don’t like much at all. Answer: A Using statistics is one of the best ways to avoid a subjective or biased tone. Note that this revision does not suggest that teachers are providing an “incomplete” education, or that either genre is neglected. The revision includes only facts that can be supported and confirmed with evidence.

Review sentence (3) from the passage. Which revision most effectively improves the style of the writing?

A. Unlike the Transcendentalists who celebrated the inherent perfection of the human condition and the notion that each individual could reach his own spirit and connect it to those of all other people and nature, the Dark Romantics asserted that mankind was flawed, fallible, and prone to sin and self destruction. B. The Transcendentalists thought people were most good, while the Dark Romantics thought people were generally evil. C. The Transcendentalists celebrated man’s spirit and how it could be connected to the spirits of nature, while the Dark Romantics insisted that mankind was flawed, fallible and prone to sin and self destruction. D. Unlike the Dark Romantics, who saw that self-destruction was inevitable for mankind, the Transcendentalists celebrated how perfect the human condition was and how all living things are connected.

Take  your  S   AT  prep  to  the  next  level.  Visit  www.learnerator.com   257  

258     Answer: A Revision A corrects the passive form of the sentence by suggesting that the “individual” can connect his spirit, rather than the passive construction, “the spirit, if it could be reached, would connect…” Review the sentence below from the passage: The works of Poe, and to a lesser extent1 Hawthorne and Melville, touch on the depraved2, the grotesque3, and the horrific, all topics that seem elicit4, yet appealing to the adolescent mind. Which section includes an error in diction?

A. 1 B. 2 C. 3 D. 4 Answer: D In option D (4), the writer has made a word-choice error. “Elicit” means to evoke or drawn a reaction or result. It is likely this author intended to use the word “illicit,” which means forbidden. Review the sentence below from the passage: His focus on motive1, on what made criminals commit crime, frequently led his detectives to identify deep-seeded madness2 within a criminal, and to suggest that all humans are capable of such depravity3, but that some are more successful than others at suppressing it4.

Take  your  S   AT  prep  to  the  next  level.  Visit  www.learnerator.com   258  

259     Which section includes an error in diction?

A. 1 B. 2 C. 3 D. 4 Answer: B This is an error in the use of a phrasal adjective. The correct phrase is “deepseated,” meaning that something is “seated” or firmly in place within a context. In this case, the madness noted is firmly in place within the criminal mind. You can determine the style and tone of a piece by paying attention to the words used -- particularly if they elicit a specific emotion, communicate a bias, or argue a point. As you read, pay attention to the diction of a piece, and consider its intended audience. Remember too that all written works have some sort of style, whether it's formal or informal, objective or subjective, or somewhere in between. This will help you accurately answer questions that the new SAT may pose about style, tone, and grammar.

Take  your  S   AT  prep  to  the  next  level.  Visit  www.learnerator.com   259  

260    

Syntax Syntax   Syntax  questions  require  readers  to  consider  a  text  at  the  sentence  level.  That  is,  the   reader  should  be  able  to  identify  and  apply  necessary  changes  to  sentences  in  order   to  improve  the  clarity,  precision,  or  logic  of  the  writing.  Questions  on  this  topic   might  require  students  to:     •

Combine  short  or  related  sentences  



Identify  sentence  revisions  that  improve  logical  connections  between  ideas.  



Identify  sentence  revisions  that  correct  grammar  errors.  



Identify  sentence  revisions  that  improve  clarity.    

Read  the  passage  below,  and  consider  the  relationships  between  the  sentences  as   you  do  so:   Puritan  literature  is  rooted  in  the  belief  that  God  is  always  at  work  in  the  daily   lives  of  humans.  Most  texts  describe  the  discovery  of  evidence  of  God’s   intervention  or  participation  in  human  lives.1Texts  often  include  allusions  to   classical  literature,  as  Puritans  valued  education.2Because  they  eschewed   flowery  prose,  the  allusions  are  straightforward  and  direct.  3Puritan  literature   is  the  seed  of  the  American  identity.4It  stresses  self-­‐reliance  and  overcoming   hardship  to  achieve  a  goal.5These  values  are  often  associated  with  pursuit  of   the  American  Dream.6     Which  transition  added  to  the  beginning  of  sentence  (2)  would  best   indicate  the  relationship  to  the  preceding  sentence?   A.  Although   B.  As  a  result   C.  To  begin  with  

Take  your  S   AT  prep  to  the  next  level.  Visit  www.learnerator.com   260  

261     D.  Moreover     Answer:  B     The  first  two  sentences  of  the  passage  indicate  a  cause/effect  relationship.  The  first   sentence  describes  the  “root”  of  Puritan  literature.  The  second  sentence  describes   the  focus  of  the  literature  resulting  from  that  root-­‐belief.     Which  is  the  best  revision  of  sentence  (3)?   A.  Because  the  Puritans  eschewed  flowery  prose,  the  allusions  are   straightforward  and  direct.   B.  To  avoid  flowery  prose,  the  allusions  are  straightforward  and  direct.   C.  They  eschewed  flowery  prose;  the  allusions  they  preferred  are   straightforward  and  direct.   D.  In  lieu  of  flowery  prose,  the  allusions  are  straightforward  and  direct.      Answer:  A     To  answer  this  type  of  question  the  reader  should  begin  by  confirming  what  the   sentence  intends  to  say.  In  this  case,  there  are  two  main  points:  the  Puritans   rejected  (eschewed)  flower  prose,  and,  as  a  result  of  that  rejection,  the  allusions  are   straightforward  and  direct.  This  sentence  has  a  dangling  modifier  error,  caused  by   the  lack  of  an  antecedent  for  the  pronoun  “they”  in  the  first  clause.  That  is,  this   sentence  seems  to  say  that  “they”  refer  to  the  allusions  in  the  second  clause.  In  fact,   “they”  refers  to  the  Puritans.  So,  replacing  “they”  with  “Puritans,”  corrects  the   sentence  and  makes  it  clear.     Which  sentence  most  effectively  combines  sentence  (5)  and  sentence   (6)?   A.  It  stresses  self-­‐reliance  and  overcoming  hardship  to  achieve  a  goal,  values   often  associated  with  pursuit  of  the  American  Dream.   Take  your  S   AT  prep  to  the  next  level.  Visit  www.learnerator.com   261  

262     B.  It  stresses  self-­‐reliance,  overcoming  hardship  to  achieve  a  goal,  and  values   often  associated  with  the  American  Dream.   C.  It  stresses  self-­‐reliance  and  overcoming  hardship  to  achieve  a  goal;  values   often  associated  with  pursuit  of  the  American  Dream.   D.  It  stresses  values  associated  with  the  pursuit  of  the  American  Dream  like   self-­‐reliance  and  overcoming  hardship  to  achieve  a  goal.     Answer:  A     To  choose  the  most  effective  sentence  the  reader  must  understand  the  context  of  the   passage.  The  main  point  of  the  last  three  sentences  is  that  Puritan  literature  was  a   “seed”  from  which  the  “American  identity”  grew  –  meaning  it  came  before  the   American  identity,  which  makes  sense,  since  there  was  no  America  when  it  was   written.       Understanding  the  context  enables  the  reader  to  dismiss  choice  D  as  incorrect,   because  it  suggests  that  the  Puritans  were  in  pursuit  of  the  American  Dream,  which   was  not  the  case.  Choice  B  indicates  that  the  “values  often  associated  with….”  are   different  from  self-­‐reliance  and  overcoming  hardship,  which  is  not  the  case.  Choice  C   is  not  correct  because  one  should  use  a  semicolon  (;)  only  between  two  independent   clauses.  The  second  clause  in  this  sentence  could  not  stand-­‐alone;  it  is  not  an   independent  clause.  Choice  A  is  correct.     Review  the  passage  below:   Most  people  are  familiar  with  catnip’s  effects  on  cats,  but  many  are  not   aware  that  catnip  has  a  long  history  of  use  as  a  medicinal  and  culinary   herb.1A  member  of  the  mint  family,  catnip  can  also  be  used  in  tea,  and   as  a  savory  accent  to  soups  and  stews.2While  American  cooks  prefer   more  subtle  seasonings,  French  cooking  uses  the  sharp,  minty  taste  of   catnip  in  many  well-­‐known  dishes.3One  popular  French  dish  that  uses  

Take  your  S   AT  prep  to  the  next  level.  Visit  www.learnerator.com   262  

263     catnip  is  bouillabaisse,  a  fish  stew.4      

Take  your  S   AT  prep  to  the  next  level.  Visit  www.learnerator.com   263  

264     Review  the  sentence  below:   Catnip  is  used  in  traditional  medicines  to  treat  insomnia,  stomach  upset,  and   headaches.     Where  would  this  sentence  best  fit  within  the  given  passage?   A.  After  sentence  (1)   B.  Before  sentence  (1)   C.  After  sentence  (2)   D.  Before  sentence  (4)      Answer:  A     Sentence  (1)  is  the  topic  sentence,  and  it  establishes  two  focus  areas:  medicinal  and   culinary  uses  for  catnip.  Since  “medicinal”  is  mentioned  first,  it  would  logically  be   elaborated  on  first.  Note,  as  well,  that  sentence  (2)  reads,  “catnip  can  also  be  used…”   This  transitional  word  indicates  a  comparison.  Adding  the  given  sentence  before   sentence  (2)  provides  the  point  of  comparison.     Which  is  the  most  effective  revision  of  sentence  (3)?   A.  While  American  cooks  prefer  more  subtle  seasonings,  French  chefs  use  the   sharp,  minty  taste  of  catnip  in  many  well-­‐known  dishes.   B.  While  American  cooks  prefer  more  subtle  seasonings,  the  French  use  the   sharp,  minty  taste  of  catnip  in  many  well-­‐known  dishes.   C.  Although  American  cooks  prefer  more  subtle  seasonings,  French  cooking   uses  the  sharp,  minty  taste  of  catnip  in  many  well-­‐known  dishes.   D.  While  American  cooks  prefer  more  subtle  seasonings,  but  French  cooking   uses  the  sharp,  minty  taste  of  catnip  in  many  well-­‐known  dishes.     Answer:  A     Sentence  (3)  includes  an  error  in  comparison.  The  sentence  begins  with  “American   Take  your  S   AT  prep  to  the  next  level.  Visit  www.learnerator.com   264  

265     cooks,”  so  it  refers  to  the  people  who  do  the  cooking.  In  the  existing  sentence,  cooks   are  compared  to  cooking,  which  is  flawed.  To  correct  the  sentence,  the  comparison   must  be  between  cooks  or  between  types  of  cooking.  Only  A  revises  the  sentence  so   that  cooks  are  compared  to  chefs  (cooks).     Understanding  syntax  is  an  important  part  of  honing  your  verbal  skills.  As  you  read,   consider  the  relationships  that  each  sentence  sets  up.  Are  the  sentences  all   grammatical?  Do  they  introduce  or  reinforce  direct  and  logical  relationships  to  one   another,  or  to  the  content  of  the  text?  Paying  close  attention  and  thinking  critically   about  these  ideas  will  help  you  to  score  high  on  the  SAT.      

 

Take  your  S   AT  prep  to  the  next  level.  Visit  www.learnerator.com   265  

266    

            Part  V:  Standard  English  Conventions   (Writing)    

 

 

Take  your  S   AT  prep  to  the  next  level.  Visit  www.learnerator.com   266  

267    

Sentence Formation Basic  Parts  of  a  Sentence   A  simple  sentence  consists  of  a  subject  and  a  predicate.  The  subject  of  the  sentence   is  a  person,  place,  thing,  or  idea.  The  predicate  of  the  sentence  contains  a  verb  that   states  something  about  the  subject.     •

He  walks.  



John  writes.  



They  talk.    

The  first  words  in  the  examples  above  are  subjects,  and  the  final  words  in  these   examples  are  predicates.  This  is  the  basic  structure  of  all  simple  sentences.     The  Subject   The  subject  of  a  sentence  can  be  a  noun,  a  pronoun,  or  a  noun  phrase.  It  usually   appears  at  or  near  the  beginning  of  a  sentence.  The  subjects  in  the  following   examples  have  been  underlined.     These  sentences  have  nouns  as  their  subjects.  Nouns  can  be  common,  proper,  or   collective.  Always  remember  that  the  noun  that  functions  as  a  subject  will  NEVER  be   part  of  a  prepositional  phrase.     •

The  dude  lied.  (common  noun)  



Lilith  dances  beautifully.  (proper  noun)  



The  pack  of  wolves  howled  at  the  moon.  (collective  noun)    

These  sentences  have  pronouns  as  their  subjects.  Always  use  subjective  pronouns  as   subjects,  and  never  use  an  objective  pronoun.    

Take  your  S   AT  prep  to  the  next  level.  Visit  www.learnerator.com   267  

268     •

He  giggles.  (singular  pronoun)  



Yesterday,  they  fell  down  the  stairs.  (plural  pronoun)  



Me  and  Joe  are  very  good  friends.  (incorrect  usage  of  an  objective  pronoun)    

These  sentences  have  noun  phrases  as  their  subjects.  Although  a  noun  phrase  is   made  up  of  multiple  words,  they  collectively  form  a  single  component  that  functions   as  a  normal  noun  in  the  sentence.  Noun  phrases  may  be  gerund  phrases  or  infinitive   phrases.     •

Lying  about  yourself  is  bad.  (gerund  phrase)  



To  learn  is  a  good  thing.  (infinitive  phrase)    

Verbs   All  predicates  must  contain  at  least  one  verb.  The  three  primary  types  of  verbs  are   action  verbs,  linking  verbs,  and  auxiliary  verbs.  The  verbs  in  the  following  examples   have  been  underlined.     Action  verbs  are  expressive  of  an  activity  being  performed.  An  action  verb  can  be   either  transitive  or  intransitive.  A  transitive  verb  is  followed  by  a  direct  object.  An   intransitive  verb  is  not  followed  by  a  direct  object.  A  direct  object  cannot  be  found  in   a  prepositional  phrase.     •

The  dog  runs.  (intransitive  verb)  



A  fireman  climbs  the  ladder.  (transitive  verb-­‐-­‐the  word  ladder  is  the  direct   object)  



The  dolphins  swim  in  the  water.  (intransitive  verb-­‐-­‐the  word  water  is  not  the   direct  object  because  it  is  the  object  of  a  prepositional  phrase)    

A  linking  verb  attaches  a  subject  compliment  to  the  subject.  The  subject   compliments  in  these  sentences  are  in  italics.  A  subject  compliment  can  be  a  noun  

Take  your  S   AT  prep  to  the  next  level.  Visit  www.learnerator.com   268  

269     that  renames  the  subject  or  an  adjective  that  describes  the  subject.     •

The  food  on  the  table  smells  delicious.  



The  old  man  looks  very  happy.  



The  fireman  is  Joe.    

Auxiliary  verbs  are  smaller  verbs  placed  in  front  of  the  main  verb.  Up  to  three   auxiliary  verbs  may  be  used  in  a  sentence  along  the  main  verb.  Auxiliary  verbs   express  tense,  mode,  and  voice.   •

Jane  is  baking.  (expresses  tense  -­‐  present  participle)  



Jane  could  have  baked  the  cake.  (expresses  mode  -­‐  possibility)  



The  cake  might  have  been  baked  by  Jane.  (expresses  voice  -­‐  passive)    

Direct  Objects   The  predicate  of  the  sentence  may  also  contain  a  direct  object,  an  indirect  object,  or   a  subject  compliment.  The  direct  object  receives  the  action  of  the  verb,  and  the   indirect  object  receives  what  is  being  given  or  done.  The  subject  compliment  is  an   adjective  or  a  noun  and  either  describes  or  renames  the  subject.   The  italicized  words  are  examples  of  direct  objects.  Direct  objects  can  be  nouns,   pronouns,  noun  phrases,  or  subordinate  clauses.     •

Robert  reads  books.(noun)  



Jane  hit  him.  (pronoun)  



She  loves  running  in  the  morning.  (noun  phrase)  



Bill  hates  when  she  doesn't  listen.  (subordinate  clause)    

Indirect  Objects   The  words  in  italics  are  examples  of  indirect  objects.  Indirect  objects  can  be  either   nouns  or  pronouns.  Indirect  objects  can  also  be  placed  within  prepositional  phrases.     •

Phil  brings  Jane  candy.  (noun)  

Take  your  S   AT  prep  to  the  next  level.  Visit  www.learnerator.com   269  

270     •

He  finds  you  money.  (pronoun)  



Bill  gives  food  to  the  needy.  (inside  a  prepositional  phrase)    

Subject  Compliments   The  words  in  italics  are  subject  compliments.  A  subject  compliment  can  be  a  noun  or   a  pronoun  that  renames  the  subject  or  an  adjective  that  describes  the  subject  (use   subject  form  of  pronoun  as  a  subject  compliment-­‐-­‐it  sounds  strange,  but  it  is   correct).  A  subject  compliment  is  always  followed  by  a  linking  verb.  A  subject  can   also  be  a  participial  phrase  or  an  infinitive  phrase).     •

Jim  is  the  principal.  (noun)  



The  principle  is  he.  (pronoun  -­‐  subject  form)  



Mountains  are  beautiful.  (adjective)  



She  seems  to  be  distracted.  (infinitive  phrase)  



She  is  floating  on  a  cloud.  (participial  phrase)    

Clauses   A  clause  is  formed  with  a  subject  and  a  predicate  like  a  sentence;  however,  it  is  used   as  a  building  block  for  a  larger  sentence  that  is  more  complex.  A  clause  may  be   either  independent  or  subordinate.     Independent  clauses  are  capable  of  standing  alone  as  complete  sentences.  Every   sentence  must  contain  at  least  one  clause.  An  independent  clause  expresses  a   complete  thought.     •

He  sat  on  his  porch  while  his  cat  played  on  his  lawn.  



Since  they  were  so  thirsty,  they  bought  some  bottled  water  to  drink.  



Bob  was  riding  on  his  green  tractor  because  he  needed  to  plow  his  field.    

Subordinate  clauses  begin  with  a  subordinating  conjunction  and  are  not  capable  of   standing  alone  as  complete  sentences.  The  subordinating  conjunctions  in  the   Take  your  S   AT  prep  to  the  next  level.  Visit  www.learnerator.com   270  

271     following  examples  have  been  italicized.  Subordinate  clauses  begin  with  either  a   subordinate  conjunction  or  a  relative  pronoun.  In  addition,  subordinate  clauses   make  the  reader  want  more  information     •

He  shot  his  rifle  because  he  felt  like  it.  (subordinate  conjunction)  



He  was  on  the  team,  which  had  won  every  game.  (relative  pronoun)  

  Four  Types  of  Sentences   There  are  four  types  of  sentences:  simple,  compound,  complex,  and  compound-­‐ complex.   Simple  sentences  are  formed  from  a  single  independent  clause  that  stands  on  its   own.  No  further  information  is  needed  in  order  to  completely  understand  the  idea   that  is  being  asserted  by  the  independent  clause.     •

Jay  walks.  



The  gorilla  weighed  800  pounds.  



The  preacher  spoke  for  several  hours.  

  Compound  sentences  are  formed  from  two  independent  clauses  that  are   connected  by  a  coordinating  conjunction.  There  are  only  seven  coordinating   conjunctions,  and  they  are  for,  and,  nor,  but,  or,  yet,  and  so.  They  can  be   remembered  by  using  the  acronym  FANBOYS.   Compound  sentences  MUST  use  a  comma  right  before  the  coordinating  conjunction   unless  the  clauses  are  very  short.  This  is  the  most  common  punctuation  mistake  in   English.  The  coordinating  conjunctions  in  the  following  examples  are  underlined.     •

He  went  the  grocery  store  yesterday,  and  he  went  to  the  zoo  last  week.  



Bob  ran  all  the  way  home,  but  he  forget  his  key  at  work.  



The  big  lion  wandered  out  of  its  cage,  so  the  zookeeper  shot  it  with  a   tranquilizer  dart.  

  Take  your  S   AT  prep  to  the  next  level.  Visit  www.learnerator.com   271  

272     Complex  sentences  are  formed  from  an  independent  clause  and  a  dependent   clause.  When  the  dependent  clause  comes  first,  a  comma  is  used  to  separate  the  two   clauses.  When  the  independent  clause  comes  first,  nothing  is  used  to  separate  the   two  clauses.     •

Because  he  likes  the  taste  of  ice  cream,  he  eats  his  desserts  very  quickly.  



He  eats  his  desserts  very  quickly  because  he  likes  the  taste  of  ice  cream.  

  Compound-­‐complex  sentences  are  formed  from  two  independent  clauses  and  a   dependent  clause.  There  are  many  possibilities  and  combinations  that  are  possible   with  these  types  of  sentences.     •

He  went  to  the  stadium  because  he  wanted  to  see  the  game,  but  he  fell  asleep   in  his  chair.  



Since  he  was  a  model  citizen,  he  picked  up  the  smelly  garbage,  and  he  threw   it  away  in  a  nearby  trash  can.  

Shifts in Construction Certain  components  of  a  sentence  need  to  remain  consistent  and  parallel.  However,   there  are  inappropriate  shifts  in  tense,  voice,  mood,  person,  number,  and  discourse   that  make  it  hard  for  the  reader  to  grasp  the  intended  meaning  of  a  sentence.  The   most  common  inappropriate  shifts  in  sentence  construction  will  now  be  examined.     Tense   Verb  tense  expresses  the  time  that  the  action  of  the  verb  takes  place.  If  the  tense   within  the  sentence  is  changed,  then  the  meaning  of  the  sentence  will  often  become   illogical.  However,  unnecessary  shifts  in  tense  are  easy  to  notice  and  repair.     Incorrect  Shift  of  Tense  

Take  your  S   AT  prep  to  the  next  level.  Visit  www.learnerator.com   272  

273     •

He  caught  the  fish  and  then  throws  in  back  into  the  water.  (past  to  present   tense)  



She  will  buy  the  chicken  and  cooked  it  in  her  oven.  (future  to  past  tense)      

Correct  Repair   •

He  caught  the  fish  and  then  threw  it  back  into  the  water.  (both  past  tense)  



She  bought  the  chicken  and  cooked  it  in  her  oven.  (both  past  tense)  

  Voice   A  verb  can  be  used  to  express  either  active  or  passive  voice.  Active  voice  is  when  the   subject  of  the  sentence  performs  the  action  of  the  sentence.  Passive  voice  is  when   the  subject  of  the  sentence  does  not  perform  the  action  of  the  sentence.  When  a   sentence  uses  two  verbs  they  usually  need  to  use  the  same  voice.     Incorrect  Shift  of  Tense   •

Some  people  want  candy,  but  sometimes  bananas  are  received.  (active  to   passive)  



When  the  lady  turned  up  the  radio,  her  favorite  song  was  heard.  (active  to   passive)      

Correct  Repair   •

Some  people  want  candy  but  sometimes  receive  bananas.  (both  active)  



When  the  lady  turned  up  the  radio,  she  heard  her  favorite  song.  (both  active)  

  Mood   A  verb  may  also  be  used  to  express  mood.  There  are  three  types  of  mood:  indicative,   imperative,  and  subjunctive.  The  errors  in  mood  shift  occur  when  either  indicative  is   shifted  to  imperative  within  a  sentence  or  when  imperative  is  shifted  to  indicative.     Incorrect  Shift  of  Mood  

Take  your  S   AT  prep  to  the  next  level.  Visit  www.learnerator.com   273  

274     •

Stand  up  in  the  room,  and  you  speak  to  the  audience.  (imperative  to   indicative)  



Take  your  medicine,  and  you  call  your  doctor.  (imperative  to  indicative)      

Correct  Repair   •

Stand  up  in  the  room,  and  speak  to  the  audience.  (both  imperative)  



Take  your  medicine,  and  call  your  doctor.  (both  imperative)  

  Person   There  are  three  perspectives  (or  points  of  view)  in  English.  They  are  first  person,   which  is  the  person  speaking.  The  next  is  second  person,  which  is  the  person  spoken   to.  The  last  one  is  third  person,  which  is  the  person  spoken  about.  Most  unnecessary   shifts  in  person  usually  occur  when  the  third  person  changes  to  the  second  person.       Incorrect  shift  in  person   •

Individuals  like  being  busy,  so  when  a  job  is  slow,  you  are  sad.  (third  to   second)  



Dogs  want  dog  food,  so  while  they  eat,  you  are  satisfied.  (third  to  second)    

Correct  Repair   •

You  like  being  busy,  so  when  a  job  is  slow,  you  are  sad.  (both  second)  



Dogs  want  dog  food,  so  while  they  eat,  they  are  satisfied.  (both  third)  

  Number   An  antecedent  is  a  word  or  a  phrase  that  is  represented  by  a  pronoun.  One  should   use  singular  pronouns  to  refer  to  singular  antecedents  and  use  plural  pronouns  to   refer  to  plural  antecedents.  Errors  that  involve  unnecessary  shift  of  number  are   usually  very  easy  to  detect  and  repair.     Incorrect  shift  in  number  

Take  your  S   AT  prep  to  the  next  level.  Visit  www.learnerator.com   274  

275     •

If  Janet  goes  out  in  the  sun,  then  they  need  to  wear  sunscreen.  (singular  to   plural)  



While  he  rode  in  the  car  together,  the  boys  cleaned  their  shoes.  (singular  to   plural)      

Correct  Repair   •

If  Janet  goes  out  in  the  sun,  then  she  needs  to  wear  sunscreen.  (both  singular)  



While  they  rode  in  the  car  together,  the  boys  cleaned  their  shoes.  (both   plural)  

  Discourse   Indirect  discourse  is  when  dialogue  is  reported  and  not  quoted.  Direct  discourse   uses  quotation  marks  and  actually  quotes  what  is  said.  Most  errors  involving   discourse  occur  when  direct  discourse  is  not  enclosed  in  quotation  marks  and   appears  as  indirect  discourse.     Incorrect  expression  of  discourse   •

He  stated  he  was  really  happy  and  let's  to  go  to  the  park.  (both  indirect)      

Correct  Repair   •

He  stated  he  was  really  happy  and  said,  "Let's  go  to  the  park."  (indirect  and   direct)  



He  said,  "I  am  really  happy.  Let's  go  to  the  park."(both  direct)    

Take  your  S   AT  prep  to  the  next  level.  Visit  www.learnerator.com   275  

276    

Pronouns Pronouns   A  pronoun  is  a  word  that  is  substituted  for  a  noun  or  noun  phrase.  The  word  for   which  it  is  used  as  a  substitute  is  named  or  understood  within  the  context  of  the   prose.   There  are  eight  different  types  of  pronouns:     •

personal  -­‐  related  to  a  singular  entity  or  group  



indefinite  -­‐  refers  to  a  non-­‐specific  item(s),  being(s),  or  location(s)  



relative  -­‐  begins  a  relative  clause  that  functions  as  either  a  noun  or  an   adjective  



interrogative  -­‐  asks  a  question  



reflexive  -­‐  refers  back  to  the  subject  of  the  sentence  



possessive  -­‐  indicates  ownership  



demonstrative  -­‐  distinguishes  things  that  are  spoken  about    

  PERSONAL  PRONOUNS   This  type  of  pronoun  specifies  either  a  person,  animal,  thing,  or  idea.  A  personal   pronoun  will  use  different  forms  depending  on  case,  number,  gender,  and   grammatical  person.     Number   Singular  and  plural  pronouns  are  used  in  order  to  denote  the  number  of  people  or   items  that  are  referenced.     •

I  will  go  fishing  with  him.  (Singular  pronouns)  



They  have  seen  us  going  to  the  river  with  them.  (Plural  pronouns)    

Gender   Singular  pronouns  referring  to  a  person  will  take  on  different  forms  in  order  to   Take  your  S   AT  prep  to  the  next  level.  Visit  www.learnerator.com   276  

277     denote  gender.  The  pronoun  it  is  gender  neutral.  Only  third  person  pronouns  specify   gender.     •

She  walks  with  her.  (Feminine  gender)  



He  plays  with  him.  (Masculine  gender)  

  Case   Personal  pronouns  may  be  either  subjective  or  objective,  depending  on  their  use  in  a   sentence.  The  subjective  form  is  used  as  either  as  a  subject  of  a  sentence  or  as  a   subject  compliment.  The  objective  form  is  used  either  as  a  direct  object  (or  indirect   object)  of  a  sentence  or  as  the  object  of  a  prepositional  phrase.     •

We  will  go  to  the  aquarium.    (Subjective  -­‐  used  as  the  subject  of  a  sentence)  



Bob  has  left  Martha  for  her.    (Objective  -­‐  object  of  the  preposition  for)  

  Person   Personal  pronouns  have  three  different  forms  to  denote  the  concept  of  the   grammatical  "person":     •

First  person:  Person  or  persons  speaking  



Second  person:  Person  or  persons  spoken  to  



Third  person:  Person  or  persons  spoken  about    

Here  are  some  examples  of  this  concept:     •

We  like  football.  (first  person  plural  -­‐  subjective)  



You  watch  football  on  television.  (second  person  singular  or  plural  -­‐   subjective)  



Football  annoys  her.  (third  person  singular  -­‐  objective)    

Here  is  a  list  of  the  personal  pronouns  used  in  English:   Take  your  S   AT  prep  to  the  next  level.  Visit  www.learnerator.com   277  

278       Subjective  Case   •

First  person  -­‐  I  (singular),  we  (plural)  



Second  person  -­‐  you  (all  forms)  



Third  person  -­‐  he  (singular  masculine),  she  (singular  feminine),  it  (singular   neutral),  they  (plural  neutral)    

Objective  Case   •

First  person  -­‐  me  (singular),  us  (plural)  



Second  person  -­‐  you  (all  forms)  



Third  person  -­‐  him  (singular  masculine),  her  (singular  feminine),  it  (singular   neutral),  them  (plural  neutral)  

  INDEFINITE  PRONOUNS   This  type  of  pronoun  refers  to  a  non-­‐specific  item(s),  being(s),  or  location(s).  This  is   the  largest  group  of  pronouns  in  the  English  language.  Indefinite  pronouns  can  be   easily  classified  by  the  types  of  verbs  they  use  (singular  and/or  plural).     •

Nobody  knows  everything.  (singular)  



Everybody  is  at  risk.  (singular)  



Others  play  with  that  team.  (plural)  



Many  were  selected.  (plural)  

  There  are  also  possessive  and  compound  forms  of  indefinite  pronouns.  Possessive   forms  end  with  an  apostrophe  and  the  letter  s.  Compound  forms  fuse  two  indefinite   pronouns  together  so  that  they  may  act  like  a  singular  entity.     Possessive  forms  of  indefinite  pronouns   •

It  could  be  anybody's  fault  



It  was  everyone's  responsibility  

Compound  forms  of  indefinite  pronouns   Take  your  S   AT  prep  to  the  next  level.  Visit  www.learnerator.com   278  

279     •

They  despised  one  another.  



They  loved  each  other.    

Here  is  a  list  of  the  indefinite  pronouns  used  in  English:     Singular   •

no  one,  nobody,  everyone,  everybody,  someone,  somebody,  anyone,  anybody,   nothing,  everything,  something,  anything,  each,  another,  other,  whatever,   whichever,  whoever,  whomever,  enough,  little,  less,  much,  more,  most,  plenty,   one,  either,  neither      

Plural   •

both,  others,  none,  all,  some,  any,  several,  few,  fewer,  many,  more,  most  

  RELATIVE   Relative  pronouns  begin  relative  clauses  that  function  as  either  nouns  or  adjectives.     •

He  saw  the  man  who  was  running  away  from  the  robber.  (adjective  clause)  



She  saw  the  bag  that  fell  from  the  robber's  hands.  (adjective  clause)  



She  despised  whomever  stood  in  her  way.  (noun  clause)    

In  first  two  examples  above,  the  relative  clauses  that  begin  with  relative  pronouns   serve  as  adjectives  that  describe  the  direct  objects  of  the  sentences.  In  the  final   example,  the  relative  pronoun  whomever  forms  a  noun  clause  that  functions  as  the   direct  object  of  the  sentence.   There  are  nine  relative  pronouns  in  English:   Adjective  clauses:      "who",  "whom",  "which",  "what",  "that"   Noun  clauses:                    "whoever",  "whomever",  "whichever",  "whatever"     INTERROGATIVE  

Take  your  S   AT  prep  to  the  next  level.  Visit  www.learnerator.com   279  

280     Interrogative  pronouns  ask  a  question.  There  are  five  interrogative  pronouns,  and   they  are  "which",  "what",  "who",  "whom",  and  "whose".  The  word  who  is  the   subjective  form  and  the  word  whom  is  used  as  the  objective  form.     •

What  is  she  going  to  do?  



Who  did  that?  (subjective  form  used  as  the  subject)  



To  whom  does  that  belong?  (objective  form  used  as  the  object  of  the   prepositional  phrase  that  begins  with  the  word  to)    

The  biggest  error  with  interrogative  pronouns  is  the  incorrect  usage  of  the  word   who  as  the  objective  case.  Most  individuals  are  not  sure  how  to  use  the  words  who   and  whom.       Incorrect  usage   •

The  letter  was  addressed  to  who?  (incorrect  usage  of  subjective  form)  



To  who  was  the  package  delivered?  (incorrect  usage  of  subjective  form)      

Correct  usage   •

The  letter  was  addressed  to  whom?  (correct  usage  of  objective  form)  



To  whom  was  the  package  delivered?  (correct  usage  of  objective  form)  

  REFLEXIVE   These  types  of  pronouns  refer  back  to  the  subject  of  the  sentence.  Every  subjective   personal  pronoun  has  a  reflexive  form.  Thus,  the  reflexive  pronouns  in  English  are   "myself",  "yourself",  "himself",  "herself",  "itself",  "ourselves",  "yourselves",  and   "themselves".  In  addition,  the  pronoun  one  also  has  a  reflexive  form  (oneself).     •

I  needed  it  myself.  (first  person)  



You  bought  the  purse  yourself.  (second  person)  



He  brought  all  of  that  on  himself.  (third  person  masculine)    

Take  your  S   AT  prep  to  the  next  level.  Visit  www.learnerator.com   280  

281     Most  of  the  grammatical  errors  that  occur  with  reflexive  pronouns  happen  when   they  are  used  as  the  subject  instead  of  referring  back  to  the  subject.       Incorrect  Usage   •

Myself  is  to  blame.  (incorrect  usage  of  the  singular  reflexive  form)  



Ourselves  have  caused  it.  (incorrect  usage  of  the  plural  reflexive  form)    

Correct  Usage   •

I  am  to  blame.  (correct  usage  of  first  person  singular)  



We  have  caused  it.  (correct  usage  of  first  person  plural)  

  POSSESSIVE   Possessive  pronouns  indicate  ownership.  Every  personal  pronoun  has  an  objective   form.  The  personal  pronouns  he  and  him  both  use  the  pronoun  his  for  their   possessive  form.   Therefore,  the  possessive  pronouns  in  English  are:  "its",  "my",  "mine",  "your",  "her",   "hers",  "his",  "our",  "ours",  "their",  "theirs",  and  "yours".     •

We  rode  in  their  car.  (third  person  plural  -­‐  objective)  



Your  car  is  black.  (second  person  singular  -­‐  subjective)    

By  far,  the  biggest  and  most  common  mistake  is  the  misuse  of  the  possessive   pronoun  "its".  Quite  often,  it  is  confused  with  the  contraction  it's,  which  is  a   combination  of  the  word  it  and  is.     Incorrect  usage   •

The  green  car  lost  it's  wheel.  (incorrect  usage  of  contraction)  



Its  all  in  a  days  work.  (incorrect  usage  of  pronoun)    

Correct  usage   •

The  green  car  lost  its  wheel.  (correct  usage  of  possessive  pronoun)  

Take  your  S   AT  prep  to  the  next  level.  Visit  www.learnerator.com   281  

282     •

It's  all  in  a  day's  work.  (correct  usage  of  contraction)  

  DEMONSTRATIVE   There  are  four  demonstrative  pronouns:  "this",  "that",  "these",  and  "those".  They   stand  on  their  own  and  distinguish  the  entity  or  entities  that  the  speaker  is  speaking   about.     •

This  is  what  I  mean.  (singular)  



Who  needs  those?  (plural)    

One  of  the  most  common  mistakes  when  using  demonstrative  pronouns  is  the   incorrect  use  of  the  pronouns  this  and  these.  The  pronoun  this  is  singular,  and  the   pronoun  these  is  plural.  Here  are  some  examples  of  correct  and  incorrect  usage  of   these  terms.     Incorrect  Usage   •

These  is  not  what  I  mean.  (incorrect  usage  of  the  plural  form)  



This  are  not  the  proper  kinds  of  napkins.  (incorrect  usage  of  the  singular   form)    

Correct  Usage   •

This  is  not  what  I  mean.  (correct  usage  of  the  singular  form)  



These  are  not  the  proper  kinds  of  napkins.  (correct  usage  of  the  singular   form)    

  Sample  Test  Questions   Select  the  answer  that  correctly  identifies  the  pronoun  (or  pronouns)  in  the   sentence.     1)  The  grumpy  old  man  fell  down  the  stairs  and  hurt  his  knee.  

Take  your  S   AT  prep  to  the  next  level.  Visit  www.learnerator.com   282  

283     A.  grumpy   B.  old   C.  fell   D.  and   E.  his     2)  The  cats  smelled  the  fish  and  ran  out  the  door   A.  cats   B.  and   C.  ran   D.  out   E.    There  is  no  pronoun  in  the  sentence.     3)  This  is  good  advice  because  she  really  likes  to  run.   A.  This,  because   B.  This,  she   C.  because,  she   D.  she,  to   E.  she,  likes     4)  Those  are  all  of  the  items  in  the  blue  basket  that  they  brought  to  the  picnic.   A.  Those,  all,  that   B.  Those,  in,  they   C.  all,  that,  to   D.  Those,  all,  they   E.  all,  in,  they     ANSWERS:      1)  E            2)  E            3)  B          4)  D  

Take  your  S   AT  prep  to  the  next  level.  Visit  www.learnerator.com   283  

284    

Possessive Determiners Also  referred  to  as  possessive  adjectives,  possessive  determiners  assign  possession   to  a  grammatical  element  that  functions  as  a  noun  in  a  sentence.   There  are  two  types  of  possessive  determiners  in  the  English  language:  weak   possessive  pronouns  and  nouns  that  end  with  an  apostrophe  and  the  letter  "s".     Pronouns  As  Possessive  Determiners   The  group  of  seven  weak  possessive  pronouns  form  the  first  group  of  possessive   determiners.  They  are  called  weak  because  they  cannot  stand  alone  like  other   pronouns  and  must  have  some  type  of  noun  to  modify.   The  possessive  pronouns  "mine,  hers,  theirs,  ours",  and  "yours"  are  considered  to  be   strong  possessive  pronouns  and  stand  on  their  own;  they  do  not  modify  anything.   Thus,  they  are  NOT  possessive  determiners.   There  are  seven  weak  possessive  pronouns  that  function  as  possessive  determiners   in  the  English  language:     •

"my"  -­‐  first  person  singular  



"your"  -­‐  second  person  singular  and  plural  



"his"  -­‐  third  person  singular  (masculine)*  



"her"  -­‐  third  person  singular  (feminine)  



"its"  -­‐  third  person  singular  (neutral)  



"our"  -­‐  first  person  plural  



"their"  -­‐  third  person  plural    

*The  pronoun  "his"  may  function  as  either  a  strong  or  weak  possessive  pronoun.   There  is  also  another  pronoun  that  functions  as  a  possessive  determiner;  it  is  the   pronoun  "whose",  and  it  is  interrogative  in  nature.     What  Weak  and  Strong  Possessive  Pronouns  Imply   The  weak  possessive  pronouns  always  imply  the  definite  article  "the"  because  they   Take  your  S   AT  prep  to  the  next  level.  Visit  www.learnerator.com   284  

285     refer  to  specific  things  and  do  not  imply  a  sense  of  generality.  The  weak  possessive   pronouns  are  found  in  both  the  subject  and  the  predicate  of  the  sentence.   In  contrast,  the  strong  possessive  pronouns  can  be  used  to  imply  either  the   indefinite  article  "a"  or  the  definite  article  "the".  They  are  primarily  found  in  the   predicate  of  the  sentence  and  are  usually  used  as  subject  compliments  but  may  also   be  used  as  subjects;  however,  this  second  type  of  usage  is  not  very  common.   These  two  types  of  possessive  pronouns  may  be  modified  by  the  adverbs  "more,  less,   as,  much,  as  much",  and  "mostly".   Here  are  some  examples  of  strong  and  weak  pronouns:     1. Randy  was  her  friend.  (weak  possessive  pronoun  in  the  predicate)   2. Her  friend  was  Randy.  (weak  possessive  pronoun  in  the  subject)   3. Randy  was  a  friend  of  hers.  (strong  possessive  pronoun  in  the  predicate)   4. Hers  was  a  gift  that  belonged  to  Randy.  (strong  possessive  pronoun  in  the   subject)     In  the  first  two  examples,  the  pronoun  "her"  implies  the  definite  article  "the"  and   makes  Randy  her  specific  friend.  In  the  third  example,  the  pronoun  "hers"  implies   the  indefinite  article  "a"  and  makes  Randy  one  of  many  friends.   While  the  usage  of  a  strong  possessive  pronoun  as  the  subject  in  the  last  sentence  is   grammatically  correct,  it  is  a  type  of  sentence  construction  that  is  not  used  very   often  in  English.   Here  are  some  examples  of  weak  possessive  pronouns  used  as  possessive   determiners:     1. My  car  is  a  very  good  car.   2. Their  car  is  more  her  car  than  it  is  his.   3. Whose  car  is  it?     In  the  first  example,  the  word  "my"  is  used  as  a  possessive  determiner  to  modify  the   word  "car".  In  the  second  example,  the  word  "their"  is  used  as  a  possessive   Take  your  S   AT  prep  to  the  next  level.  Visit  www.learnerator.com   285  

286     determiner  to  modify  the  word  car,  and  the  word  "her"  is  used  to  modify  the  second   usage  of  the  word  "car";  the  adverb  "more"  is  then  used  to  modify  the  pronoun  "her".   The  pronoun  "his"  is  used  as  a  strong  possessive  pronoun  and  stands  alone;   therefore,  it  is  NOT  a  possessive  determiner.  In  the  final  example,  the  word  "whose"   is  used  as  an  interrogative  possessive  determiner  that  modifies  the  word  "car".     Common  Grammatical  Errors   Many  common  errors  in  the  English  language  occur  because  of  the  misuse  of  the   words  "their"  and  "there".  The  word  "their"  is  a  possessive  determiner,  and  the  word   "there"  is  an  adverb;  these  two  words  are  constantly  confused  with  each  other.   These  two  words  are  also  confused  with  the  contraction  "they're",  which  is  a   reduction  of  the  phrase  "they  are".   Here  are  several  examples  of  common  errors  with  these  words:     Incorrect  Usage   •

It's  not  there  fault.  (adverb  used  as  a  determiner)  



She  cannot  tell  when  their  lying.  (determiner  used  as  a  contraction)  



The  two  boys  did  not  want  to  do  there  chores.  (adverb  used  as  a  determiner)  



Jack  wants  to  go  and  see  what  there  doing.  (adverb  used  as  a  contraction)      

Correct  Usage   •

It's  not  their  fault.  (correct  use  of  determiner)  



She  cannot  tell  when  they're  lying.  (correct  use  of  contraction)  



The  two  boys  did  not  want  to  do  their  chores.  (correct  use  of  determiner)  



Jack  wants  to  go  and  see  what  they're  doing.  (correct  use  of  contraction)  

  Nouns  as  Possessive  Determiners   Nouns  can  be  used  as  possessive  determiners  when  they  are  spelled  with  an   apostrophe  and  the  letter  "s".  However;  if  a  noun  already  ends  with  the  letter  "s"   because  it  is  plural,  then  only  an  apostrophe  is  used  to  denote  possession.   Here  are  some  examples:   Take  your  S   AT  prep  to  the  next  level.  Visit  www.learnerator.com   286  

287       •

That  is  John's  house.  



The  swans  like  to  bathe  in  the  pond's  fragrant  waters.  



The  babies'  beds  are  all  in  a  row.  



The  lions'  food  source  is  almost  gone,  and  soon  they  will  grow  hungry.    

These  types  of  possessive  determiners  are  very  similar  to  the  weak  possessive   pronouns  that  are  used  as  possessive  determiners.  They  always  imply  the  definite   article  "the"  because  they  always  refer  to  specific  things.  In  addition,  they  may  also   be  modified  with  adverbs  in  the  same  way  that  the  weak  possessive  pronouns  are.     Common  Errors   Many  mistakes  happen  when  the  apostrophe  is  not  placed  correctly.  Quite  often,  the   word  is  misspelled  and/or  the  meaning  of  the  word  is  unintentionally  changed.     Incorrect  Usage   •

The  mariachi  band  loved  Johns'  generous  tip.  (incorrect  plural)  



They  destroyed  the  walruses's  habitat.  (incorrect  plural)  



The  old  mans'  dog  ate  the  kitten's  food  before  they  could  finish  it.  (incorrect   singular  and  plural)  

   

Take  your  S   AT  prep  to  the  next  level.  Visit  www.learnerator.com   287  

288     Correct  Usage   •

The  mariachi  band  loved  John's  generous  tip.  (correct  singular)  



They  destroyed  the  walruses'  habitat.  (correct  plural)  



The  old  man's  dog  ate  the  kittens'  food  before  they  could  finish  it.  (correct   plural  and  singular)  

  Sample  Test  Questions   Select  the  answer  that  correctly  identifies  the  possessive  determiner  (or   determiners)  in  the  sentence.     1)  That  was  her  idea  of  a  great  artistic  presentation.   A.  that   B.  was   C.  her   D.  of   E.  great     2)  Nobody  thought  it  was  John's  idea.   A.  nobody   B.  thought   C.  was   D.  John's   E.  There  is  no  possessive  determiner  in  the  sentence.    

Take  your  S   AT  prep  to  the  next  level.  Visit  www.learnerator.com   288  

289     3)  His  idea  was  that  she  should  try  Jane's  recipe  and  not  hers.   A.  his,  she   B.  was,  she   C.  she,  hers   D.  that,  hers   E.  his,  Jane's     4)  Its  main  flaw  was  that  her  idea  was  not  as  good  as  his.   A.  its,  his   B.  was,  his   C.  her,  his   D.  its,  her   E.  that,  not     5)  Their  solution  was  a  combination  of  Jane's  recipe  and  John's  idea.   A.  their,  was,  Jane's   B.  was,  Jane's,  John's   C.  their,  Jane's,  John's   D.  their,  was,  and   E.  was,  and,  idea     Solutions   1)  C  -­‐  The  word  "her"  is  a  weak  possessive  pronoun  that  functions  as  a  possessive   determiner  and  modifies  the  word  "idea".   2)  D  -­‐  The  word  "John's"  is  a  possessive  determiner  formed  from  the  proper  noun   "John";  this  possessive  determiner  modifies  the  word  "idea".   3)  E  -­‐  The  word  "his"  is  a  weak  possessive  pronoun  that  functions  as  a  possessive   determiner  and  modifies  the  word  "idea".  The  word  "Jane's"  is  a  possessive   determiner  formed  from  the  proper  noun  "Jane";  this  possessive  determiner   modifies  the  word  "recipe".  The  word  "hers"  at  the  end  of  the  sentence  is  a  strong  

Take  your  S   AT  prep  to  the  next  level.  Visit  www.learnerator.com   289  

290     possessive  pronoun  in  the  predicate  and  stands  alone;  thus,  it  is  NOT  a  possessive   determiner.   4)  D  -­‐  The  word  "its"  is  a  weak  possessive  pronoun  that  functions  as  a  possessive   determiner  and  modifies  the  word  "flaw".  The  word  "her"  is  a  weak  possessive   pronoun  that  functions  as  a  possessive  determiner  and  modifies  the  word  "idea".   The  word  "his"  at  the  end  of  the  sentence  is  a  strong  possessive  pronoun  in  the   predicate  and  stands  alone;  thus,  it  is  NOT  a  possessive  determiner.   5)  C  -­‐  The  word  "their"  is  a  weak  possessive  pronoun  that  functions  as  a  possessive   determiner  that  modifies  the  word  "idea".  The  word  "Jane's"  is  a  possessive   determiner  formed  from  the  proper  noun  "Jane";  this  possessive  determiner   modifies  the  word  "recipe".  The  word  "John's"  is  a  possessive  determiner  formed   from  the  proper  noun  "John";  this  possessive  determiner  modifies  the  word  "idea".  

Take  your  S   AT  prep  to  the  next  level.  Visit  www.learnerator.com   290  

291    

Grammatical Agreement In  English,  there  are  three  major  types  of  agreement  that  are  required  in  order  to  be   grammatically  correct:     •

Agreement  of  pronoun  and  its  antecedent  



Agreement  of  subject  and  verb  



Agreement  of  nouns  

  Agreement  of  Pronoun  and  Antecedent   The  antecedent  of  a  pronoun  is  the  person,  thing,  location,  or  idea  in  a  sentence  to   which  a  pronoun  refers.  The  antecedent  may  be  either  a  noun  or  another  pronoun.   Therefore,  a  pronoun  must  agree  both  in  number  and  gender  with  its  antecedent  in   order  for  a  sentence  to  be  grammatically  correct.     Agreement  of  number   A  singular  pronoun  is  used  with  a  singular  antecedent,  and  a  plural  pronoun  is  used   with  a  plural  antecedent.  Here  are  several  examples:     1. The  dog  thought  that  its  bone  was  in  the  backyard.   2. The  kittens  ate  all  of  their  food  in  the  kitchen.   3. The  dog  and  the  cat  were  glad  that  they  were  friends.     In  the  sentences  above,  the  number  of  each  pronoun  agrees  with  its  antecedent.  In   the  first  example,  the  possessive  pronoun  its  is  singular  because  its  antecedent  is  the   singular  noun  dog.  In  the  second  example,  the  possessive  pronoun  there  is  plural   because  its  antecedent  is  the  plural  noun  kittens.     In  the  final  example,  The  plural  antecedent  of  the  plural  pronoun  they  is  a  compound   noun  formed  from  the  nouns  dog  and  cat.  In  this  case,  the  compound  noun  takes  a   plural  pronoun.  This  brings  us  to  our  next  topic,  which  is  compound  nouns.     Take  your  S   AT  prep  to  the  next  level.  Visit  www.learnerator.com   291  

292     Compound  nouns   Compound  nouns  are  created  from  two  nouns,  two  pronouns,  or  a  noun  and  a   pronoun;  the  two  elements  of  a  compound  noun  are  connected  by  the  conjunctions   and,  or,  or  nor.  When  the  two  words  are  connected  by  the  conjunction  and,  a  plural   pronoun  is  used.     When  the  two  words  are  connected  by  the  conjunction  or  or  nor,  the  element  of  the   compound  noun  that  is  closest  to  the  pronoun  determines  whether  a  singular  or  a   plural  pronoun  is  used.     1. Either  the  dogs  or  I  will  eat  my  food.   2. Either  the  duck  or  the  pigs  will  eat  their  food.   3. The  turkey  and  the  chicken  will  drink  their  water.     The  compound  nouns  in  the  examples  are  in  agreement  with  the  pronouns  that  are   using  them  as  antecedents.  In  the  first  example,  the  compound  noun  consists  of  a   plural  noun  and  a  singular  pronoun  connected  by  the  conjunction  or;  therefore,   since  the  singular  pronoun  I  is  closer  to  the  pronoun  that  is  using  the  compound   noun  as  an  antecedent,  the  singular  form  is  used.   In  the  second  example,  a  singular  noun  and  a  plural  noun  are  connected  by  the   conjunction  or;  thus,  the  pronoun  using  the  compound  noun  as  an  antecedent  is   plural  since  the  plural  noun  is  closer  to  the  antecedent.   In  the  last  example,  two  singular  nouns  are  connected  by  the  conjunction  and;   therefore,  the  pronoun  using  the  compound  noun  as  an  antecedent  will  be  plural.     Collective  Nouns   Collective  nouns  are  singular  nouns  that  are  used  to  identify  various  kinds  of   groups.  When  a  collective  noun  is  used  as  an  antecedent  by  a  pronoun,  the  singular   form  of  the  pronoun  is  used.     1. The  team  won  its  game.   Take  your  S   AT  prep  to  the  next  level.  Visit  www.learnerator.com   292  

293     2. The  herd  began  its  stampede.   3. A  colony  of  ants  was  about  to  meet  its  doom.       In  all  three  examples,  collective  nouns  are  used  as  antecedents  by  singular   pronouns.     Common  Errors   Most  of  the  errors  with  compound  and  collective  nouns  occur  when  an  incorrect   pronoun  is  chosen  to  be  the  antecedent.     Incorrect  Usage   •

The  cats  eat  its  food.  (plural  antecedent  with  singular  pronoun)  



That  pair  of  animals  eats  their  food.  (singular  collective  noun  with  plural   pronoun)  



The  mice  are  eating  its  food.  (plural  noun  with  singular  pronoun)      

Correct  Usage   •

The  cats  eat  their  food.  (plural  antecedent  with  plural  pronoun)  



That  pair  of  animals  eats  its  food.  (singular  collective  noun  with  singular   pronoun)  



The  mice  are  eating  their  food.  (plural  noun  with  plural  pronoun)  

  Agreement  of  Gender   A  feminine  pronoun  is  used  with  a  feminine  antecedent,  and  a  masculine  pronoun  is   used  with  a  masculine  antecedent.  The  same  rules  for  compound  nouns  apply,   depending  on  whether  or  not  the  two  words  are  connected  by  the  conjunction  and   or  by  the  conjunctions  nor  and  or.     1. William  likes  to  think  that  he  is  always  right.   2. Mary  knows  in  her  heart  that  William  is  always  wrong.   3. William  and  Mary  do  not  think  that  they  have  a  good  relationship.   Take  your  S   AT  prep  to  the  next  level.  Visit  www.learnerator.com   293  

294     4. Does  either  Mary  or  William  think  that  he  has  a  good  relationship.     In  the  above  sentences,  the  gender  of  each  pronoun  agrees  with  its  antecedent.  In   the  first  example,  the  pronoun  he  is  masculine  because  its  antecedent  is  William,  and   he  is  a  male.   In  the  second  example,  the  possessive  pronoun  her  is  feminine  because  its   antecedent  is  Mary,  and  she  is  a  female.  In  the  third  example,  the  plural  pronoun   they  is  gender  neutral.   In  the  last  example,  the  compound  noun  because  it  has  a  compound  noun  connected   by  the  conjunction  or,  so  the  pronoun  using  the  compound  noun  as  its  antecedent   uses  the  proper  name  William  (the  element  of  the  compound  noun  closer  to  the   pronoun)  to  determine  its  number  and  gender.     Indefinite  Pronouns   Indefinite  pronouns  refer  to  things  in  a  non-­‐specific  manner.  Indefinite  pronouns   may  be  singular  or  plural  (some  indefinite  pronouns  may  be  used  either  way)  when   used  as  antecedents  within  a  sentence.     The  following  indefinite  pronouns  use  singular  antecedents:     •

no  one,  nobody,  everyone,  everybody,  someone,  somebody,  anyone,  anybody,   nothing,  everything,  something,  anything,  each,  another,  other,  whatever,   whichever,  whoever,  whomever,  enough,  little,  less,  much,  more,  most,  plenty,   one,  either,  neither  

  These  pronouns  use  plural  antecedents:   •

both,  few,  many,  others,  several  

  These  pronouns  may  use  either  singular  or  plural  antecedents:   •

all,  any,  more,  most,  none,  some  

  Take  your  S   AT  prep  to  the  next  level.  Visit  www.learnerator.com   294  

295     Here  are  a  few  examples  of  indefinite  pronouns  being  used  as  antecedents:     1. All  of  the  monkeys  were  let  out  of  their  cage.   2. Everything  finds  its  place.   3. Both  of  them  know  their  purpose.     In  the  first  example,  the  indefinite  pronoun  all  is  used  in  its  plural  form  as  the   antecedent  of  the  indefinite  pronoun.  In  the  second  example,  the  indefinite  pronoun   everything  is  used  as  the  singular  antecedent  of  the  pronoun  its.  In  the  final   example,  the  plural  indefinite  pronoun  both  is  used  as  the  antecedent  of  the   pronoun  their.     Controversial  usage  of  plural  third  person  to  insure  gender  neutrality   In  the  late  20th  century,  various  writers  began  to  change  the  practice  of  using  the   masculine  singular  third  person  pronoun  as  the  antecedent  for  singular  indefinite   pronouns  and  substituted  the  neutral  plural  third  person  instead.   By  doing  this,  there  would  be  no  gender  discrimination.  In  other  words,  the  pronoun   his  was  paired  with  the  singular  indefinite  pronouns,  and  certain  individuals  began   to  substitute  it  with  the  pronoun  their.  Others  would  use  the  words  his  or  her.       •

Everybody  knew  his  place.  (older  style  -­‐  instantly  defaults  to  masculine   pronoun)  



Everybody  knew  their  place.  (modern  style  -­‐  uses  plural  pronoun  instead)  



Everybody  knew  his  or  her  place.  (modern  style  -­‐  uses  masculine  and   feminine)  

  All  three  of  these  forms  are  considered  correct  by  different  entities.  However,  the   first  example  is  being  shunned  by  more  and  more  people  as  time  progresses.    

Take  your  S   AT  prep  to  the  next  level.  Visit  www.learnerator.com   295  

296     Subject  and  Verb  Agreement   In  English,  the  subject  and  verb  must  agree  in  number.  Thus,  a  singular  subject   requires  a  singular  verb  and  a  plural  subject  requires  a  plural  verb.     1. Jack  throws  the  knife  at  the  wall.   2. Jack's  friends  throw  him  out  of  the  pub.   3. Everyone  is  throwing  a  fit.     In  the  first  example,  a  singular  subject  (Jack)  uses  a  singular  verb.  In  the  second   example,  a  plural  subject  (friends)  is  using  a  plural  verb.  In  the  final  example,  a   singular  subject  (the  singular  indefinite  pronoun  everyone)  is  using  a  singular  verb.   In  addition,  the  rules  for  compound  subjects  used  as  antecedents  for  pronouns  are   identical  to  those  used  for  subject  and  verb  agreement.  Therefore  when  the  two   elements  of  a  compound  subject  are  connected  by  the  conjunction  and,  they  require   a  plural  verb.     Likewise,  when  the  two  elements  of  a  compound  subject  are  connected  by  the   conjunction  or  (or  the  conjunction  nor),  the  element  that  is  closer  to  the  verb   determines  the  number  to  be  used.     1. The  pony  and  the  dog  no  longer  run  through  the  fields   2. The  pony  nor  the  dogs  run  through  the  fields.   3. The  ponies  nor  the  dog  runs  through  the  fields.     In  the  first  example,  the  compound  subject  is  connected  with  the  conjunction   and  uses  a  plural  verb.  In  the  second  example,  the  elements  of  the  compound   sentence  are  connected  with  the  conjunction  nor;  therefore,  the  word  closer   to  the  verb  is  used  to  determine  number.  The  same  is  true  for  the  third   example.      

Take  your  S   AT  prep  to  the  next  level.  Visit  www.learnerator.com   296  

297     Common  Errors   The  most  popular  errors  that  occur  are  when  there  is  a  noun  or  pronoun  of  a   different  number  that  is  placed  between  the  subject  and  the  verb.  This  sort  of   occurrence  is  most  common  with  prepositional  phrases  beginning  with  the   preposition  of.  The  verb  is  erroneously  matched  with  the  other  word  that  is  closer.     Incorrect  Usage   •

One  of  the  fryers  are  broken.  (singular  subject  with  plural  verb)  



John's  friends  standing  by  the  policeman  is  yelling.  (plural  subject  with   singular  verb)  



The  candy  that  she  put  in  the  jars  are  gone.  (singular  subject  with  a  plural   verb)      

Correct  Usage   •

One  of  the  fryers  is  broken.  (subject  and  verb  are  singular)  



John's  friends  standing  by  the  policeman  are  yelling.  (subject  and  verb  are   plural)  



The  candy  that  she  put  in  the  jars  is  gone.  (subject  and  verb  are  plural)  

  Agreement  of  Nouns   When  a  sentence  uses  a  form  of  the  verb  be  as  a  linking  verb,  the  subject  and  the   subject  compliment  must  agree  in  number  and  gender.     1. James  is  a  big  man.   2. John  was  a  very  small  individual.   3. These  men  were  regular  sized  people.   4. This  lady  is  a  highly  intelligent  professor.   In  the  first  example,  a  singular  masculine  subject  is  paired  with  a  singular  masculine   subject  compliment.  In  the  second  example,  a  singular  masculine  subject  is  paired   with  a  singular  neutral  subject  compliment.  In  the  third  example,  a  plural  masculine  

Take  your  S   AT  prep  to  the  next  level.  Visit  www.learnerator.com   297  

298     subject  is  paired  with  a  plural  neutral  subject  compliment.  In  the  last  example,  a   singular  feminine  subject  is  paired  with  a  singular  neutral  subject  compliment.     Common  Errors   Most  errors  occur  when  the  subject  compliment  is  a  noun  that  is  used  or  spelled   incorrectly.  This  sort  of  error  can  easily  be  spotted  and  corrected  without  much   difficulty.     Incorrect  Usage   1. Greg  is  a  lady.  (feminine  subject  compliment  with  masculine  subject)   2. Frank  is  a  cowards.  (plural  subject  compliment  with  singular  subject)   3. Samoans  are  big  peoples.  (incorrect  spelling  of  plural  noun)       Correct  Usage   1. Greg  is  a  man.  (gender  agreement  between  nouns)   2. Frank  is  a  coward.  (number  agreement  between  nouns)   3. Samoans  are  big  people.  (correct  spelling  of  noun)     Sample  Test  Questions   In  the  following  questions,  select  the  answer  that  contains  an  INCORRECT  usage  of   agreement:     1)   A.    They  love  their  team.   B.    He  loves  his  team.   C.    We  loves  his  team.   D.    I  love  our  team.     2)     A.    Greg  drinks  from  his  cup.   B.    Greg  and  I  drink  from  our  cups.   Take  your  S   AT  prep  to  the  next  level.  Visit  www.learnerator.com   298  

299     C.    I  drink  from  my  cup.   D.    You  drinks  from  your  cup.     3)     A.    The  men  and  the  boy  are  playing  golf.   B.    The  boys  or  the  men  are  playing  golf.   C.    The  men  or  the  boy  is  playing  golf.   D.    The  boy  or  the  men  is  playing  golf.     4)     A.    One  of  the  ducks  is  walking.   B.    One  of  the  ducks  are  walking.   C.    The  ducks  are  walking.   D.    The  duck  is  walking.     Answers   1. C)  This  sentence  is  incorrect  because  it  has  a  plural  pronoun  as  its  subject   and  uses  a  singular  verb.   2. D)  This  sentence  is  incorrect  because  it  has  a  second  person  pronoun  and   uses  the  incorrect  verb  tense.   3. D)  This  sentence  is  incorrect  because  it  has  a  compound  subject  with  a   singular  noun  and  a  plural  noun  connected  by  the  conjunction  or.  Since  the   plural  noun  men  is  closer  to  the  verb,  a  plural  verb  must  be  used  instead.   4. B)  This  sentence  is  incorrect  because  it  has  the  singular  indefinite  pronoun   one  for  its  subject.  This  singular  subject  requires  a  singular  verb.  The   confusion  in  this  example  is  the  result  of  the  prepositional  phrase  that   contains  the  plural  noun  ducks.  Since  this  plural  noun  directly  precedes  the   verb  are,  it  tricks  the  writer  into  thinking  that  a  plural  verb  should  be  used.  

Take  your  S   AT  prep  to  the  next  level.  Visit  www.learnerator.com   299  

300    

Frequently Confused Words 1)  Affect  -­‐  Effect   Affect  -­‐  to  influence  (verb);  an  emotional  response  (noun)   •

She  did  not  want  to  affect  his  decision.  (most  common  usage)  



He  displayed  a  happy  affect.  (used  as  a  noun  primarily  in  psychology-­‐-­‐very   rare)    

Effect  -­‐  a  result  (noun);  to  cause  (verb)   •

It  did  not  have  much  of  an  effect.  (most  common  usage)  



He  wanted  to  effect  change  within  the  system.  (used  as  a  verb-­‐-­‐not  as   common)    

  2)  Advice  -­‐  Advise   Advice  -­‐  a  recommendation  (noun)   •

He  gave  her  some  good  advice.  



She  did  not  like  his  advice.    

Advise  -­‐  to  recommend  (verb)   •

They  did  not  advise  him  about  the  situation.  



John  would  advise  him  about  the  problem.  

  3)  Complement  -­‐  Compliment   Complement  -­‐  something  that  completes  (noun);  to  complete  something  else  (verb)   •

The  purse  was  a  perfect  complement  to  her  outfit.  (noun)  



The  table  and  the  chair  complement  each  other  well.  (verb)    

Compliment  -­‐  praise,  flattery  (noun);  to  say  nice  things  about  (verb)   •

He  appreciated  her  compliment.  (noun)  



She  complimented  his  performance  (verb-­‐-­‐past  tense)  

  Take  your  S   AT  prep  to  the  next  level.  Visit  www.learnerator.com   300  

301     4)  Council  -­‐  Counsel   Council  -­‐  governing  body  (noun)   •

The  council  met  every  month.  



She  was  a  member  of  the  council.    

Counsel  -­‐  advice  (noun);  a  lawyer  or  lawyers  (noun);  to  give  advice  (verb)   •

He  sought  the  counsel  of  his  teacher  (noun-­‐-­‐advice)  



Jane  is  serving  as  counsel  for  the  defendant.  (noun-­‐-­‐lawyer)  



All  counsel  must  obey  the  rules  of  the  court  system.  (noun-­‐-­‐lawyers)  



He  tried  to  counsel  her.  (verb)  

  5)  Discreet  -­‐  Discrete   Discreet  -­‐  prudent  (adjective);  modest  (adjective);  unobtrusive  (adjective)   •

He  was  very  discreet  and  only  said  what  was  required.  (adjective-­‐-­‐prudent)  



She  was  impressed  by  the  discreet  elegance  of  the  home.  (adjective-­‐-­‐modest)  



He  followed  at  a  discreet  distance.  (adjective-­‐-­‐unobtrusive)    

Discrete  -­‐  separate,  distinct  (adjective)   •

The  book  has  several  discrete  sections.  



That  thing  has  many  discrete  parts.  

  6)  Eminent  -­‐  Imminent  -­‐  Immanent   Eminent  -­‐  prominent  (adjective)   •

She  was  an  eminent  artist.  



There  are  many  eminent  doctors  in  the  city.    

Imminent  -­‐  about  to  happen  (adjective)   •

The  situation  was  imminent.  



He  was  in  imminent  danger.  

Immanent  -­‐  inherent  (adjective)   •

Beauty  is  not  something  acquired;  it  is  immanent.  

Take  your  S   AT  prep  to  the  next  level.  Visit  www.learnerator.com   301  

302     •

Kindness  is  immanent  in  all  people.  

  7)  Farther  -­‐  Further   Farther  -­‐  at  a  greater  distance  (adjective)   •

She  threw  the  ball  farther  than  he  did.  



He  ran  farther  down  the  trail.    

Further  -­‐  in  greater  depth  (adjective)   •

This  requires  further  analysis.  



They  needed  further  detail.  

  8)  Its  -­‐  It's   Its  -­‐  possessive  form  of  the  word  it  (pronoun)   •

The  whale  lost  its  way.  



The  monkey  found  its  cage.    

It's  -­‐  contraction  for  the  clause  it  is  (pronoun  and  verb)   •

It's  clear  to  see  that  he  didn't  have  a  clue.  



It's  time  for  tea.  

  9)  Principal  -­‐  Principle   Principal  -­‐  foremost  (adjective);  administrator  of  a  school  (noun)   •

The  tomato  was  the  principal  ingredient  in  the  stew.  (adjective)  



She  was  the  principal  of  the  local  elementary  school.  (noun)    

Principle  -­‐  moral  rule  (noun);  basic  truth  (noun)   •

She  was  a  woman  of  principle.  (noun-­‐-­‐moral  rule)  



It  was  a  principle  of  human  behavior.  (noun-­‐-­‐basic  truth)  

  10)  Than  -­‐  Then   Than  -­‐  in  comparison  to  (conjunction,  preposition)   Take  your  S   AT  prep  to  the  next  level.  Visit  www.learnerator.com   302  

303     •

He  is  older  than  you  are.  (conjunction)  



He  is  older  than  you.  (preposition)    

Then  -­‐  next  (adverb);  that  time  (noun)   •

He  then  traveled  all  over  the  world.  (adverb)  



Since  then,  she's  been  more  prudent.  (noun  

  11)  Their  -­‐  There  -­‐  They're   Their  -­‐  possessive  form  of  the  pronoun  they  (pronoun)   •

It  was  their  decision.  



Their  goal  was  to  succeed.    

There  -­‐  in  that  place  (adverb);  introduces  a  sentence  (pronoun);  that  place  (noun);   relied          on  for  support  (adjective)   •

Stand  over  there.  (adverb)  



There  was  a  time.  (pronoun)  



She  took  it  from  there.  (noun)  



She  was  always  there  for  him.  (adjective)    

They're  -­‐  contraction  for  the  clause  they  are.  (contraction)   •

They're  bad.  



They're  good.  

  12)  To  -­‐  Too   To  -­‐  toward;  begins  an  infinite  phrase  (preposition)   •

He  went  to  the  movies.  (preposition-­‐-­‐toward)  



Bill  went  to  see  her  house.  (preposition-­‐-­‐infinitive)    

Too  -­‐  also;  very  (adverb)   •

They  wanted  to  see  her,  too.  (adverb-­‐-­‐also)  



She  was  too  smart  for  them.  (adverb-­‐-­‐very)  

Take  your  S   AT  prep  to  the  next  level.  Visit  www.learnerator.com   303  

304       13)  Whose  -­‐  Who's   Whose  -­‐  interrogative  possessive  determiner  denoting  ownership  (pronoun)   •

Whose  wallet  is  that?  



Randy  did  not  know  whose  purse  that  was.    

Who's  -­‐  contraction  for  the  clause  who  is  (contraction)   •

Who's  at  the  door?  



She  wants  to  see  who's  who.  

  14)  Your  -­‐  You're   Your  -­‐  second  person  possessive  (pronoun)   •

Your  car  is  green  



I  do  not  like  your  green  car.    

You're  -­‐  contraction  for  the  clause  you  are  (contraction)   •

You're  dumb.  



You're  falling.  

  15)  Capital  -­‐  Capitol   Capital  -­‐  assets  (noun);  related  to  assets  (adjective);  upper  case  (adjective);  major   city  (noun)   •

The  business  needed  more  capital.  (noun-­‐-­‐assets)  



He  paid  taxes  on  his  capital  gains.  (adjective-­‐-­‐assets)  



Start  the  sentence  with  a  capital  letter.  (adjective-­‐-­‐upper  case)  



The  capital  of  Japan  is  Tokyo.  (noun-­‐-­‐major  city)    

Capitol  -­‐  government  building  (noun  or  adjective)   •

She  went  to  the  capitol  building.  (adjective)  



He  works  at  the  Texas  State  Capitol.  (noun)  

  Take  your  S   AT  prep  to  the  next  level.  Visit  www.learnerator.com   304  

305     16)  Die  -­‐  Dye   Die  -­‐  to  lose  life  (verb);  a  cube  that  functions  as  a  random  number  generator  (noun)   •

Tyrone  did  not  want  to  die.  (verb)  



Rudy  rolled  the  die.  (noun)    

Dye  -­‐  to  change  or  add  color  by  applying  liquid  dye  (verb)   •

Tyrone  did  not  want  to  dye  his  shirt  blue.  



Rudy  was  out  of  dye.  

  17)  Gorilla  -­‐  Guerilla   Gorilla  -­‐  animal  in  ape  family  (noun)   •

The  gorilla  was  small.  



The  gorilla  was  hungry.    

Guerrilla  -­‐  a  type  of  soldier  (noun);  indicative  of  a  guerilla  (adjective)   •

The  guerilla  was  small.  (noun)  



The  guerilla  soldier  was  hungry.  (adjective)  

  18)  Knew  -­‐  New   Knew  -­‐  to  have  been  familiar  with  (verb)   •

The  girl  knew  the  dog.  



The  dog  knew  the  girl.    

New  -­‐  fresh,  not  yet  old  (adjective)   •

Randy  wore  a  new  shirt.  



His  new  girlfriend  bought  it  for  him.  

  19)  Maybe  -­‐  May  be   Maybe  -­‐  perhaps  (adverb)   •

Maybe,  she  would  be  interested.  



John  might  be  interested,  maybe.  

Take  your  S   AT  prep  to  the  next  level.  Visit  www.learnerator.com   305  

306       May  be  -­‐  might  be  (verb)   •

She  may  be  interested.  



John  may  be  interested.  

  20)  Personal  -­‐  Personnel   Personal  -­‐  intimate  (adjective);  owned  by  a  person  (adjective)   •

It  was  a  very  personal  question.  (adjective-­‐-­‐intimate)  



Those  were  his  personal  belongings.  (adjective-­‐-­‐owned  by  a  person)    

Personnel  -­‐  employees  (collective  noun)   •

He  fired  all  of  his  personnel.  



The  store's  personnel  were  all  given  Christmas  bonuses.  

  21)  Plain  -­‐  Plane   Plain  -­‐  simple  (adjective)   •

It  was  plain  to  see  



That  was  plain  enough.    

Plane  -­‐  to  shave  wood  (verb);  aircraft  (noun)   •

It  was  time  for  the  carpenter  to  plane  the  board.  



It  was  a  plane  up  in  the  sky.  

  22)  Presence  -­‐  Presents   Presence  -­‐  attendance  (noun);  being  at  hand  (noun)   •

His  presence  was  requested.  (noun-­‐-­‐attendance)  



Her  presence  was  felt  by  all.  (noun-­‐-­‐being  at  hand)    

Presents  -­‐  gifts  (noun)   •

George  received  lots  of  presents.  



Bill  gave  George  many  presents.  

Take  your  S   AT  prep  to  the  next  level.  Visit  www.learnerator.com   306  

307       23)  Stationary  -­‐  Stationery   Stationary  -­‐  standing  still  (adjective)   •

Tom  was  in  a  stationary  position.  



Tina  was  stationary.    

Stationery  -­‐  writing  paper  (noun)   •

The  stationery  was  pink  with  red  trim.  



She  ran  out  of  stationery,  so  she  didn't  write  all  of  her  letters.  

  24)  Through  -­‐  Threw   Through  -­‐  finished  (adjective);  into  and  out  of  (adverb)   •

He  was  through.  (adjective)  



He  went  through  the  field.  (adverb)    

Threw  -­‐  toss  (verb)   •

He  threw  the  red  ball.  



She  threw  it  back  at  him  and  knocked  him  over.  

  25)  Waist  -­‐  Waste   Waist  -­‐  midsection  of  the  body  (noun)   •

He  had  a  slim  waist.  



Her  waist  was  much  slimmer.    

Waste  -­‐  discarded  material  (noun);  to  squander  (verb)   •

The  garbage  truck  was  full  of  waste.  (noun)  



They  did  not  want  to  waste  their  money.  (verb)  

  Sample  Questions   In  the  questions  below,  identify  the  sentence  that  is  correct:   1)   Take  your  S   AT  prep  to  the  next  level.  Visit  www.learnerator.com   307  

308     A)  He  threw  the  stationery  into  the  trash.   B)  He  threw  the  stationary  into  the  trash.   C)  He  through  the  stationary  into  the  trash.   D)  He  through  the  stationery  into  the  trash.   2)   A)  The  gorilla  was  flying  in  the  knew  plane  that  was  taking  it  to  the  zoo.   B)  The  guerilla  was  flying  in  the  new  plain  that  was  taking  it  to  the  zoo.   C)  The  guerilla  was  flying  in  the  knew  plane  that  was  taking  it  to  the  zoo.   D)  The  gorilla  was  flying  in  the  new  plane  that  was  taking  it  to  the  zoo.   3)   A)  There  problem  is  that  their  not  multiplying  they're  numbers  correctly.   B)  They're  problem  is  that  their  not  multiplying  there  numbers  correctly.   C)  There  problem  is  that  they're  not  multiplying  their  numbers  correctly.   D)  Their  problem  is  that  they're  not  multiplying  their  numbers  correctly.   E)  Their  problem  is  that  they're  not  multiplying  there  numbers  correctly.   4)   A)  Its  not  plain  too  see  the  affect  of  your  presents.   B)  Its  not  plane  to  see  the  effect  of  you're  presence.   C)  Its  not  plain  to  see  the  affect  of  your  presence.   D)  It's  not  plane  too  see  the  effect  of  your  presents.   E)  It's  not  plain  to  see  the  effect  of  your  presence.   F)  It's  not  plain  to  see  the  affect  of  your  presence.     Solutions   1. A)  The  word  threw  must  be  a  verb,  and  the  word  stationary  must  be  a  noun.   2. D)  The  animal  flying  to  the  zoo  is  a  gorilla  and  not  a  guerilla  soldier.  Also,  the   word  new  must  be  an  adjective,  and  the  word  plane  must  be  a  noun.   3. D)  The  word  at  the  beginning  of  the  sentence  must  be  a  pronoun,  and  the   word  before  the  word  not  must  be  a  contraction  of  the  clause  they  are.  In   addition,  the  word  before  the  word  numbers  must  also  be  a  pronoun.  

Take  your  S   AT  prep  to  the  next  level.  Visit  www.learnerator.com   308  

309     4. E)  The  word  at  the  beginning  of  the  sentence  must  be  a  contraction  of  the   clause  it  is,  and  the  third  word  of  the  sentence  must  be  an  adjective.  Also,  the   word  before  the  word  of  must  be  a  noun  that  means  "a  result."  In  addition,   the  last  word  of  the  sentence  must  be  a  noun  that  means  "being  at  hand."  

 

 

Take  your  S   AT  prep  to  the  next  level.  Visit  www.learnerator.com   309  

310    

Logical Comparisons Logical  comparisons  that  occur  between  two  or  more  items  must  have  these  three   characteristics  in  order  to  be  correct:     •

completeness  



consistency  



clarity  

  Completeness   When  making  any  type  of  logical  comparison,  there  must  be  two  things  that  are   compared.  The  most  popular  error  with  this  aspect  of  logical  comparison  occurs   when  a  claim  of  comparison  is  made  and  only  one  element  of  the  comparison  is   mentioned  in  the  sentence.     Incorrect  Usage   1. The  diet  product  contains  33%  less  fat.   2. The  rock  is  ten  pounds  lighter.     Correct  Usage   1. The  diet  product  contains  33%  less  fat  than  the  regular  product.   2. The  rock  is  ten  pounds  lighter  than  the  brick.     In  the  above  examples,  only  one  item  is  mentioned  in  a  statement  that  is  attempting   to  make  a  logical  comparison.  Therefore,  another  item  must  be  included  so  that  a   true  logical  comparison  is  correctly  made.     Incorrect  Usage   1. Mary  walked  so  slow.   2. John  swam  so  fast.    

Take  your  S   AT  prep  to  the  next  level.  Visit  www.learnerator.com   310  

311       Correct  Usage   1. Mary  walked  so  slow  that  she  was  late  for  work.   2. John  swam  so  fast  that  he  won  the  race.     In  the  above  examples,  the  word  so  needs  to  be  qualified  in  order  to  have  logical   comparison.  Both  sentences  can  be  corrected  by  comparing  speeds  of  movement   with  specific  events.     Consistency       Similarity   The  items  being  compared  need  to  have  some  type  of  similarity  that  allows  for  a   logical  comparison  to  be  made  so  that  consistency  is  maintained.     Incorrect  Usage   1. The  chips  at  the  corner  store  are  cheaper  than  the  supermarket.     Correct  Usages   1. The  chips  at  the  corner  store  are  cheaper  than  the  supermarket's  chips.   2. The  chips  at  the  corner  store  are  cheaper  than  the  chips  at  the  supermarket.   3. The  chips  at  the  corner  store  are  cheaper  than  those  at  the  supermarket.     In  the  incorrect  example  above,  chips  are  being  compared  to  a  supermarket.   Therefore,  the  sentence  may  be  corrected  if  the  chips  are  compared  to  other  chips.   The  three  correct  usages  present  different  ways  that  this  correction  can  be  made.     Specificity   The  sentence  must  be  specific  with  the  exact  elements  that  are  being  compared.   Quite  often,  the  first  element  mentioned  is  also  included  in  the  second  element.  

Take  your  S   AT  prep  to  the  next  level.  Visit  www.learnerator.com   311  

312     Therefore,  the  writer  needs  to  be  specific  about  the  various  elements  that  are  being   compared  in  order  for  the  logical  comparison  to  be  consistent.     Incorrect  Usage   1. The  boy  runs  faster  than  anyone.     Correct  Usage   1. The  boy  runs  faster  than  anyone  else.   2. The  boy  runs  faster  than  any  one  of  the  others.   3. The  boy  runs  faster  than  any  one  of  the  other  boys.     In  the  above  example,  the  boy  is  being  compared  to  himself  because  he  is  part  of  the   universal  pronoun  anyone.  This  illogical  comparison  occurs  because  the  terms  in  the   sentence  are  not  providing  an  adequate  amount  of  specificity.  Thus,  all  correct   usages  need  to  be  more  specific  in  order  to  remedy  this  situation.  This  type  of  repair   is  usually  made  by  including  the  words  else  or  other(s).     Clarity   There  are  also  many  errors  with  logical  comparison  that  occur  because  of  clarity   issues.  These  types  of  problems  are  the  result  of  sentences  that  lack  precision.   Oftentimes,  an  ambiguous  meaning  is  not  meant  but  is  expressed  because  the   correct  words  are  not  employed.     Incorrect  Usage   1. She  gave  her  best  friend  more  candy  than  her  mother.     Correct  Usages   1. She  gave  her  best  friend  more  candy  than  her  own  mother  did.   2. She  gave  her  best  friend  more  candy  than  she  gave  her  own  mother.   3. She  gave  her  best  friend  more  candy  than  her  friend's  mother  did.     Take  your  S   AT  prep  to  the  next  level.  Visit  www.learnerator.com   312  

313     The  incorrect  usage  in  the  example  above  demonstrates  how  the  lack  of   precision  causes  ambiguity  and  affects  the  clarity  of  the  sentence.  Thus,  the   logical  comparison  is  uncertain  because  is  unclear  which  things  are  being   compared.  The  correct  usages  use  specific  terminology  in  order  to  precisely   clarify  the  meaning  of  the  sentence.     Sample  Test  Questions   In  the  questions  below,  please  select  the  answer  that  contains  an  INCORRECT  usage   of  logical  comparison.     1)   A)  The  diet  candy  contains  half  the  amount  of  calories  than  the  regular  candy   contains.   B)  The  apple  pie  is  better  than  all  of  the  others.   C)  He  ran  faster  than  anyone.   D)  She  baked  her  cookies  more  quickly  than  everyone  else.   2)   A)  She  cried  so  hard  that  her  tears  dripped  onto  her  shirt.   B)  They  ran  so  fast  that  they  fell.   C)  The  very  large  boy  in  the  blue  shirt  ran  so  fast.   D)  The  tiny  dog  walked  so  fast  that  it  quickly  lost  sight  of  its  owner.   3)   A)  The  grapes  at  the  farmers'  market  were  more  expensive  than  the  store.   B)  The  grapes  at  the  farmers'  market  were  more  expensive  than  the  store's   grapes.   C)  The  grapes  at  the  farmers'  market  were  more  expensive  than  the  grapes  at   the  store.   D)  The  grapes  at  the  farmers'  market  were  expensive.   4)   A)  Linda  typed  faster  than  her  mother.   B)  Linda's  mother  typed  faster  than  anyone.   Take  your  S   AT  prep  to  the  next  level.  Visit  www.learnerator.com   313  

314     C)  Linda's  mother  typed  faster  than  anyone  else  her  age.   D)  They  both  typed  faster  than  all  of  the  others  in  the  room.   E)  All  of  the  sentences  are  correct.     Solutions   1. C)  The  subject  of  the  sentence  ran  faster  than  anyone.  However,  the  subject  is   included  in  the  pronoun  anyone.  Since  the  subject  cannot  run  faster  than   himself,  it  is  logically  incorrect.   2. C)  The  subject  of  this  sentence  ran  so  fast  that  something  happened.   However,  the  sentence  does  not  explain  what  this  something  is;  therefore,  it   is  logically  incorrect.   3. A)  The  grapes  are  being  compared  to  the  store.  This  unintended  comparison   is  illogical;  thus,  the  sentence  contains  an  incorrect  usage  of  comparison.   4. B)  Linda's  mother  is  included  in  the  word  anyone;  therefore,  an  incorrect   usage  of  logical  comparison  has  occurred.  

Take  your  S   AT  prep  to  the  next  level.  Visit  www.learnerator.com   314  

315    

Conventional Expressions There  are  many  conventional  expressions  that  are  inconsistent  with  standard   written  English.  While  we  are  not  going  to  cover  them  all  here,  several  of  these   incorrect  expressions  are  given  are  given  below  along  with  their  proper  corrections.     1)  For  all  intensive  purposes  –  For  all  intents  and  purposes   This  phrase  is  commonly  misused  when  a  writer  wishes  to  express  an  idea  meaning   that  in  every  practical  sense,  something  has  occurred.  The  misuse  of  this  phrase   means  that  the  purposes  with  which  something  has  occurred  were  extremely   intense.     Incorrect  Usage   For  all  intensive  purposes,  the  marriage  was  over.     Correct  Usage   For  all  intents  and  purposes,  the  marriage  was  over.     In  the  incorrect  example  above,  the  writer  is  expressing  that  the  purposes   with  which  the  marriage  had  ended  were  very  intense.  In  the  correct   example,  the  writer  is  expressing  the  intended  notion  that  for  all  practical   purposes,  the  marriage  was  over.     2)  I  could  care  less  –  I  couldn’t  care  less   This  phrase  is  misused  when  a  writer  tries  to  express  the  idea  of  apathy,  but  instead   expresses  the  idea  of  caring  for  something  and  then  being  capable  of  reducing  the   amount  of  this  concern.     Incorrect  Usage   I  could  care  less  if  she  loves  him.     Take  your  S   AT  prep  to  the  next  level.  Visit  www.learnerator.com   315  

316     Correct  Usage   I  couldn’t  care  less  if  she  loves  him.     In  the  incorrect  example  above,  the  writer  actually  does  care  that  she  loves  him;   however,  the  writer  is  able  to  reduce  the  amount  of  this  care  to  a  lesser  degree.  In   the  correct  example,  the  writer  has  absolutely  no  concern  about  whether  or  not  she   loves  him.     3)  It’s  a  doggy-­‐dog  world  –  It’s  a  dog-­‐eat-­‐dog  world.   This  phrase  is  misused  when  the  writer  wishes  to  express  the  fact  that  the  world  is   enormously  competitive  but  instead  expresses  the  unusual  notion  that  the  world  is   like  that  of  a  tiny  little  dog.     Incorrect  Usage   My  father  always  told  me  that  it’s  a  doggy-­‐dog  world  out  there.     Correct  Usage   My  father  always  told  me  that  it’s  a  dog-­‐eat-­‐dog  world  out  there.   In  the  incorrect  usage  above,  the  father  of  the  writer  has  informed  his  child  that  the   world  is  similar  to  a  cute,  little  puppy.  In  the  correct  example,  the  father  of  the   writer  has  informed  his  child  that  the  world  is  a  very  competitive  place  because   friends  and  colleagues  will  often  betray  or  destroy  each  other  in  order  to  get  ahead     4)  Nip  it  in  the  butt  –  Nip  it  in  the  bud   This  phrase  is  misused  when  a  writer  wishes  to  express  that  it  is  time  to  put   something  to  an  end  but  instead  states  that  it  is  time  to  literally  bite  the   hindquarters  of  something.     Incorrect  Usage   She  saw  the  way  that  she  came  on  to  him  and  decided  it  was  time  to  nip  it  in   the  butt  before  he  fell  prey  to  her  amorous  advances.   Take  your  S   AT  prep  to  the  next  level.  Visit  www.learnerator.com   316  

317       Correct  Usage   She  saw  the  way  that  she  came  on  to  him  and  decided  it  was  time  to  nip  it  in   the  bud  before  he  fell  prey  to  her  amorous  advances.     In  the  incorrect  example,  she  decided  to  bite  another  lady’s  flirtation  in  its  figurative   hindquarters  before  he  fell  for  this  lady’s  advances.  In  the  correct  example,  she   decided  to  put  an  end  to  another  lady’s  flirting  before  he  fell  for  this  lady’s  advances.     5)  Statue  of  Limitations  –  Statute  of  Limitations   This  phrase  is  misused  when  a  writer  wishes  to  express  that  there  is  a  deadline  for   filing  a  law  suit  but  instead  states  that  there  is  a  stone  carving  that  is  somehow   shaped  like  limitations  (whatever  that  shape  may  be).  The  writer  could  also  be   referring  to  a  statue  created  by  an  artist  named  Limitations.   Incorrect  Usage   The  statue  of  limitations  has  expired.   Correct  Usage   The  statute  of  limitations  has  expired.   In  the  incorrect  example,  the  writer  is  expressing  an  illogical  notion  that  somehow  a   stone  carving  has  expired.  In  the  correct  example,  the  writer  is  stating  that  a   criminal  charge  can  no  longer  be  legally  prosecuted  because  the  deadline  to  file  a   lawsuit  has  passed.    

Take  your  S   AT  prep  to  the  next  level.  Visit  www.learnerator.com   317  

318     6)  One  in  the  same  –  One  and  the  same   This  phrase  is  most  often  misused  when  the  writer  wishes  to  express  the  concept   that  two  things  are  alike  but  instead  expresses  the  illogical  notion  that  something  is   contained  within  an  object  referred  to  as  “the  same.”     Incorrect  Usage   They  are  one  in  the  same.     Correct  Usage   They  are  one  and  the  same.     In  the  incorrect  example  above,  multiple  entities  are  united  in  some  form  or  fashion   while  they  are  inside  of  an  object  known  as  “the  same.”  In  the  correct  example,  two   people,  two  animals,  or  two  things  are  actually  the  same.     7)  Got  off  scott  free  –  Got  off  scot  free   This  phrase  is  most  often  misused  when  the  writer  wishes  to  express  the  concept   that  someone  has  escaped  punishment  for  a  wrong  doing  or  misdeed  but  instead   claims  that  someone  charged  nothing  to  get  off  of  an  individual  named  Scott.     Incorrect  Usage   He  got  off  scott  free.     Correct  Usage   He  got  off  scot  free.     In  the  incorrect  example  above,  the  writer  states  that  a  man  removed  himself  from   an  individual  named  Scott  for  no  charge.  In  the  correct  example,  the  writer  states   that  a  man  has  escaped  punishment  for  a  crime.     8)  Windshield  –  Wind  chill   Take  your  S   AT  prep  to  the  next  level.  Visit  www.learnerator.com   318  

319     A  windshield  is  a  large  piece  of  glass  placed  near  the  front  of  an  automobile.  The   wind  chill  is  the  perceived  decrease  in  the  temperature  felt  by  the  human  body.     Incorrect  Usage   The  windshield  factor  was  30  degrees.     Correct  Usage   The  wind  chill  factor  was  30  degrees.     In  the  incorrect  example  above,  an  illogical  concept  is  expressed.  In  the  correct   example  above,  an  expression  of  the  perceived  outdoor  temperature  is  asserted.     9)  Irregardless  –  Regardless   The  word  "irregardless"  is  not  accepted  by  most  teachers  of  English;  however,  it  is   used  a  lot  in  common  speech  of  an  informal  nature.  It  is  a  type  of  double  negative   that  is  intended  to  mean  “without  regard;”  however,  the  term  "regardless"  means   “without  regard,”  so  the  prefix  ir-­‐  is  unnecessary.     Incorrect  Usage   One  should  use  words  correctly,  irregardless  of  how  other  people  may  use   them.     Correct  Usage   One  should  use  words  correctly,  regardless  of  how  other  people  may  use   them.     In  the  incorrect  example  above,  the  word  irregardlessis  used  and  could  be  viewed  as   a  type  of  double  negative.  In  the  correct  example  above,  the  word  regardless  is  used   and  the  sentence  is  grammatically  and  logically  perfect.       10)  Literally  -­‐  Figuratively   Take  your  S   AT  prep  to  the  next  level.  Visit  www.learnerator.com   319  

320     The  word  literally  is  often  used  when  the  word  figuratively  is  meant.  Oftentimes,  the   error  caused  by  this  type  of  usage  can  be  repaired  by  simply  removing  the  word   literally.     Incorrect  Usage   She  literally  drowned  herself  in  a  river  of  tears.     Correct  Usage   She  drowned  herself  in  a  river  of  tears.     In  the  incorrect  example  above,  the  writer  is  stating  that  an  individual  actually   drowned  in  a  real  river  that  was  created  by  tears.  This  would  also  imply  that  this   person  died-­‐-­‐literally.  However,  in  the  correct  usage,  the  “river  of  tears”  can  be   perceived  as  being  metaphorical;  therefore,  the  actual  meaning  of  the  sentence  is  a   creative  expression  of  a  person  crying  so  hard  that  an  above  average  amount  of   tears  were  shed  in  the  process.     Sample  Test  Questions   In  the  following  questions;  please  select  the  answer  that  CORRECTLY  expresses  the   idea  that  is  intended  by  the  conventional  expressions  used.    

Take  your  S   AT  prep  to  the  next  level.  Visit  www.learnerator.com   320  

321     1)   A)  The  man  was  literally  thrown  to  the  wolves.   B)  The  windshield  factor  was  20  below  zero.   C)  They  stood  together,  irregardless  of  the  outcome.   D)  Scott  got  off  scot  free.     2)   A)  For  all  intents  and  purposes,  I  could  care  less.   B)  For  all  intensive  purposes,  I  could  care  less.   C)  For  all  in  tents  and  porpoises,  I  couldn’t  care  less.   D)  For  all  intents  and  purposes,  I  couldn’t  care  less.     3)   A)  You  need  to  nip  it  in  the  bud,  because  it’s  a  doggy-­‐dog  world.   B)  You  need  to  nip  it  in  the  bud,  because  it’s  a  dog-­‐eat-­‐dog  world.   C)  You  need  to  nip  it  in  the  butt,  because  it’s  a  dog-­‐eat-­‐cat  world.   D)  You  need  to  nip  it  in  the  butt,  because  it’s  a  doggy-­‐dog  world.     4)   A)  The  statue  of  limitations  had  just  expired,  so  he  got  off  scott  free.   B)  The  statue  of  limitations  had  just  expired,  so  he  got  off  scot  free.   C)  The  statute  of  limitations  had  just  expired,  so  he  got  off  scott  free.   D)  The  statute  of  limitations  had  just  expired,  so  he  got  off  scot  free.   E)  All  of  the  sentences  are  incorrect.     ANSWERS   1. D)  An  individual  named  Scott  did  not  get  punished  for  a  crime  the  he   committed.   2. D)  For  all  practical  purposes,  the  writer  is  completely  apathetic  about  the   situation  at  hand.  

Take  your  S   AT  prep  to  the  next  level.  Visit  www.learnerator.com   321  

322     3. B)  You  need  to  stop  the  undesirable  behavior,  because  the  world  is  an   extremely  competitive  place.   4. D)  The  deadline  for  filing  a  lawsuit  has  passed,  so  the  individual  will  not  be   punished  for  a  crime  that  he  has  committed.  

Take  your  S   AT  prep  to  the  next  level.  Visit  www.learnerator.com   322  

323    

Conventions of Punctuation There are many types of uses for the various forms of punctuation in the English language. There are also many times when these punctuation marks are misused. The more popular types of these misuses will now be discussed.

End of sentence punctuation There are three different punctuation marks used at the ends of sentences. They are the period (declarative and imperative), the exclamation point (exclamatory and imperative), and the question mark (interrogative). Using a different punctuation mark at the end of a sentence can drastically alter its meaning. •

I  went  to  the  store.  (declarative  sentence  -­‐  states  a  fact)  



I  went  to  the  store!  (exclamatory  sentence  -­‐  expresses  extreme  emotion)  



I  went  to  the  store?  (interrogative  sentence  -­‐  asks  a  question)  

In addition, there are also imperative and imperative/exclamatory hybrid sentences. •

Go  to  the  store.  (imperative  sentence  -­‐  a  command)  



Go  to  the  store!  (imperative/exclamatory  -­‐  a  command  expressing  emotion)  

There are many instances when the context requires that a sentence uses a specific form of punctuation, or else the intended meaning may be altered. Incorrect Usages 1. Did  he  go  to  the  store.   2. "I  went  to  the  store!"  he  whispered  quietly  so  that  no  one  would  hear.   3. Will  she  go  to  the  store  after  she  gets  back  from  work.  

Take  your  S   AT  prep  to  the  next  level.  Visit  www.learnerator.com   323  

324     Correct Usages 1. Did  he  go  to  the  store?   2. "I  went  to  the  store,"  he  whispered  quietly  so  that  no  one  would  hear.   3. Will  she  go  to  the  store  after  she  gets  back  from  work?   In the first example above, the sentence is worded in a way that only allows it to be an interrogative sentence; therefore, it MUST end with a question mark. In the second example above, the context makes it clear that the exclamation point is inappropriate for the quoted material within the sentence. In the final example, the sentence is constructed in a manner that requires a question mark at the end.

Punctuation within the sentence There are three types of punctuation used to indicate sharp breaks of thought within a sentence: colons, semicolons, and em dashes. Colons

Colons are used immediately after independent clauses in order to present a list, an individual thing, or a quote. •

He  wanted  three  things  from  the  store:  grapes,  nuts,  and  avocadoes.  



John  said  it  the  best:  "I  don't  care  if  you  care  about  me  caring."  



The  was  only  one  thing  that  mattered:  his  money.  

By far, the most common colon error is committed when the colon does not follow an independent clause. Incorrect Usage 1. The  dessert  was  made  from:  bananas,  cherries,  and  oranges.   2. The  one  thing  we  can  do  is:  wait  for  the  firemen  to  arrive.   3. Henry  spoke  these  words  and  said  that:  "I  am  waiting  for  you  to  wait  for  me.  

Take  your  S   AT  prep  to  the  next  level.  Visit  www.learnerator.com   324  

325     Correct Usage 1. The  dessert  was  made  from  three  different  fruits:  bananas,  cherries,  and   oranges.   2. There  is  one  thing  we  can  do:  wait  for  the  firemen  to  arrive.   3. Henry  spoke  these  words:  "I  am  waiting  for  you  to  wait  for  me."   In all of the above examples, the errors are corrected by reconstructing the sentence so that the colon follows an independent clause that can stand alone as a complete sentence. Semicolons

There are two uses for semicolons: to connect two independent clauses and to separate lists of three or more items if there are commas used in the components of the list. This second type of usage will be discussed in an upcoming portion of this article. Semicolons can connect two independent clauses and can be used to correct comma splices. The two clauses that are connected should be related. •

He  fled  the  country  because  he  lost  the  election;  he  feared  for  his  life.  



The  little  kitty  was  playing  in  the  grass;  it  was  so  cute.  

The most common semicolon error occurs when at least one of the items that it connects is not a complete independent clause. Incorrect Usage 1. John  knew  that  he  was  funny;  people  laughing  at  his  jokes.   2. The  way  to  the  store;  five  blocks  to  the  south.   Correct Usage 1. John  knew  that  he  was  funny;  people  were  always  laughing  at  his  jokes.   2. The  way  to  the  store  was  obvious;  it  was  five  blocks  to  the  south.   Take  your  S   AT  prep  to  the  next  level.  Visit  www.learnerator.com   325  

326     In the first example above, the final clause of the sentence is only a phrase; it is corrected by converting it into an independent clause. In the second example above, both elements of the sentence are phrases and need to be converted into independent clauses. Em dashes

Em dashes are used to distinguish parenthetical expressions from the rest of the sentence. Commas and parenthesis are also used to perform this function. •

The  boy  was  not  scared  ̶  his  father  was  only  minutes  away.  



Kate  joined  in  on  the  fun  ̶  she  had  just  finished  her  chores.  

This usage of em dashes, parenthesis, and commas will be discussed in more detail in an upcoming section of this article.

Possessive Nouns and Pronouns When a noun or pronouns is possessive, an apostrophe and the letter s are attached to the end of the word. When a word is plural, then an apostrophe is added to the end of the word. Collective nouns (herd, flock, crowd, etc.) are singular. •

It  was  John's  fault.  



They  loved  all  of  the  oranges'  flavors;  there  were  two  dozen  of  them  in  the   bag.  



The  crowd's  applause  was  deafening.  

It is also important to note that there is no strict rule for adding an apostrophe and the letter s or just an apostrophe to singular nouns that end with the letter s. Thus, all of the sentences below could be considered correct depending on which authority is consulted.

Take  your  S   AT  prep  to  the  next  level.  Visit  www.learnerator.com   326  

327     •

It  was  Mrs.  Jones's  cat.  



It  was  Mrs.  Jones'  cat.  



The  canvas's  size  was  enormous.  



The  canvas'  size  was  enormous.  

The most common error with apostrophes occurs when a collective noun is treated as though it were plural. Also, many errors occur when a plural noun is pluralized as though it is singular. Incorrect Usage 1. The  ferocity  of  the  herds'  members  was  incredible.   2. The  apple's  colors  ranged  from  light  green  to  dark  red.   Correct Usage 1. The  ferocity  of  the  herd's  members  was  incredible.   2. The  apples'  colors  ranged  from  light  green  to  dark  red.   In the first example, the word herd is a collective noun that needs to be treated like a singular noun when it is converted into possessive form; thus, an apostrophe followed by the letter s is needed in order for the sentence to be correct. In the last example, the word apples is a plural noun and only needs an apostrophe at the end in order to be correct.

Items in a series The various components of lists of two or more items need to be separated with commas. If the Oxford (serial) comma is used, then the final item of a list of three or more items also needs to be separated with a comma just before the conjunction near the end of the list. The majority of most English writers now utilize the Oxford comma.

Take  your  S   AT  prep  to  the  next  level.  Visit  www.learnerator.com   327  

328     •

She  loved  to  play  with  puppies,  kitties,  and  bunnies.  (with  the  Oxford  comma)  



She  loved  to  play  with  puppies,  kitties  and  bunnies.  (without  the  Oxford   comma)  

The sample essay on the official ACT website uses the Oxford comma in the second and final paragraphs. http://www.actstudent.org/writing/sample/six.html Semicolons are used to separate the items in a list when the various items in the list contain commas. •

He  loved  to  eat  sweet,  ripe  oranges;  small,  delicate  peas;  and  big,  juicy  steaks.  



They  had  lived  in  San  Diego,  California;  Portland,  Oregon;  and  Houston,   Texas.  

A common error occurs when the Oxford comma is used with a list of two that employs a conjunction or when a comma is omitted in a list of two items that does not use a conjunction. In addition, a semicolon is often incorrectly used to separate the items of a list when a comma is required. Incorrect Usage 1. The  man  liked  to  eat  bananas,  and  peaches.   2. She  loved  sweet  chewy  raisins.   3. The  girl  liked  to  draw  pictures  of  monkeys,  horses;  whales,  dogs,  and  cats.   Correct Usage 1. The  man  liked  to  eat  bananas  and  peaches.   2. She  loved  sweet,  chewy  raisins.  OR  She  loved  sweet  and  chewy  raisins.   3. The  girl  liked  to  draw  pictures  of  monkeys,  horses,  whales,  dogs,  and  cats.   In the first example, the comma is unnecessary. In the second example, the list of two can be separated with either a conjunction or a comma. In the last example, the semicolon needs to be replaced with a comma. Take  your  S   AT  prep  to  the  next  level.  Visit  www.learnerator.com   328  

329     Nonrestrictive and parenthetical elements Nonrestrictive and parenthetical elements are set off from the rest of the sentence by using parentheses, em dashes, or commas. Parenthesis are the strongest method of setting of such elements that have little if any relation to the rest of the sentence. Em dashes are not quite as strong as parenthesis and are usually used for material that is at least somewhat relevant to the context of the sentence. Commas are the weakest and set off material that is very relevant to the sentence. •

The  mouse  played  (although  it  had  been  distracted  numerous  times)  with  the   cheese.  



The  mouse  played  ̶  it  was  quite  fond  of  playing  with  its  food  ̶  with  the  cheese.  



The  mouse  played,  and  kept  playing,  with  the  cheese.  

In the above examples, all of the parenthetical elements have been correctly set off from the sentences by using the three types of punctuation used for this type of situation. Incorrect Usage •

The  mouse  (in  the  garage)  was  killed  by  a  mousetrap.  



His  cousin  ̶  Bob  ̶  was  a  very  large  man.  

Correct Usage •

The  mouse  in  the  garage  was  killed  by  a  mousetrap.  



His  cousin  Bob  was  a  very  large  man.  (If  subject  has  more  than  one  cousin)  



His  cousin,  Bob,  was  a  very  large  man.  (If  subject  has  only  one  cousin)  

In the first example, the prepositional phrase in the garage is a restrictive element of the sentence and should not be set off with any type of punctuation. In the second example, comma usage is determined by how many cousins the subject

Take  your  S   AT  prep  to  the  next  level.  Visit  www.learnerator.com   329  

330     has. If the subject more than one cousin, then the word Bob is a restrictive element and should not be separated with commas. If the subject has only one cousin, then the word Bob is an nonrestrictive element and needs to be set off with commas. Restrictive or essential elements

Restrictive elements are necessary elements of a sentence that are needed in order for the meaning of the sentence to be perfectly clear. They are not separated from the rest of the sentence with commas. •

The  store  that  was  near  the  pizza  restaurant  was  burned  down.  



A  woman  who  was  in  the  store  saw  the  arsonist.  

In the first example above, the phrase that was near the pizza store tells the reader which particular store was burned down. In the second example above, the phrase who was in the store lets the reader know which woman saw the arsonist. Both of these phrases are essential so that the reader has all of the detail needed to understand the concepts expressed by these sentences. Incorrect Examples 1. The  security  guard,  who  worked  the  night  shift  at  the  mall,  was  fat.   2. The  mall,  which  was  near  the  city  zoo,  was  being  renovated  since  the  other   malls  in  town  were  doing  much  better.   Correct Examples 1. The  security  guard  who  worked  the  night  shift  at  the  mall  was  fat.   2. The  mall  that  was  near  the  city  zoo  was  being  renovated  since  the  other  malls   in  town  were  doing  much  better.  

Take  your  S   AT  prep  to  the  next  level.  Visit  www.learnerator.com   330  

331     In the first example above, the phrase who worked the night shift at the mall is an essential element that lets the reader know which security guard is the subject of the sentence; therefore, it should not be separated from the rest of the sentences with commas. In the second example above, the phrase which was near the city zoo is a restrictive element that lets the reader know which mall is being talked about in the sentence; thus, it should not be separated from the rest of the sentence with commas.

Unnecessary punctuation There are many ways that punctuation can be used incorrectly in a sentence; however, there are a few situations that are very common. These more popular errors will now be discussed. Unnecessary apostrophes

Oftentimes, a writer will add an apostrophe when creating the plural form of a word. Incorrect Examples 1. The  girl's  played  in  the  backyard.   2. There  were  many  different  kinds  of  football's  at  the  store.   Correct Examples 1. The  girls  played  in  the  backyard.   2. There  were  many  different  kinds  of  footballs  at  the  store.   In the first example above, the word girls has an unnecessary apostrophe. In order to fix this sentence, the apostrophe needs to be removed. In the second example, the word footballs contains an unnecessary apostrophe that needs to be removed.

Take  your  S   AT  prep  to  the  next  level.  Visit  www.learnerator.com   331  

332     Unnecessary quotation marks

A writer will often use quotation marks on material that is not a direct quote. These unnecessary quotes are placed so that these words will receive more emphasis. Incorrect Examples 1. He  said  that  he  saw  a  "light  at  the  end  of  the  tunnel."   2. He  said  that  the  "party"  which  he  attended  last  night  was  very  lame.   Correct Examples 1. He  said  that  he  saw  a  light  at  the  end  of  the  tunnel.   2. He  said  that  the  party  which  he  attended  last  night  was  very  lame.   In the above examples, there are no direct quotations; however, quotations marks are used to add emphasis. Merely remove these unnecessary quotation marks, and both sentences will be grammatically correct. Incorrect comma usage

Commas are frequently misused by either omitting them in necessary places or by placing them in unnecessary places. Commas are often omitted in necessary places when they are not placed before a coordinating conjunction in a compound sentence. This is probably the most frequent mistake made by writers of the English language. If

the writer does not have the seven

coordinating conjunctions memorized, then this mistake will continue throughout everything that a writer creates. However, if the independent clauses are very short, then the comma becomes optional. •

He  decided  to  go  to  the  store,  and  then  he  decided  to  go  to  the  movies.  



She  went  to  the  downtown  museum,  so  she  could  photograph  some  of  the   art.  

Take  your  S   AT  prep  to  the  next  level.  Visit  www.learnerator.com   332  

333     •

He  slept  so  she  cooked  breakfast.  

In addition, writers often omit commas after the end of a dependent clause that begins a complex sentence. These commas are very necessary in order for a sentence to maintain clarity. •

After  she  went  to  the  store,  she  went  to  the  movies.  



Before  he  went  to  the  movies,  he  took  a  walk  in  the  park.  

Writers will often place unnecessary commas in sentences. This situation usually occurs when a writer places a comma before a second verb in a sentence that has only one subject; however, this sort of error also occurs when a writer places a comma before a subordinating conjunction that precedes a subordinating clause at the end of a complex sentence. •

She  went  to  the  store  and  then  took  a  walk  in  the  park.  



He  went  to  the  museum  before  he  went  to  the  store.  

In the first example above, there is only one subject, so there is no need for a comma before the conjunction and. In the second example, the conjunction before is a subordinating conjunction and does not require a comma directly before it. Incorrect Usages 1. He  went  to  the  downtown  zoo  and  then  he  went  to  the  pet  store.   2. He  went  to  the  downtown  zoo,  and  then  went  to  the  pet  store.   3. After  she  went  to  the  downtown  zoo  she  went  to  the  pet  store.   4. She  went  to  the  downtown  zoo,  after  she  went  to  the  pet  store.   Correct Usages 1. He  went  to  the  downtown  zoo,  and  then  he  went  to  the  pet  store.   2. He  went  to  the  downtown  zoo  and  then  went  to  the  pet  store.   Take  your  S   AT  prep  to  the  next  level.  Visit  www.learnerator.com   333  

334     3. After  she  went  to  the  downtown  zoo,  she  went  to  the  pet  store.   4. She  went  to  the  downtown  zoo  after  she  went  to  the  pet  store.   In the first example, two independent clauses are connected by the coordinating conjunction and; therefore, a comma is required before the word and. In the second example, there is only one subject, so there is no need for a comma before the word and. In the third example, a subordinating clause begins a complex sentence; therefore, a comma is needed after the clause. In the last example, a subordinating clause ends a complex sentence; therefore, no comma is needed. In order to know when to correctly use commas in compound and complex sentences, the writer MUST have the seven coordinating conjunctions memorized. The seven coordinating conjunctions are for, and, nor, but, or, yet, and so. These seven words can be easily memorized by using the acronym FANBOYS.

All other conjunctions that are commonly used to connect sentences are

subordinating conjunctions. Important exception

It is imperative to note that the word so is a coordinating conjunction; however, when it is combined with the word that, it becomes a two-word subordinating conjunction. Therefore, correct comma placement is indicated in these two examples. •

Rebecca  drove  to  the  lake  so  that  she  could  see  her  uncle.  



Rebecca  drove  to  the  lake,  so  she  could  see  her  uncle.  

Excess exclamation points

Only one exclamation point is needed at any single point in a sentence. To use more than one at a single location is grammatically incorrect. Incorrect Usage 1. They  went  to  the  store,  and  then  they  went  to  the  movies!!!!   Take  your  S   AT  prep  to  the  next  level.  Visit  www.learnerator.com   334  

335     2. "Hey!!!"  she  yelled.  "Get  over  here  right  now!!"   Correct Usage 1. They  went  to  the  store,  and  then  they  went  to  the  movies!   2. "Hey!"  she  yelled.  "Get  over  here  right  now!"   In the examples above, multiple exclamation points are used at single locations. In order to correct these sentences, omit the extra exclamation points. Incorrect quotation mark placement

Oftentimes, a writer will incorrectly place punctuation outside of a set of quotation marks at the end of a quote. Incorrect Usage 1. "Hello  there,"  he  said.  "Let's  go  to  the  store".   2. "Hey  there"!  she  said.  "Let's  go  to  the  mall."   Correct Usage 1. "Hello  there,"  he  said.  "Let's  go  to  the  store."   2. "Hey  there!"  she  said.  "Let's  go  to  the  mall."   In the first example above, a period is placed outside of the quotation marks; therefore, in order to correct the sentence, the period should be placed within the quotes. In the second example, an exclamation point is placed outside of the quotes; therefore, it needs to be placed inside the quotes so that the sentence will be correct. Hyphen, En dash, and Em dash usage

There is much confusion over these three closely related punctuation marks, and they are commonly confused an a wide variety of ways. Therefore, the specific

Take  your  S   AT  prep  to  the  next  level.  Visit  www.learnerator.com   335  

336     usages for each type of mark will be presented so that a comprehensive understanding may be gained. The hyphen is used to connect two adjectives in a way that allows for a combined meaning of the two words. It is also used to connect two or three words that have been combined to produce a single idea. •

It  was  a  low-­‐budget  project  that  required  clear-­‐headed  thinking.  



Her  mother-­‐in-­‐law  had  just  bought  an  up-­‐to-­‐date  gardening  manual.  

The hyphen is also used to connect numbers and fractions as words. It is also used after the prefixes ex-, self-, and all-. •

One-­‐third  of  twenty-­‐one  is  seven.  



His  ex-­‐wife  has  an  all-­‐inclusive  plan  for  self-­‐righteousness.  

The en dash is used to connect two items that are related to each other by distance. •

He  read  pages  34  ̵  56  



He  read  from  the  September  ̵  December  issue  of  the  journal.  

The em dash is used to set off nonrestrictive elements in a sentence. This welldefined usage ̶ as well as its relationship to parentheses and commas ̶ was explained on pages 4 ̵ 5 of this article. Incorrect Usages 1. He  thought  that  he  was  finished  with  pages  56-­‐58  of  the  script.   2. Doug  bred  rabbits  in  the  fall-­‐and  guinea  pigs  in  the  spring-­‐on  his  farm.   3. His  ex  ̶  wife  was  very  upset  with  him.   4. They  found  thirty  ̵  four  errors  in  the  text.   Correct Usages Take  your  S   AT  prep  to  the  next  level.  Visit  www.learnerator.com   336  

337     1. He  thought  that  he  was  finished  with  pages  56  ̵  58  of  the  script.   2. Doug  bred  rabbits  in  the  fall  ̶  and  guinea  pigs  in  the  spring  ̶  on  his  farm.   3. His  ex-­‐wife  was  very  upset  with  him.   4. They  found  thirty-­‐four  errors  in  the  text.   In the first example, a hyphen is used to connect two items that are related by distance; thus, the sentence requires an en dash in order to be correct. In the second example, hyphens are used to set off a parenthetical expression from the rest of the sentence; thus, en dashes may be used in order to make this sentence correct. In the third example, an em dash is used in a common expression that requires a hyphen. In the final example, an en dash is used for a written expression of numbers; therefore, a hyphen needs to be used. Sample Test Questions

1) Identify which sentence contains an INCORRECT usage of punctuation. A) Her mother read the book. B) Her mother read the book? C) Her mother read the book D) Her mother read the book! 2) Identify which sentence contains an INCORRECT usage of punctuation. A) Elizabeth only wanted three things: money, fun, and candy. B) She knew what she wanted; she spent her time acquiring them. C) There was no time for amateurs; however, they insisted on being there. D) The middle of the night; it was a time for reflection. 3) Identify which sentence contains an INCORRECT usage of punctuation.

Take  your  S   AT  prep  to  the  next  level.  Visit  www.learnerator.com   337  

338     A) It was John's fault. B) It was Johns fault. C) It is John's fault. D) It is his fault. 4) Identify which sentence contains an INCORRECT usage of punctuation. A) They wanted beets, yams, and radishes. B) They wanted beets; yams and radishes. C) They wanted beets, yams, radishes, and cucumbers. D) They wanted three things: beets, yams, and radishes. 5) Identify which sentence contains an INCORRECT usage of punctuation. A) The cat over by the window was the cutest cat in the house. B) The cat, over by the window, was the only cat in the house. C) The cat was the cutest cat in the house. D) The cat over by the window was the only cat in the house. E) The cat was the only cat in the house. 6) Identify which sentence contains an INCORRECT usage of punctuation. A) The girl's played in the field. B) The girls played in the field. C) The girls played and played in the field. D) The girls played. E) The girls played baseball in the field. ANSWERS

Take  your  S   AT  prep  to  the  next  level.  Visit  www.learnerator.com   338  

339     1. C)  This  sentence  needs  some  type  of  punctuation  at  its  end.   2. D)  The  first  section  of  this  sentence  is  not  an  independent  clause;  therefore,  it   cannot  be  separated  by  another  independent  clause  with  a  semicolon.   3. B)  The  word  Johns  is  the  possessive  form  of  a  proper  noun  and  requires  an   apostrophe  before  the  letter  s.   4. B)  A  semicolon  is  used  incorrectly  in  a  list  of  three  items.   5. D)  Since  the  cat  is  the  only  cat  in  the  house,  the  phrase  over  by  the  window  is   nonrestrictive  and  should  be  separated  from  the  rest  of  the  sentence  with   commas.   6. A)  The  word  girl's  is  not  possessive  and  does  not  need  an  apostrophe.    

Take  your  S   AT  prep  to  the  next  level.  Visit  www.learnerator.com   339  

340    

Sign up for Learnerator SAT Prep!

 

    We’ll  give  you  everything  you  need  to  start  mastering  the  SAT  today.   Our  online  test  prep  offers:   •

Over 300 SAT Integrated Reading & Writing practice questions



Over 300 SAT Math practice questions



Hundreds of practice questions for SAT IIs: Biology, Chemistry, French, Literature, Math 1 & 2, Physics, US History, World History



Detailed personalized statistics based on your performance



A competitive online leaderboard to see how you stand compared to others



Convenience with anytime access from an Internet-enabled device Visit  http://www.learnerator.com     Use  SAT2016  for  10%  off  any  Learnerator  SAT  Subject  Guide  

  Take  your  S   AT  prep  to  the  next  level.  Visit  www.learnerator.com   340  

View more...

Comments

Copyright ©2017 KUPDF Inc.
SUPPORT KUPDF